Download as pdf or txt
Download as pdf or txt
You are on page 1of 637

Page i

Clinical Cases
for General
Practice
Exams

4e
Page ii

Acknowledgments
I am delighted that Dr Andrew Moreton, Dr Leela Arthur, Dr Rebekah
Ledingham and Dr Lucas Wheatley were willing to join me in writing this
fourth edition. They are experienced and compassionate clinicians with a
great desire and ability to help doctors learn through the process of passing
exams. Their cases challenged and taught me when I tried them out, and they
are a very welcome addition.
I would also like to express my appreciation to all those who helped with
the previous editions of this book. Dr Linda Samera, Professor John
Wakerman, Dr Ethel Gilbert, Dr Patrick Mutandwa, Dr Nina Kilfoyle, Dr
Chris Lesnikowski, Dr Rob Roseby, Dr Peter Tait and Dr Rupa Vedantam
were a great help with the first edition. The second edition benefited from
input from Dr Pat Giddings, Dr Louise Baker, Dr Trudi Cullinan, Dr Tyler
Schofield, Dr Daniel Cloughton, Dr Danielle Butler, Dr Katrina Page, Dr
Louise Butler, Dr Sarah Koh, Dr Charles Mutandwa, Dr Ann Dunbar, Angela
Beilby and Dr Tim Henderson. For the third edition I was helped by Dr
Andrew Moreton, Dr Genevieve Yates, Dr Bambi Ward, Dr Robert Menz, Dr
Sarah Kloeden and Dr Sally Banfield.
The photographs in Figures 1 and 3 were supplied by iStockphoto; the
Eczema Association of Australasia Inc. provided the photograph for Figure 2;
Dr Andrew Moreton supplied the photographs in Figures 4, 6 and 7; and Dr
Brendan Bell supplied the ECG in Figure 5.
Page iii

CLINICAL CASES SERIES

Clinical Cases
for General
Practice
Exams
4e

ASSOCIATE PROFESSOR SUSAN


WEARNE
Page iv

NOTICE
Medicine is an ever-changing science. As new research and clinical experience broaden our knowledge,
changes in treatment and drug therapy are required. The editors and the publisher of this work have
checked with sources believed to be reliable in their efforts to provide information that is complete and
generally in accord with the standards accepted at the time of publication. However, in view of the
possibility of human error or changes in medical sciences, neither the editors, nor the publisher, nor any
other party who has been involved in the preparation or publication of this work warrants that the
information contained herein is in every respect accurate or complete. Readers are encouraged to
confirm the information contained herein with other sources. For example, and in particular, readers are
advised to check the product information sheet included in the package of each drug they plan to
administer to be certain that the information contained in this book is accurate and that changes have
not been made in the recommended dose or in the contraindications for administration. This
recommendation is of particular importance in connection with new or infrequently used drugs.
This fourth edition published 2019
First published 2005, second edition 2010, third edition 2015
Reprinted 2011, 2012, 2013 (twice), 2016
Text © 2019 Susan Wearne
Illustrations and design © 2019 McGraw-Hill Australia Pty Ltd
Every effort has been made to trace and acknowledge copyrighted material. The authors and publishers
tender their apologies should any infringement have occurred.
Reproduction and communication for educational purposes
The Australian Copyright Act 1968 (the Act) allows a maximum of one chapter or 10% of the pages of
this work, whichever is the greater, to be reproduced and/or communicated by any educational
institution for its educational purposes provided that the institution (or the body that administers it) has
sent a Statutory Educational notice to Copyright Agency (CA) and been granted a licence. For details
of statutory educational and other copyright licences contact: Copyright Agency, 66 Goulburn Street,
Sydney NSW 2000. Telephone: (02) 9394 7600. Website: www.copyright.com.au
Reproduction and communication for other purposes
Apart from any fair dealing for the purposes of study, research, criticism or review, as permitted under
the Act, no part of this publication may be reproduced, distributed or transmitted in any form or by any
means, or stored in a database or retrieval system, without the written permission of McGraw-Hill
Australia including, but not limited to, any network or other electronic storage.
Enquiries should be made to the publisher via www.mcgraw-hill.com.au or marked for the attention
of the Rights and Permissions Manager at the address below.

National Library of Australia Cataloguing-in-Publication Entry


Author: Wearne, Susan
Title: Clinical cases for general practice exams
Edition: 4th edition
ISBN: 9781743767450 (paperback)
eBook ISBN: 9781743767467

Published in Australia by
McGraw-Hill Australia Pty Ltd
Level 33, 680 George Street, Sydney NSW 2000
Publisher: Diane Gee-Clough
Production Editor: Lara McMurray
Editor: Yani Silvana
Cover design: ChristaBella Designs
Internal design: SPi, India
Proofreader: Meredith Lewin
Typeset in 10/12 pt Nimbus by SPi, India
Page v

Contents

Acknowledgments
Preface
About the author and contributing authors
List of abbreviations
Introduction
How to use this book

Section 1 Aboriginal health


Case 1 Kasey Kox
Case 2 Sharon Price

Section 2 Adolescent health


Case 3 Erin Campbell
Case 4 Amanda Porter

Section 3 Aged care


Case 5 Elsie Humphries
Case 6 Flora McMillan
Case 7 Nell Worthington
Case 8 Margaret Wilson

Section 4 Cardiovascular system


Case 9 Helen Berkovic
Case 10 Dilip Patel
Case 11 Jackie Maloney
Case 12 Eric Schmidt

Section 5 Challenging consultations


Case 13 Doug Sullivan
Case 14 Jeanette Wilkinson
Case 15 Craig Kelly
Case 16 Wazza Wainright
Case 17 Hope Briganza

Section 6 Child health


Case 18 Kylie Chong
Case 19 Brandon Harkness
Case 20 Natalie Jones
Case 21 Latu O’Donnell

Section 7 Dermatology
Case 22 Sammy Burnside
Case 23 Robert Kerslake Page vi

Case 24 Ken Anderson

Section 8 Ear, nose and throat


Case 25 Ruby Chan
Case 26 Jane Matthews
Case 27 Pamela Taylor
Case 28 Trevor Watts
Case 29 Clayton Dixon

Section 9 Emergency medicine


Case 30 Catriona Chryssides
Case 31 Carrie Patterson

Section 10 Endocrinology
Case 32 Veronica Richards

Section 11 Eyes
Case 33 Edward Galloway
Case 34 Henrik Schneider
Case 35 Roger Chin

Section 12 Gastroenterology
Case 36 Jenna Banks
Case 37 Enrico Castallani
Case 38 Kirrilee DeMarco
Case 39 Mohammed Noor
Case 40 Annie Nguyen
Case 41 Kathy Jones
Case 42 Jack Kingsley
Case 43 Neil Dawson

Section 13 Men’s health


Case 44 Kim Hosking
Case 45 Jock Palmer
Case 46 Costa Rinaldi

Section 14 Mental health


Case 47 Phyllis Brown
Case 48 Shirley Hill
Case 49 Monica Middlethorpe
Case 50 Tom Newton

Section 15 Musculoskeletal medicine


Case 51 Anthony Campbell
Case 52 Martin Chatterjee
Case 53 Sarah Cosgrove
Case 54 Jeremy King
Case 55 Geoff Sharp
Case 56 Anna Wong

Section 16 Neurology Page vii

Case 57 Wilma Burns


Case 58 Sybil Clarke
Case 59 Rosie Inkamala
Case 60 Joe Summers

Section 17 Palliative care


Case 61 Liz Ross
Case 62 Frank Stanley
Case 63 Katrina Carroll

Section 18 Preventive health


Case 64 Bill Ferguson
Case 65 Taylor Jordan
Case 66 Ali Turnbull

Section 19 Professional practice


Case 67 Vincent Butler
Case 68 Stephanie Clark
Case 69 Debra and Declan Poole

Section 20 Respiratory medicine


Case 70 Andrew Bond
Case 71 Kerrie Griffiths
Case 72 Paul Jackson
Case 73 Nicholas Morris
Case 74 Jonty McLeod

Section 21 Sexual health


Case 75 Ben Ramsay
Case 76 Vinay Singh
Case 77 Samantha Heyward

Section 22 Travel health


Case 78 Tanya Hardy
Case 79 Betty Ward

Section 23 Women’s health


Case 80 Jenny Butterfield
Case 81 Vikki Nicolaides
Case 82 Shantelle Kickett
Case 83 Zahra Mohammed Ibrahim
Section 24 Vivas
Case 84 Lori Dalton
Case 85 Kaitlin Johansen

Section 25 Vulnerable populations


Case 86 Jill Krecher
Case 87 Marcus Petrovic
Page viii

Preface

This is the book that I looked for when I first moved to Australia. A group of
GP registrars wanted help preparing for their Fellowship exam and examples
of clinical cases did not exist, so I began writing questions. This book is the
teaching material that I have developed and refined since then.
It has been rewarding to hear many stories of doctors who found the book
useful. Doctors have persuaded non-medical friends, partners and spouses to
role-play patients and the instructions have proved adequate to provide
realistic exam practice.
One GP registrar expressed concern that the book was too ‘cook book’ or
formulaic. Forgive me if I have unintentionally promoted uniformity. Each
time I see a GP at work or in a role-play, I see new ways of being a GP. Each
of us brings our personality and experience to our role, but some core
principles apply to each consultation. I hope that my practice will forever
include the formula of an introduction, building rapport to understand
something of the person’s life and situation, hearing their concerns, assessing
the problem in bio–psycho–social terms and working with them to create a
plan that they understand and want to implement.
We have to judge each situation, without being judgemental, listen intently
and be supportive without being patronising. How I achieve this varies for
different patients and makes the job both challenging and satisfying. This
‘how’ of calibrating words and actions to individuals is crucial in exams and
in practice; the ‘what’ of creating therapeutic relationships, and explaining
medical terms is different for fractious children, over-stretched middle-aged
professionals or an elderly person facing losing health and their
independence.
Most of the cases are adapted from my clinical practice and identifying
details have been changed to protect the patient’s identity. If you think you
can spot yourself or a friend it is only because these cases cover common
clinical conditions. I appear in them and, with their permission, so do many
of my family.
I hope that this book assists medical students, GP registrars and GPs to
learn more about the art and science of being an effective GP.
Page ix

About the author and contributing


authors

Associate Professor Susan Wearne


BM PhD MMedSci FRACGP FACRRM MRCGP DRCOG DCH DFFP
GCTEd Susan Wearne is a GP in Canberra and a Clinical Associate
Professor at the Australian National University (ANU). She is Senior Medical
Adviser for the Commonwealth Department of Health, in the Division which
manages general practice training.
Susan is from England; she grew up in Alderley Edge, Cheshire and
qualified in medicine from Southampton University. After GP training in
Kettering, Northamptonshire and Wilmslow, Cheshire, she was a partner in
training practices in Manchester and York. She started her academic career
teaching communication skills at Leeds University, and then moved to the
Centre for Remote Health in Alice Springs. Susan was a medical educator for
the Royal Australian College of General Practitioners, Northern Territory
General Practice Education and the Remote Vocational Training Scheme. She
is an examiner for ANU, the Royal Australian College of General
Practitioners and the Australian College of Rural and Remote Medicine. She
has published over 40 articles on general practice and medical education.
Susan is married to Tim Henderson, an ophthalmologist at Alice Springs
Hospital. Their married daughter Charlotte lives in England and their son
Michael works in Adelaide. Susan is a member of the Alice Aussi Swimming
Club and enjoys needlework. Since her job moved to Canberra she has taken
up rowing and knitting—but not simultaneously!
Contributing authors
Dr Andrew Moreton Andrew Moreton graduated from the University of
Queensland in 1987 and became a Family Medicine trainee with the Royal
Australian College of General Practitioners (RACGP) in Bourke, New South
Wales, where he went on to practise for 15 years. Having completed his
FRACGP he became involved in case-writing, examining and quality
assurance for the FRACGP exams. He is passionate about teaching and
learning in medicine and has enjoyed roles teaching GP registrars, GP
supervisors and medical students for almost 20 years. He has special interests
in communication skills, Indigenous and rural health, exam preparation and
doctor’s self-care. His cases are drawn from the rich tapestry of life that is
general practice.

Dr Leela Arthur
Leela Arthur BSc (Hons 1), MBBS, FRACGP, DCH graduated from the
University of Queensland in 2008, then completed her FRACGP in Hervey
Bay in 2015. She has since moved back to her home town of Brisbane Page x
where she is now working both in general practice, and as a ‘GP with
Special Interest’ in an irritable bowel syndrome clinic within the hospital
system. She has a keen interest in teaching medical students and registrars
and has been a medical educator for Queensland Rural Medical Education for
several years. More recently, she has become an examiner for the RACGP.
She has a particular interest in chronic disease management and refugee
health.
In her spare time, Leela enjoys going on adventures with her husband and
children, and singing women’s barbershop with her quartet Kit’n’Kaboodle,
and chorus Brisbane City Sounds.

Dr Rebekah Ledingham
Rebekah Ledingham Bsc (Nursing); BMBS; DRANZCOG; DCH; FRACGP
is a GP and medical educator in Broome, Western Australia. She is
passionate about Aboriginal health and committed to Closing the Gap and
supporting an increasing number of Aboriginal and Torres Strait Islander
people in the health workforce. She is also a keen advocate for the well-being
of medical students and doctors in training, and would like to see a more
nurturing, flexible system in which to grow our future workforce. She
founded the online group Medical Mums and Mums to be, now a support
network for more than 7000 doctors, after struggling with the prospect of
being a junior doctor and a mum simultaneously, but remains eternally
thankful that she ignored the (ridiculous) advice not to have babies, as they
are growing into fabulous humans who help it all make sense.

Dr Lucas Wheatley
Lucas Wheatley BBiomedSc, MBBS, FRACGP, GDipClinEd,
GDipSurgAnat, MPH qualified in Medicine in 2009 and entered service with
the Royal Australian Air Force. After completing his FRACGP and MPH,
with several tours overseas working in Primary Care and Aeromedical
Retrievals, Lucas returned to Brisbane, Queensland. Furthering his scope of
practice with additional training in surgery and critical care, Lucas splits his
time between working for Queensland Health and the Australian Defence
Force. Focusing on clinical and procedural training, Lucas has an interest in
medicine in austere and rural environments, with a passion for registrar and
medical student education, examining for the University of Queensland and
Bond University.

Dr Genevieve Yates
Genevieve Yates is Associate Director of Training for North Coast GP
Training, a general practitioner and medical writer. She also delivers medico-
legal education sessions and develops resource materials for MDA National
and is an examiner for the RACGP and a member of the RACGP Fellowship
Support Panel. In 2014, she was named GPET Medical Educator of the Year.
She has particular interests in doctors’ health and wellbeing, ethics and
professionalism, and the medical humanities.
Page xi

List of abbreviations

Abbreviation Meaning
24hrECG 24 hour electrocardiogram
ABPI ankle blood pressure index
ACE angiotensin converting enzyme
ACR albumin-creatinine ratio
ACRRM Australian College of Rural and Remote Medicine
ACS acute coronary syndrome
ADHD attention deficit hyperactivity disorder
ADLs activities of daily living
ADT adult diphtheria-tetanus vaccine
AFB acid fast bacilli (tuberculosis)
AF atrial fibrillation
alb albumin
Anti CCP anti-cyclic citrullinated peptide antibody
ARB angiotensin receptor blocker
b.d. twice a day (bis die)
bHCG beta human chorionic gonadotropin
bili bilirubin
BMI body mass index
BP blood pressure
bpm beats per minute
BSL blood sugar level
CABG coronary artery bypass grafting
CHA2DS2VASc calculates risk of ischaemic stroke
Chol cholesterol
CI confidence interval
CK creatine kinase
Cl chloride
Coags coagulation tests
COPD chronic obstructive pulmonary disease
COX2 inhibitors cyclo-oxygenase 2 inhibitors
CPD continuing professional development
Creat creatinine
CRP C-reactive protein
CST cervical screening test
CT computerised tomography
CXR chest X-ray
dTpa diphtheria, tetanus and pertussis vaccine adult formulation
DNA did not attend Page xii
DMARDs disease-modifying anti-rheumatic drugs
DVT deep venous thrombosis
ECG electrocardiogram
ED emergency department
EDNOS eating disorder not otherwise specified
ELISA enzyme linked immunosorbent assay
EPDS Edinburgh Postnatal Depression Score
ERCP endoscopic retrograde cholangiopancreatography
ESR erythrocyte sedimentation rate
FAI free androgen index
FBC/E full blood count/examination
FEV1 forced expiratory volume in 1 second
FODMAPs fermentable oligosaccharides, disaccharides, monosaccharides
and polyols
FSH follicle stimulating hormone
FVC forced vital capacity
GA general anaesthetic
GCS Glasgow coma scale
GGT gamma-glutamyl transferase
GP general practitioner
GPM gravida, parity, miscarriages
GTN glyceryl trinitrate
GTT glucose tolerance test
HAS-BLED calculates the risk of bleeding from anticoagulation in patients
with atrial fibrillation
Hb haemoglobin
HbA1c haemoglobin A1c
HCO3 bicarbonate
HDLChol high density lipoprotein cholesterol
Hep B hepatitis B
Hep C hepatitis C
Hib haemophilus influenzae type B vaccination
HIV human immunodeficiency virus
HPV human papillomavirus
HRT hormone replacement therapy
HZV herpes zoster vaccine
IBS irritable bowel syndrome
IM intramuscular injection
INR international normalised ratio
IUCD intrauterine contraceptive device
IV intravenous
JVP jugular venous pulse
K potassium
kg kilogram
LBP lower back pain Page xiii
LDH/LH lactate dehydrogenase
LDLChol low density lipoprotein cholesterol
LFTs liver function tests
LH luteinising hormone
LMP first day of last menstrual period
LSCS lower segment Caesarean section
LUTS lower urinary tract symptoms
m metre
mane in the morning
MCS microscopy, culture and sensitivity
MCP metacarpophalangeal joints
MCV mean cell volume
mg milligram
MRI magnetic resonance imaging
MSU midstream urine
MTHFR methylenetetrahydrofolate reductase
MTP metatarsophalangeal joints
MVA motor vehicle accident
Na sodium
NAAT nucleic acid amplification test
NAD no abnormality detected
NH&MRC National Health and Medical Research Council
NIDDM non-insulin dependent diabetes mellitus
NIPT non-invasive pre-natal testing
nocte at night
NPS National Prescribing Service
NRT nicotine replacement therapy
NSAIDs non-steroidal anti-inflammatory drugs
OA osteoarthritis
OCP oral contraceptive pill
ocp ova, cysts and parasites
od once a day
OGTT oral glucose tolerance test
OTC over the counter—medication bought from a pharmacy without
a prescription
Pap Papanicolaou
PBS Pharmaceutical Benefit Scheme
PCOS polycystic ovarian syndrome
PCR polymerase chain reaction
PEFR peak expiratory flow rate
PMH past medical history
PND paroxysmal nocturnal dyspnoea
PR per rectum
PrEP pre-exposure prophylaxis
prn as and when needed (pro re nata) Page xiv
Prot protein
PSA prostate specific antigen
qds four times a day (quarter die sumendus)
RACGP Royal Australian College of General Practitioners
RDW red cell distribution width
RFDS Royal Flying Doctor Service
RPR rapid plasma reagin test for syphilis
RR relative risk
sc subcutaneous
SHBC steroid hormone binding globulin
SLR straight leg raising
SOB shortness of breath

SSRI selective serotonin reuptake inhibitor


STI sexually transmitted infection
SVD spontaneous vaginal delivery
TB tuberculosis
TCA tricyclic antidepressant
tds three times a day (ter die sumendus)
temp temperature
TFTs thyroid function tests
TIA transient ischaemic attack
TMJ temporomandibular joint
TSH thyroid stimulating hormone
TURP transurethral resection of prostate
UEC urea, electrolytes and creatinine
Vit D Vitamin D
WHO World Health Organization
WONCA World Organization of National Colleges, Academies and
Academic Associations of General Practitioners/Family
Physicians aka World Organization of Family Doctors
Page xv

Introduction

The aim of this book is to help you prepare for Australian general practice
exams using clinical case role-plays.
General practice activity data1 indicate that general practitioners’ work has
become increasingly complicated: patients have multiple comorbidities and
are prescribed more drugs. My impression is that more research information
can counter-intuitively create less certainty. GPs spend more time assessing
patients’ personal risk–benefit of any particular intervention. Screening for
prostate cancer and prescribing hormone replacement therapy are prime
examples of where there is a high risk of iatrogenic disease. The GP needs to
be an expert in reading patients’ personalities and fears, as well as scientific
data. The cases in this book test the skill of taking a holistic approach to
managing uncertainty in a complex context.
Most doctors find the time restrictions of the exam difficult. Three minutes
is not long to read a patient’s notes but may be longer than many doctors take
to do this in clinical practice. Likewise, dealing with a problem in eight
minutes, challenges some candidates, while others expertly breeze through
with time to spare to include opportunistic health promotion, confirming that
the examiners do not ask the impossible. Cases should be quicker than in real
life as the ‘patient’ knows in advance the answers to the important questions
and candidates are not required to make clear, contemporaneous written
notes.

Definitions of general practice


Before you take a general practice exam you need to have a clear
understanding of general practice. Let’s look at some definitions of general
practice (also called family practice in some countries).
The Royal Australian College of General Practitioners (RACGP) says:2
General practice is part of the Australian health care system and operates
predominantly through private medical practices, which provide universal
unreferred access to whole person medical care for individuals, families and
communities. General practice care means comprehensive, coordinated and
continuing medical care drawing on biomedical, psychological, social and
environmental understandings of health.
The World Organization of Family Doctors (WONCA Europe) defines the
competencies and characteristics of general practitioners as:3
• normally the point of first medical contact within the health care system,
providing open and unlimited access to its users, dealing with all health
problems regardless of the age, sex or any other characteristic of the
person concerned
• making efficient use of health care resources through coordinating care,
working with other professionals in the primary care setting and by
managing the interface with other specialties, taking an advocacy role for
the patient when needed
• developing a person-centred approach, orientated to the Page xvi
individual, his/her family and their community
• having a unique consultation process, which establishes a relationship
over time through effective communication between doctor and patient
• being responsible for the provision of longitudinal continuity of care as
determined by the needs of the patient
• having a specific decision-making process determined by the prevalence
and incidence of illness in the community
• managing simultaneously both acute and chronic health problems of
individual patients
• managing illness that presents in an undifferentiated way at an early stage
in its development, which may require urgent intervention
• promoting health and wellbeing both by appropriate and effective
intervention
• having a specific responsibility for the health of the community
• dealing with health problems in their physical, psychological, social,
cultural and existential dimensions.
Doctors working in rural and remote Australia can face clinical situations
that require skills in a wider scope of practice. Doctors can now choose to
qualify as a general practitioner via the Australian College of Rural and
Remote Medicine (ACRRM) Fellowship examination. ACRRM expects
Fellows to be performing all the functions of a GP listed by RACGP and
WONCA plus:
• responding to emergencies including stabilisation and definitive
management as appropriate; providing hospital-based secondary care
where required; delivering obstetric care; and undertaking a range of
population health interventions at the practice and community level.4
Those planning to sit the ACRRM exam may still find the cases useful by
thinking through what they would do in each case if the nearest hospital or
referral centre was more than an hour’s flight away, or they were the only
doctor in town running both a general practice and a small hospital.
The above definitions demonstrate the breadth of the general practice
specialty and remind me why it is such an interesting profession. The
illnesses we see are common but we are also alert for potentially serious
conditions. For this we need a system of diagnosis that differentiates the
headache caused by a brain tumour from a tension headache or a hangover
and, if it is a tension headache, we need ways of exploring possible causes of
the tension and discussing with the patient how to manage them.

Consultation frameworks
The traditional consultation framework comprises history, examination,
investigation, diagnosis, treatment and follow-up. While suitable for
emergency care, this model is inadequate for the complexity of general
practice.
I recommend the following texts that suggest some task-based and some
process-based consultation frameworks:
• Pendleton, D, Schofield, T, Tate, P, et al. 2003, The new consultation:
developing doctor–patient communication, Oxford.
• Neighbour, R 2004, The inner consultation: how to develop an Page xvii
effective and intuitive consulting style, 2nd ed, Oxford.
• Stewart, M, Brown, J. B., Weston, W. W., et al. 2003, Patient-centred
medicine: transforming the clinical method, 2nd ed, Oxford.
In 2001 a meeting of medical educators and communication skills experts
reached a consensus on the essential elements of communication in medical
encounters (Kalamazoo Consensus Statement).5 In this book I have adopted
the elements of their suggested approach to cases, summarised as:
• build the doctor–patient relationship
• open the discussion
• gather information
• understand the patient’s perspective
• share information
• reach agreement on problems and plans
• provide closure.

General practice exams


General practice exams need to ensure that, to be effective, GPs have the
appropriate attitude and a broad range of knowledge and skills. So prepare to
be tested on any of the characteristics outlined in the above definitions.
Reading a textbook on clinical medicine is not enough, you have to apply the
textbook knowledge to a particular person in their particular context, making
the best use of the resources available.
One way to do this is to consider the domains of general practice. The
RACGP domains are:6
1. Communication skills and the doctor–patient relationship
2. Applied professional knowledge and skills
3. Population health and the context of care
4. Professional and ethical roles
5. Organisational and legal dimensions.
The ACRRM domains are:7
1. Core clinical knowledge and skills
2. Extended clinical practice
3. Emergency care
4. Population health
5. Aboriginal and Torres Strait Islander health
6. Professional, legal and ethical practice
7. Rural and remote contexts.

Applying the domains to a particular situation


Imagine that you have just completed your reading on diabetes. You are now
up-to-date with the latest research and the signs of early diabetic retinopathy.
How will you apply this as a GP?

RACGP domain 1. Communication skills and the Page xviii


doctor–patient relationship
Domain 1 reminds you that diabetes affects a person. In consultations you
will focus on the patient and their experience of life with diabetes. You will
communicate clearly in terms they understand and that are free of medical
jargon.
A trap for doctors, particularly those from non-English speaking
backgrounds, is to assume that patients whose first language is English
understand all English. This is a myth: it is said that first-year medical
students learn more new words than first-year language students. We need to
constantly distinguish between medical English and colloquial Australian
English, and use each when appropriate.
Another aspect of practice that varies between countries is how much
patients direct consultations. In Australia, GPs do not automatically have
patients’ agreement to give advice or recommendations. This right has to be
earned by first listening carefully to patients’ ideas, concerns, and
expectations. Only then can doctors outline their ideas and plans, and
negotiate with patients a management plan. There are exceptions, such as
emergencies, but in general shared decision-making is expected(1). Learning
this skill is essential and requires a considerable shift in approach if you
trained in a system where doctors were expected to give advice and patients
passively acquiesced.
Domain 1 also includes health promotion, so discuss immunisations,
smoking, alcohol and maintaining health, as well as the illness of diabetes,
with your patient.

RACGP domain 2. Applied professional knowledge and skills


This is the most familiar domain, in which you ensure your patient has good
diabetic control and you work to prevent complications.

RACGP domain 3. Population health and the context of


general practice
‘Population health’ requires you as a GP to broaden your thinking beyond
your patient to encompass the wider community. What is your role in
preventing or screening for diabetes? Is there a group of patients with an
increased risk of diabetes—Torres Strait Islander people, for instance—who
need culturally appropriate screening?
‘The context of general practice’ includes consideration of your patient’s
life circumstances. Can they get fresh fruit and vegetables easily? What
exercise can they safely do? Does their family or religious background
influence their health? For example, how would you advise a Muslim patient
with diabetes on hypoglycaemics to cope during Ramadan?

RACGP domain 4. Professional and ethical role


Your role as a GP requires you to look at the standard of care that you
provide for patients with diabetes. Use clinical audit to check that you are
following the national guidelines. Consider how your care integrates with the
rest of the health care system: what are your links with the local optometrist
who could screen your patients for diabetic retinopathy? Are you Page xix
making the best use of resources? Could a practice nurse do routine
diabetic care so that you can focus on complex cases?
Your ethical role might include such dilemmas as a case where a diabetic
patient whose livelihood depends on holding a commercial driving licence
also requires insulin.

RACGP domain 5. Organisational and legal dimensions


Are you using information technology to improve the quality of care for your
patients with diabetes? What are the recall systems? How do you ensure
patient confidentiality when they attend for repeat prescriptions? When did
you last calibrate your glucometer and sphygmomanometer? Do you make
comprehensive and comprehensible notes? Is yours an accredited practice
eligible for Service Incentive Payments?

ACRRM domains. Emergency care, rural and remote context


and Aboriginal and Torres Strait Islander health
For diabetes this may mean managing diabetic ketoacidosis in a small rural
hospital, starting newly diagnosed diabetic children on insulin or running a
screening program at a football carnival in a remote Aboriginal community.

Long and short cases


In this book, long cases usually test a whole consultation. Short cases usually
test part of a consultation, perhaps focusing on taking a history, performing a
physical examination, giving a diagnosis or negotiating management. This
challenges doctors who are used to conducting full consultations.
In cases that require management only, both students and doctors often
still begin with a history and examination. This is a normal (and good)
clinical response but it wastes precious time because it isn’t required. Several
cases in this book give you practice at picking up the consultation halfway
through.
GPs would never finish our day’s work if we took a full history and
performed a complete examination on every patient. We take appropriate
short cuts and focused histories, examining the patient for specific
information to rule a diagnosis in or out. Our expertise ‘is the ability to access
knowledge and make connections across seemingly disparate fields and life
experiences’.8 The art of exams is to demonstrate this expertise.
Medical students can eventually reach the answer but only experienced
clinicians can connect the disparate elements of a scenario into a diagnostic
approach in the time available. This ‘elaborated knowledge’ is essential for
an independent general practitioner.9
For example, what diagnosis comes to mind when you read ‘headache’,
‘fever’, ‘rash’? The safe, experienced GP asks, ‘Is this meningitis?’ The
clinical encounter will focus on excluding meningitis by a short, focused
history and an immediate look at the rash. If it is purpuric they would exclude
an allergy, give parenteral antibiotics and arrange immediate hospital
admission. By contrast, a student might approach this case using a
systematic, hypothetico-deductive method,10 first taking a detailed history of
the presenting symptoms, then a review of current medication and allergies,
followed by a social history, family history, systems review and Page xx
finally conducting a top-to-toe clinical examination. Once all the data
is gathered the student will begin to piece together a possible hypothesis or
diagnosis and deduce the most likely diagnosis from the evidence available,
or perhaps decide that further information is needed. By this time the
experienced GP will have given the patient antibiotics. In this extreme
example, the speed of the GP’s diagnosis may be life-saving. This is rare in
clinical practice but it illustrates the efficiency of pattern recognition as a
diagnostic process. There is no short cut to this. It requires considerable study
and experience and underlines the need for GPs to undergo supervised
clinical practice.

Summary
Clinical practice is still the best preparation for general practice and general
practice exams. My hope is that this book will supplement your clinical work
so that during exams you can demonstrate that you have the knowledge, skills
and attitudes necessary for general practice in Australia.

References
1. Britt, H, Miller, G. C., Henderson, J, Bayram, C, Valenti, L, Harrison,
C, et al. 2014, A Decade of Australian General Practice Activity 2004–
05 to 2013–14, General Practice Series no. 37.
2. Royal Australian College of General Practitioners, ‘Definition of
general practice and general practitioner’. Available at: www.racgp.org
.au/ whatisgeneralpractice, accessed 1 June 2010.
3. WONCA Europe, ‘The European definition of general practice/family
medicine’. Available at: www.euract.org/index.php?folder_id=24,
accessed 1 June 2010.
4. Australian College of Rural and Remote Medicine, ‘The ACRRM
position on the specialty of general practice’. Available at: www.acrrm.
org.au, accessed 22 April 2015.
5. Participants in the Bayer-Fetzer conference on physician–patient
communication in medical education 2001, ‘Essential elements of
communication in medical encounters: the Kalamazoo consensus
statement’. Academic Medicine vol. 76, pp. 390–3.
6. Royal Australian College of General Practitioners 2011, ‘The RACGP
Curriculum for Australian General Practice’. Available at: http://curric
ulum.racgp.org.au/, accessed 27 February 2015.
7. Australian College of Rural and Remote Medicine 2013, Primary
Curriculum, Fourth ed.
8. Fraser S. W. and Greenhalgh, T 2001, ‘Coping with complexity:
education for capability’, British Medical Journal, vol. 323, pp. 799–
803.
9. Bordage G 1994, ‘Elaborated knowledge: a key to successful
diagnostic thinking’, Academic Medicine, vol. 69, issue 11, pp. 883–5.
10. Hays, R 1999, Practice-based Teaching: a Guide for General
Practitioners, pp.19–21.
Page xxi

How to use this book

This book is designed to help medical students and doctors practise clinical
cases likely to be seen by general practitioners in Australia. Each case
consists of instructions for the doctor and role-playing patient, a suggested
approach, case commentary and references, or recommended further reading.
The cases are written in note form as is usual practice in clinical records.
It is based on my experience of facilitating case-based learning groups for
medical students and general practice registrars. Trial and error helped
establish what worked and what did not, but everyone is different, so if you
find a new way of practising please let me know.

Don’t read this book!


You will learn more by using this book rather than reading it from cover to
cover. Come to each case fresh, so that you can test your knowledge and
skills in general practice and identify your learning needs. If you read through
the book when you practise cases you will distract yourself with trying to
remember the suggested approach. This would test your recall rather than the
application of your knowledge and clinical skills to a particular context.

Do read the instructions


The easiest way to fail a clinical exam is to ignore the instructions as marks
are pre-allocated according to the instructions. Time management is crucial to
ensure you cover the requirements. For example, you need to take a ‘good
enough’, safe history before moving on to the clinical examination and
planning management, if all three elements are requested in the scenario.
Taking eight minutes to do a complete history will lead to a fail if the
examination and management are omitted.

Find a group
Most people find it is easier to study with a group rather than in isolation. A
group motivates you to persevere and learning from your mistakes in a safe
environment is far better than learning from making mistakes with patients.
If you are in isolated practice you can still join a group. I have run practice
consultation groups over the phone, via Skype or video-conference. It
presents some challenges but can be done. I do not know of any technology
for remote physical examination, but all other aspects of the consultation can
be practised. If you have not been observed conducting physical
examinations you could videorecord consultations with patients (with Page xxii
their written consent), friends or relatives. Send password-protected
copies to your group and ask them for constructive feedback. Your physical
examination of each system and body part must be so automatic that you can
focus and interpret what you find.
Groups work better if group rules have been agreed to. The group needs to
develop trust so that ignorance leads to learning rather than embarrassment.
Confidentiality is an important issue. Participants will learn more if learning
is derived from and related to their own clinical practice, so everyone needs
to know that they can safely share their experiences.

Allocate roles
Allocate the roles to the role-players ahead of the session if possible. This lets
the role-players familiarise themselves with the scenario and search out more
information about the clinical problem, which adds to their learning. Props
can help role-players give a more authentic look and mimic the sorts of clues
that prompt GPs in clinical practice.
These scenarios are about common problems; at some stage one of the
scenarios could be about a condition that the role-player, or a friend or
relative, suffers from. This may enhance their performance but equally it
could create a tense situation. By allocating roles in advance, the role-player
can decline the case before major issues develop.

Observers
Observers of the role-play will learn more if they see only the ‘Instructions
for the doctor’. Encourage observers to make notes on how they would
approach the case, which helps them to be active learners rather than passive
observers. They can also give constructive feedback to the role-playing
doctor and patient.

Facilitator
One person can facilitate the case and have access to all the information. The
checklists are designed for the facilitator to tick off as the doctor deals with a
particular aspect of the case.
Please give the doctor the examination findings only on specific request.
Doctors should demonstrate their reasoning by asking for the findings that
will confirm or refute the diagnosis. For example ‘what are the neurological
findings’ is too broad a question to ask about a patient with a suspected
stroke. Instead questions should be asked about the expected changes in tone,
power, sensation, coordination and reflexes in the affected limbs.

Timing
Allocate a timekeeper for each scenario. Students and junior general practice
registrars may decide to use the material without a time limit but more
experienced doctors should complete the short cases in eight minutes Page xxiii
and the long cases in nineteen minutes, following three minutes to
read through the case scenario.
Allow time for discussion after each scenario: usually twenty minutes for
short cases and thirty minutes for long cases is adequate.

Room setup
Set up the room like a consulting room, with chairs for the patient and doctor,
a desk, examination couch and standard consultation equipment. The patient
should be sitting on the chair at the beginning of the consultation even if they
will need to be examined. It gives too big a clue if the doctor walks in and
finds the patient dressed ready for a physical examination! Role-playing
patients may prefer to wear bathers or shorts under their clothes to ease their
potential embarrassment at being examined BUT remember that this is about
learning, not humiliation. If the patient role-player does not wish to be
examined, please accommodate this.
In physical examination cases you should demonstrate in a deliberate way
that you are performing the examination correctly. Talking yourself through
the examination, making a running commentary of your actions and findings,
can increase your awareness of what you are doing and why, and demonstrate
your clinical reasoning to observers.

Doctors training in the Australian College of


Rural and Remote Medicine pathway
One component of the Australian College of Rural and Remote Medicine
(ACRRM) assessment is ‘StAMPS’: structured assessment using multiple
patient scenarios. This is an examination via video-conference where each
scenario lasts for ten minutes. Doctors are asked questions in a viva or role-
play format. Physical examination and practical skills are assessed separately
from StAMPS.
The cases in this book were primarily designed for doctors preparing for
the Royal Australian College of General Practitioners (RACGP) Clinical
Examination but doctors preparing for the ACRRM Fellowship may find
them a useful resource to practise via video-conference or webcam (see
Introduction on page xv).
Suggested approach
The suggested approach to each case includes the essential elements of
medical consultations as outlined in the Kalamazoo Consensus Statement1
but does not exhaust all the possibilities. When you are reading this section
you will probably think of your own additions, but for each addition you need
to remove an item to avoid the cases becoming impossibly long for the time
available. You will find that additions are easy but taking something out is
not; discuss these issues with your group—the process of considering what to
do in each case greatly improves your clinical reasoning. If your group does
agree on a significant addition, please let me know.
Mnemonics pose a particular risk. They help us to remember what Page xxiv
to ask but are best used as a framework for a conversation. Too often
a doctor is working hard to remember the mnemonic rather than focusing on
the patient’s response to the questions. While this is understandable we must
be more than questionnaires on legs! There is no point asking the right
questions if you don’t listen and respond to the answers.

Case commentary
There is a commentary on the relevant issues in each case as well as
references and suggestions for further reading.

Let the show go on


Doctors will usually learn more from the scenario if it runs for the allotted
eight or nineteen minutes. However, maintaining someone’s self-esteem may
be more important. The response to a doctor getting stuck or suffering from
stage fright mid-consultation will depend on their level of experience. For
doctors early in training, you can ask for volunteers to take over the
consultation. For more experienced doctors, the facilitator can make
suggestions such as, ‘Try reading through the scenario again; it might prompt
you on how to proceed,’ or ‘If this happened in your surgery, what would you
do?’
This gives the doctor a framework for what to do if they get stuck in either
an exam situation or in clinical practice.
If the role-playing patient is disturbed or upset, the scenario may need to
be stopped.

Guidelines for giving constructive feedback


Giving feedback constructively is a skill that will be useful for all general
practitioners. Confidence can quickly evaporate in someone who has done
badly in a scenario and it helps to have guidelines to avoid feedback being
perceived as judgemental and destructive. It may feel contrived but it is safe.
Once the group has grown and is learning well together you can focus more
quickly on the aspects for improvement.

Pendleton’s guide to giving feedback


• Briefly clarify matters of fact
• The doctor says what was done well, and how
• The rest of the group says what was done well, and how
• The doctor says what could be done differently, and how
• The rest of the group says what could be done differently, and how.
I follow Pendleton’s guide and ask the doctor to start the Page xxv
discussion by outlining what they did well.2 Doctors invariably focus
on their mistakes and fail to notice what worked well. I have waited on
occasion for several minutes for some doctors to say just one good thing
about their consultations. This is important, so that learning builds on what is
already good and gives practitioners a realistic framework for self-reflection
throughout their career.
Once the positives have been recognised I ask the doctor to outline what
they might do differently if they had another chance. This gives me insight
into their perceptions of their performance, and allows them to self-correct
before group members comment. I know that I am facing a major teaching
challenge when someone is unaware that they have performed poorly.
If there is time and the doctor has noticed an error, you can re-run the role-
play and allow them to practise their new learning. Once they have shared
their ideas on how to improve their consultations, the observers can join in.
Again, it is a good principle to ensure that feedback is balanced. Descriptive
feedback should reduce defensiveness and promote learning. It is:3
• non-judgemental
• specific
• directed towards behaviour rather than personality
• checked with the recipient
• outcome-based
• problem-solving
• in the form of suggestions rather than prescriptive comments.
Let me give you an example. You have just watched a colleague’s
consultation.
During the consultation the doctor wrote a script for an anti-hypertensive,
asking the patient about her exercise regime at the same time. You observed
the patient go red in the face and mumble an answer. The doctor did not raise
the issue of exercise again and did not notice the patient blush.
Judgemental feedback might be something like this:
Facilitator: ‘You missed your chance to hear about her exercise. It is bad
practice to ask questions while you are writing scripts. You’ll never make a
decent doctor. Don’t they teach you anything at medical school?’
The alternative descriptive feedback might be:
Facilitator: ‘Well done for asking about exercise. She did not say very
much. Why do you think that might be?’
Doctor: ‘Maybe she was embarrassed because she does so little. Why do
you ask?’
Facilitator: ‘I noticed that you asked about exercise while you were
writing the script. When you asked the question she blushed, looked
embarrassed and mumbled an answer. What do you think that was about?’
Doctor: ‘I guess asking about exercise is a delicate issue. Maybe I gave the
impression that I wasn’t interested in the answer when I asked her at the same
time as writing the script. No wonder she didn’t say much. In future I’ll try to
give patients my full attention when I ask important questions.’
Facilitator: ‘Would you like to re-run the scenario?’

Reluctant participants Page xxvi

Role-play has an established place in facilitating learning in general practice


and in assessing the competency of practitioners in Australia.4 However,
many students and doctors are reluctant to participate. This is understandable.
Consulting is central to our effectiveness as practitioners but it is also very
personal.
International medical schools may have different approaches to teaching
consultation skills. Several doctors that I have taught had not been observed
consulting during their training. When they moved to Australia they found
this method of learning new and frightening—an added challenge was
consulting in English. My tip for this situation is to suggest that international
medical graduates come to groups and just watch at first. When they are more
familiar with the process they may be prepared to role-play as patients. Once
confident in this, they can be invited to play the doctor.

Have fun
Lastly, do enjoy your practice sessions. This is NOT about ritual humiliation.
I learn something new each session about clinical practice, which improves
the quality of care I give as a GP and increases my interest in the profession.

References
1. Elwyn, G, Frosch, D, Thomson, R, Joseph-Williams, N, Lloyd, A,
Kinnersley, P, et al. 2012, ‘Shared decision making: a model for clinical
practice’, Journal of general internal medicine, vol. 27, issue 10, pp.
1361–7.
2. Participants in the Bayer-Fetzer conference on physician–patient
communication in medical education 2001, ‘Essential elements of
communication in medical encounters: the Kalamazoo consensus
statement’, Academic Medicine, vol. 76, pp. 390–3.
3. Pendleton, D, Schofield, T, Tate P & Havelock, P 2003, ‘The new
consultation: developing doctor–patient communication, Oxford.
4. Silverman, J, Draper, J & Kurtz, S. M. 1997, ‘The Calgary–Cambridge
approach to communication skills teaching II: the SET-GO method of
descriptive feedback’, Education for General Practice, vol. 8, pp. 16–23.
Page 1
Section 1
Aboriginal health
Page 2

Case 1
Kasey Kox

Instructions for the doctor


This is a long case.
Please take a history from Kasey. When you are ready, request
examination findings and the results of surgery tests from the examiner and
outline your differential diagnosis. Outline your immediate investigations and
negotiate a management plan with Kasey. Please answer the examiner’s
questions.

Scenario
Kasey is a 17-year-old Aboriginal girl who has not been seen at your
Melbourne practice before.

The following is on her summary sheet:


Past medical history
Grommets as a child
Medications
Etonorgestrel contraceptive implant (Implanon)
Allergies
Nil known
Immunisations
As per schedule
Family history
Nil recorded
Social history
From Balgo in WA
Moved to Melbourne last year for boarding school.

Instructions for the patient, Kasey Kox Page 3

You are a 17-year-old Aboriginal girl from Balgo in WA who has a


scholarship to complete year 12 in Melbourne. Over the past few days you
have been unwell with aches and pains. Five days ago, your left wrist was
sore and warm. Then both wrists were affected, but they have improved.
Now your right knee has become really painful. It is difficult to walk or
bend your knee so you couldn’t go to school today. You have a headache
and feel thirsty.
You feel cold even though it’s warm outside. You have had no skin
sores, no diarrhoea or vomiting, no abdominal pain, and no urinary
symptoms.
You like boarding school but you do get homesick. You spend a lot of
time on social media connecting with your family back home.
You’ve recently been at home for the holidays and, if asked, you did
have a sore throat a few weeks ago. As usual your three-bedroom house
was packed; there are often 12 people living there, depending on who is in
the community.

Sexual health
You were sexually active with a boyfriend last year (aged 16) and would
accept an STI check if offered.
You had the cervical cancer vaccines at school and haven’t had any
screening tests yet.
Your periods are regular and very light since the contraceptive implant
went in.

Social (home and education)


You are one of five children. Your older sister is studying medicine in
Melbourne and you hope to be a teacher. You are fit and a keen Aussie
rules footballer.

Drugs and alcohol


You don’t smoke.
You’ve tried marijuana and alcohol at parties but take nothing regularly.

Family history
Your dad has diabetes and kidney problems.
Your mum had a heart operation in her 30s for her valves. If asked for
more detail say, ‘rheumatic heart’ and that she takes blood thinners.
Your sister also has heart problems; if asked specifically, say she gets a
monthly injection to prevent it getting worse.

The following is on your summary sheet: Page 4


Past medical history
Grommets as a child
Medications
Etonorgestrel contraceptive implant (Implanon)
Allergies
Nil known
Immunisations
As per schedule
Family history
Nil recorded
Social history
From Balgo in WA
Moved to Melbourne last year for boarding school.

Questions for the doctor


Will I need to go to hospital?

Information for the facilitator


Vital signs
Temperature 38.3°C
Pulse 80 bpm
BP 100/60
Sp02 99% room air
RR 20/min
BMI 22
ENT throat NAD
No cervical lymphadenopathy
Ears—bilateral scarred tympanic membranes
Chest clear
Heart sounds—systolic murmur
JVP normal
Abdomen soft and non-tender
Right knee warm to touch, obvious effusion, mild limitation in range of
motion in all directions
Antalgic gait, significant limp but can weight bear
Left wrist—warm, mild tenderness to palpation
Skin NAD
U/A normal
BSL 4.5 mmol Page 5
ECG—see page 6.
With three minutes to go, or when the candidate finishes speaking, ask the
following:
1. What are your differential diagnoses? Tell me your favoured diagnosis.
2. What blood tests are you going to do?
3. What immediate management will you provide?
4. What are the criteria for diagnosing acute rheumatic fever?

Suggested approach to the case


History
History of presenting complaint and risk factors for acute rheumatic fever
need to be solicited.
A HEADDSSS (Home situation; Employment, Education, Economic
situation; Activities, Affect, Ambition, Anxieties; Drugs, Depression;
Sexuality; Suicide, Self-esteem, Stress) approach is also valuable for this
young woman who finds herself far from home and is unknown to the
practice. Offering a chaperone or an Aboriginal health worker if available
would be helpful.

Examination
Vital signs
Joint examination—including other joints
Skin examination looking for skin sores (alternative source of Group A Strep)
and erythema marginatum or subcutaneous nodules (both rare)
Cardiovascular, particularly auscultation of the heart and looking for signs of
heart failure.

Investigations
FBC, UEC, CRP/ESR
Blood culture
Throat swab and swab any infected skin sores
Anti-DNase B
Anti-streptolysin O titre
Echocardiogram
STI screen
Surgery tests
ECG
Urinalysis.

Major criteria (need two major or one major and two minor for Page 6
diagnosis plus evidence of preceding Group A streptococcal infection)
— Carditis, including subclinical evidence of rheumatic vasculitis on
echocardiogram
— Polyarthritis or aseptic monoarthitis or polyarthralgia
— Erythema marginatum
— Subcutaneous nodules
— Chorea (Sydenham’s chorea does not need evidence of preceding
Group A streptococcal infection, providing other causes of chorea are
excluded).
Minor criteria Page 7
— Monoarthralgia
— Fever (>38°C)
— ESR > 30 or CRP >30
— Prolonged PR interval on ECG.

Management
Given the differentials in this case, hospital referral for further workup is
appropriate. If septic arthritis can be excluded, benzathine penicillin would be
given for acute rheumatic fever and aspirin or NSAIDs would be first-line
treatment for pain.
Ongoing management would include four-weekly benzathine penicillin as
secondary prophylaxis of recurrent episodes of acute rheumatic fever with the
aim of preventing rheumatic heart disease.

COMMON PITFALLS

Doctors working outside areas with a high prevalence of acute


rheumatic fever may not have this diagnosis at the forefront of
their minds. Untreated acute rheumatic fever leads to rheumatic
heart disease, which causes significant morbidity and mortality,
so it is an important disease not to miss.

Online resources
ARF/RHD Guideline app
www.rhdaustralia.org.au
http://kamsc.org.au/wp-content/uploads/2016/11/Acute-Rheumatic-Fever-Oct
ober-2016.pdf.

References
Bossingham, D 2015, ‘Case study: Atypical arthritis: a young woman
presents with fever and joint pains.’ How to Treat—Australian Doctor,
Available at: https://www.howtotreat.com.au/case-report/case-study-aty
pical-arthritis, accessed 18 March 2019.
Karthikeyan, G & Guilherme, L 2018, ‘Acute rheumatic fever’, Lancet, vol.
392 (10142), pp. 161–74.
Page 8

Case 2
Sharon Price

Instructions for the doctor


This is a short viva.
Please answer the questions of a GP supervisor about your consultation with
Sharon.

Scenario
Sharon, a 39-year-old Aboriginal woman who works in retail, has just
moved to your area. She was seen by a colleague last week for some
scripts and your colleague suggested Sharon book in for an Aboriginal
adult health check.

The limited summary in her file is as follows:


Past medical history
Diabetes
Medications
Metformin XR 2 g daily
Sitagliptin 100 mg daily.

Instructions for the facilitator


The doctor is expected to talk with you as a professional colleague. During
the viva please ensure you ask the following questions:

1. Why is it important to identify Aboriginal and Torres Strait Islander


patients in your practice?
2. What would you do to check Sharon’s diabetes?
3. Sharon has just moved interstate as a newly single parent. The cost of her
diabetes medications is overwhelming. What could you do to assist?
4. What other screening would be important for Sharon at this point? Page 9
5. You realise the practice does not see many Aboriginal patients.
How could you ensure your practice is culturally safe and welcoming to
Aboriginal and Torres Strait Islander patients?
6. What resources or guidelines might help you deliver the most appropriate
care for Sharon?

CASE COMMENTARY

1. Identifying Aboriginal and Torres Strait Islander patients is


important for many reasons, including:
• providing culturally safe medical care, e.g. male or
female doctors to provide care for men’s and women’s
business respectively
• offering interpreters or cultural mentors to attend
appointments
• different disease prevalence leading to
— Aboriginal and Torres Strait Islander specific
immunisation and chronic disease screening
schedules
— Medicare incentives for practices and patients to
improve access to care and treatment.
2. Sharon has diabetes and needs:
• assessment of her current wellbeing and how she is
handling her diabetes
• consideration of the suitability of her medication regime,
her coherence and any barriers to taking medications
• exploration of her modifiable risk factors: smoking,
nutrition, alcohol, physical activity (SNAP) and check of
BMI, waist circumference, lipid profile
• screening for end-organ damage of her eyes (retinal
screening), heart (BP, ECG), kidneys (UEC, urine ACR)
and feet (skin, peripheral pulses, monofilament testing of
sensation).
3. Sharon could register for the Medicare ‘Closing the Gap’
initiative. This reduces the cost of her medications if she has
no Health Care Card or eliminates costs if she has one. A
combined preparation of her medications would reduce the
cost and the number of pills she takes.
She is eligible for a Medicare rebate on an Aboriginal and
Torres Strait Islander adult health check (715) and GP
Management Plan and Team Care Arrangement (721/723).
The latter would give her subsidised access to a diabetes
educator and/or podiatrist.
4. Documentation of past medical history, immunisation Page 10
status, family and social history, allergies and
physical examination if this was not done when seen last
week. Plus cervical and STI screening if appropriate.
5. To create a culturally safe practice, all staff should undertake
cultural training, ideally offered by local Aboriginal or
Torres Strait Islander providers. Aboriginal and Torres Strait
Islander flags or acknowledgement of the traditional owners
can make the practice more welcoming as can local
Aboriginal artworks and culturally appropriate pamphlets
and resources. Increasing the numbers of Aboriginal and
Torres Strait Islander health professionals and staff members
is also important for ongoing improvements in providing
culturally safe and appropriate care.
6. There are resources designed to assist GPs to provide
appropriate care for Aboriginal and Torres Strait Islander
people. These include:
• National resources, for example the National guide to a
preventative health assessment for Aboriginal and Torres
Strait Islander people.1
• Local guidelines developed in collaboration with local
Aboriginal people and community controlled health
organisations, for example the CARPA manual (NT)2 and
Kimberley Aboriginal Medical Services guidelines.3

References
1. National Aboriginal Community Controlled Health Organisation and The
Royal Australian College of General Practitioners 2018, National guide
to a preventive health assessment for Aboriginal and Torres Strait
Islander people, 3rd ed, RACGP, East Melbourne, Vic.
2. Centre for Remote Health, CARPA standard treatment manual, 7th ed.,
Alice Springs, NT.
3. Kimberley Aboriginal Health Planning Forum 2018, Kimberley
Aboriginal Medical Services guidelines. Available at:
https://kahpf.org.au/clinical-protocols, accessed 18 March 2018.

Further reading Page 11


Australian Indigenous Health InfoNet at healthinfonet.ecu.edu.au, accessed
10 February 2019.
Liaw, ST, Hasan, I, Wade, V, Canalese, R, Kelaher, M, Lau, P & Harris, M
2015, ‘Improving cultural respect to improve Aboriginal health in
general practice: a multi-methods and multi-perspective pragmatic
study’, Australian Family Physician, vol. 44, no. 6, pp. 387–92.
RACGP Aboriginal and Torres Strait Islander Health, Five steps toward
excellent Aboriginal and Torres Strait Islander healthcare. Retrieved
from: www.racgp.org.au/FSDEDEV/media/documents/Faculties/ATSI/
Five-steps-guide.pdf, accessed 26 February 2019.
The Royal Australian College of General Practitioners 2016, ‘General
practice management of type 2 diabetes: 2016–18’, RACGP, East
Melbourne, Vic. Page 12
Page 13
Section 2
Adolescent
health
Page 14

Case 3
Erin Campbell

Instructions for the doctor


This is a short case.
Please take a history from Erin. A clinical photograph of Erin’s face will
be available to you on request (Refer to Figure 1, centre insert page A).1
Please then outline the most likely diagnosis and negotiate a management
plan with her.

Scenario
Erin Campbell is a 14-year-old girl who presents to you with moderate
facial acne. Her mum has come with her to the surgery but lets Erin see
you on her own.

The following information is on her summary sheet:


Past medical history
Nil significant
Medication
Nil
Allergies
Nil
Immunisations
Up-to-date
Social history
Lives with parents.

Instructions for the patient, Erin Campbell


You are 14 years old and attend the local high school. Your pimples
dominate your life. Each morning your mum shouts at you to get dressed and
ready for school, while you stare at your pimples in the mirror. You Page 15
are convinced that you will never have a boyfriend like all your other
friends. You used to take comfort in eating chocolate and cheese but have
stopped since a friend said that was causing your pimples.
You are embarrassed to be going to see the GP and hope the doctor will be
kind.

The following information is on your summary sheet:


Past medical history
Nil significant
Medication
Nil
Allergies
Nil
Immunisations
Up-to-date
Social history
Lives with parents.
Suggested approach to the case
Establish rapport with Erin
Open-ended questions to explore Erin’s concerns and expectations about her
acne.

Specific questions
Duration of acne
Location of acne
Impact of acne on social life and relationships
What has she tried so far as treatment?
What does she think causes the acne?
General health, e.g. are her periods regular?
Request permission to examine.

Examination
Examine the face
— Confirm acne
— Describe signs: comedones, pustules, erythema or scarring.

Management Page 16

Explain medical understanding of cause of acne without using jargon


Reassure that acne can be controlled, should not stop her socialising
Advise against picking or squeezing
Offer treatment depending on what Erin has already tried
Advise regular washing with soap
Avoid oily or greasy skin preparations
Recommend healthy diet
Treatment takes four to eight weeks to be effective, can combine topical and
oral treatments

Reduce excess cells (hyperkeratinisation)2


Topical retinoids, azelaic acid
Reduce bacteria (propionibacterium acnes)
Benzoyl peroxide, strength according to skin type
Course of topical antibiotics (combine with Benzoyl peroxide or topical
retinoids).3 Courses of oral antibiotics are indicated for severe cystic acne
—tetracyclines not recommended until bone and teeth development
complete

Rebalance hormones (reduce androgen excess)


Oral contraceptives

Reduce sebum hypersecretion


Oral isotretinoin—high risk of teratogenicity

Arrange follow-up.

CASE COMMENTARY

Adolescent acne is a common problem and can lead to physical


and psychological scarring.4 Acne is a problem of pilosebaceous
ducts caused by, duct obstruction from keratinocytes, androgen-
induced increase in sebum, Propionibacterium acnes
proliferation and inflammation. The mainstay of treatment is
topical retinoids as there is concern that the prolonged use of
topical and oral antibiotics is contributing to the problem of
antimicrobial resistance.2 Antibiotics can be used for courses of
three to four months to control cystic acne; once under control
topical retinoids are used to maintain control.3
Typically many treatments will have been tried at home prior
to consulting a GP. It is important to find out what Erin has tried
so you are offering something new rather than a treatment Page 17
that has already been tried and failed. Diets with a high
glycaemic index may exacerbate acne so it is worth encouraging
a healthy diet.2
For some teenagers, the hidden agenda of an acne
presentation is a request for contraception. This requires
sensitive exploration. A phrase that I find useful is: ‘Some
people who come to see me know that the contraceptive pill
helps with their skin; is this the case with you?’ The question
about Erin’s periods is to assess the likelihood of polycystic
ovarian syndrome.

COMMON PITFALLS

Teenagers determine the severity and impact of their acne, not


doctors. This case tests the doctor’s ability to practise in a
patient-centred framework. Low marks will be given to doctors
who dismiss Erin as having a few pimples which are
insignificant and which she should learn to live with.

References
1. Usatine, RP, Smith, MA & Mayeaux Jr, EJ 2009, The Color Atlas of
Family Medicine, McGraw-Hill, New York, NY, p. 439.
2. Dawson, AL & Dellavalle, RP 2013, ‘Acne vulgaris’, British Medical
Journal, vol. 346, p. f2634.
3. Zaenglein, AL 2018, ‘Acne vulgaris’, New England Journal of Medicine,
vol. 379, no. 14, pp. 1343–52.
4. Goodman, G 2006, ‘Acne and acne scarring: the case for active and early
intervention’, Australian Family Physician, vol. 35, no. 7, pp. 503–4.
Page 18

Case 4
Amanda Porter

Instructions for the doctor


This is a short case.
Please take a history from Amanda and then negotiate an initial
management plan with her. Your management plan will include explaining
the need for a physical examination.

Scenario
Amanda Porter is a 16-year-old girl who has been coming to the practice
for a long time. Last week during a school trip her class teacher caught
Amanda making herself sick after the evening meal. With Amanda’s
consent the teacher phoned to make this appointment.

The following information is on her summary sheet:


Past medical history
Greenstick fracture left radius aged eight
Medication
Nil
Allergies
Nil
Immunisations
Fully immunised
Social history
Lives with parents.

Instructions for the patient, Amanda Porter


Your family have always had ‘big bones’. As a child you accepted the fact
that your mum and dad were on the sidelines at sports day rather than Page 19
entering in the parents’ races. You are used to being part of a big
family and shopping at the plus size stores. Whenever friends visit they can’t
finish the amount of food they are given.
About three months ago you had a really bad attack of gastroenteritis. You
were amazed to find that you lost weight. Losing weight made you feel great.
You found that if you made yourself vomit just after the evening meal at
home you could lose weight without having to diet. Although the vomiting is
not pleasant it makes you feel good because your figure is better and you are
getting attention from the boys at the youth club.
Last week during a school trip your class teacher caught you making
yourself sick after the evening meal. Your teacher phoned to make this
appointment with your consent. You are 1.67 m tall and weighed 95 kg three
months ago, giving you a BMI of 34 kg/m2. You now weigh 89 kg (BMI 29
kg/m2).
You are very scared about seeing the GP.

The following information is on your summary sheet:


Past medical history
Greenstick fracture left radius aged eight
Medication
Nil
Allergies
Nil
Immunisations
Fully immunised
Social history
Lives with parents.

Suggested approach to the case


Establish rapport
Confirm confidentiality assured—unless disclosure required by law
Sensitively explore Amanda’s ideas, concerns and expectations regarding the
consultation.

Specific questions
Establish facts about the vomiting and weight loss
Exclude significant physical pathology
— Energy and general health
— Diet, appetite
— Fever, cough, sputum Page 20
— Nausea, vomiting, diarrhoea, bowel disturbance, abdominal
pain, amenorrhoea
— Thirst, polyuria
Explore symptoms of an eating disorder using the SCOFF questionnaire:1
1. Do you make yourself Sick because you feel uncomfortably full?
2. Do you worry you have lost Control over how much you eat?
3. Have you recently lost more than One stone (approx 6.4 kg) in a three-
month period?
4. Do you believe yourself to be Fat when others say you are too thin?
5. Would you say Food dominates your life?
Exercise regime
Drugs such as laxatives
Or HEADSS1 screen:
— H Home situation
— E Employment, Education, Economic situation
— A Activities, Affect, Ambition, Anxieties
— D Drugs, Depression
— S Sexuality
— S Suicide, Self-esteem, Stress
Explain that a physical examination is needed.

Most likely diagnosis


Explain most likely diagnosis—purging disorder.

Management
Explain reasons for concern about vomiting
Raise possibility of an eating disorder
Perform physical examination and urinalysis
Recommend blood tests to exclude electrolyte disturbance
Assure of ongoing support and follow-up
Reassure that assistance is available.

CASE COMMENTARY

The aim of this case is for the doctor to develop a trusting


relationship with Amanda and establish that she has an eating
disorder. Sensitive questioning and active listening is needed to
get Amanda to admit that she is making herself vomit. Page 21
She does not meet the criteria for either anorexia or
bulimia; however, the self-induced vomiting, or purging
disorder, is classified as Other specified feeding or eating
disorder (OFSED).2 Eating disorders affect 2–3% of people in
Australia and are presenting earlier and in more boys than
previously.3
The SCOFF questionnaire provides a framework for asking
about food and its place in people’s lives.1 Appropriate
management is a combination of the medical aspects, such as a
physical examination and investigations, and the psychosocial
aspects.2 HEADSS4 is another useful framework for
conversation with adolescents.
A poor or inexperienced doctor will focus solely on the
medical implications of her purging and not consider the person
and their situation and the emotion surrounding the problem.
Feedback to the teacher can only be given with Amanda’s
permission.
The doctor is not expected to give details of the proposed
management in the time available, but should reassure Amanda
that there are options for assistance which usually involve her
family and a psychologist, and maybe a psychiatrist,
paediatrician, dietitian, social worker or psychiatric nurse.2

References
1. Morgan, JF, Reid, F & Lacey, JH 1999, ‘The SCOFF questionnaire:
assessment of a new screening tool for eating disorders’, British Medical
Journal, vol. 319, pp. 1467–8.
2. Rowe, E 2017, ‘Early detection of eating disorders in general practice’,
Australian Family Physician, vol. 46, no. 11, pp. 833–8.
3. Madden, S, Morris, A, Zurynski, YA, Kohn, M & Elliot, E. 2009,
‘Burden of eating disorders in 5–13-year-old children in Australia’,
Medical Journal of Australia, vol. 190, no. 8, pp. 410–4.
4. Goldenring, J & Cohen, E 1988, ‘Getting into adolescents’ heads’,
Contemporary Paediatrics, July pp. 75–80.
Further reading
Hay, P, Chinn, D, Forbes, D, Madden, S, Newton, R, Sugenor, L et al. 2014,
‘Royal Australian and New Zealand College of Psychiatrists clinical
practice guidelines for the treatment of eating disorders’, Australian &
New Zealand Journal of Psychiatry, vol. 48, no. 11, pp. 977–1008.
Page 22
Page 23
Section 3
Aged care
Page 24

Case 5
Elsie Humphries

Instructions for the doctor


This is a long case.
Please take a history from Mrs Humphries. Then ask the facilitator for the
results of the physical examination. Discuss your management plan with Mrs
Humphries.

Alternative instructions
Medical students and junior doctors might learn more from this scenario if
they are asked to conduct the physical examination and negotiate a
management plan with Mrs Humphries without a time restriction.

Scenario
You are called to see Mrs Elsie Humphries, an 85-year-old woman, at
home. Her family phoned for the visit because she has told them that she
has had several falls recently. They run a business interstate and visit her
twice a year. The family want reassurance that Mrs Humphries is safe.

The following information is on her summary sheet:


Current medical problems
Diverticular disease
Congestive heart failure
Atrial fibrillation (AF)
Past medical history
Burnt-out rheumatoid arthritis
First child 1959
Second child 1961
Medication Page 25
Warfarin as directed
Digoxin 0.625 mg od
Frusemide 20 mg od
Perindopril 5 mg od
Allergies
Nil
Immunisations
Up-to-date
Social history
Widowed 2007
Husband was an Australian Football League coach
Non-smoker
Alcohol intake—two standard drinks per week.

Instructions for the patient, Elsie Humphries


You are a rural battler. You have survived several droughts and floods and
had rheumatoid arthritis during middle life. You are determined not to let old
age beat you. You have coped since you were widowed and, while you do not
enjoy getting old, you are proud of managing despite having only your cat for
company.
You gave up expecting your children to be of any help to you long ago.
They visit for just long enough to ease their guilt and run back to the city. If
the children had their way they would parcel you up and put you in a home. It
has worried them that you have mentioned a couple of recent falls. The
children have asked your doctor to make a home visit to check that you are
still safe.
You have fallen three times in your home in the past two months. The
most recent fall was last week when you tripped over a door mat going to
hang your washing out. You hit the side of your head on the door frame, but
did not sustain any other injuries or lose consciousness. Your other falls have
been similar. You have had no dizziness and are currently otherwise well.
You have noticed it is harder to see details on the TV, and you stopped
buying the newspaper a couple of years ago when you couldn’t read the small
print. You haven’t noticed any changes in your hearing.
You have given up driving and catch a taxi once per week to the local
shops to buy your groceries, have your blood test for warfarin and pay your
bills at the post office. You manage to cook simple meals at home, although
you are buying microwave meals more and more often. You struggle with
your housework, particularly heavy work such as vacuuming, and you Page 26
haven’t changed the sheets on your bed since you fell changing them a
few months ago. You are mostly continent of urine with the very occasional
accident when you struggle to get out of your chair easily, and you shower
daily although this tires you out. Your doctor has mentioned getting some
home help in the past, but you have brushed off these suggestions, feeling
that it would just be a step towards losing your independence and ending up
in a nursing home.
You have had limited local supports since your close friend passed away
shortly after your husband.
You don’t smoke but you do still enjoy your 50-year habit of a sherry on a
Friday and Saturday night before bed.
Your house is full of a lifetime of collected trinkets and memorabilia,
which makes moving without tripping difficult. You have an alarm, which
you carry round your neck.
Clinical examination findings
You have cataracts and your toenails are so long you cannot wear shoes. You
are wearing slippers, and a headscarf to cover up the bruise on your left
temple from a recent fall.
Your blood pressure is 140/90 mmHg with no postural drop and your
pulse is 72 and irregularly irregular. There is no evidence of congestive
cardiac failure. You have some ulnar deviation of the fingers following your
rheumatoid arthritis.
All other physical examination findings are within normal limits.

The following information is on your summary sheet:


Current medical problems
Diverticular disease
Congestive heart failure
Atrial fibrillation (AF)
Past medical history
Burnt-out rheumatoid arthritis
First child 1959
Second child 1961
Medication
Warfarin as directed
Digoxin 0.625 mg od
Frusemide 20 mg od
Perindopril 5 mg od
Allergies
Nil
Immunisations Page 27
Up-to-date
Social history
Widowed 2007
Husband was an Australian Football League coach
Non-smoker
Alcohol intake—two standard drinks per week.
Information for the facilitator
Clinical examination findings
Mrs Humphries has cataracts, her visual acuity is 6/18 left eye 6/12 right eye
not correctable by glasses, and her toenails are so long she cannot wear shoes.
She is wearing slippers, and a headscarf to cover up the bruise on her left
temple from a recent fall.
Her blood pressure is 140/90 mmHg with no postural drop and her pulse is
72 and irregularly irregular. There is no evidence of congestive cardiac
failure. She has some ulnar deviation of the fingers following her rheumatoid
arthritis.
All other physical examination findings are within normal limits.
Mrs Humphries’ house is full of trinkets and memorabilia, which makes
moving without tripping difficult. She has an alarm, which she carries round
her neck.

Suggested approach to the case


Establish rapport
Open-ended questions to explore Mrs Humphries’ ideas, concerns and
expectations.

Specific questions
Ask about the falls
— Frequency of the falls
— Timing of the falls
— Mrs Humphries’ ideas on the cause of the falls
— Have any of the falls been witnessed?
— Have any injuries been sustained?
— Any history of fits, incontinence or loss of consciousness?
Obtain further history exploring possible causes of falls
— Environment related/accommodation
— Impaired sensory input
— Drugs and alcohol Page 28
— Locomotor disorders
— Lower cerebral perfusion—postural hypotension, syncope,
uncontrolled atrial fibrillation
— Epilepsy
— Systemic illness, e.g. infection
Explore her response to the falls, how she sees her future
Social history—current support mechanisms, how is she managing activities
of daily living, driving, continence
Request permission to examine.

Examination
Ask the facilitator for specific examination findings or seek permission to
examine
Assess the home
• Causes of falls—loose rugs, obstacles, poor lighting, footwear
• Safety items—personal alarm
Assess Mrs Humphries
• Temperature
• Inspect for evidence of injury such as multiple bruises of different ages
• Cardiovascular system
— Pulse, rate and rhythm
— BP standing and sitting
— Jugular venous pulse
— Heart sounds
— Carotid bruits
— Ankle oedema
• Respiratory system
— Respiratory rate
— Chest auscultation
• Neurological system
— Cognitive function
— Hearing and visual acuity, including type of glasses worn
— Tremor
— Weakness
— Proprioception and balance: Romberg’s test
— Coordination
• Movements
— Walking
— Neck movements Page 29
— Falls risk-assessment tests, e.g. sit-to-stand test, 6-metre walk
• Mental health
— Depression
— Anxiety
• Miscellaneous
— Thyroid
— Toenails
— Metabolic.

Investigations
FBC ECG
UEC 24 hr ECG
BSL TFTs
Vitamin D
LFTs Exclude underlying urine infection—MSU for MCS
CT head To exclude cerebral haemorrhage.

Management
Outline to Mrs Humphries that her cataracts and her toenails may be causing
the falls
Explain treatment is available for both
Seek Mrs Humphries’ consent for referral and treatment:
• ophthalmologist for cataracts
• podiatrist for toenails.
Drug therapy:
• monitor current therapy
• consider risk–benefits of warfarin with CHA2DS2–VASc and HAS-
BLED calculators.1
Maintain activity and mobility
With Mrs Humphries’ permission, she should be referred to My Aged Care (
www.myagedcare.gov.au) for assessment. Some reassurance that services
can help her maintain her independence may encourage this.
Other options to consider will depend on the services available locally but
could include occupational therapy, physiotherapy, group
exercise/balance programs, continence services, social services, volunteer
services such as Meals on Wheels, trial of hip protectors, referral to a falls
clinic.
The issues to include regarding home safety are:
• current likelihood of fall—is she safe staying at home?
• ability to feed, dress and bathe safely
• clear passageways of trinkets and memorabilia Page 30
• remove loose rugs
• get frame/walking aid
• telephone near bed
• regular visitors.
Bone density scan should be considered
Arrange follow-up
Ask Mrs Humphries if she wishes you to discuss the outcome of the visit
with her family.

CASE COMMENTARY

This is a common general practice scenario: an elderly,


independent lady who is at risk of losing her independence.
Assessing Mrs Humphries at home is key to determining her
level of risk and seeing life from her perspective. This richer
understanding of the person and their life, compared to the
impression gleaned in a clinic room, provides a good foundation
for discussing her ability to continue living independently.
Doctors are expected to demonstrate a systematic approach to
the problem of falls in the elderly, addressing the physical,
psychological and social issues. Interventions can help elderly
people to live safely and independently and can delay admission
to hospital or nursing home care. The history should focus on
the details of the falls and on identifying a potential cause
together with a risk assessment of the likelihood of further falls
and significant injury for Mrs Humphries. Mrs Humphries’
perspective on the falls must be explored. Is she frightened and
seeking residential care? Or would she prefer to stay at home
despite the risks? Mrs Humphries’ past history of rheumatoid
arthritis is a risk factor for osteoporosis.
In this case the examination reveals cataracts and long
toenails. The facilitator will only mention this pathology if
asked. These are treatable and the doctor can be optimistic that
Mrs Humphries can maintain her independence once these
issues are dealt with appropriately. I have seen one doctor offer
to cut the toenails that day. Before the cataracts are treated, the
doctor may need to offer short-term residential care. If Mrs
Humphries goes outside, it is worthwhile for her to have a
single-lens pair of spectacles to avoid falls that can be caused by
multifocal lenses.
Referral to allied health services through the Enhanced
Primary Care items can follow completion of a care plan. Mrs
Humphries might benefit from a Home Medicines Review; it is
imperative that her warfarin is reviewed—does the risk of
warfarin therapy now outweigh its potential benefits?

COMMON PITFALLS Page 31


This case requires the doctor to focus their questions and
examination on the common causes of falls in the elderly.
Doctors who take an overly-detailed history or examination will
run out of time and not progress to discussing management with
Mrs Humphries.
Mrs Humphries is the doctor’s patient but the family have
asked for the visit. It is important to involve her family in her
care, however, her consent is needed before discussing the
outcome of the visit with them.
There is a risk of either bowing to the relatives’ pressure and
arranging admission for Mrs Humphries or colluding with Mrs
Humphries’ denial of the issues and not taking action where it is
needed.

Reference
1. National Prescribing Service 2017, Predicting risk with oral
anticoagulants. Available at: www.nps.org.au/medical-info/clinical-
topics/news/predicting-riskwith-oral-anticoagulants, accessed 24 June
2018.

Further reading
Gillespie, LD, Robertson, MC, Gillespie, WJ, Sherrington, C, Gates, S,
Clemson, LM, et al. 2012, ‘Interventions for preventing falls in older
people living in the community’, Cochrane Database of Systematic
Reviews, doi: 10.1002/14651858.CD007146.pub3.
Mackenzie, L & Clemson, L 2014, ‘Can chronic disease management plans
including occupational therapy and physiotherapy services contribute to
reducing falls risk in older people?’, Australian Family Physician, vol.
43, pp. 211–5.
The Royal Australian College of General Practitioners 2018, Guidelines for
preventive activities in general practice, 9th ed, updated, RACG, East
Melbourne, Vic.
Waldron, N, Hill, AM & Barker, A 2012, ‘Falls prevention in older adults—
assessment and management’, Australian Family Physician, vol. 41, pp.
930–5.
Page 32

Case 6
Flora McMillan

Instructions for the doctor


This is a short case.
Please explain the test results to Flora and develop a management plan
with her for the prevention of further osteoporotic fractures.

Scenario
In January of this year Flora McMillan, aged 73, fell on her left arm and
sustained a Colles fracture. She recently had a bone densitometry scan
done and the results are:

Scan BMD T- Fracture Z- Peer


g/cm2 score risk score relationship
Lumbar 0.854 −2.6 Marked −1.2 Lowest
spine quartile
Total femur 0.686 −2.6 Marked −1.5 Lowest
(L) quartile

T-score—comparison with young normal adult


Z-score—comparison with aged matched controls

The following information is on her medical record:


Past medical history
Hyperlipidaemia
Medication
Simvastatin 40 mg od
Allergies
Nil known
Immunisations
Up-to-date
Family history Page 33
Father died of a myocardial infarction, aged 45
Social history
Teacher, semi-retired
Non-smoker
Height 1.6 m
Weight 53 kg
BMI 20.7 kg/m2.

Instructions for the patient, Flora McMillan


You are a 73-year-old teacher. You work part-time in the school library. In
January of this year, you tripped over while on a walking holiday and broke
your left wrist. (You did not quite hear the name of the fracture—it sounded
like a Collie fracture but you though Collies were dogs.)
You are now back at work. Your wrist is apparently healed but it feels stiff
in the morning and aches after a long day of carrying books.
Your GP arranged a bone density scan. You are here to find out the results
and want to hear about anything you can do to prevent further fractures.
The following information is on your medical records:
Past medical history
Hyperlipidaemia
Medication
Simvastatin 40 mg od
Allergies
Nil known
Immunisations
Up-to-date
Family history
Father died of a myocardial infarction, aged 45
Social history
Teacher, semi-retired
Non-smoker
Height 1.6 m
Weight 53 kg
BMI 20.7 kg/m2.

Suggested approach to the case


Establish rapport
Check reason for attendance—to obtain results of bone density scan
Give results Page 34
— Explain results in terms that Flora understands—scan
demonstrates osteoporosis (‘thinning of the bones’)
— Link osteoporosis with recent fracture
— Explain risk of further fractures and need to prevent
Maintain/improve bone density
— Calcium intake—recommended 1000–1500 mg calcium per day,
ideally through low-fat dietary sources; may need supplement if
unable to meet requirements in diet
— Vitamin D—assess sun exposure, check and replace Vitamin D if at
risk
— Weight-bearing exercise 30–60 minutes, three to four days per week
— Initiate treatment with anti-osteoporosis medication, e.g.
bisphosphonate or denosumab
— Avoid excess alcohol and caffeine. Don’t start smoking
Fracture prevention
— Preventing falls—regular exercises to maintain sense of balance and
proprioception, appropriate footwear, safety at home, caution at night.
(For higher risk patients, consider occupational therapy assessment,
walking aids and hip protectors)
— Review of medications that predispose to bone loss/falls, e.g. steroids,
sleeping tablets, anti-hypertensives can cause postural drop
Consider referrals to allied health professionals, e.g. exercise
physiologist, dietician, occupational therapist to provide further
assistance with addressing the above issues
Plan follow-up to review medication and reinforce lifestyle advice
— A repeat BMD is generally not required unless considering a change
to or cessation of anti-osteoporosis medication.

CASE COMMENTARY

This case requires the GP to interpret Flora’s bone density scan


results and give her understandable, practical information about
osteoporosis and preventing falls and further fractures. The Z-
score compares Flora’s bone mineral density to that of other
women of her age whereas the T-score compares Flora’s bone
mineral density to that of normal young women. The T-score
relates more closely to fracture risk. A T-score less than –2.5 is
consistent with the diagnosis of osteoporosis and each standard
deviation reduction in bone density is associated with an
approximate doubling of the risk of hip fracture.
Osteoporosis prevention and treatment characterises much of
modern general practice: the evidence regarding best practice is
changing rapidly and can be contradictory for different Page 35
conditions. Flora’s osteoporosis needs good calcium and vitamin
D levels, but in the low-fat diet needed for her hyperlipidaemia
many of the foods that contain these are reduced. An association
between calcium supplements and heart disease adds further
complications. Prevention of skin cancer requires patients to
avoid sun exposure—just the very thing needed to produce
vitamin D! Balancing these competing guidelines is difficult.
Pharmacological treatment options include an oral or
intravenous bisphosphonate or denosumab, and second-line
options are raloxifene, strontium or teriparatide. Careful
consideration should be given to prescribing strontium for
people at increased risk of cardiovascular disease. The
recommended daily intake of calcium for women over 50 is
1300 mg/day. This requires at least three serves of dairy foods
per day, with one of these being calcium fortified.

COMMON PITFALLS

There is a risk of seeing osteoporosis and fracture prevention


purely in terms of medication. Drugs play an important role but
are only one aspect of management. Simple advice about
maintaining fitness and balance, safety at home, a healthy diet
and adequate but not excessive sun exposure is also needed. The
challenge is to say this in a way that Flora does not feel
patronised. She sees herself as a working teacher, not an old
lady!

Further reading
Ewald, D 2012, ‘Osteoporosis—prevention and detection in general practice’,
Australian Family Physician, vol. 41, pp. 104–8.
The Royal Australian College of General Practitioners and Osteoporosis
Australia 2017, Osteoporosis prevention, diagnosis and management in
postmenopausal women and men over 50 years of age, 2nd ed, RACGP,
East Melbourne. Available at: www.osteoporosis.org.au/sites/default/file
s/files/20439%20Osteoporosis%20guidelines.pdf, accessed 23
November 2018.
Therapeutic Drugs Administration 2014, ‘Strontium ranelate (Protos) and risk
of adverse events’. Available at: www.tga.gov.au/alert/strontium-ranelat
e-protosand-risk-adverse-events-0, accessed 23 November 2018.
Therapeutic Guidelines 2014, ‘Treating osteoporosis: calcium and vitamin D
supplements and osteoporosis therapy’, in eTG complete [Internet], TG,
Melbourne.
Winzenberg, T, van der Mei, I, Mason, RS, Nowson, C & Jones, G 2012,
‘Vitamin D and the musculoskeletal health of older adults’, Australian
Family Physician, vol. 41, pp. 92–9.
Page 36

Case 7
Nell Worthington

Instructions for the doctor


This is a short case.
Please take a history as if you were in clinical practice and then outline
your plans to Mrs Worthington.

Scenario
Mrs Nell Worthington is an 80-year-old retired legal secretary. She has
hypertension. She has just moved to your area to be nearer to her
daughter. She has brought her summary sheet from her previous practice.

The following information is on her summary sheet from her previous


practice:
Past medical history
Hypertension diagnosed 2013
Osteoarthritis of neck and hips
Vaginal hysterectomy for prolapse 1995
Four children
Medication
Spironolactone 25 mg od
Ramipril 10 mg od
Potassium chloride 600 mg slow release 1 tab od
Allergies
Nil known
Immunisations
Up-to-date
Social history
Widowed 2008.

Instructions for the patient, Nell Page 37

Worthington
You are an 80-year-old retired legal secretary. You have just moved to be
nearer your daughter. Life has been hard since Vince, your husband, died in
2008. You are feeling lost as you try to establish your new life in this town.
You had attended the same doctor back home for years and are nervous about
coming here today.
You have been feeling lethargic recently but have put this down to the
move. Your opening line will be, ‘Doctor I’m feeling tired, but maybe this is
because I have just moved here.’

The following information is on your summary sheet:


Past medical history
Hypertension diagnosed 2013
Osteoarthritis of neck and hips
Vaginal hysterectomy for prolapse 1995
Four children
Medication
Spironolactone 25 mg od
Ramipril 10 mg od
Potassium chloride 600 mg slow release 1 tab od
Allergies
Nil known
Immunisations
Up-to-date
Social history
Widowed 2008.

Suggested approach to the case


Establish rapport
Welcome Mrs Worthington to the practice
— Ask about the move, is she settling in?
Explore patient’s ideas, concerns and expectations regarding this
consultation.

Specific questions
Enquire about current symptoms
Brief systems review—any problems with your breathing, your appetite, your
bowels, your ‘waterworks’, sleep, energy level, weight changes?
Sensitively explore any mood symptoms
Review information on summary sheet with Mrs Worthington Page 38
Confirm current medication
Explore other cardiac risk factors
— Exercise
— Smoking
Other preventive health measures
— Immunisation status
— Alcohol consumption
Full physical examination—needed, but not part of this consultation.
Investigations
Explain need for investigations
Surgery tests: BSL or urine dipstick, ECG
Lab tests
— Arrange urgent UEC—drug regimen risks hyperkalaemia, can cause
tiredness
— Non-urgent BSL, FBC, LFTs, TFTs.

Management
Cease potassium supplement and explain why
Arrange follow-up—how Mrs Worthington will get the results.

CASE COMMENTARY

The first consultation for a new patient at a practice provides a


good opportunity for a thorough medical review. Mrs
Worthington’s drug regimen puts her at risk of hyperkalaemia
and she needs urgent assessment of her electrolytes.
Mrs Worthington’s lethargy could be due to the recent
relocation but, given her age, it is important to exclude other
pathology. Doctors are expected to deal with the immediate
issue of potential hyperkalaemia and then make arrangements
for further review. This prioritisation of ‘what do I need to do
today, what do I need to do in the next week, what do I need to
do in the next month’ is an important skill for GPs to learn and
demonstrate. The cost of admission creates a tendency to do
everything at once for hospitalised patients, but in general
practice the best use of resources and the best way to help
patients to change is to establish priorities and work through
them systematically over time.
This case is based on a true story. The patient was Page 39
admitted to hospital because of the hyperkalaemia—a
preventable admission. The rates of morbidity and mortality
associated with hyperkalaemia have risen with the increased use
of angiotensin-converting enzyme inhibitors, angiotensin
receptor blockers and aldosterone receptor antagonists. Adverse
drug reactions were reported in 10% of patients seen in
Australian general practice over a six-month period, in 16% of
patients presenting to emergency departments, and in up to 20%
of hospital admissions.

COMMON PITFALLS

Doctors should avoid unfair criticism of the doctor who


instigated Mrs Worthington’s drug regimen. The ideal would be
to discreetly contact the prescriber and find out why this regimen
was chosen, enabling the person to learn from the experience.

Further reading
Kalisch, LM, Caughey, GE, Barratt, JD, Ramsay, EN, Killer, G, Gilbert, AL
et al. 2012, ‘Prevalence of preventable medication-related
hospitalizations in Australia: an opportunity to reduce harm’,
International Journal for Quality in Health Care, vol. 24, pp. 239–49.
Miller, GC, Britt, HC & Valenti, L 2006, ‘Adverse drug events in general
practice patients in Australia’, Medical Journal of Australia, vol. 184,
pp. 321–4.
Nyirenda, MJ, Tang, JI, Padfield, PL & Seckl, JR 2009, ‘Hyperkalaemia’,
British Medical Journal, vol. 339, p. b4114.
Phillips, AL, Nigro, O, Macolino, KA, Scarborough, KC, Doecke, CJ,
Angley, MT et al. 2014, ‘Hospital admissions caused by adverse drug
events: an Australian prospective study’, Australian Health Review, vol.
38, pp. 51–7.
Page 40

Case 8
Margaret Wilson

Instructions for the doctor


This is a short case.
Please take a history from Margaret Wilson and discuss your management
plan with her.

Scenario
Margaret and Don Wilson have been your patients for many years. Don
has developed Alzheimer’s disease. You made the diagnosis on the basis
of a typical history and an unremarkable organic screen. Don has been
seeing a geriatrician who has started him on donepezil (Aricept) with
little response to date. Margaret is generally healthy but is carrying
Don’s care almost single-handedly and you are concerned about how she
is coping.
The following information is on her summary sheet:
Age
74
Past medical history
Osteoarthritis
Medication
Paracetamol 665 mg 2 tds
Allergies
Nil known
Immunisations
Up-to-date
Social history
Married
Non-smoker
Alcohol intake—two standard drinks per week.

Instructions for the patient, Margaret Page 41

Wilson
You and your husband, Don, have been coming to see this doctor for many
years. Don is 76 and has recently been diagnosed with Alzheimer’s disease.
He sees a visiting geriatrician (Dr Sue Davies), who has prescribed a
medication—Aricept—but so far there doesn’t seem to be any improvement.
You are finding his care very draining and are worried about how you are
going to manage. Don is anxious when you are not nearby, so he follows you
around the house or garden and seems to need your reassurance all the time.
He asks the same things over and over again and needs assistance with all but
the most basic activities. You have found yourself getting increasingly
frustrated with him and have lost your temper with him lately. You have
never been physically violent, but you worry about what might happen if
things continue the way they are. Dr Davies suggested Don go to home group
once or twice a week, but he doesn’t want to go and usually refuses. You
have stopped going out to most of your usual activities (walking, croquet and
bridge) because you don’t feel you can leave Don alone. You have lost
contact with most of your friends and can feel yourself being worn down
emotionally.
Your appetite is normal and you are sleeping reasonably well, but you
often worry about what is going to happen to Don. Your mood is dominated
by worry about Don but is otherwise OK and your energy levels seem
normal. Your children live interstate and are busy with their own lives. You
don’t have anyone you feel you can share your worries with, and you are
grieving the relationship you previously had with Don where you could talk
about things. If the doctor raises possible placement in care, you respond by
saying, ‘Oh, I could never do that’. Don needs almost constant help, and you
have heard stories about people being neglected in nursing homes. You can’t
bear to imagine Don wasting away by himself in one of those places—
besides you took a vow to stay with him in sickness and in health. You have
not had any suicidal thoughts but you feel lonely and trapped in an
impossible situation with no way out. You are hopeful the doctor will have
some answers for you.

The following information is on your summary sheet:


Age
74
Past medical history
Osteoarthritis
Medication
Paracetamol 665 mg 2 tds
Allergies
Nil known
Immunisations Page 42
Up-to-date
Social history
Married
Non-smoker
Alcohol intake—two standard drinks per week.

Suggested approach to the case


Establish rapport
Open questions to explore Margaret’s concerns.

Specific questions
Explore Margaret’s ability to cope
Establish whether Don and Margaret are safe
How is she currently managing her stress
Explore Margaret’s supports
Sensitive questioning regarding option of applying for carer’s pension.

Management
Reflect on your concerns about Margaret’s current stress
Demonstrate empathy and acknowledge the difficulties her role entails
Education about carer stress and its management principles:
• need for regular respite, offer referral to arrange
• involvement in a carer support group
• making time for herself for social and physical activities and connecting
with others.
Ensure regular and close follow-up.

CASE COMMENTARY

About 1.2 million Australians care for someone with dementia.


GPs play a crucial role in managing the needs of people with
dementia and caregivers, by providing ongoing support and by
facilitating access to evidence-based care. Scheduled, regular
reviews of people with dementia and their caregivers should be
standard practice, and this case allows those skills to be
demonstrated. It is common for caregivers to feel trapped as
their partner’s demands increase and their own coping ability
diminishes. Margaret’s situation has many of the Page 43
recognisable risk factors for elder abuse, and the safety of
both patients needs to be sensitively explored. Margaret needs
education, resources and practical assistance in order to become
a healthy caregiver.
Many carers find that the experience of a carer support group
is invaluable, mainly because it connects them with others who
have faced similar circumstances and challenges and who can
share their struggles and solutions. This can help combat a
carer’s feelings of isolation, guilt, uncertainty and helplessness.
Margaret needs assistance in developing a strong support
network of her own, as well as regular, quarantined timeout for
herself. She should be encouraged to resume her activities—both
social and physical—that have been set aside as the demands of
caring for Don have increased.

COMMON PITFALLS

If we approach the situation with the single focus of convincing


Margaret to place Don in residential care, we miss the
opportunity to explore other fruitful areas like carer stress and its
management. Elder abuse is a serious and all-too-common
problem and needs to be explored in a situation like this.
Utilising the various community supports for Margaret without
losing the central, caring role of the GP can also be a challenge.

Further reading
Alzheimer’s Association 2014, ‘Caregiver stress’. Available at: www.alz.org/
care/alzheimers-dementia-caregiver-stress-burnout.asp, accessed 25
September 2014.
Brooks, D, Ross, C & Beattie, E 2015, Caring for someone with dementia:
the economic, social and health impacts of caring and evidence based
supports for carers, Alzheimers Australia. Available at: www.dementia.
org.au/files/NATIONAL/documents/Alzheimers-Australia-Numbered-P
ublication-42.pdf, accessed 16 August 2018.
Burke, D 2012 ‘Carer stress’, Australian Doctor, 15 May 2012. Available at:
www.ausdoc.com.au/therapy-update/carer-stress, accessed 23 May
2019.
Commonwealth of Australia 2006, Dementia: the caring experience. A guide
for families and carers of people with dementia, Canberra, ACT.
Strivens, E & Craig, D 2014, ‘Managing dementia-related cognitive decline
in patients and their caregivers’, Australian Family Physician, vol. 43,
no. 4, pp. 170–4. Page 44
Page 45
Section 4
Cardiovascular
system
Page 46

Case 9
Helen Berkovic

Instructions for the doctor


This is a short case.
Please negotiate a management plan with Helen.

Scenario
Helen Berkovic is a 53-year-old prison officer. You have just returned
from leave and in your absence Helen has been seeing your locum. The
locum has diagnosed essential hypertension as blood pressure readings
have been over 180/110 on three different occasions.

The following information is on her summary sheet:


Past medical history
Nil
Medication
Nil
Allergies
Nil
Immunisations
Up-to-date
Family history
Father—on blood pressure medication
Social history
Smokes 10 cigarettes per day.

Instructions for the patient, Helen Berkovic


You are a 53-year-old prison officer. Your own GP has been on leave and
you have been seeing the locum. The locum has diagnosed high blood
pressure but you don’t know what this means or what can be done about it.
Your dad has been taking medication for his blood pressure for years.
You have now come to see your own doctor to plan what to do. Page 47

The following information is on your summary sheet:


Past medical history
Nil
Medication
Nil
Allergies
Nil
Immunisations
Up-to-date
Family history
Father—on blood pressure medication
Social history
Smokes 10 cigarettes per day.
The locum conducted a physical examination that was normal. Your BMI
is 27 kg/m2 and urine dipstick was normal, with no evidence of micro/macro-
albuminuria. The CXR, ECG, echocardiogram, FBC, UEC, LFTs, fasting
blood glucose and fasting lipids were normal.

Suggested approach to the case


Management
Re-establish rapport
Confirm diagnosis of essential hypertension
Confirm with Helen that investigations and examination were normal
Explore Helen’s understanding of hypertension and treatment.

Treatment options
Non-pharmacological
— Exercise
— Weight loss
— Salt reduction
— Reduce alcohol intake
— Address potential life stressors—work
— Treatment of obstructive sleep apnoea
Discuss smoking and assess readiness to quit
Pharmacological
— Start medication.

Follow-up Page 48

Document BP and suggest mechanism for patient to record readings, for


example, manually on a card or using an app on a smartphone or tablet Add
to recall system to recheck UEC and BP on treatment Provide sources for
further information for the patient.

CASE COMMENTARY
This is a common general practice situation. The first aim of this
case is to ensure that the doctor has the skill to take over
management of a patient from a colleague.
Traditional medical training is for doctors to plan patient
management after taking a history and conducting an
examination themselves. The increase in team practice requires
doctors to pick up a consultation at different stages of the
process. Reading the patient’s notes was considered ‘cheating’
prior to seeing patients as a medical student: now it is a
prerequisite of every consultation. Scanning notes with an
attitude of critical trust is needed.
The second aim is to ensure the doctor can think holistically
about hypertension and cardiovascular risk. Helen needs
tailored, practical assistance to lower her risk of cardiovascular
disease both through medication and via non-pharmacological
interventions such as quitting smoking and increasing physical
activity.
Helen meets the criteria for starting medication for her blood
pressure. ACEI, ARB, calcium-channel blockers, thiazide-like
diuretics are drugs of first choice for uncomplicated
hypertension in non-pregnant adults. The actual choice is
influenced by individual patient factors such as associated
medical conditions and the risks of adverse effects of the drug.
Lastly, arrangements for ongoing monitoring and follow-up
are needed.

COMMON PITFALLS

While lifestyle advice is recommended for all patients, in


patients at low absolute CVD risk (<10% five-year risk) with
persistent blood pressure > 160/100 mmHg, antihypertensive
therapy should be started once the diagnosis is confirmed.
Further reading Page 49

Banks, E, Crouch, SR, Korda, RJ, Stavreski, B, Page, K, Thurber, KA &


Grenfell, R 2016 ‘Absolute risk of cardiovascular disease events, and
blood pressure- and lipid-lowering therapy in Australia’, Medical
Journal of Australia, vol. 204, no. 8, p. 320.
Blood Pressure Lowering Treatment Trialists Collaboration, Sundstrom, J,
Arima, H, Woodward, M, Jackson, R, Karmali, K et al. 2014, ‘Blood
pressure-lowering treatment based on cardiovascular risk: a meta-
analysis of individual patient data’, Lancet, vol. 384, pp. 591–8.
National Heart Foundation of Australia 2016, Guidelines for the diagnosis
and management of hypertension in adults, National Heart Foundation
of Australia. Available at: www.heartfoundation.org.au/images/uploads/
publications/PRO-167_Hypertension-guideline-2016_WEB.pdf,
accessed 18 February 2019.
O’Callaghan, CJ, Goh, MY & Rong, P 2013, ‘Hypertension—the difficult
decisions’, Australian Family Physician, vol. 42, pp. 376–9.
Therapeutic Guidelines Ltd 2018, ‘Elevated blood pressure: pharmacological
management’. In eTG Complete [Internet], Therapeutic Guidelines Ltd,
Melbourne, Vic.
The Royal Australian College of General Practitioners 2015, Smoking,
nutrition, alcohol, physical activity (SNAP): A population health guide
to behavioural risk factors in general practice, 2nd ed, RACGP, East
Melbourne, Vic.
The Royal Australian College of General Practitioners 2018, Guidelines for
preventive activities in general practice, 9th ed, updated, RACGP, East
Melbourne, Vic, pp. 85–91.
Page 50

Case 10
Dilip Patel

Instructions for the doctor


This is a short case.
Please respond appropriately to Dilip’s questions.

Scenario
Dilip Patel is a 44-year-old plumber who lives in your rural town. Four
weeks ago he had severe chest pain while working and was admitted to
hospital with acute coronary syndrome. Tests confirmed myocardial
infarction and triple vessel disease for which he had coronary artery
bypass graft (CABG) surgery in a metropolitan tertiary hospital. Dilip is
now on the following medications:
• Atorvastatin 80 mg od
• Glyceryl trinitrate spray prn
• Aspirin 100 mg od
• Clopidogrel 75 mg od
• Perindopril 5 mg od
• Metoprolol 50 mg od.

The following information is on his summary sheet:


Past medical history
Nil
Allergies
Nil
Immunisations
Nil recorded
Family history
Nil recorded
Social history
Plumber
Ex-smoker.

Instructions for the patient, Dilip Patel Page 51

You are a 44-year-old plumber in a rural town. Four weeks ago you had
severe chest pain while working and you were admitted as an emergency to
the local hospital. You were told that you had had a heart attack. After lots of
tests you had urgent bypass surgery in a big city hospital.
You are now back home. On discharge from the hospital you were given a
long list of medications to take. You have never needed tablets before and
keep forgetting to take them.
You are unsure what the tablets are for as you assumed the bypass surgery
cured the problems. You have all the tablets with you in a shopping bag in
case the GP asks you what you are taking. You have the following questions
for the GP:
• why do I need to take tablets when the operation fixed the problem?
• what does each tablet do?
• how long will I be on these tablets?
• do they have any side effects?
You take:
• Atorvastatin 80 mg once per day
• Glyceryl trinitrate spray as required for chest pain
• Aspirin 100 mg once per day
• Clopidogrel 75 mg once per day
• Perindopril 5 mg once per day
• Metoprolol 50 mg once per day.
You quit smoking 10 years ago and do not drink alcohol. You do no
regular exercise.
Once the GP has discussed your worries about the medication you will be
interested to hear any other advice the GP offers. If the GP starts to give other
advice before answering your concerns about the medication you will get
quite cranky and irritable.

The following information is on your summary sheet:


Past medical history
Nil
Allergies
Nil
Immunisations
Nil recorded
Family history
Nil recorded
Social history Page 52
Plumber
Ex-smoker
Alcohol—nil.

Suggested approach to the case


Establish rapport
Explore Dilip’s understanding and concerns about his medication
Ask if there are any other queries today:

Why do I need to take tablets when the operation fixed the problem?
The operation has bypassed the blocked blood vessels or ‘pipes’. You need to
keep the new vessels or ‘pipes’ open and prevent blockages in any other of
your blood vessels. This is best done with a healthy lifestyle and medication.

What does each tablet do?


• Atorvastatin 80 mg tablets help to stop fat build-up in the blood vessels
along with a healthy diet that includes five serves of vegetables and two
serves of fruits per day.1
• Glyceryl trinitrate spray prn helps to open up the blood vessels to get
more oxygen to the heart if you have any chest pain.2
• Aspirin 100 mg od and Clopidogrel 75 mg od both ‘thin’ the blood. They
reduce the risk of blood clots and another heart attack.1
• Perindopril 5 mg od is an ACE inhibitor. It works on a chemical in the
kidney that makes your blood pressure go up. An ACE inhibitor protects
the kidneys and keeps your blood pressure down, which reduces the risk
of another heart attack.1
• Metoprolol is a beta-blocker. This blocks any excess adrenaline from
damaging your heart and blood vessels by slowing your heart rate and
lowering your blood pressure.1

How long will I be on these tablets?


You will be able to stop the Clopidogrel a year after your surgery provided
there are no further risks. Normally it is best for people to stay on the other
medication. This will be reviewed at regular intervals.

Do they have any side effects?


All medications have potential side effects. If you are concerned if something
might be a side effect either phone or come in for advice. The common ones
are:2
• Atorvastatin—muscle pains
• Glyceryl trinitrate spray prn—headaches, dizziness
• Aspirin—stomach pains (gastritis)
• Clopidogrel—bleeding, diarrhoea Page 53
• Perindopril—cough, reduced kidney function
• Metoprolol—tiredness, cold hands and feet, impotence.
If you have time, raise other common concerns—understanding of procedure
done, return to driving (minimum four weeks post-CABG for private car),
return to work, advice about sexual activity, mood disturbance post-heart
attack.

Summarise information
Surgery was needed to bypass the blocked blood vessels
Aim now to get fit and stay healthy by:
• taking medication as prescribed
• regular check-ups—blood pressure, lipids
• not smoking
• graded increase in exercise
• healthy diet—offer dietician referral
• establish cardiac rehabilitation program.2
Offer to provide printed patient handout on any of these topics.

CASE COMMENTARY

Despite overwhelming evidence of the effectiveness of


secondary prevention therapies, surveys indicate poor adherence
to medical treatments and lifestyle recommendations after an
acute coronary syndrome.3, 4
Dilip is a plumber who spends his life fixing blocked pipes.
He understands the principles of coronary artery bypass grafting
well but needs to also understand the risk of re-blockage and
what can be done to prevent it.
The GP should be able to explain the role of each medication
and its potential side effects in straightforward lay terms,
avoiding jargon, as this will increase Dilip’s chances of
following the advice being given.5 The GP will also need to
discuss other cardiovascular risk factors such as levels of
activity and smoking.6 Patients recovering from surgery have
many concerns. Dilip firstly wants his questions answered about
the drugs but will also be grateful to a GP who raises more
sensitive issues, such as mood disorders5 and when it is safe to
resume sexual intercourse, work and driving.

COMMON PITFALLS

Dilip wants to be treated with respect. He needs more than just


being told to take the tablets because ‘the hospital said so’.
The GP may be tempted to criticise the hospital for not Page 54
giving Dilip enough information about his medication.
Experience suggests that patients remember less than half of
what they are told. Factors that make this worse are fear, pain,
recent anaesthesia and anxiety, all of which may apply to Dilip.
It’s important to give him information politely and not to assume
that the hospital made no effort. But if this is a consistent gap
with a particular hospital unit it is worth phoning the consultant
to let them know your observation and ask what systems are in
place. If Dilip continues to be perplexed or uncertain about his
medication, the GP can ask if he would like a Home Medicines
Review with a pharmacist.

References
1. National Heart Foundation of Australia 2012, Reducing risk in heart
disease; an expert guide to clinical practice for secondary prevention of
coronary heart disease. Available at:
www.heartfoundation.org.au/images/uploads/publications/Reducing-risk-
in-heart-disease.pdf, accessed 18 February 2019.
2. Therapeutic Guidelines Ltd 2018, ‘Secondary prevention of
cardiovascular events’. In: eTG complete [Internet], Melbourne, Vic.
3. Thakkar, JB & Chow, CK 2014, ‘Adherence to secondary prevention
therapies in acute coronary syndrome’, Medical Journal of Australia, vol.
201, pp. S106–9.
4. Stroke Foundation 2012, Guidelines for the management of absolute
cardiovascular disease risk; National Vascular Disease Prevention
Alliance. Available at:
www.heartfoundation.org.au/images/uploads/publications/Absolute-
CVD-Risk-Full-Guidelines.pdf, accessed 24 June 2018.
5. Stafford, L, Jackson, HJ & Berk M 2008, ‘Illness beliefs about heart
disease and adherence to secondary prevention regimens’, Psychosomatic
Medicine, vol. 70, pp. 942–8.
6. The Royal Australian College of General Practitioners 2018, Guidelines
for preventive activities in general practice, 9th ed, updated, RACGP,
East Melbourne, Vic.
Page 55

Case 11
Jackie Maloney

Instructions for the doctor


This is a short case.
Please take a history from Jackie. Ask for the results of a focused
examination from the facilitator. Tell Jackie the most likely diagnosis and
your management plan.

Scenario
Jackie Maloney is a 50-year-old taxi driver. She smokes 30 cigarettes a
day. She has asked for an appointment to see you today because she has
noticed some pain in her chest when she lifts heavy suitcases or has to go
up stairs.

The following information is on her summary sheet:


Past medical history
Mechanical low back pain—recurrent
Hiatus hernia—gastro-oesophageal reflux disease
Medication
Omeprazole 40 mg od
Allergies
Nil
Immunisations
Nil known
Family history
Irish family
Mother died of myocardial infarction aged 60
Social history
Taxi driver.

Instructions for the patient, Jackie Maloney


You are a 50-year-old taxi driver. You feel heaviness in your chest on
exercise. The heaviness lasts for less than a minute if you stop what you are
doing.
You are worried that this heaviness comes from your heart. It has been Page 56
getting worse and you have finally admitted that you need help. At the
back of your mind is the fact that your mother died of a heart attack. You
have not had any palpitations but when pressed admit that you have some
shortness of breath at the same time as you get the chest pain. You last had
pain two days ago.
The most likely diagnosis is angina.

The following information is on your summary sheet:


Past medical history
Mechanical low back pain—recurrent
Hiatus hernia—gastro-oesophageal reflux disease
Medication
Omeprazole 40 mg od
Allergies
Nil
Immunisations
Nil known
Family history
Irish family
Mother died of myocardial infarction aged 60
Social history
Taxi driver.

Instructions for the facilitator


Clinical examination findings
Looks well
No cyanosis
Height 1.65 m
Weight 87 kg
BMI 32 kg/m2
Waist circumference 94 cm
Blood pressure 130/80 mmHg
Pulse 72
Apex beat not displaced
Heart sounds normal, no added sounds
Chest clear
Peripheral pulses present, no oedema.

Please have a copy of Assessing Fitness to Drive available and pass to the
candidate on request. Please keep the copy out of sight until requested.

Suggested approach to the case Page 57

Establish rapport
Open questions to explore patient’s ideas, concerns and expectations.
Specific questions
Detail about the heaviness/pain
Shortness of breath
Palpitations
Cough
— Relieving factors
Assess cardiac risk factors
— Smoking
— Diabetes
— Hypertension
— Hyperlipidaemia
— Family history
Explore lifestyle contributing factors, diet and exercise
Request permission to examine.

Examination
Looks well
No cyanosis
Height 1.65 m
Weight 87 kg
BMI 32 kg/m2
Waist circumference 94 cm
Blood pressure 130/80 mmHg
Pulse 72
Apex beat not displaced
Heart sounds normal, no added sounds
Chest clear
Peripheral pulses present, no abnormalities found, no oedema.

Most likely diagnosis


Angina—sensitive explanation without using medical jargon.

Planned management
Investigations
FBC, UEC, LFTs, fasting blood glucose, lipids—fasting, resting ECG,
consider troponin (see Case Commentary).

Treatment Page 58

— GTN spray
— Aspirin low dose
— Beta-blocker or nitrate or long-acting calcium-channel blocker
Check emotional impact of possible diagnosis
Arrange cardiology referral and further testing according to local protocols
and facilities (see below)
Recommend urgent attendance at hospital/call ambulance if pain not settling
at 10 minutes1
Check and discuss implication for driving2
Ensure follow-up and offer health promotion (smoking cessation,
immunisations, check alcohol intake, advice regarding exercise).

CASE COMMENTARY

This is a case of angina. In the time available the doctor should


be able to state angina as the most likely diagnosis. When this
probable diagnosis is mentioned for the first time, a good doctor
will watch carefully for Jackie’s reaction and allow her to vent
her feelings.
Aspirin is recommended in angina but caution is needed
because of the past history of gastro-oesophageal reflux and
Jackie will need to be asked about recurrences of pain in follow-
up. Beta-blockers, nitrates and calcium-channel blockers appear
to be equally effective for relieving angina.
Knowing when to test troponin is difficult. It is indicated in
suspected acute coronary syndrome (ACS)3 but not in chronic
stable angina. Urgent hospital admission and assessment are
appropriate for the management of ACS, not troponin testing in
the surgery. Jackie’s symptoms do not meet the criteria for ACS
and her angina is not yet classified as stable, so it may be
reasonable to do a troponin test to rule out ACS, provided
follow-up is in place to ensure the result arrives and is actioned
urgently if indicated.4 The role of imaging in ischaemic heart
disease is evolving,5 and decision rules have limited validation
in primary care.5, 6
Jackie needs an early referral to a specialist cardiologist for
probable exercise stress test, stress echocardiography, coronary
angiography or computerised tomography (CT) coronary
angiography to assess her potential for coronary
revascularisation.7 A telehealth consultation is an option in rural
or remote areas without ready access to cardiology services.
A safety point is to consider the impact of angina on Jackie’s
work as a taxi driver. Doctors are not expected to remember the
rules regarding angina and commercial vehicle drivers, but must
be aware that this is an issue and tell the facilitator that Page 59
they would check the regulations and advise Jackie
accordingly. Jackie meets the standards for private driving but
not the commercial standard. Jackie would need to be off work
until a cardiologist assesses the severity of the angina.2

COMMON PITFALLS

Jackie’s livelihood is her ability to drive a taxi; the candidate


should approach the medical and licensing considerations for
cardiovascular disease delicately. The seriousness of Jackie’s
symptoms should be emphasised, not trivialised; an excellent
candidate will be able to alert but not alarm the patient.
Critically, the candidate should provide a safety net for the
patient and outline circumstances where ACS is a risk and they
should seek immediate medical attention.
References
1. Heart Foundation. Heart attack symptoms. Will you recognise your heart
attack? (serial online). Available at: www.heartfoundation.org.au/your-
heart/heart-attack-symptoms/, accessed 1 November 2018.
2. Austroads and National Transport Commission 2017, Assessing Fitness to
Drive, 5th ed, Sydney.
3. Parsonage, WA, Cullen, L & Younger, JF 2013, ‘The approach to
patients with possible cardiac chest pain’, Medical Journal of Australia,
vol. 199, pp. 30–4.
4. Mauro, M, Nelson, A & Stokes, M 2017, ‘Troponin testing in the primary
care setting’, Australian Family Physician, vol. 46, pp. 823–6.
5. Storey, P 2014, ‘Imaging for cardiac disease: a practical guide for general
practitioners’, Australian Family Physician, vol. 43, pp. 260–3.
6. Stokes, M, Nerlekar, N, Moir, S & Teo, K 2016, ‘The evolving role of
cardiac magnetic resonance imaging in the assessment of cardiovascular
disease’, Australian Family Physician, vol. 45, pp. 761–4.
7. Therapeutic Guidelines Ltd 2018, ‘Secondary prevention of
cardiovascular events’. In: eTG complete (Internet), Melbourne, Vic.

Further reading
Australian cardiovascular risk charts, Australian absolute cardiovascular
disease risk calculator. Available at: www.cvdcheck.org.au, accessed 18
February 2019.
Stroke Foundation 2012, Guidelines for the management of absolute
cardiovascular disease risk, National Vascular Disease Prevention
Alliance. Available at: www.heartfoundation.org.au/images/uploads/pub
lications/Absolute-CVD-Risk-Full-Guidelines.pdf, accessed 18 February
2019.
Page 60

Case 12
Eric Schmidt

Instructions for the doctor


This is a long case.
Please take a history and conduct an appropriate examination. Outline the
most likely diagnosis and negotiate a management plan with Mr Schmidt.

Scenario
Mr Eric Schmidt is a retired farmer aged 76. He is used to hard physical
work and is worried that he can no longer walk very far. He did not want
to bother the doctor about it but finally let his daughter make an
appointment when she saw him struggling at the weekend.

The following information is on his summary sheet:


Past medical history
Deep laceration to right arm from farm machinery 1984
Declined to attend recent men’s health screening held at surgery
Medication
Nil recorded
Allergies
Nil recorded
Immunisations
Nil recorded
Family history
Father died of myocardial infarction aged 50
Mother died of ‘old age’
Social history
Farmer
Married with two children.

Instructions for the patient, Eric Schmidt Page 61

You are a retired farmer aged 76. You are used to hard physical work and are
worried because you can no longer walk very far. You did not want to bother
the doctor about it but finally let your daughter make an appointment when
she saw you struggling at the weekend.
You get a cramp-like pain in your right leg after walking a certain
distance. Once you stop and rest for five minutes you can continue on further.
Currently you can get to the first paddock, about 500 m, but then have to lean
on the gate for a while until the pain in your leg goes away.
You do not have any nocturnal pain or any other symptoms of
cardiovascular disease.
You roll your own cigarettes and get through about 20 a day. You drink
two or three cans of beer at the weekend. You have considered giving up
smoking and would like advice from the doctor on how to do this,
particularly if the doctor thinks that this will improve your leg pain.

Instructions for the facilitator


The candidate should perform a cardiovascular examination.

Clinical examination findings


On examination, there are absent popliteal, posterior tibial and dorsalis pedis
pulses on the right leg. There are no bruits. All other findings are within the
normal range.

The following information is on your summary sheet:


Past medical history
Deep laceration to right arm from farm machinery 1984
Declined to attend recent men’s health screening held at surgery
Medication
Nil recorded
Allergies
Nil recorded
Immunisations
Nil recorded
Family history
Father died of myocardial infarction aged 50
Mother died of ‘old age’
Social history
Farmer
Married with two children.

Suggested approach to the case Page 62

Establish rapport
Open-ended questions to explore Mr Schmidt’s ideas, concerns and
expectations.

Specific questions
Establish claudication distance and how quickly pain eases with rest
Absence of pain at night or at rest
Check for other symptoms of cardiovascular disease—angina, TIAs,
shortness of breath, fatigue
Effect of low temperature on symptoms
Impact of symptoms on function
Recent worsening of symptoms
Cardiovascular risk factors—smoking history and readiness to change
Past medical history
Family history
Medication—ask about over-the-counter medication, complementary or
alternative medicines
Request permission to examine.

Examination
Height 1.80 m
Weight 80 kg
BMI 24.7 kg/m2
Waist circumference 82 cm
Blood pressure 135/80 mmHg right arm, 132/78 mmHg left arm
Pulse 65 Apex beat not displaced
Heart sounds normal
No signs of heart failure
No carotid bruits
Peripheral pulses
Skin
— Note colour, distribution of hair
— Postural colour change
— Capillary filling
Urinalysis
— Glycosuria, proteinuria.

Most likely diagnosis Page 63

Intermittent claudication.

Investigations
FBC
UEC
Lipids—fasting with ratio
ECG
Fasting BSL
Exclude arteritis/thrombophlebitis—ESR or CRP
Doppler study of peripheral arteries—ankle-brachial pressure index.

Management
‘Keep walking, stop smoking’
Explain the pathophysiology of intermittent claudication and the rationale for
exercise and giving up smoking
Offer practical advice on quitting smoking; schedule follow-up
Treat any dyslipidaemia, diabetes, hypertension
Start aspirin1
Advise on urgent symptoms that would need early review: critical ischaemia
(gangrene, rest pain, ulcers) and features of acute coronary syndrome
Plan follow-up including review of need for immunisations, particularly
tetanus
Referral for intervention such as angioplasty, stenting or surgery if femoral
pulses absent, critical ischaemia (urgent) or persistent symptoms despite
medical treatment.

CASE COMMENTARY

Mr Schmidt gives a clear history and typical examination


findings of a person with intermittent claudication. Doctors need
to demonstrate that the leg is not critically ischaemic and
measure/discuss the ankle-brachial pressure index (ABPI) to
confirm peripheral arterial disease—an ABPI less than 0.9 is
diagnostic.1, 2 The ‘risk to the limb is low but the risk to life is
high’3 as intermittent claudication is an independent marker for
other cardiovascular problems, such as myocardial infarction or
cerebrovascular disease.4 The focus of management should be
on the reduction of cardiovascular risk1 by treating any Page 64
dyslipidaemia, diabetes and hypertension and by quitting
smoking, exercising regularly5 and dietary modification.2
The doctor will need to explain the link between the current
pain and cardiovascular risk factors and use motivational
interviewing techniques to discuss smoking cessation.
Vasoactive medication may be used if symptoms persist after
initial interventions.1, 2
Mr Schmidt has an occupational risk of tetanus but there is no
information in the summary sheet about his immunisations.
Good doctors will notice this and arrange a review of Mr
Schmidt’s immunisations at another appointment.

References
1. Au, T, Golledge, J, Walker, P, Haigh, K & Nelson, M 2013, ‘Peripheral
arterial disease, diagnosis and management in general practice’,
Australian Family Physician, vol. 42, pp. 397–400.
2. Peach, G, Griffin, M, Jones, KG, Thompson, MM & Hinchliffe, RJ 2012,
‘Diagnosis and management of peripheral arterial disease’, British
Medical Journal, vol. 345, p. e5208.
3. Burns, P, Gough, S & Bradbury A 2003, ‘Management of peripheral
arterial disease in primary care’, British Medical Journal, vol. 326, pp.
584–8.
4. Lakshmanan, R, Hyde, Z, Jamrozik, K, Hankey, GJ & Norman, PE 2010,
‘Population-based observational study of claudication in older men: the
Health in Men Study’, Medical Journal of Australia, vol. 192, pp. 641–5.
5. Handbook of Non-Drug Intervention (HANDI) Project Team 2013,
‘Exercise for intermittent claudication and peripheral arterial disease’,
Australian Family Physician, vol. 42, pp. 879.
Page 65
Section 5
Challenging
consultations
Page 66

Case 13
Doug Sullivan

Instructions for the doctor


This is a short case.
Please take a focused history from Doug. Request the findings of an
appropriate physical examination and investigations. Explain your
provisional diagnosis and initial management plan to Doug.

Scenario
Doug Sullivan is a 32-year-old man who has been a patient of the
practice for 12 months.
He has a history of intravenous drug use (mainly opiates) and alcohol
dependence. He has been off illicit drugs and alcohol for two years and is
compliant with a methadone maintenance program. He is on telmisartan
(Micardis) for hypertension and esomeprazole (Nexium) for gastro-
oesophageal reflux.
He is divorced with three children who live with their mother on the
coast. He is not working consistently but sometimes helps his father who
runs a contract construction crew.
He presented last month with back pain, which he attributed to having
to sleep in his car on a recent trip back from the coast. There were no
neurological symptoms and physical examination was largely
unremarkable apart from paraspinal muscle spasm, so you managed him
with simple measures.
Last week he attended the emergency department with the same pain
and the discharge letter is attached. He was asked to see you if the pain
persisted.

The following information is on his summary sheet: Page 67


Age
32
Past medical history
Hypertension
Gastro-oesophageal reflux
Alcohol abuse/dependence
Opiate dependence IV drug use
Dental caries/poor dentition
Medication
Telmisartan 40 mg mane
Esomeprazole 40 mg mane
Methadone syrup 5 mg/ml 3ml po per day
Paracetamol 500mg 2 qid prn
Allergies
Nil known
Immunisations
Up-to-date
Social history
Divorced—has shared custody of his three children
Smokes 20 cigarettes per day
Non-drinker
Family history
Hypertension
Osteoarthritis.

Below is the letter from the local emergency department.


Dear Doctor
Re: Mr Douglas Sullivan
This 32-year-old man was seen at the emergency department with back
pain. He described three to four weeks of pain in the lumbar region which
was not responding to conservative measures. Further history and physical
examination were consistent with mechanical back pain and there were no
red flags.
He was given oxycodone (Endone) tabs 5 mg 2 tds prn and meloxicam
(Mobic) 15 mg daily and was referred for physiotherapy.
He has been instructed to return to see you if his symptoms do not resolve.
Kind regards,
Dr David Jones RMO

Instructions for the patient, Douglas Page 68

Sullivan
You have now had almost five weeks of pretty constant upper lumbar pain,
which you describe as deep and throbbing. If asked to score it, it is seven or
eight out of ten. It started the day after you slept in your car returning from
visiting your children on the coast. It’s worse with movement but also hurts
when lying down. You find yourself walking around or sitting instead of
lying down. Night-times are very difficult and you often wander around
unable to sleep, or try sleeping in a lounge chair. You’ve tried paracetamol
and ibuprofen as well as a heat pack and some topical creams, none of which
seem to help much.
The oxycodone from the doctor in emergency has not helped, nor has the
meloxicam. You have had low back pain before but this feels different. It is
higher, more constant and more severe. You worry that something else might
be going on.
In answer to specific questioning: There has been no disturbance of
bladder or bowel and no radiation of the pain. There’s been no trauma.
You’ve not had any fever and in fact have been otherwise pretty well other
than having ‘rotten teeth’ which the dentist has started to work on. You have
had two bad teeth removed and need several more extracted, but you are
stretching it out partly because of finances and partly because there’s so
much work to be done. You think you might have lost a couple of kilos in the
last month or two but you put that down to your rotten teeth and not being
able to chew well.
There’s no weakness or altered sensation. You have been tempted to try
alcohol to see if it helps with the pain or with sleep but you feel you’ve
worked hard to get on top of your drinking and don’t want to go back again.
Similarly, with respect to drugs, you have been tempted—especially when
the pain has been bad but have resolved to stay clean for the sake of your
children. You are down to 15 mL of methadone and are hoping to get off it
by the end of this year.

Suggested approach to the case


A focused history and physical examination should explore possible serious
causes of back pain. Most clinicians are familiar with the list of ‘red flags’ for
low back pain but many patients with benign back pain have at least one red
flag. The challenge is to evaluate the red flags in the context of the clinical
presentation.
Doug has a number of risk factors for infection, and further investigation
will yield the diagnosis. An initial discussion about Doug’s condition and an
overview of management should follow.

Points to address in the history include: Page 69


• nature of the pain
• trauma
• neurological symptoms
• constitutional symptoms (fever, weight loss)
• bladder or bowel dysfunction
• general health screen.
Physical examination should include:
• general appearance
• gait and posture
• range of movement
• palpation of the spine
• tendon reflexes, motor and sensory testing.
Further investigation:
• FBC, EUC, LFTs
• ESR/CRP
• imaging—CT lumbar spine.
Management:
• explanation of Doug’s condition in simple language (spinal infection)
• outline of management—may require drainage; parenteral antibiotics
followed by oral antibiotics based on specific organism(s) identified
• organise admission and communicate with receiving hospital.

Physical examination
General appearance: man of stated age in mild distress at rest
Gait: walks slowly bracing his back in the upper lumbar region Vital signs:
• HR 85/min regular
• BP 134/85 mmHg sitting in both arms
• Temp 37.1°C
• BMI 24 kg/m2
• RR 14/min
Spine examination: (each item to be asked for individually)
Appearance: normal
Range of movement: normal
Palpation: locally tender over L2/3 spinous processes with some paraspinal
muscle spasm
Deep tendon reflexes: normal
Straight leg raising: normal
Neurological examination: power, tone and sensation testing all normal
Cardiovascular examination: normal Page 70
Respiratory examination: normal
Surgery tests: normal
Remainder of physical examination is normal.

Investigation results
FBC/EUC/LFTs: normal
ESR 52 CRP 68
CT lumbar spine: There is bony destruction at L2/3 level extending well into
the body of L3 with a small intervertebral/interosseous abscess. There is
loss of the disc space with circumferential soft tissue swelling at L2/3
level. No other abnormality is noted.
CONCLUSION: Acute discitis at L2/3 level with vertebral osteomyelitis and
intervertebral abscess.
X-ray lumbar spine: There is loss of the disc space with circumferential soft
tissue swelling at L2/3 level. There is bony destruction of the body of L3
consistent with osteomyelitis with a possible small intervertebral abscess.
No other abnormality is noted.
Recommend further investigation with a CT or MRI.
Other investigations are normal.

Management
Explain that Doug has an uncommon but serious infection involving his
lumbar vertebra and disc. He will require admission to hospital where he may
undergo surgical drainage of his abscess to ascertain the organism(s)
involved. He will require prolonged (at least several weeks) of antibiotic
treatment, initially parenteral, then oral.

CASE COMMENTARY

Low back pain is the most common musculoskeletal complaint


seen in general practice in Australia. There is a growing
understanding that inappropriate imaging can have a negative
impact on patient attitudes and beliefs and can influence pain
behaviours.1 The concept of red flags alerts doctors to
potentially serious causes among the more common benign
conditions and therefore limiting imaging and further
investigations to those cases where they are indicated.2–4 This
case is about applying the red flags in the overall context of a
patient. Rather than learned as a list of unrelated items, red flags
should be considered in clusters or groups relating to a Page 71
specific serious pathology. Doug has several red flags
associated with spinal infection including his serious ongoing
oral sepsis, a worsening of his pain when supine and at night, as
well as his previous intravenous drug use. In addition, opiates
(including methadone) are known to impair immune function.
In spinal infection symptoms are frequently non-specific and
fewer than 50% have an elevated white cell count. However, C-
reactive protein and erythrocyte sedimentation rate are elevated
in approximately 90% of patients with spinal infection and can
be used to follow patient progress on treatment.5
Any imaging in this case would yield the diagnosis and a
brief discussion of the condition and its management should
follow.

COMMON PITFALLS

Candidates will need to manage their time well in order to


complete the case. Any inefficiencies in history-taking, physical
examination or investigation will likely result in the candidate
running out of time and not reaching management.
Some candidates may confuse this situation with that of a
drug-seeker and lose important time explaining that opiates are
not indicated. Although Doug is in significant pain, he is more
concerned that there may be something serious going on.
It is not sufficient to rely on the assessment done on the
patient in the emergency department the week before. Although
the absence of red flags in the history and examination is listed,
this is clearly not the case.
Doug’s relatively normal physical examination should not
prevent candidates from further investigation.
It is essential to identify the causative organism and
antibiotics should not be commenced in the general practice
setting. In addition, although Doug is not systemically unwell
there should be no contemplation of outpatient management at
this stage as it would significantly impact his chance of full
recovery.
A variation of the case is to ask candidates to perform the
examination of the spine themselves and conduct it as a long
case. The significant findings of spinous tenderness can be
effectively role-played.

References
1. Wheeler, L, Karran, E & Harvie, D 2018, ‘Low back pain: can we
mitigate the inadvertent psycho-behavioural harms of spinal imaging?’,
Australian Family Physician, September, vol. 47, no. 9. Available Page 72
at: www1.racgp.org.au/ajgp/2018/september/low-back-pain,
accessed 25 February 2019.
2. Bratton, R 1999, ‘Assessment and management of acute low back pain’,
American Family Physician, 15 November, vol. 60, no. 8, pp. 2299–306.
3. Jensen, S 2004, ‘Back pain—clinical assessment’, Australian Family
Physician, June, vol. 33, no. 6. Available at:
www.racgp.org.au/afpbackissues/2004/200406/20040601jensen.pdf,
accessed 25 February 2019.
4. Traeger, A, Buchbinder, R, Harris, I & Maher, C 2017, ‘Diagnosis and
management of low-back pain in primary care’, Canadian Medical
Association Journal, 13 November, vol. 189, no. 45, pp. E1386–95.
5. Nagashima, H, Tanishima, S & Tanida, A 2018, ‘Diagnosis and
management of spinal infections’, Journal of Orthopaedic Science,
January, vol. 23, issue 1, pp. 8–13.
Page 73

Case 14
Jeanette Wilkinson

Instructions for the doctor


This is a long case.
Please take a history from Jeanette. Examination findings will be available
from the facilitator. Discuss with Jeanette your differential diagnosis and
identify her issues that need to be addressed. Negotiate a management plan
with her. No investigations will be available but a request for these may be
part of your management plan.

Scenario
Mrs Jeanette Wilkinson is a 48-year-old personal care worker at a
residential aged care facility. Jeanette and her family have been attending
your practice, in a small regional town, for several years, although
Jeanette rarely attends for herself.
The following information is on her summary sheet:
Past medical history
G3P2M1: Two spontaneous vaginal deliveries (healthy babies), one
miscarriage
Medication
Nil regular
Allergies
Nil known
Immunisations
None recorded
Family history
Father myocardial infarction age 65
Social history
Personal care worker at a residential aged care facility
Lives on a farm outside of town Page 74
Cervical screening
Up-to-date and normal.

Instructions for the patient, Jeanette Wilkinson


You are coming to talk to your GP about a test for the MTHFR gene. A
friend of yours mentioned it and, from what you’ve read on the internet, it
sounds like this could explain some of your problems. You have been feeling
fatigued and run-down, getting progressively worse over the last two to three
years. You are worried, as the website you read states the MTHFR gene can
be associated with many of the symptoms and problems you or your family
have experienced. For example: miscarriages (you’ve had one), heart attacks
(your dad), fatigue, concentration problems and ADHD (your 12-year-old son
has been diagnosed with this and you have struggled with his behaviour for
many years). You are hopeful that there might be an answer to all your
problems.
You have another child, a 15-year-old daughter. You have some worries
about the friends she is starting to hang around with and the way she is
dressing, but she is generally a good kid. You work full-time doing long,
irregular shifts in the residential aged care facility.
You live 20 km out of town and run a small family farm with some cattle
and sheep. Your time away from ‘real work’ is taken up with farm work. The
farm is a major stress for you and is losing money financially. You are
working outside of the farm to be able to cover the bills. Your husband has
finally agreed to try to sell it, and it is on the market, but so far you have not
had any interested buyers. You haven’t been on a holiday longer than one
night away in over ten years.
You feel you are depressed, probably worsening over the last six months,
and you often cry in the mornings when you are alone getting ready for the
day. You don’t seem to enjoy the things you previously enjoyed, and you
have no time for any interests outside your family and your work.
Your concentration is suffering, and you find it difficult to fall asleep. You
tend to use food as a comfort when you’re feeling down, and your weight has
increased 5–10 kg over the last two years. You feel guilty sometimes that
you’re not there for your kids. You feel a sense of hopelessness that the farm
will never be sold. You’ve had brief moments where you feel that you would
be better-off dead but no real suicidal intent. You would never harm yourself
because of your children. You are open to talking to a psychologist or trying
medication if this is suggested.
Your relationship with your husband has deteriorated. You have a Page 75
lot of resentment towards him for getting you into the mess with the
farm. You have no libido and have not been sexually active in many months.
You think your husband is depressed also and you tend to snap at each other
rather than communicate. Your husband has taken to sleeping in a different
room. Although your husband tells you this is because of your snoring, you
think the relationship issues may also be contributing.
If asked, you wake feeling unrefreshed, you’re fatigued during the day and
you have had a couple of microsleeps driving the 20 km home from work
after a 12-hour shift which have scared you.
You don’t drink alcohol or smoke and have never used recreational drugs.
Your diet consists of frozen/easy meals due to time constraints, and few fruit
and vegetables. You do no formal exercise but are active in your roles as a
personal carer and on the farm.
Your periods remain regular and you have not experienced hot flushes.
Support-wise, you have your parents in town who help sometimes with your
kids.
Rest of systems review is normal.

Prompts if needed:
• there are so many things worrying me, it’s really getting me down
• things with my husband haven’t really been so great
• I’m so tired all the time, I even wake up feeling tired.

The following information is on your summary sheet:


Past medical history
G3P2M1: Two spontaneous vaginal deliveries (healthy babies), one
miscarriage
Medication
Nil regular
Allergies
Nil known
Immunisations
None recorded
Family history
Father myocardial infarction age 65
Social history
Personal care worker at a residential aged care facility
Lives on a farm outside of town
Cervical screening
Up-to-date and normal.

Information for the facilitator Page 76

Clinical examination findings


General appearance: slightly agitated, overweight woman of middle-aged
appearance
No pallor, jaundice or skin changes
Height 1.65 m
Weight 82 kg
BMI 30.1 kg/m2
Blood pressure 152/89 mmHg
Pulse 68 beats per minute
Temperature 36.8°C
Cardiovascular and respiratory exams are normal
Thyroid exam is normal
No lymphadenopathy or hepatosplenomegaly
Finger-prick blood sugar level is 5.2 mmol/L
Urinalysis is normal
K10 score 32.

Suggested approach to the case


Establish rapport
Open-ended questions to explore Jeanette’s ideas, concerns and expectations.

Specific questions
Explore why she feels she may have the MTHFR mutation
Depression—including mood, anhedonia, sleep, appetite, fatigue, motivation,
concentration, thoughts of worthlessness/hopelessness/suicidal ideation
and suicide risk assessment
Sleep history—falling asleep, sleep interruptions, snoring, daytime fatigue
Exploration of stressors—home, relationship, work, financial
Exclude menopause or perimenopause as a cause of symptoms
Other systems review, e.g. fever, weight loss, headaches, joint symptoms,
thirst/polyuria, abdominal pains/bowel changes, genitourinary symptoms,
chest pain and shortness of breath
Preventative health—smoking, alcohol, nutrition, physical activity,
immunisations.

Examination
General appearance, height, weight and BMI
Blood pressure/pulse/temperature
Cardiovascular and respiratory examination
Thyroid examination Page 77
Lymph nodes/liver/spleen
Office tests including finger-prick glucose and urinalysis
Objective measure of psychological distress such as K10.

Differential diagnosis/issues that need addressing


Depression
Possible sleep apnoea, needs sleep studies
Overwork/stress
Poor libido and relationship dysfunction
Obesity
Hypertension.

The candidate should recognise the interplay between all these issues and
focus on depression as the biggest issue for the patient. The candidate should
recognise possible sleep apnoea as something that may be causing
microsleeps and hence be potentially serious for the patient.

Management
Addressing the request for MTHFR screening—candidate should be able to
explain why this is not appropriate in a sensitive manner, while leaving the
consult open to exploring her issues and concerns Management of
depression:
• explanation of problem
• options for treatment—exercise, mindfulness, mental health apps/online
psychoeducation, psychology (consider Mental Health Care Plan),
medication, support groups
• patient safety—discuss pros/cons of time off work, provide information
about emergency assistance, such as Lifeline
Plan to investigate for sleep apnoea (recognising that this needs to be
reasonably urgent given driving with microsleeps)
Plan to address her weight, assess her risk of cardiovascular disease, manage
BP, check for diabetes, cholesterol and urine ACR in future consults
Further investigations for fatigue (such as thyroid function or iron studies) are
not necessary at this stage without any red flags. They should not be
requested unless fatigue persists, despite management of the above issues
1
Address the relationship difficulties and poor libido. She and her husband
may benefit from a referral for relationship counselling and/or assistance
with parenting a child with ADHD
Extras for case—offer preventative health care, including immunisations (e.g.
hepatitis B and Q fever, given her work).

CASE COMMENTARY Page 78

Fatigue is a common presentation in general practice and is often


multifactorial. Candidates should have a good approach at taking
a systematic history and working through the differentials of
fatigue. Cases where more than one issue needs identification
and management are likewise common! Time management can
be difficult, both in real life and in an exam situation. Candidates
should be able to identify and prioritise issues and develop a
plan to manage them in subsequent consults.
Often, when patients have symptoms of fatigue or non-
specific symptoms, they will turn to the internet for answers.
Jeanette presents asking for the MTHFR gene test after finding
information on the internet. The MTHFR gene is responsible for
the production of the enzyme methylenetetrahydrofolate
reductase (MTHFR), which is important in the folate pathway.
Polymorphisms of this gene are common, and it is purported by
many that these polymorphisms are responsible for a raft of
common symptoms, including depression and fatigue. There is,
however, no good evidence that the MTHFR gene
polymorphisms have any clinically important effects.2 Often
when patients turn to complementary or alternative approaches,
they have not received help from conventional medicine in the
past. Candidates should be able to explore the underlying
reasons Jeanette is requesting this test and explain in a sensitive
manner why this is not an appropriate test to order.3

COMMON PITFALLS

Failing to take a systematic history in this long case could mean


that candidates focus on one problem only and fail to identify
and manage potentially serious co-morbidities, including sleep
apnoea and hypertension.

References
1. Wilson, J, Morgan, S, Magin, PJ & van Driel, M 2014, ‘Fatigue—a
rational approach to investigation’, Australian Family Physician, vol. 43,
pp. 457–61.
2. Long, S & Goldblatt, J 2016, ‘MTHFR genetic testing: controversy and
clinical implications’, Australian Family Physician, vol. 45, pp. 237–40.
3. Royal Australian College of General Practitioners, ‘Position statement:
Responding to patient requests for tests not considered clinically
appropriate’. Available at: www.racgp.org.au/your-
practice/guidelines/position-statementon-responding-to-patient-requests-
for-tests-not-considered-clinically-appropriate, accessed 31 January 2018.
Page 79

Case 15
Craig Kelly

Instructions for the doctor


This is a short case.
Please take a history from Craig. Examination findings will be available
from the facilitator. You are expected to tell Craig the most likely diagnosis
and negotiate a management plan with him.

Scenario
Craig Kelly is a 22-year-old man. He has not previously been seen at the
clinic.

Craig filled out a new patient questionnaire with the following


information:
Past medical history
Kidney stones
Current medication
Nil regularly
Allergies
Nil
Immunisations
No idea
Social history
Smokes 10 cigarettes per day
Unemployed.

Instructions for the patient, Craig Kelly


Over the past few years you have become dependent on oxycodone 5 mg
tablets, which you take several times per day. You see multiple different
doctors every week trying to obtain scripts.
Today you are seeing this new doctor with the intention of saying Page 80
you had acute renal colic last night and that you need oxycodone 5 mg
to relieve the pain. You say that the pain was right-sided and was severe
enough to have you rolling around in agony. You had kidney stones
previously about three years ago and were prescribed oxycodone for your
pain. (This started your dependence on opioids, but do not volunteer this
information straight away.) You state you had two pills still left in your
cupboard from your original presentation. These helped you get through the
night, but you now need more. When asked, you have some blood in your
urine but no other urinary symptoms. If asked, you are happy for the doctor
to contact a GP who saw you for your initial presentation. You state you have
been well since and have not needed a regular GP. You are insistent that you
need a ‘strong painkiller’ and that paracetamol/ibuprofen will not work.
If the doctor recognises you as a drug seeker, you may start to open up
about your background. The initial prescription of oxycodone helped ease
your physical pain but also blotted out other pains in your life. You had a
difficult childhood and were sometimes beaten and often ignored. You did
not achieve well at school and were an angry teen with few close friends.
You have been on various youth schemes but have never had a regular job.
You moved to the city so that you could see lots of different doctors to get
the oxycodone that you crave.
You have never tried other drugs but have sometimes wondered whether
injecting would give you more of a hit than the tablets. You haven’t
considered giving up because you can’t see any better way of easing the pain
in your life. It is becoming harder and harder to obtain new scripts, and you
are open to exploring help.
You have no other relevant past medical history. You have filled out the
new patient questionnaire as below:
Past medical history
Kidney stones
Current medication
Nil regularly
Allergies
Nil
Immunisations
No idea
Social history
Smokes 10 cigarettes per day
Unemployed.

Information for the facilitator Page 81

Temperature 37°C
Pulse 88
BP 112/70 mmHg
Abdominal examination normal
No signs of opiate use or withdrawal
Urinalysis normal.

Suggested approach to the case


Establish rapport
Open-ended questions to explore Craig’s ideas, concerns and expectations.
Specific questions
About renal colic
— Frequency, duration, site, radiation etc., of pain
— Severity of pain
— What else has been tried?
— Previous investigations
— Urinary symptoms
Previous treating doctor.

Ask for the examination findings


Temperature, pulse, BP
Abdominal and renal angle examination
Urinalysis
Check for signs of opiate use or withdrawal: injection sites, pupil size, red
eyes, runny nose.

Management
Establish/suspect that patient is a drug seeker
Negotiate permission to speak to previous GPs, specialists
Ask whether Craig has ever injected drugs
Phone Prescription Shopping Information Service
Advise of local resources to assist with addiction
Mention contacting the State/Territory poisons branch or drug dependence
unit
Manage any anger or other emotion in the consultation
Advise on needle exchange scheme if injecting mentioned
Demonstrate commitment to provide ongoing care if the patient Page 82
wishes
Consider opportunistic health promotion about smoking and need for
Hepatitis B and other immunisations.
CASE COMMENTARY

Craig has a substance-use disorder and is seeking a script from


the doctor. GPs need to be cautious when prescribing opiates
and should have a high index of suspicion that patients are drug
seeking. Warning signs of patients seeking drugs are:
• reception and appointment: new patient recently moved,
walk-ins, last-minute appointments
• history: very knowledgeable about condition and drugs
requested, requests drug by name, previous doctor
‘unavailable’, multiple allergies or reactions to non-addictive
medications, carries letter from other doctors about opiate or
benzodiazepine prescriptions with dubious rationale
• examination: exaggerated appearance of pain, inconsistent
behaviour and signs.
Patients who are prescribed opiates either for pain or as part of a
rehabilitation scheme (usually methadone) will carry
documentation with them regarding their condition and should
not be offended by probing questions from a GP. A wise GP will
still check that the supporting letter is valid or check the
patient’s MyHealth electronic record, particularly given the
increase in deaths from prescription drugs obtained from
multiple prescribers.
This case tests the doctor’s skill in refusing a patient’s
request. It also tests the professional ethics involved in wishing
to relieve pain yet avoid perpetuating drug dependence or
inadvertently facilitating the diversion of drugs. A firm but
polite no is needed. A practice policy clearly displayed in the
waiting room stating that drugs of dependence are not prescribed
on an initial visit can help. Other options for managing this case
would be offering alternative analgesia, or prescribing enough
morphine capsules to last for only the next 24 hours while
gathering further information from the previous GP.
COMMON PITFALLS

Inexperienced or poor doctors may fail to see through Craig’s


story and will issue a script for the oxycodone as requested;
alternatively, they may say no so aggressively that there Page 83
is no opportunity to work with Craig on his addiction or
other problems, such as the past history of abuse. Manipulative
behaviour can be a symptom of substance-use disorder;
recognising this is an important step in developing the rapport
needed to offer treatment to these patients. Over half of
dependent substance users will eventually achieve stable
recovery, so it is worth offering him assistance.

Further reading
Best, DW & Lubman, DI 2012, ‘The recovery paradigm—a model of hope
and change for alcohol and drug addiction’, Australian Family
Physician, vol. 41, pp. 593–7.
Degenhardt, L, Whiteford, HA, Ferrari, AJ, Baxter, AJ, Charlson, FJ, Hall,
WD et al. 2013, ‘Global burden of disease attributable to illicit drug use
and dependence: findings from the Global Burden of Disease Study
2010’, Lancet, vol. 382, pp. 1564–74.
James, J 2016, ‘Dealing with drug-seeking behaviour’, Australian Prescriber,
vol. 39, no. 3, pp. 96–100.
Kotalik, J 2012, ‘Controlling pain and reducing misuse of opioids: ethical
considerations’, Canadian Family Physician, vol. 58, pp. 381–5, e190–
5.
Monheit, B 2010, ‘Prescription drug misuse’, Australian Family Physician,
vol. 39, pp. 540–6.
The Royal Australian College of General Practitioners 2015, ‘Prescribing
drugs of dependence in general practice’. Available at: www.racgp.org.a
u/your-practice/guidelines/drugs-landing/, accessed 26 January 2018.
Page 84

Case 16
Wazza Wainright

Instructions for the doctor


This is a short case.
Please review the scenario below and take a focused history from Wazza.
No further examination is required. Please discuss the issues raised and
outline your management plan to Wazza. The Austroads Assessing Fitness to
Drive1 standards will be available in the consultation room in either hard or
electronic copy.

Scenario
Wazza Wainright, aged 48, drives road trains for a living. He has booked
in to see you for his commercial driving licence renewal. Wazza attends
your practice but this is the first time that you have seen him. He has
completed part of his driving assessment with your practice nurse, and
relevant sections of this are available below.
The following information is on his summary sheet:
Past medical history
Nil significant
Medication
Nil
Allergies
Nil known
Immunisations
Up-to-date
Social history
Road train driver
Married
Non-smoker
Alcohol intake—two standard drinks weekly
The below assessments have been handed to you by your practice Page 85
nurse.

Epworth Sleepiness Scale


How likely are you to doze Would Slight Moderate High
off in the following never chance chance chance
situations, in contrast to just doze of of dozing of
feeling tired? off (0) dozing (2) dozing
(1) (3)

a) Sitting and reading

b) Watching TV

c) Sitting inactive in a public


place (e.g. a theatre or a
meeting)

d) As a passenger in a car for


an hour without a break

e) Lying down to rest in the


afternoon when
circumstances permit

f) Sitting and talking to


someone

g) Sitting quietly after a lunch


without alcohol

h) In a car, while stopped for a


few minutes in the traffic

Observations
Blood pressure 136/82 mmHg
Pulse 76, regular
Height 180 cm
Weight 149 kg
BMI 46 kg/m2

The rest of Wazza’s fitness to drive health assessment is unremarkable.

Instructions for the patient, Wazza Wainright


You are a 48-year-old road train driver. You enjoy the freedom of the open
road but staying awake on long stretches has become more difficult. You
keep yourself going with energy drinks, such as Red Bull, and strong coffee.
Some of the younger drivers have tried to sell you amphetamines but you’ve
resisted. Your diet is dependent on what you can buy at roadhouses, and you
do little exercise.
You look forward to your days off at home but recently all you Page 86
seem to do is fall asleep during the day when you try to relax. You get
to sleep OK but your partner seems to spend half the night telling you to stop
snoring or shouting at you to start breathing.
You have booked this consultation to renew your commercial driving
licence. You are expecting the doctor to sign-off on the licence without too
much fuss, as previous GPs have in the past. If the doctor refuses to sign-off
on it today, you become worried and frustrated, as losing your commercial
driver’s licence means you risk your job.

Suggested approach to the case


Diagnostic phase
Establish rapport
Open-ended questions to explore Wazza’s ideas, concerns and expectations
Calculate Epworth Sleepiness Scale score
Summarise findings of high score on Epworth Sleepiness Scale and very high
body mass index and suspected obstructive sleep apnoea
Check Austroads Assessing Fitness to Drive, commercial standards.

Management phase
Explain that if commercial vehicle standards are not met, licence cannot be
renewed
Observe for response and acknowledge any anger or frustration
Explain need for referral to specialist physician for sleep studies
Explain likely relationship between sleep apnoea and obesity
Encourage initiation of increased exercise and decreased calorie intake
Assure him of ongoing support, arrange follow-up and offer health check
Document advice in records.

CASE COMMENTARY

Assessing fitness to drive can be a challenging aspect of general


practice as the GP’s public health role may need to take
precedence over the role of caring for an individual patient.
Doctors are not expected to remember the Fitness to Drive1
standards but should be familiar with the document and find the
information that shows Wazza does not meet the commercial
licence standards. For Wazza this will mean loss of his
livelihood until he can be treated, for example, with continuous
positive airways pressure. There is no one preferred Page 87
treatment for obstructive sleep apnoea but rather a range
of options of proven effectiveness that can be applied
individually or in combination, depending on patient preference,
symptoms, severity, co-morbidities and other health risk factors.
2
Severe obstructive sleep apnoea is strongly associated with an
increased risk of stroke and cardiovascular disease. Despite this
association, there is no clear evidence that treatment of sleep
apnoea with continuous positive airways pressure reduces
metabolic risk.3 Wazza should be advised regarding weight
reduction, as this will ameliorate obstructive sleep apnoea and is
also important in managing his high metabolic risk. Wazza
should be offered a preventative health check, although
maintaining a therapeutic relationship when you have just
removed someone’s driving licence tests communication skills!

References
1. Austroads and National Transport Commission 2016, Assessing Fitness to
Drive, 5th ed, Sydney.
2. Mansfield, DR, Antic, NA & McEvoy, RD 2013, ‘How to assess,
diagnose, refer and treat adult obstructive sleep apnoea: a commentary on
the choices’, Medical Journal of Australia, vol. 199, pp. S21–6.
3. Hamilton, GS & Joosten, SA 2017, ‘Obstructive sleep apnoea and
obesity’, Australian Family Physician, vol. 46, pp. 460–3.

Further reading
Kee, K & Naughton, MT 2009, ‘Sleep apnoea: a general practice approach’,
Australian Family Physician, vol. 38, no. 5, pp. 284–8.
Page 88

Case 17
Hope Briganza

Instructions for the doctor


This is a short case.
Please take a focused history. Request the findings of an appropriate
physical examination from the facilitator and explain your management plan
to Hope. No further investigations are available.

Scenario
Hope Briganza is a 26-year-old child care worker who is a regular
patient of the practice. She is generally healthy. She presented to your
colleague 10 days ago and he diagnosed a lower back strain.

Your colleague’s clinical notes from her previous visit reads:


• three days right LBP without red flags
• works in child care—lifts children all day
• past mild episodes only
• otherwise well; only on OCP
• physical examination unremarkable
• assessment: acute mechanical LBP/muscle strain
• plan: rest (medical certificate 1/7); simple analgesia prn; local heat prn
• r/v no better.

The following information is on her summary sheet:


Age
26
Past medical history
Oral contraception
Tonsillectomy (age 12)
Medication Page 89
Ethinyloestradiol 30 mcg/levonorgestrel 150 mcg (Levlen ED) 1
mane
Allergies
Nil known
Immunisations
Up-to-date
Cervical screening
Up-to-date
Social history
Non-smoker
Alcohol—one or two drinks a week
Family history
Hypertension
Ischaemic heart disease
Osteoarthritis.

Instructions for the patient, Hope Briganza


You have had about two weeks of right-sided lower back pain. You came to
see another doctor at this practice 10 days ago. He diagnosed a back strain
and suggested rest, heat packs and paracetamol. He thought it could be
related to lifting children (you work in child care) so he gave you a day off
work. The pain is in your right flank (loin) and is dull and deep. It is constant
and is not affected by anything you do. It occurs at rest as well as with
movement. You don’t recall any particular incident or injury but you do bend
and lift children all day. You’ve taken some paracetamol but it doesn’t help
much. You have had episodes of mild lower back pain previously, but they
were brief and usually only when you had done too much in the garden or at
sport. This pain feels different as it is higher up, and constant—you’re
worried it might be something serious.
If asked, you do have mild urinary frequency (day and night) and urinary
urgency but no incontinence. If asked about fever or temperatures you do
sometimes feel hot and cold but you haven’t checked your temperature. You
have been feeling tired and washed out over the last week or so. There has
been some nausea and anorexia but no vomiting or change in your bowels.
You live with your boyfriend who has been your only sexual partner for
the past four years. You are on the contraceptive pill (Levlen) and remember
to take this every day. Your last period was about a week ago and was
normal. There are no symptoms of pregnancy and your cervical screening is
up-to-date.
You work full-time in child care and enjoy your work. You have an Page 90
active social life playing sport (netball and touch football) with your
friends regularly. You are generally healthy and well apart from the
symptoms above.
When it comes to diagnosis and management, simply ask the doctor the
sort of questions you think Hope would.

The following information is on your summary sheet:


Age
26
Past medical history
Oral contraception
Tonsillectomy (age 12)
Medication
Ethinyloestradiol 30 mcg/levonorgestrel 150 mcg (Levlen ED) 1 mane
Allergies
Nil known
Immunisations
Up-to-date
Cervical screening
Up-to-date
Social history
Non-smoker
Alcohol—one or two drinks a week
Family history
Hypertension
Ischaemic heart disease
Osteoarthritis.

Suggested approach to the case


Based on the scenario, the candidate should already be thinking about
alternative diagnoses to mechanical low back pain in a young woman. The
focused history is aimed at separating these, and the physical examination
(along with surgery tests) should confirm that the doctor is dealing with a
case of acute pyelonephritis. A decision will need to be made about inpatient
versus outpatient treatment based on the severity of the condition and
possible complications. A simple explanation of the condition and its
management to Hope is required along with appropriate answers to her
questions.

Points to address in the history include:


• details of the pain
• presence of any urinary symptoms
• presence of fever Page 91
• abdominal pain, nausea, vomiting
• possibility of pregnancy
• brief general health screen.
Physical examination should include:
• general appearance
• vital signs (esp. temperature, heart rate, BP)
• costovertebral angle tenderness
• possible abdominal or suprapubic tenderness
• urinalysis, pregnancy test.
Management should include:
• explanation of the significance of her symptoms and the likely diagnosis
of acute pyelonephritis
• instruction regarding collection of a midstream urine for culture and
antimicrobial susceptibility testing to guide possible adjustment of the
initial regimen if there is no clinical improvement
• commencement of an appropriate oral antibiotic (such as
amoxycillin/clavulanate, cephalosporin or
trimethoprim/sulfamethoxazole) for 10 days
• imaging or blood cultures are generally not required in uncomplicated
cases
• follow-up and safety netting should be outlined. It would be reasonable to
review Hope in one to two days to ensure she is improving. A repeat
urine culture is not usually necessary where there has been clinical
resolution of the infection.

Physical examination
General appearance: young woman in no distress at rest
Vitals:
HR 94/min regular
BP 104/68 mmHg sitting
Temp 38.1°C
BMI 24 kg/m2
RR 12/min
Remainder of cardiovascular examination unremarkable
Abdominal exam: abdomen soft; no masses; mild suprapubic tenderness;
bowel sounds active
Musculoskeletal: lumbar spine normal
Costovertebral angle tenderness on the right
Remainder of physical examination is normal.
Surgery tests Page 92
U/A: leucocytes ++; blood +
Urine Beta HCG: negative
BSL (random): 5.6 mmol/L
ECG: Sinus rhythm normal.

CASE COMMENTARY

Acute unilateral back pain in a young woman should raise the


possibility of pyelonephritis—a common bacterial infection of
the renal pelvis and kidney that results from ascent of a bacterial
pathogen up the ureter from the bladder. Because of the
frequency and severity of the infection, GPs must be able to
make the diagnosis, decide between inpatient and outpatient
care, and select an appropriate antibiotic. In addition, because
pyelonephritis is associated with adverse maternal and foetal
outcomes in pregnancy, it is important to rule out pregnancy in
women of child-bearing age.1, 2
Flank pain and lower urinary tract symptoms along with
fever, costovertebral tenderness and positive urinalysis all
confirm the diagnosis. Urine culture is recommended in all cases
prior to empirical antibiotic prescribing. In the majority of cases
in young women, Escherichia coli is the responsible pathogen
and the spectrum of pathogens involved in acute pyelonephritis
is similar to that of cystitis.1, 2
Australian antibiotic guidelines for mild, uncomplicated
pyelonephritis differ from US and European guidelines in
restricting fluoroquinolones to cases where the causative
organism is resistant to first-line drugs.3, 4
Given the severity described in Hope’s case and the absence
of co-morbidities, hospitalisation is unnecessary.1 Similarly,
imaging and blood cultures are not required in uncomplicated
cases and repeat urine cultures are not usually necessary where
there has been clinical resolution of symptoms.

COMMON PITFALLS

If candidates don’t reconsider the initial diagnosis, significant


time can be lost exploring the various red flags for low back
pain with a series of closed questions. Beginning with open
questions should elicit enough information to raise the
possibility of pyelonephritis as an alternative diagnosis.
We shouldn’t be defensive about our colleague’s Page 93
management in the previous consultation, nor should we
be drawn into being critical of their care. Clinical presentations
evolve, symptoms develop and conditions declare themselves,
making the diagnosis more obvious over time. The patient is not
upset about her management and the initial incorrect diagnosis is
not an issue for her. Although the oral contraceptive is listed on
her patient summary, the possibility of pregnancy should be
specifically ruled out.
Hospitalising Hope, although unnecessary, would not result
in a failure for the case providing the diagnosis is made correctly
and appropriate treatment discussed. Similarly, a discussion
about various strategies for preventing recurrent urinary tract
infections is probably premature following her first episode.

References
1. Colgan, R, Williams, M & Johnson, J 2011, ‘Diagnosis and treatment of
acute pyelonephritis in women’, American Family Physician, 1
September, vol. 84, no. 5, pp. 519–26.
2. Wagenlehner, FM, Lichtenstern, C & Rolfes, C 2013, ‘Diagnosis and
management for urosepsis’, International Journal of Urology, vol. 20, no.
10, pp. 963–70.
3. Therapeutic Guidelines: Antibiotics 2019, ‘Acute pyelonephritis in
adults’. In: eTG Complete [Internet]. Available at:
https://tgldcdp.tg.org.au/viewTopic?topicfile=acute-pyelonephritis-adults,
accessed 23 May 2019.
4. Emergency Care Institute NSW 2017, ‘Management of pyelonephritis’,
Available at:
https://www.aci.health.nsw.gov.au/networks/eci/clinical/clinical-
resources/clinical-tools/renal/pyelonephritis, accessed 23 May 2019.
Page 94
Page 95
Section 6
Child health
Page 96

Case 18
Kylie Chong

Instructions for the doctor


This is a short case.
Kylie does not attend this appointment. Please take a history from her
mother, Ilana, and make suggestions for management. You are required to
focus on the presenting complaint only.

Scenario
Kylie Chong is an 8-year-old girl who has seen you a few times for
minor illnesses in childhood. Kylie’s mum, Ilana, has booked to see you
today because ‘Kylie is out of control and won’t do what she is told’.
Ilana wants you to prescribe the drug that is used in attention deficit
hyperactivity disorder (ADHD).
Instructions for Kylie’s mother, Ilana Chong
You have three children and work part-time as a cleaner. You were brought
up in Australia and so was your husband. His family migrated to Australia
from China. Your husband and his family consider child rearing to be the
mother’s main responsibility. You are keen to have support and will be open
to any suggestions from the doctor if you feel respected not blamed.
You had a normal pregnancy and birth. Kylie reached all her milestones.
Kylie is the middle child of the family. From the beginning she has been
the most challenging but recently you feel things are out of your control. She
is doing OK at school but her behaviour is very disruptive at home. It is so
frustrating that the teachers report her to be good, while at home Kylie is
disobedient and refuses to join in with the family. Getting her up to go to
school in the mornings is a nightmare and at weekends all she does is watch
TV and eat junk food. You have tried a strict diet free from preservatives but
that made no difference. A recent hearing check was normal.
You have come to the doctor today because you have heard there is Page 97
a drug for attention deficit hyperactivity disorder (ADHD) and you
think it might be the answer for Kylie.
If the doctor asks about the three main symptoms of ADHD of inattention,
hyperactivity and impulsiveness, you are to answer that Kylie does NOT
have these symptoms.

Suggested approach to the case


Establish rapport
Open-ended questions to explore Kylie’s mum’s ideas, concerns and
expectations.

Specific questions
Kylie’s behaviour—define the issues
Pregnancy, birth and developmental history
Interests
Friendships
Relationship with siblings and family
Sleep
School performance, reports from teachers
General health, appetite, mood
Test against criteria expected in ADHD
— Inattention
— Hyperactivity
— Impulsiveness.

Most likely diagnosis


Outline that ADHD doesn’t seem likely
Sensitively suggest that it is a behavioural issue
Important to address the problem now
Praise Ilana for coming along now—lots of options for change.

Management
Outline available options
— Positive parenting groups
— Agreed family rules—reward appropriate behaviour, ignore
undesirable behaviour
— Negotiate with whole family about the issue—involve Kylie’s Page 98
father
— Family therapy
— Offer online resources, e.g. https://raisingchildren.net.au
Assure of continued support.

CASE COMMENTARY

Kylie is causing problems but the doctor needs to work out


whether she has ADHD. This is characterised by inattention,
hyperactivity and/or impulsiveness. This does not fit with
Kylie’s story of being able to concentrate on the TV for much of
the day and her reasonable performance and behaviour at school.
The doctor will need to demonstrate empathy with Ilana
about Kylie’s problem and sensitively explain that the treatment
for ADHD will not help, as it seems to be more of a behavioural
issue. The doctor can then outline the options available to assist
Ilana and her family. This could entail support and guidance
from the GP, parenting courses or referral to more specialised
children’s services, depending on local availability and Ilana’s
choice.

Further reading
Feldman, HM & Reiff, MI 2014, ‘Clinical practice. Attention deficit-
hyperactivity disorder in children and adolescents’, New England
Journal of Medicine, vol. 370, pp. 838–46.
Furlong, M, McGilloway, S, Bywater, T, Hutchings, J, Smith, SM &
Donnelly, M 2012, ‘Behavioural and cognitive-behavioural group-based
parenting programmes for early-onset conduct problems in children
aged 3 to 12 years’, Cochrane Database of Systematic Reviews, vol.
2:CD008225.
Jarman, R 2015, ‘Finetuning behaviour management in young children’,
Australian Family Physician, vol. 44, no. 12, pp. 896–9
Halasz, G 2009, ‘Attention deficit hyperactivity disorder: time to rethink’,
Medical Journal of Australia, vol. 190, pp. 32–3.
Page 99

Case 19
Brandon Harkness

Instructions for the doctor


This is a short case.
You are expected to take a history from Brandon’s mother, Julie. Results
of the examination will be available from the facilitator. Outline your
differential diagnosis to Julie and negotiate a management plan.

Scenario
Brandon is a lively two-year-old Caucasian boy. He has been a patient at
the surgery since he was born and has attended only for coughs and
colds, and immunisations. His growth has been along the 75th centile.
Four weeks ago, Julie brought Brandon in concerned that he had
‘gastro’. He had vomited three times in the night and had profuse watery
diarrhoea. He was not dehydrated. You advised fluids and expected that
the illness would soon resolve. Today Julie has come to see you because
Brandon’s diarrhoea is continuing.

The following information is on Brandon’s summary sheet:


Medication
Nil recorded
Allergies
Nil recorded
Immunisations
Fully immunised as per schedule.

Instructions for Brandon’s mother, Julie


Harkness
You work as a pharmacy assistant three days a week and spend the other days
at home with your children. Your daughter, aged four, attends preschool and
Brandon, your two-year-old, attends child care.
Four weeks ago, Brandon had ‘gastro’. He was vomiting and had Page 100
diarrhoea. The vomiting soon settled and, as advised, you continued
with fluids and reintroduced solids when Brandon seemed hungry. He no
longer has any fever.
Brandon is thriving with a good appetite and seems well but the diarrhoea
has not stopped. He passes a light-coloured, loose, watery, foul-smelling
stool about five times a day and you can see undigested food in it. Previously
Brandon passed one to three stools per day. The situation is beginning to get
you down because the staff at child care are worried about him and keep
suggesting that he is not well enough to be there. When you talked with the
pharmacist at work, she suggested you go back to the GP for a check-up.
The following information is on Brandon’s medical record:
Medication
Nil recorded
Allergies
Nil recorded
Immunisations
Fully immunised as per schedule.
Information for the facilitator
Clinical examination findings
Clinical examination is entirely normal. Brandon’s weight and height are on
the 75th centile and tracking well.

Suggested approach to the case


Establish rapport
Open-ended questions to explore Julie’s ideas, concerns and expectations.

Specific questions
Details about the episode of gastro
Details about the diarrhoea—frequency, smell, blood, colour, consistency,
mucus
Systemic upset
Abdominal pain
Appetite and thirst
Current diet—any observed relationship between diet and diarrhoea
Consumption of fruit, fruit juice, milk
Medications used
Known allergies Page 101
Anyone else in family with diarrhoea
What else has Julie tried?
Any recent travel?
Request permission to examine.

Examination
Ask the facilitator for specific examination findings
Level of alertness, responsiveness and hydration
Weight and height plotted on centile chart (75th centile)
Pulse
Temperature
Ears, nose, throat
Absence of jaundice or rash
Abdominal examination
— No tenderness elicited
— No masses found
— No organomegaly
— No perianal rash or ulceration.

Differential diagnoses
Toddler’s diarrhoea (also known as chronic non-specific diarrhoea)
Temporary lactose or other disaccharide intolerance
Worth excluding infectious cause, e.g. Giardia.

Management
Reassure
Regular weighing and follow-up
Check for pathogens—faecal MCS and PCR for enteric pathogens
Advise balanced diet, avoid snacks, reduce consumption of fruit juice
If not settling, consider further testing to include ESR, FBE, coeliac
antibodies, stool pH (5.5 or less and sugars suggest carbohydrate
intolerance).

CASE COMMENTARY

The most important clinical feature is that Brandon is thriving.


The fact that he is still on the 75th centile excludes any
potentially serious cause of the diarrhoea and the doctor should
emphasise this with Julie.
Brandon’s symptoms are characteristic of toddler or Page 102
chronic nonspecific diarrhoea, the commonest cause of
diarrhoea in children aged one to five years. Some speculate this
is an iatrogenic problem that starts when parents are advised to
push fluids, including fruit juices, and reduce fibre and fat
intake, in children with gastroenteritis.1 Children then enter a
vicious cycle of diarrhoea and further dietary restrictions aimed
at curing the diarrhoea. Treatment is rebalancing the ‘four Fs’—
Fat, Fibre, Fluid and Fruit juices—by reintroducing well-defined
meals and snacks. Fruit juices, in particular clear apple juice,
and other squashes should be limited. Toddler diarrhoea might
be prevented by advising that children with acute diarrhoea
require initial rehydration and then resumption of a normal
feeding pattern as soon as it is tolerated.
Other common possible causes of Brandon’s diarrhoea are
residual infection (such as with Giardia), carbohydrate
intolerance or temporary lactose intolerance.2 Carbohydrate
intolerance results in unabsorbed disaccharides in the stool
which show positive on Clinitest or glucose testing strips.3
Fermentation of unabsorbed sugars produces acids which yield a
stool pH of <5.5. In temporary lactose intolerance, reduction of
milk-based products is recommended for a short time. If the
symptoms do not resolve in a week then this is not the cause of
the problem and milk can be reintroduced. More complex
changes to his diet would need supervision from a dietician to
ensure any benefit in reducing the diarrhoea is not achieved at
the expense of malnutrition.
The concern of the day care centre about Brandon’s diarrhoea
should be discussed. Once a stool test is confirmed as clear,
Brandon can attend again and will not pose a risk to the health of
the other children or the carers. The GP can offer to put this in
writing to the day care centre.

References
1. Kneepkens, C & Hoekstra, J 1996, ‘Chronic nonspecific diarrhea of
childhood: pathophysiology and management’, Pediatric Clinics of North
America, vol. 43, pp. 375–90.
2. Zella, GC & Israel, EJ 2012, ‘Chronic diarrhea in children’, Pediatrics in
Review, vol. 33, no. 5, pp. 207–17.
3. Klish, WJ 2006, ‘Chronic non specific diarrhoea of childhood’. In: JA
McMillan, RD Feigin & CD De Angelis et al. (eds), Oski’s Pediatrics
Principles and Practice, Lippincott Williams & Wilkins, pp. 1924–6.
Page 103

Case 20
Natalie Jones

Instructions for the doctor


This is a short case.
Please conduct this consultation as you would in your clinical practice.

Scenario
Jacinta Jones comes in to see you to discuss Natalie, her six-year-old
daughter. She had taken Natalie to a paediatrician because her asthma
was not well controlled on her medication of prophylactic cromoglycate
and salbutamol.
You have read the letter from the paediatrician suggesting that Natalie
start on inhaled steroids. You are not quite sure why she has booked this
appointment to see you today, particularly as she did not come to see you
about Natalie before going to the paediatrician.
Instructions for Natalie’s mother, Jacinta
Jones
You are an articulate business executive in your late thirties. You live in a
large house in a prestigious area of Sydney and travel a substantial amount
for work. Natalie is your only child and you had her ‘later on’ so that you
could establish your career first. Your husband is also in business and leaves
the child care arrangements to you.
Natalie is six years old and has asthma. You are often up at night when she
coughs and this is exhausting for both of you. Natalie uses a salbutamol
inhaler and cromoglycate as a preventer. You recently took Natalie to see a
private paediatrician about the asthma. You did not ask your GP for a referral
and just paid the full price. You have precious little time for appointments
and a friend’s child sees a paediatrician for their asthma so you thought it
would be best.
You are concerned that the paediatrician has decided to start Page 104
Natalie on steroids. You know they are banned in athletes and do not
understand why they are being given to Natalie. You have made an
appointment to see the GP and have the following questions on your mind:

1. Why has Natalie been recommended steroids? How do they work?


2. Is there any alternative medication that could be tried?
3. How will the GP know if the medication is helping Natalie?
4. Are there any side effects to worry about?
5. The paediatrician gave you a spacer for helping Natalie use the
medication. You didn’t take in the instructions after a sleepless night and
would like to be shown again.

Suggested approach to the case


Establish rapport
Open-ended questions to establish Jacinta’s ideas, concerns and expectations
about this consultation and Natalie’s asthma
Answer Jacinta in terms that are understood, in a non-judgemental manner.
1. Explain that asthma is due to inflammation. Steroids are anti-
inflammatory and will reduce the amount of wheezing and coughing and
make it easier for Natalie to breathe.
Explore why Jacinta is concerned about steroids: what has she heard
about them and from whom? Tailor further answers about steroids to
Jacinta’s specific concerns.
Jacinta has heard that steroids are banned in athletes. The doctor can
reassure her that there are different types of steroids and that the body-
building steroids abused by some athletes are different from the anti-
inflammatory steroids used in asthma.
2. Explore with Jacinta what medication has already been tried. Outline the
rationale for obtaining good asthma control and for helping Natalie to feel
better. Natalie has previously tried regular cromoglycate as a preventer
and uses salbutamol for symptom relief. She has not tried other
medication. The next step according to current guidelines is to introduce
inhaled corticosteroids.
3. Explain the role of monitoring Natalie’s asthma symptoms. Asthma
control is monitored by:
• assessment of symptoms—cough, wheeze, nocturnal cough, shortness
of breath, exercise tolerance
• absence of signs—wheeze, peak expiratory flow rate, spirometry
• function—can Natalie do all the sport and activities that she would
like to or does her asthma stop her?
4. Side effects such as oral thrush and dysphonia are possible with Page 105
inhaled corticosteroids. These can be reduced by using a spacer
and rinsing the mouth after inhalation. Longer-term side effects include
the development of cataracts, reduced bone density, depression of the
hypothalamic—pituitary—adrenal axis and delay in growth.
The list of potential problems from inhaled corticosteroids is long. The
doctor needs to give a balanced description of the potential risks and
benefits of this medication. The doctor can inform Jacinta that the lowest
effective dose of inhaled steroids will be used and that Natalie’s growth
and general development will be monitored.
5. Demonstrate use of spacer device with advice regarding cleaning.
6. Suggest a written asthma management plan.

Any remaining time could be used to review preventive measures, such as


encouraging exercise, and avoiding environmental triggers to Natalie’s
asthma, such as passive smoking.

CASE COMMENTARY

This case tests the doctor’s communication skills and


understanding of the management of childhood asthma. Another
twist is the potential for interprofessional issues to interfere with
Natalie’s care.
In the Australian health system, patients are referred to
specialists by their GPs. However, many people are willing to
pay for direct access to specialists. Natalie’s asthma is not severe
and would normally be dealt with by a GP. The GP may feel
undermined that Jacinta has taken Natalie to a specialist.
Wounded professional pride is compounded by having to deal
with Jacinta’s response to the suggestions of inhaled
corticosteroids. The GP should put aside their own reactions and
approach Jacinta professionally and in Natalie’s best interests.
The doctor needs to explore why Jacinta is concerned about
giving Natalie steroids. While the doctor could guess at the
causes for her concern from their general knowledge of patients’
reactions, the reassurance will be more effective if it addresses
Jacinta’s particular fears.
Some doctors may feel threatened by Jacinta’s questioning of
medical advice. A good doctor will be able to discuss the issue
with Jacinta as if in a partnership, whereas a poor doctor may
show irritation and resort to medical dominance or cave in and,
in an effort to appease her mum, deny Natalie the medication she
needs.
Spacers come with simple written instructions. If you Page 106
are not confident in using a spacer or in instructing
others how to use one, ask a local pharmacist if you can look at
one and read the instructions for use.

Further reading
National Asthma Council 2016, Australian Asthma Handbook, version 1.2,
National Asthma Council, Melbourne, Vic. Available at: www.asthmaha
ndbook.org.au, accessed 19 February 2019.
Robinson, PD & Van Asperen, P 2009, ‘Asthma in childhood’, Pediatric
Clinics of North America, vol. 56, no. 1, pp. 191–226.
Van Asperen, PP, Mellis, CM, Sly, PD & Robertson, CF 2011, ‘Evidence-
based asthma management in children—what’s new?’, Medical Journal
of Australia, vol. 194, no. 8, pp. 383–4.
Zhang, L, Prietsch, S & Ducharme, F 2014, ‘Inhaled corticosteroids in
children with persistent asthma: effects on growth’, Cochrane Database
of Systematic Reviews, no. 7.
Page 107

Case 21
Latu O’Donnell

Instructions for the doctor


This is a short case.
Please take a history from Latu’s mother Mele. The findings of a relevant
physical examination will be available from the facilitator. Negotiate a
management plan with Mele. You are not required to take a history from
Latu.

Scenario
Mele and her 11-year-old son Latu have been coming to your surgery
since they moved to the area three years ago. Mele’s ex-husband, Robert,
moved interstate after their marriage ended. Mele is from Tonga and her
parents live nearby. She works as a receptionist for the local council and
has another part-time job as a cleaner. Latu has previously been healthy,
presenting only for minor illnesses. Mele presents today to discuss her
concerns about Latu’s increasing weight and sedentary lifestyle.

The following information is on Latu’s summary sheet:


Medication
Nil recorded
Allergies
Nil recorded
Immunisations
Up-to-date
Social history
Lives with his mother
Attends school.

Instructions for the patient’s mother, Mele Page 108


Kafoa
You and Latu have come to the doctor today to get some help with his
weight. Latu is 11 and generally healthy, but since you moved to this area his
weight has been steadily increasing. He is less physically active and you have
talked to him several times about getting more exercise and eating better, but
each time he reacts angrily and tells you to stop nagging him. What
motivated you to bring him in was an episode last week when he was upset
and told you about being bullied at school about his weight. Latu is naturally
shy but increasingly he has been withdrawing into his computer gaming
world, which seems to be the main place that he is happy and engaged.
Your break-up with Robert was very unpleasant and you were glad to
make a new start with Latu when you moved back to the area where your
parents live. Unfortunately, Latu hasn’t really made good friends here,
preferring to play online games with some of his previous friends. He goes to
stay with Robert roughly one weekend a month and for some school holidays.
From your point of view, Robert seeks to ‘buy’ Latu’s love by giving him
lots of presents (computer games, tablet, new phone) and basically letting
him do what he wants when Latu is with him. You feel that this undermines
your efforts to help him be healthier but haven’t spoken to Robert about it.
You would like to see Latu get outside more, play sport and be socially active
but you’re having trouble reaching him. You are conscious that this role
would normally be performed by Latu’s brothers or cousins, but in their
absence you are doing the best you can.
You work hard both as a receptionist and a cleaner and are often not home
until dinner time. You lack the energy to cook and often grab some takeaway
on the way home. Both of you eat your dinner in front of the TV or the
computer and most evenings you relax on the couch. You shop each Saturday
and usually buy Latu some of his favourite foods (biscuits, chips, chocolates)
for snacks and rewards. Working hard to pay the bills and keep the cupboard
stocked takes all your energy—everything else feels overwhelming and tends
to go into the ‘too hard’ basket. Your own weight has increased by several
kilograms since moving. You put this down to not being physically active
outside of work and to a habit of comfort eating (ice cream, chocolate) when
you get down. You are close with your parents, who are a good support for
you. Every Sunday after church they have you and Latu for lunch, which lasts
for much of the day with lots of food. You have spoken to your mother about
Latu’s weight but she said, ‘it is better to be bigger than skinny’. You are
interested in the doctor’s thoughts about whether you should be concerned
about Latu and, if so, what practical steps you can take to help him.

The following information is on Latu’s summary sheet: Page 109


Medication
Nil recorded
Allergies
Nil recorded
Immunisations
Up-to-date
Social history
Lives with his mother
Attends school.

Information for the facilitator


Clinical examination findings
General appearance: overweight boy sitting quietly in no distress
Height 141.5 cm (50th centile)
Weight 51 kg (96th centile)
BMI 26 kg/m2 = 97th centile
Remainder of physical examination normal.

Suggested approach to the case


Open questions to explore Mele’s concerns, without Latu present.

Specific questions for Mele


Latu’s general health and co-morbidities
— Sleep/snoring
— Shortness of breath
— Fatigue
— Skin and musculoskeletal problems
Latu’s psychosocial issues
— Mood
— Self-esteem
— Body image
— Disordered eating
— Behavioural problems
— Teasing and bullying
— School avoidance
— Social isolation
Family dynamics Page 110
— Cultural beliefs about weight
— Parenting difficulties
— Parental coping
— Social support
— Financial difficulties
— Parental motivation
The family’s eating habits and food choices
Latu’s physical activity and interests.

Examination
General appearance
BMI percentiles
Heart rate
Blood pressure
BSL
Urinalysis.

Management
Use a BMI chart to show Mele that Latu is bordering on obese.
Briefly outline the health consequences of obesity.
Affirm Mele’s decision to present and existing positive aspects of her care,
and build confidence in Latu’s successful weight management.
Interventions with strong evidence in Latu’s age group include limiting
screen time, increasing water consumption, physical activity interventions
with a home component and diet interventions with a community and
home component.
Lifestyle interventions should ideally include the whole family. Mele has
admitted to gaining weight herself so a program for her and Latu would
be beneficial.
Ongoing follow-up with a health care team and monitoring weight and
growth velocity will be important.

CASE COMMENTARY

The prevalence of overweight and obesity in Australia is high


and continues to increase. Obesity is particularly prevalent
among those in the most disadvantaged socioeconomic groups,
Aboriginal and Torres Strait Islander peoples and many Page 111
people born overseas. The prevalence is also higher in
rural and remote areas compared to urban areas. Pacific
Islanders rank among those with the highest rates of obesity.
Culturally, large physical size is considered a mark of beauty
and social status in many Pacific Island countries. There can be
resistance to the view that obesity is a health problem.
Childhood obesity is a sensitive issue—made even more
complex by family and cultural factors, as in this case. The
family doctor is well-placed to gently explore, educate and
manage childhood obesity.
Due to the changes in body composition as children grow,
single-point cut-offs are usually not suitable for establishing a
child’s weight status. BMI percentile curves specific for gender
and age should be used. A BMI between the 85th and 95th
percentile is considered to be overweight, and a BMI above the
95th percentile is considered obese. Weight maintenance as the
child grows rather than weight loss is recommended for most
children.
Generally, it is better to discuss a child’s weight with parents
when the child is not present, but from adolescence it is
important to work primarily with the patient. Before this time,
the GP has a key role to play in enhancing parental motivation,
which is essential for effective weight management in children.
Intervention is unlikely to be effective if parents are not
motivated to change. Parents are more likely to be motivated to
make changes to manage their child’s weight when they have
sufficient knowledge about children’s weight and health, believe
that their child’s excess weight is important to control, believe
that the required changes are worthwhile, and have confidence in
their ability to make the necessary changes. Parents like Mele
who have high levels of readiness for change can be offered
specific and practical interventions.
Supporting Mele in an empathic and non-judgemental way
will enhance her engagement, motivation and readiness for
change. If we emphasise her family’s strengths and avoid parent
blame and criticism we can create an alliance that will combat
the isolation generally felt by those caught between cultures and
families. Respectful exploration of traditional beliefs and
attitudes concerning weight will be needed and Mele will have
to resolve how she is going to respond to these with her family.

COMMON PITFALLS

It is easy to fall into a dynamic where the doctor is suggesting


strategies for change and the patient (in this case, the mother) is
responding to each by explaining why they won’t work. It has
been shown that such a dynamic actually works against Page 112
behavioural change and can be avoided by the
application of motivational interviewing skills.

Further reading
Curtis, M 2004, ‘The obesity epidemic in the Pacific Islands’, Journal of
Development and Social Transformation, vol. 1, November.
Kirk, SFL, Cockbain, AJ & Beazley, J 2008, ‘Obesity in Tonga: a cross-
sectional comparative study of perceptions of body size and beliefs
about obesity in lay people and nurses’, Obesity Research & Clinical
Practice, vol. 2, no. 1, pp. 35–41.
Mavoa, HM & McCabe, M 2008, ‘Sociocultural factors relating to Tongans,
and Indigenous Fijians’ patterns of eating, physical activity and body
size’, Asia Pacific Journal of Clinical Nutrition, vol. 17, no. 3, pp. 375–
84.
Miller, WR & Rollnick, S 2002, Motivational Interviewing: Preparing
People for Change, The Guilford Press, New York.
Mihrshahi, S, Gow, ML & Baur, LA 2018, ‘Contemporary approaches to the
prevention and management of paediatric obesity: an Australian focus’,
Medical Journal of Australia, vol. 209, no. 6, pp. 267–74.
National Health and Medical Research Council 2013, ‘Clinical practice
guidelines for the management of overweight and obesity in adults,
Adolescents and Children in Australia’, NHMRC, Melbourne, Vic.
Available at: www.nhmrc.gov.au/_files_nhmrc/publications/attachments
/n57_obesity_guidelines_130531.pdf, accessed 15 October 2014.
Page 113
Section 7
Dermatology
Page 114

Case 22
Sammy Burnside

Instructions for the doctor


This is a short case.
Please take a history from Jessica, Sammy’s mum. A clinical photograph
of Sammy’s skin problem will be available for you on request. (Refer to Figu
re 2, centre insert page A.) Please discuss the diagnosis with Jessica and
advise on the management of the condition.

Scenario
Sammy is a four-month-old boy who has an itchy rash.

The following information is on his summary sheet:


Past medical history
Normal vaginal delivery following uncomplicated pregnancy
Medication
Nil prescribed
Allergies
Nil known
Immunisations
Up-to-date
Family history
Mother asthmatic and allergic rhinitis
Father eczema as a baby.

Instructions for Sammy’s mother, Jessica


Burnside
You had a busy life as a teacher at the local preschool. You are on maternity
leave following Sammy’s birth but are soon due to return to work. Sammy
has had an itchy rash on his face, arms, legs and trunk for a few weeks.
You’ve been using creams your Mum bought for you, plus tea-tree Page 115
oil. You hate seeing your beautiful baby so distressed by the
scratching and are worried about the thought of putting Sammy into child
care when you return to work. You have asthma and allergic rhinitis and your
partner had eczema as a baby.

The following information is on Sammy’s medical record:


Past medical history
Normal vaginal delivery following uncomplicated pregnancy
Medication
Nil
Allergies
Nil known
Immunisations
Up-to-date
Family history
Mother asthmatic and allergic rhinitis
Father eczema as a baby.
Suggested approach to the case
Establish rapport
Explore Jessica’s ideas, concerns and expectations.

Specific questions
Duration of the rash
Site of the rash
Scratching
Precipitating and relieving factors
Impact on Sammy and family
Previous treatment tried
Current treatment—including use of soaps
Request permission to examine.

Examination
Expose down to nappy
— Dry skin with excoriations
— Widespread patches of eczema
— Exclude secondary infection.

Diagnosis Page 116

Eczema
Plus possible contact allergy to tea-tree oil.

Management
Discuss and explain the diagnosis of eczema
Explain that eczema can be controlled but not cured
Often improves as child grows up.

Prevention
Avoid heat
— Loose fitting, cotton clothing and bedding
— Tepid comfortable bath water preferred over hot water
Avoid prickle and irritants
— Wool, nylon, seams and clothing labels
— Chlorine, sand and grass
— Soaps and bubble baths, perfumed creams or other products
— Tea-tree oil is a common cause of contact allergy and should be
stopped
Avoid dryness
— Soap-free washes
— Regular emollient regime, e.g. 50% soft 50% liquid paraffin four
times per day
— Note that lotions are drying and are best avoided. Likewise, sorbolene
and aqueous cream can cause stinging and irritation and are no longer
recommended
Consider gloves to reduce scratching
Keep fingernails short.

Treat inflammation
Intermittent topical steroid to control flares
— For example, hydrocortisone 1% ointment bd to affected areas
— A more potent steroid for non-sensitive areas of the body may be
required to control more severe flares
— Provide guidance on how much to use, e.g. one fingertip unit covers
the area of two adult palms
— Topical steroids should be used until the skin is smooth and Page 117
itch and inflammation has settled
— Addressing any fear of topical steroid use is important to ensure that
adequate steroids are used to control the eczema
Treat infection if present
Plan follow-up
Consider referral if no improvement.
CASE COMMENTARY

The key issue in this case is to demonstrate competence in the


management of a common skin condition. There are no hidden
twists in this case. A good doctor will establish a good
relationship with Jessica and explore the social and
psychological impact of the problem as well as discuss medical
management. For example, the doctor could allow Jessica to
express her concerns about returning to work and having to ask
staff at the childcare centre to put on creams. Or ask what impact
is the problem having on her relationship with Sammy or her
partner?
About 75% of children grow out of their eczema by the age
of five, but the strong family history of atopy makes this less
likely in Sammy’s case. Regular use of emollients and
intermittent topical steroids are the mainstay of therapy; severe
eczema may need systemic immunomodulatory drugs, which
require close monitoring by a doctor experienced in their use.
Eczema can be a challenging condition to manage well, and
there are often multiple factors contributing to treatment failure.
For example:
• inadequate topical steroid use (conflicting advice from
medical practitioners, fear of side effects, lack of
education/awareness of importance)
• failure to address preventative measures, e.g. avoiding
soaps and other triggers
• recurrent infection and staphylococcal carriage
• allergies to dust mites or food allergens.
Bleach baths twice weekly have been shown to reduce
severity of eczema in children with recurrent infections.
Food allergens may play a role in about 35 per cent of
children with eczema, in particular milk, egg, wheat, soy and
peanuts. However, rates of false positive allergen testing are
high. Elimination diets and food challenges could be considered
in difficult-to-control eczema, under the supervision of an
allergy specialist and dietician.

COMMON PITFALLS Page 118

Pitfalls to avoid are the prescription of steroid creams without


explanation regarding judicious use—avoiding both under-
treatment and over-treatment—and the potential side effects, and
failing to consider that the tea-tree oil is exacerbating the
problem.

Further reading
Greenhawt, M 2010, ‘The role of food allergy in atopic dermatitis’, Allergy
and Asthma Proceedings, vol. 31, no. 5, pp 392–7.
Katelaris, CH & Peake, JE 2006, ‘Allergy and the skin: eczema and chronic
urticaria’, Medical Journal of Australia, vol. 185, no. 9, pp. 517–22.
McAleer, MA, Flohr, C & Irvine, AD 2012, ‘Management of difficult and
severe eczema in childhood’, British Medical Journal, vol. 345, p.
e4770.
Ross, T, Ross, G & Varigos, G 2005, ‘Eczema: practical management issues’,
Australian Family Physician, vol. 34, no. 5, pp. 319–24.
Strathie Page, S, Weston, S & Loh, R 2016, ‘Atopic dermatitis in children’,
Australian Family Physician, vol. 45, no. 5, pp. 293–6.
Therapeutic Guidelines Ltd 2018, ‘Atopic dermatitis’. In: eTG complete
[internet], Therapeutic Guidelines Ltd, Melbourne, Vic.
The Royal Children’s Hospital Melbourne, ‘Eczema’, Clinical Practice
Guidelines. Available at: www.rch.org.au/clinicalguide/guideline_index/
eczema, accessed 15 August 2018.
Page 119

Case 23
Robert Kerslake

Instructions for the doctor


This is a short case.
You are expected to take a brief history from Robert regarding a rash. A
photograph of his rash will be available to you. There is no need to conduct
any further examination. Outline to Robert your diagnosis and suggestions
for management.

Scenario
Robert Kerslake is a 24-year-old chef with a rash on the outside of his
right and left elbows. He has tried some 1% hydrocortisone cream
purchased over the counter and it cleared the rash for a short time;
however, it recurs each time he stops using it.
The following information is on his summary sheet:
Past medical history
Chickenpox aged eight
Medication
Nil recorded
Allergies
Nil known
Immunisations
Up-to-date
Social history
Lives alone
Smokes 10–20 cigarettes per day.

Instructions for the patient, Robert Page 120

Kerslake
You are 24 years old and work as a chef. You work long hours and the
kitchen is often very hot. A few months ago, you developed a rash on your
elbows. You bought some hydrocortisone cream from a chemist and when
you use it the rash goes; as soon as you stop using it, the rash reappears. The
rash is only on your arms.
The rash is annoying you more and more and it is getting you down.
Normally you like to socialise but the rash is making you stay home rather
than going out with your mates and you are worried customers may think the
rash is infectious.
You have no family history of skin disease and have no joint, hair or nail
problems. You are keen to find out from the doctor what the rash is and what
can be done about it.
When the doctor asks to look at the rash please show them the photograph.
(Refer to Figure 3, centre insert page B.)

The following information is on your summary sheet:


Past medical history
Chickenpox aged eight
Medication
Nil recorded
Allergies
Nil known
Immunisations
Up-to-date
Social history
Lives alone
Smokes 10–20 cigarettes per day.

Suggested approach to the case


Establish rapport
Open-ended questions to explore Robert’s concerns, ideas and expectations
for the consultation.

Specific questions
Explore the symptoms of the rash and impact on his life
Ask about other involvement, e.g. joint pain, nails or hair
Ask about treatment so far
Observe clinical photograph.

Most likely diagnosis Page 121

Psoriasis.

Management
Confirm diagnosis of psoriasis
Education regarding psoriasis
— Inform psoriasis is non-infectious
— Treatment aimed at containment, not cure
— Discuss impact of stress and smoking1
Initial topical treatments
— Sunlight—improves psoriasis; balance sun exposure to help psoriasis
while minimising risk of skin cancer
— Emollients, e.g. liquid/soft paraffin
— Salicylic acid + coal tar preparations
— Intermittent use of corticosteroids for flares
— Vitamin D analogues such as calcipotriol (usually in combination
with corticosteroids)
Other topical therapies that can be considered
— Dithranol
— Topical retinoids, such as tazarotene
— Calcineurin inhibitors, such as pimecrolimus (Elidel)
Systemic treatments may be required (under the direction of a dermatologist)
for more severe or extensive psoriasis, or for psoriasis with systemic
involvement, e.g. UV light therapy, systemic retinoids, methotrexate,
cyclosporine or newer immunomodulatory drugs
Offer information about support groups, patient education leaflet
Opportunistic health promotion
— Assess motivation to stop smoking
Arrange follow-up.

CASE COMMENTARY

Psoriasis is an inflammatory immune-based disorder with a


genetic predisposition:2 activated T-lymphocytes release
cytokines that cause keratinocyte proliferation.3 Approximately
1.5% of the population are significantly affected by it and the
overall incidence is up to 6%.2 It should be appreciated that
psoriasis is a systemic disease associated with increased risks of
cardiovascular disease, diabetes and depression, as well as
psoriatic arthritis.
The characteristic well-defined red plaques with Page 122
adherent silvery scale make the diagnosis relatively
easy.
The skill in this case, as in clinical practice, is tailoring the
answer to the patient’s circumstances. Robert’s work as a chef
means it is important to reassure him that the condition is not
infectious; he can also be sensitively advised that stress and
smoking can make psoriasis worse.
A significant range of treatments are available, but these can
be messy. Topical agents and emollients are used initially: for
example, regular moisturising plus corticosteroids and/or
calcipotriol. Systemic treatment is reserved for when more than
20% of the skin is affected or when there are associated systemic
problems.1 It is not necessary to describe all available options
during this initial consult. Doctors should focus on providing
good education and reassurance regarding the diagnosis, and
clear instructions regarding an initial treatment plan. Reviewing
response to treatment is important and follow-up should be
arranged.
Psoriasis has a high financial4 and emotional cost.5 A
biopsychosocial framework will assist GPs to cover the
important aspects of management, including the impact that the
psoriasis has on Robert’s self-image, relationships and his work.

References
1. Weigle, N & McBane, S 2013, ‘Psoriasis’, American Family Physician,
vol. 87, no. 9, pp. 626–33.
2. Clarke, P 2011, ‘Psoriasis’, Australian Family Physician, vol. 40, no. 7,
pp. 468–73.
3. Nestle, FO, Kaplan, DH & Barker, J 2009, ‘Psoriasis’, New England
Journal of Medicine, vol. 361, pp. 496–509.
4. Jenner, N, Campbell, J, Plunkett, A & Marks, R 2002, ‘Cost of psoriasis:
a study on the morbidity and financial effects of having psoriasis in
Australia’, Australasian Journal of Dermatology, vol. 43, pp. 255–61.
5. Magin, PJ, Adams, J, Heading, GS & Pond, CD 2009, ‘Patients with skin
disease and their relationships with their doctors: a qualitative study of
patients with acne, psoriasis and eczema’, Medical Journal of Australia,
vol. 190, pp. 62–4.
Page 123

Case 24
Ken Anderson

Instructions for the doctor


This is a short case.
Please take a history from Ken Anderson, a 60-year-old Caucasian farmer.
A clinical photograph of Ken’s skin problem will be available to you on
request. (Refer to Figure 4 on centre insert page B.) Please discuss with Ken
the management of the condition.

Scenario
Ken Anderson is a 60-year-old sheep and cattle farmer who is a regular
patient. He is usually healthy, but you have seen him for hypertension
and hyperlipidaemia (both well-controlled), as well as for minor physical
injuries.
The following information is on his summary sheet:
Past medical history
Hypertension
Hyperlipidaemia
Medication
Telmisartan 40 mg od
Rosuvastatin 10 mg od
Allergies
Nil known
Immunisations
Up-to-date
Social history
Married
Smokes 15 cigarettes per day.

Instructions for the patient, Ken Page 124

Anderson
You’ve noticed a scaly lesion on your right cheek, which has been there for
about six weeks. You have scratched it off a couple of times but it seems to
come back. It is not painful, tender or itchy and does not bleed. It is pink or
flesh-coloured, is slowly growing and is now about 3 mm in diameter. If
asked you don’t think you have had any previous skin lesions apart from the
usual bumps, bruises and scratches that come with farming.
You are fair skinned but tend to tan rather than burn. You have spent your
whole life in rural Australia and most of your days are in the sun. You wear a
hat and usually long-sleeved clothes but don’t use sun protection creams
regularly. If asked, your father had a number of skin lesions removed by his
doctor but you don’t think any were melanomas.
You are fit and healthy and generally pretty cavalier about your health,
although you are worried this spot might be a skin cancer.

The following information is on your summary sheet:


Past medical history
Hypertension
Hyperlipidaemia
Medication
Telmisartan 40 mg od
Rosuvastatin 10 mg od
Allergies
Nil known
Immunisations
Up-to-date
Social history
Smokes 15 cigarettes per day.

Suggested approach to the case


Establish rapport
Explore Ken’s concerns about the lesion.

Specific questions
Duration of the lesion
Bleeding, itching or pain
Any treatment applied
Any previous lesions
Sun exposure
Skin type Page 125
Family history of skin conditions
General health
Request permission to examine.

Examination
Exposure—adequate lighting and magnification
Dermatoscopy
Check rest of skin for other lesions.

Diagnosis
Hyperkeratotic actinic keratosis
Background sun damage.

Management
Explanation of actinic (also called solar) keratosis and natural history
Outline treatment options—cryotherapy with liquid nitrogen as first-line
management
Explain likely blister formation and aftercare required
Discussion about sun protection
Consider nicotinamide for prevention of further actinic keratoses
Advise regarding future skin checks.

CASE COMMENTARY

Australian rates of skin cancer rank among the highest in the


world, and farmers die from skin cancer at roughly twice the rate
of other Australians (65 years and over). This is a case to
demonstrate the competent management of a very common
condition. Actinic keratosis is a clinical diagnosis that should be
made with confidence. Ken needs advice about the natural
history of the lesion, which is likely to persist and slowly grow,
and carries a small risk of progression into a squamous cell
carcinoma. Cryotherapy with liquid nitrogen represents the
simplest treatment option for a single lesion, but the patient
should be warned about likely blistering and its management.
Freezes of 20 seconds or longer are more likely to clear the
lesion, but risk of scarring and hypopigmentation increases as
well. The goal is to eradicate the lesion without scarring.
This is also an important opportunity to instruct Ken Page 126
in ongoing sun protection and skin surveillance. In the
future he will develop more lesions and will need further
cryotherapy. Topical field treatments such as 5-fluorouracil,
diclofenac or imiquimod are all possible future therapeutic
options. Regular sunscreen application and nicotinamide (500
mg once or twice daily) have both been shown to reduce actinic
keratosis burden.

COMMON PITFALLS

It is always tempting to rush to look at skin lesions, but it is


important to take a thorough history. Other common errors
include neglecting to check the remainder of Ken’s skin, as well
as erring on the side of reassurance about a benign condition and
missing a chance to educate and change Ken’s behaviour with
regard to sun protection.

Further reading
Australian Centre for Agricultural Health and Safety, Farm Health and Safety
Toolkit for Rural General Practices. Available at: www.sydney.edu.au/
medicine/aghealth/uploaded/Health%20Workers/_gp_toolkit_booklet_lo
res.pdf, accessed 22 November 2018.
Sinclair, R 2012, ‘Skin checks’, Australian Family Physician, vol. 41, no. 7,
pp. 464–9.
Uhlenhake, E 2013, ‘Optimal treatment of actinic keratoses’, Clinical
Interventions in Aging, vol. 8, pp. 29–35.
Page 127
Section 8
Ear, nose and
throat
Page 128

Case 25
Ruby Chan

Instructions for the doctor


This is a short case.
Please take a history, conduct a focused examination and outline your
diagnosis and management plan to the patient.

Scenario
Ruby Chan is 40 years old and works long hours at the local club. She
has booked this appointment to see you because of facial pain.
You practise in a small rural town of 600 people. You are the only
resident health professional.

The following information is on Ruby’s summary sheet:


Past medical history
Two children
One termination of pregnancy aged 16
Medication
Nil
Allergies
Nil known
Immunisations
Nil known
Pap smear
Up-to-date
Social history
Works at the Memo Club
Non-smoker
Alcohol intake—special occasions only.

Instructions for the patient, Ruby Chan Page 129

You are a 40-year-old woman who has had facial pain for a few days. It is
getting worse and it kept you awake last night. You have had a cold and are
overdue for a check-up at the dentist. The nearest dentist is over 50 km away.
The pain is worse when you chew food. You have a slight headache but this
does not get worse on bending forwards or sneezing. Your sense of smell is
undisturbed.
Your father died of cancer of the tongue. You have never been sure
exactly what this is, but it does mean that you are more concerned than
normal about your pain being something serious. You would like the doctor
to find this out but will not reveal this unless asked appropriately.

Instructions for the facilitator


The candidate should perform a focused oral examination.

Clinical examination findings


Tapping of one of your molars gives you significant pain.
The following information is on your medical record:
Past medical history
Two children
One termination of pregnancy aged 16
Medication
Nil
Allergies
Nil known
Immunisations
Nil known
Pap smear
Up-to-date
Social history
Works at the Memo Club
Non-smoker
Alcohol intake—special occasions only.

Suggested approach to the case


Establish rapport
Open-ended questions to explore Ruby’s ideas, concerns and expectations.

Specific questions Page 130

Ask about the pain, e.g. using OLD CARTS acronym


— Onset
— Location
— Duration, diurnal variation
— Characteristics—worse on chewing
— Associated symptoms
— Relieving factors
— Treatment tried so far
— Severity
Exclude systemic symptoms—weight changes, energy level, sleep
disturbance
Request permission to examine.

Examination
Temperature
Inspect and/or palpate head and neck
Check teeth, lymph nodes, skin, eyes, salivary glands, temporomandibular
joint, cervical spine, nose, mouth, pharynx and post-nasal space and
sinuses.

Most likely diagnosis


Dental pain.

Management
Urgent dental consultation
Pain relief—aspirin or paracetamol, clove oil.

Extras for this case


Explore patient’s concerns resulting from her father’s cancer and previous
occupational exposure to cigarette smoke
Explain need for patient to book another appointment to discuss vaccination
status.

CASE COMMENTARY

Ruby’s pain is caused by a sore tooth. This will become obvious


as the doctor takes the history and taps the teeth.1 After hours,
and in rural areas, GPs are often required to provide care for
dental problems.2 Antibiotics are indicated if there is Page 131
possible cellulitis secondary to an infected tooth. Dental
abscesses require incision and drainage and root canal treatment
or extraction.3
There is a long list of possible features to check in the face.1
A good doctor can focus promptly on examination of the teeth,
lymph nodes and temporomandibular joints. If tapping the teeth
with a tongue depressor reproduces the pain there is no need to
proceed with further examination. The time can then be spent
explaining the diagnosis, planning management and exploring
any particular concerns that Ruby has. A doctor who does a
complete examination will not have time to explore the concerns
about Ruby’s father’s tongue cancer or have time to discuss
Ruby’s vaccination status.

References
1. Quail, G 2015, ‘Facial pain–a diagnostic challenge’, Australian Family
Physician, vol. 44, no. 12, pp. 901–4.
2. Wetherall, J, Richards, L, Sambrook, P & Townsend, G 2001,
‘Management of acute dental pain: a practical approach for primary
health care providers’, Australian Prescriber, vol. 24, no. 6, pp. 144–8.
3. Beech, N, Goh, R & Lynham, A 2014, ‘Management of dental infections
by medical practitioners’, Australian Family Physician, vol. 43, no. 5, pp.
289–91.

Further reading
Kingon, A 2009, ‘Solving dental problems in general practice’, Australian
Family Physician, vol. 38, no. 4, pp. 211–6.
Page 132

Case 26
Jane Matthews

Instructions for the doctor


This is a short case.
Please take a history and conduct an appropriate examination. Please tell
the facilitator your findings and suspected diagnosis.

Scenario
Jane Matthews is a 31-year-old woman who is pregnant for the first time.
She is coming to see you because she has noticed a slight swelling in her
neck just below her Adam’s apple.

The following information is on her summary sheet:


Past medical history
Nil significant. G1P0 18/40
Medication
Nil
Allergies
Nil known
Immunisations
Nil recorded
Cervical screening test
Normal this year
Family history
Nil significant
Social history
Married
Works as a hairdresser.

Instructions for the patient, Jane Page 133

Matthews
You are a 31-year-old married hairdresser. You are excited about being
pregnant for the first time. Everything has been going well. Last week you
were on holiday and you have just been looking at the photos again. Your
neck looked a bit odd in the photos and you have now had a closer look in the
mirror. There is a smooth swelling in the middle below your Adam’s apple.
You saw your GP last week about the pregnancy and the check-up was
fine. You have booked today’s appointment just to sort out the swelling. You
are not particularly worried about it.

Clinical examination findings


You have a diffuse goitre but no other physical findings or symptoms.

The following information is on your summary sheet:


Past medical history
Nil significant. G1P0 18/40
Medication
Nil
Allergies
Nil known
Immunisations
Nil recorded
Pap smear
Normal this year
Family history
Nil significant
Social history
Married
Works as a hairdresser.

Suggested approach to the case


Establish rapport
Open-ended questions to explore patient’s ideas, concerns and expectations.

Specific questions
Weight loss, appetite, vomiting, lumps elsewhere
Heat tolerance, energy levels, palpitations, tremor, and difficulty swallowing
solids or liquids
Change in voice, change in skin or hair, sweating, sleep pattern Page 134
Previous thyroid dysfunction, medications (specifically amiodarone,
lithium)
Family history: any family history of thyroid problems, diabetes
Request permission to examine.

Examination
Inspection of the swelling
Palpation
— Site
— Size
— Consistency
— Tenderness
Movement on swallowing
Percussion—not needed
Auscultation of swelling for bruits
Palpation of lymph nodes of the neck
Examination for thyroid signs
— Pulse
— Tremor
— Eyes—for proptosis
— Warmth of peripheries
— Reflexes
— General skin changes
There is minor diffuse thyroid swelling and no other abnormality.

CASE COMMENTARY

Benign thyroid enlargement is the most likely cause of a midline


neck swelling in a pregnant woman like Jane. A doctor should
take a history about the swelling and any local symptoms and
establish whether she is clinically hypothyroid or hyperthyroid.1
The neck examination should reveal that the swelling moves on
swallowing. Examination of the lymph nodes should show that
the doctor knows the position of the lymph nodes and should be
conducted from behind.
Routine biochemical screening for thyroid dysfunction is not
recommended in pregnancy2 but is indicated for symptomatic
patients.3 Traditional teaching is that any neck swelling or lump
requires examination of the scalp, ears, skin, face, mouth and
throat.4 This is vital if the lump is an enlarged lymph node but is
not needed in this case where the history and Page 135
examination findings are so suggestive of a thyroid
problem. This is an example of type 1 or pattern recognition
rather than type 2 or hypothetico-deductive thinking.
Experienced GPs use pattern recognition as a method of clinical
reasoning because it is much quicker, in this case, allowing the
doctor time to check on thyroid status as well as the swelling.
Doctors who use the hypothetico-deductive method reasoning
through this case will be safe but will have used valuable time
on unnecessary issues.
The doctor is not expected to discuss management of the
swelling.

References
1. Hughes, K & Eastman, C 2012, ‘Goitre—causes, investigation and
management’, Australian Family Physician, vol. 41, pp. 572–6.
2. Royal Australian and New Zealand College of Obstetricians and
Gynaecologists 2015, Testing for hypothyroidism during pregnancy with
serum TSH. Available at:
https://www.ranzcog.edu.au/RANZCOG_SITE/media/RANZCOG-
MEDIA/Women’s%20Health/Statement%20and%20guidelines/Clinical-
Obstetrics/Testing-for-hypothyroidism-during-pregnancy-with-serum-
TSH-(C-Obs-46)-Review-July-2015.pdf?ext=.pdf, accessed 19 February
2019.
3. Smith, A, Eccles-Smith, J, D’Emden, M & Lust, K 2017, ‘Thyroid
disease in pregnancy and postpartum’, Australian Prescriber, vol. 40, pp.
214–9.
4. Roland, N & Bradley, PJ 2014, ‘Neck swellings’, British Medical Journal
vol. 348, p. g1078.
Page 136

Case 27
Pamela Taylor

Instructions for the doctor


This is a short case.
Please take a history, examine this patient appropriately and tell the
examiner your diagnosis and management plan.

Scenario
You are asked to see Mrs Taylor, a 62-year-old tourist who has recently
arrived in town. She is feeling dizzy.

Instructions for the patient, Pamela Taylor


You are a 62-year-old woman who is normally well. You are caravanning
around Australia with your husband, and yesterday you were relieved to
reach a town where you could finally get to a hairdresser.
This morning you woke up as normal, but when you turned your head to
check the view from the caravan window, you suddenly experienced an
overwhelming spinning sensation in your head and marked nausea. If you
stay still you feel OK, but whenever you turn your head the spinning and
nausea return. The symptoms last less than a minute. You have not had
anything like this before.
Your opening line is, ‘Doctor, I feel really dizzy’.
You have no tinnitus and your hearing is normal. You have no headache
or any other neurological symptoms.
You regularly attend exercise classes in your home town, and your only
medication is psyllium with ispaghula, one sachet once a day. In the past you
have had surgery for a benign thyroid nodule.
You do not have any allergies, have never smoked and drink one to two
standard alcoholic drinks at weekends.

Clinical examination findings Page 137

You will experience nausea when asked to move on and off the examination
couch.
The candidate would observe nystagmus in your eyes during the Dix-
Hallpike manoeuvre.
No other abnormalities are found on clinical examination.

Suggested approach to the case


Establish rapport
Open-ended questions to explore Pamela’s ideas, concerns and expectations.

Specific questions
Dizziness—spinning feeling or as if about to faint, episodic or continuous
Vertigo—relationship to position
Nausea and vomiting, diarrhoea—exclude gastroenteritis
Tinnitus
Change in hearing
Visual symptoms
Previous history of similar episode
Exclude
— Fits
— Cardiac cause
— Head injury
Past medical history
— Medication including OTC and complementary or alternative
medicines
— Allergies
Request permission to examine.

Examination
Cardiovascular system
— Pulse
— BP lying and standing
— Heart sounds
Neurological examination
— Cognitive function—no apparent problem
— Cranial nerves Page 138
III, IV, VI eye movements, look for nystagmus
VIII otoscope examination, hearing
— Coordination—finger–nose or heel–toe test
— Gait
— Romberg’s test
— Head-thrust test1
— Dix-Hallpike manoeuvre1
Neck
— Cervical spine movements.

Most likely diagnosis


Explain to the examiner that the most likely diagnosis is benign paroxysmal
positional vertigo (BPPV)
Reassurance—expect recovery.2

Management
Instruct patient on using the Epley manoeuvre or Brandt-Daroff exercises to
treat the condition by canalith repositioning3, 4
Limited role and efficacy for antihistamine and antiemetic agents
Arrange follow-up
Advise against driving
Investigate for alternative causes and consider specialist referral if symptoms
persist.

CASE COMMENTARY

Dizziness is a common presentation with multiple potential


causes. The history is essential to accurate diagnosis. The first
step is to establish that Mrs Taylor uses the term dizziness to
describe a spinning sensation (vertigo) and does not feel faint
(suggests a cardiovascular cause).
The most likely cause in this case is benign paroxysmal
positional vertigo because the vertigo is exclusively related to
changes in posture. Being in a recumbent position for a
prolonged time, as can occur at the hairdresser, is a recognised
precipitant of BPPV.
In vestibular neuronitis the symptoms are sustained but often
worse on movement, and there may have been a preceding viral
upper-respiratory infection. The head-thrust test would be
abnormal. Menière’s disease is unlikely because there is Page 139
no tinnitus or deafness. Stroke should be considered and
is more likely if there are other new neurological symptoms and
signs. Vestibular migraine is possible but would usually have an
associated headache and, without any past history of migraine,
would need investigation to exclude an underlying cause.
If the symptoms do not resolve with treatment, further
investigation is needed to check for a cerebellar or cerebral
lesion. Pamela will benefit from written advice to take with her
on her travels.

References
1. Dommaraju, S & Perera, E 2016, ‘An approach to vertigo in general
practice’, Australian Family Physician, vol. 45, pp. 190–4.
2. Kim, JS & Zee, DS 2014, ‘Clinical practice. Benign paroxysmal
positional vertigo’, New England Journal of Medicine, vol. 370, pp.
1138–47.
3. Glasziou, P, Bennett, J, Greenberg, P et al. 2013, ‘The Epley manoeuvre
—for benign paroxysmal positional vertigo’, Australian Family
Physician, vol. 42, pp. 36–7.
4. Therapeutic Guidelines Ltd 2018, ‘Brandt-Daroff exercises, patient
handout’. In: eTG complete [Internet], Therapeutic Guidelines Ltd,
Melbourne, Vic.
Page 140

Case 28
Trevor Watts

Instructions for the doctor


This is a short case.
Please take a history from Trevor. You are not required to examine him.
The results of an appropriate examination will be available from the
facilitator on request. Outline the most likely diagnosis to the patient and
initial suggestions for management.

Scenario
Trevor Watts is a 41-year-old man. For the last few months he has
noticed a ringing sound in his right ear and sometimes there is ringing in
his left ear. At first it did not bother him but now he is finding it very
hard to live with. It is beginning to interfere with his sleep and it is
getting him down.
He has no known deafness, balance problems or pain in his ears.
He is otherwise well.

The following information is on his summary sheet:


Past medical history
Laceration left leg following motorcycle accident aged 20
Medication
Nil
Allergies
Nil known
Immunisations
Up-to-date. Last immunisation three years ago; ADT
Social history
Office manager.

Instructions for the patient, Trevor Watts Page 141

You are 41 years old and work as an office manager. For the last few months
you have noticed a ringing sound in your right ear, and more recently an
occasional ringing also in your left ear. At first it did not bother you but now
you are finding it very hard to live with. It is beginning to interfere with your
concentration and sleep and it is getting you down.
You have no known deafness, balance problems, vertigo or pain in your
ears. You are otherwise well. You do not take any medication.
In your late teens and early twenties, you were a drummer in a successful
rock and roll band. ‘The louder the music the better’ was your motto. You
have always worked in an office environment in one capacity or another.
You live with your girlfriend and her two children from a previous
relationship. You smoke 20 cigarettes a day and drink 10 stubbies of beer
every Friday and Saturday night.

The following information is on your summary sheet:


Past medical history
Laceration left leg following motorcycle accident aged 20
Medication
Nil
Allergies
Nil known
Immunisations
Up-to-date. Last immunisation three years ago; ADT
Social history
Office manager.

Information for the facilitator


Clinical examination findings
Pulse 68 regular
BP 132/82 mmHg
No objective tinnitus
Inspection of the ears, including otoscopy: normal pinnas, external auditory
canal and eardrums
Hearing—normal whisper test, normal Rinné and Weber tests
Balance—normal Romberg’s test
Audiometry—bilateral sensorineural high frequency deafness
Cranial nerves normal.

Suggested approach to the case Page 142

Establish rapport
Open-ended questions to elicit Trevor’s ideas, concerns and expectations.

Specific questions
Tinnitus—timeline, pitch, frequency, duration, character (pulsatile or not),
continuous or intermittent
Deafness
Balance
Vertigo
Pain
Medication—exclude use of ototoxic drugs such as aspirin, loop diuretics
Past noise exposure—occupational or recreational
Past history of infections, surgery or trauma
Exclude depression with screening questions
Exclude serious neurological problem.

Ask for the examination findings


Pulse
BP
No objective tinnitus
Ear inspection with otoscope
Hearing—whisper test, Rinné and Weber
Balance—Romberg’s test
Cranial nerves normal

Most likely diagnosis


Tinnitus—caused by noise exposure.

Investigations
BSL
Audiometry1—bilateral high frequency sensorineural deafness demonstrated.

Management2, 3
Reassure and explain nature of tinnitus
Aim for symptom control/habituation, cure unlikely
Suggest join support group
Tinnitus masker Page 143
Relaxation treatment
Cognitive behavioural therapy
Brief intervention regarding smoking and drinking
Arrange follow-up.
CASE COMMENTARY

Tinnitus is common and affects up to 10% of people in


industrialised societies. Approximately 0.5% report that tinnitus
has a severe effect on their daily life.2 It is thought to originate
from the central rather than peripheral auditory system. The
number of years of work-related noise exposure correlates with
increasing prevalence of tinnitus.
The doctor needs to take a history, to ask for the results of a
focused examination and to establish that the most likely cause
is noise exposure earlier in the patient’s life. A vestibular
schwannoma would need to be excluded if the tinnitus and
deafness shown on audiometry was unilateral, otherwise further
investigations or imaging is not indicated.2
Trevor needs reassurance that there is no underlying
pathology to cause concern but the doctor should not dismiss the
symptoms as insignificant or tell him glibly, ‘Don’t worry about
it’. Tinnitus is a risk factor for suicide. It is important that the
doctor accurately assess Trevor’s psychological well-being and
treat any associated anxiety or depression.2 Management focuses
on ways to habituate to the noises and using hearing protection
to prevent exacerbation.2, 3 Future options may include brain
retraining, capitalising on brain plasticity.4

References
1. Rose, E 2011, ‘Audiology’, Australian Family Physician, vol. 40, no. 5,
May, pp. 290–2.
2. Esmaili, A & Renton, J 2018, ‘A review of tinnitus’, Australian Journal
of General Practice, vol. 474, pp. 205–8.
3. Flanagan, S 2013, ‘How to treat: tinnitus’, Australian Doctor, 5 July.
Available at: www.ausdoc.com.au/therapy-update/treating-tinnitus,
accessed 23 May 2019.
4. Lozano, AM 2011, ‘Harnessing plasticity to reset dysfunctional neurons’,
New England Journal of Medicine, vol. 364, pp. 1367–8.
Page 144

Case 29
Clayton Dixon

Instructions for the doctor


This is a short case.
Please take a history from Clayton’s grandmother, Shirley Dixon. When
you are ready, ask the facilitator to give you the results of an appropriate
examination. No further investigations are available at this time.
Outline your differential diagnosis and management plan to the facilitator.
The consultation takes place as a single session.

Scenario
Clayton Dixon is an 8-year-old Indigenous boy who lives with his
grandmother, Shirley. They have been in the area for several years and
often attend the accident and emergency department (ED) of the local
hospital or a bulk-billing medical centre in town. You have some letters
summarising his presentations to the ED but have no other notes. You
have seen him for boils, scabies, chest infections and discharging ears,
and he has been seen for similar presentations in the ED as well. Past
medications include Flopen (flucloxacillin), Amoxil (amoxycillin),
Kenacomb Otic (triamcinolone acetonide, neomycin, nystatin,
gramicidin), Sofradex (dexamethasone, framycetin and gramicidin),
Ascabiol (benzyl benzoate) and Lyclear (permethrin).

The following information is on his summary sheet:


Age
Eight
Past medical history
Chronic suppurative otitis media
Recurrent boils
Scabies
Upper and lower respiratory tract infections Page 145
Medication
No regular medications
Allergies
Nil known
Immunisations
Up-to-date
Social history
Lives with his grandmother, Shirley.

Instructions for the patient’s grandmother,


Shirley Dixon
You are Shirley Dixon, a 47-year-old married Indigenous woman who is
bringing her 8-year-old grandson, Clayton, to the doctor because his ears are
running again and you need more ear drops.
Clayton is assumed to be present.
Your opening line should be, ‘I’ve got my grandson, Clayton, with me,
Doctor. His ear is running again.’
You often look after Clayton and his little sister, Taneka, who is six years
old. His mother struggles with alcohol and drugs and for the time being is
living in another city. The authorities have never had any concerns about
Clayton’s welfare because you and your husband have stepped in whenever
needed.
Clayton has always been thin. He has had recurrent boils, chesty coughs
and runny ears. You have taken him to this clinic, another medical centre and
the hospital emergency department. Currently his right ear has been running
for the past two to three weeks. It’s a thick, yellow discharge that comes out
and stains his pillow and sheets. He says it sometimes hurts when he has
infections, but he usually doesn’t get a temperature.
If asked, Clayton’s hearing hasn’t been so good for the past few weeks. He
does seem to get into trouble at school and has been suspended for poor
behaviour a couple of times. He enjoys sport and playing outside with his
friends but has never really liked school and doesn’t do very well.
If asked, you have been using some ear drops you got from another doctor
when his ears started running. You’re not sure of the name but you think it is
Keno-something. The bottle you had left over from a previous episode has
just run out. You’ve had it lots of times in the past, as well as Sofra-
something. Sometimes doctors give you an Amoxil mixture for Clayton to
take as well, but you haven’t got any at the moment. You use these Page 146
drops when his ears run: two drops in the running ear, three or four
times a day. Mostly you stop the drops when the ear stops running, but
occasionally you have kept it going for a week or so to stop the infection
coming back so quickly. You sometimes ring this clinic or the medical centre
to get scripts for more ear drops without coming in. If asked, you’ve probably
had one or two scripts for ear drops each month for the past six months.
If asked, Clayton has recently (for the last month or so) started
complaining about ringing in his right ear and also some giddiness. You have
noticed his balance isn’t as good as it usually is.
If asked, you think his immunisations are up-to-date.
He has had a few chest infections, which have also required antibiotics.
He’s not on any other medications (apart from the drops you’ve been giving
him) and has no allergies that you know of. You try to feed the children a
healthy diet, but both Clayton and Taneka are very picky eaters, with a
fondness for lollies. You sometimes suspect the healthy lunches you pack for
school are traded for less healthy snacks.
At home with you, as well as Clayton and Taneka, is your husband, Gary.
Your other children are grown and have left town—you see them for
holidays.
If asked, neither you nor Gary are smokers and are both physically
healthy.

The following information is on Clayton’s summary sheet:


Age
Eight
Past medical history
Chronic suppurative otitis media
Recurrent boils
Scabies
Upper and lower respiratory tract infections
Medication
No regular medications
Allergies
Nil known
Immunisations
Up-to-date
Social history
Lives with his grandmother, Shirley.

Suggested approach to the case


Establish rapport
Open questions to explore Shirley’s concerns.

Specific questions Page 147

History of the ear discharge


Pain
Hearing loss
Fever
Tinnitus/vertigo
Past (current) treatment
General health/diet
Social history
School performance (with respect to hearing loss)
Passive smoking
Medications/allergies.

Information for the facilitator


Clinical examination findings
General appearance: Indigenous male child of stated age in no distress
Height 135 cm (50th percentile)
Weight 27 kg (25th percentile)
BP 95/60 mmHg, no postural drop
Pulse 86/min regular
Temperature 37.4 tympanic (left ear)
ENT—throat normal, nose normal
External ear canals—right has purulent discharge; left normal
Tympanic membranes—right has suppurative otitis media with small inferior
perforation; left is normal apart from some patches of tympanosclerosis
Cervical nodes normal
If asked, mastoid is normal
Gross hearing assessment indicates moderate hearing loss R > L
Chest/abdomen—NAD
Skin—several scars from previous infections, otherwise normal
Remainder of examination is normal.

Differential diagnoses
Chronic suppurative otitis media with perforation
Aminoglycoside ototoxicity
Possibility of cholesteatoma formation or suppurative labyrinthitis.

Management1 Page 148


Cease aminoglycoside drops and caution against future use with signs of
perforation
Regular ear toilet (Betadine 0.05% washes along with tissue ear spears)
Use of non-ototoxic fluoroquinolone drops (ciprofloxacin)
Oral antibiotics may benefit (amoxycillin)
Order appropriate investigations such as audiology, tympanometry
ENT referral
Scheduled GP follow-up
Encourage healthy diet.

CASE COMMENTARY

Chronic suppurative otitis media (CSOM) is a disease of poverty


and represents a significant burden of disease for Indigenous
Australians and refugee populations, with estimates of
prevalence ranging from 15% to 80%. Complications include
hearing loss, mastoiditis, cholesteatoma, facial nerve paralysis,
meningitis, brain abscess and sigmoid sinus thrombosis. In
children, hearing loss affects speech, language and intellectual,
psychological and social development, as well as education.
Frequently, treatment is complicated by fragmented care,
poor health literacy and issues with access to and cost of health
care. In this case, Clayton is receiving treatment from a number
of sources, including the local hospital and two different GP
clinics. He has symptoms consistent with ototoxicity from
repeated aminoglycoside ear drops in the presence of
perforation. This is an uncommon but serious complication, and
Australian recommendations include using non-ototoxic
fluoroquinolone ear drops together with twice-daily ear cleaning,
with weekly GP review and referral to ENT.1
Under the Pharmaceutical Benefits Scheme, ciprofloxacin ear
drops are subsidised for Aboriginal and Torres Strait Islander
people (aged one month or older).
Prevention principles include ensuring adequate nutrition,
keeping immunisations up-to-date, encouraging nose blowing as
well as breastfeeding of infants and avoiding passive smoking.

COMMON PITFALLS

It’s easy to focus on treating the CSOM and perforation and to


miss the subtle signs of ototoxicity. Instructing the carer in the
techniques of ear cleaning is time consuming but Page 149
important in Clayton’s care. In a short case, reference
could be made to the practice nurse instructing the grandmother
how to use tissue ear spears and demonstrating the technique.
Swabbing the discharge is unlikely to be helpful. We should be
open about the iatrogenic complication of aminoglycoside
ototoxicity, avoiding either a defensive or accusatory tone.

Reference
1. Indigenous and Rural Health Division 2015, ‘Recommendations for
clinical care guidelines on the management of Otitis Media in Aboriginal
and Torres Strait Islander Populations’, Department of Health, Canberra,
ACT.

Further reading
Australian Institute of Health and Welfare 2018, ‘6.4 Ear health and hearing
loss among Indigenous children’, Australia’s Health 2018. AIHW,
Australia’s health series no. 16, AUS 221, Canberra. Available at: www.
aihw.gov.au/getmedia/12c11184-0c0a-43ad-8386-975c42c38105/aihw-a
us-221-chapter-6-4. pdf.aspx, accessed 19 February 2019.
Closing the Gap Clearinghouse (AIHW & AIFS) 2014, ‘Ear disease in
Aboriginal and Torres Strait Islander children’, resource sheet no. 35.
Produced by the Closing the Gap Clearinghouse. Canberra: Australian
Institute of Health and Welfare & Melbourne: Australian Institute of
Family Studies.
Coates, H 2008, ‘Ear drops and ototoxicity’, Australian Prescriber, vol. 31,
pp. 40–1.
‘Deadly Ears, Queensland Health’s Aboriginal and Torres Strait Islander
Ear Health Program.’ Available at:
www.childrens.health.qld.gov.au/chq/our-services/community-health-
services/deadly-ears/middle-ear-disease/, accessed 01 December 2018.
Hill, S 2012, ‘Ear disease in Indigenous Australians: a literature review’,
Australian Medical Student Journal, vol. 3, no. 1, pp. 45–9.
Gunasekera, H, O’Connor, T, Vijayasekaran, S & Del Mar, C 2009, ‘Primary
care management of otitis media among Australian children’, Medical
Journal of Australia, vol. 191, no. 9, p. 55. Page 150
Page 151
Section 9
Emergency
medicine
Page 152

Case 30
Catriona Chryssides

Instructions for the doctor


This is a short case about a new patient.
Please manage this medical emergency.

Scenario
Catriona Chryssides is a 29-year-old arts coordinator. You have been
asked to see her as an emergency because of dizziness and palpitations
on a Saturday morning.
You are doing a locum in a remote Aboriginal community. The clinic
has a treatment room with emergency equipment and medication. The
nearest hospital is 300 km away and patients requiring urgent hospital
care are evacuated by the RFDS plane. Hospital specialists provide
telephone advice if needed. An Aboriginal Health Practitioner is also
working in the clinic.
The facilitator will give you the physical examination findings and the
results of initial investigations when specifically requested.

Instructions for the patient, Catriona


Chryssides
You have just moved to the area as an arts coordinator. You wanted a change
of scene and to travel before your 30th birthday next month. It has been a
major upheaval leaving your friends and family in the city. You are finding
the work challenging and chaotic, and wonder if you will ever be able to
make a worthwhile contribution.
It’s Saturday morning. You started the day with a strong coffee and got up
to go for a walk and felt fine. Suddenly you experienced a jolt in your heart
and it feels as if it has gone haywire as it is pounding rapidly. You feel dizzy,
unwell, a bit nauseated and frightened. You told your housemate how you felt
and she promptly drove you down to the clinic.
You have never experienced anything like this before. You have Page 153
no significant past medical history.
You smoke 10 cigarettes per day but do not consume alcohol or take
recreational drugs.
Before leaving the city you made sure that you were up-to-date with your
vaccinations and cervical screening test. You have a subcutaneous
etonogestrel (Implanon) contraceptive implant.
You will suddenly feel well again when the doctor gives you the second
injection.

Instructions for the facilitator


This scenario can be changed to assess the doctor’s emergency skills in their
own clinical setting, for example, the venue could be a small rural hospital or
a suburban general practice.
The observations are:
• temperature normal (37.0°C)
• pulse 180 beats per minute, regularly regular rhythm by palpation
• 16 breaths a minute, no evidence of cyanosis
• BP 92/64 mmHg, sitting. JVP normal
• orientated to time, place and person.
Please show the candidate the ECG recording once the monitor is attached.
(Refer to Figure 5, centre insert page C.) (Note: an ECG is given rather than
just rhythm strip to enable the candidate to proceed with definitive treatment
in the time available for the case.)
Ask the candidate for their diagnosis when they have looked at the ECG.
Sinus rhythm is restored after the second (12 mg) bolus of adenosine.
When Catriona has returned to sinus rhythm ask the candidate to outline a
future management plan.

Suggested approach to the case


Primary survey
DRABC

Danger—ensure safety of patient and personnel, move through to treatment


room
Response—patient able to say name and describe symptoms of dizziness and
‘heart racing’
Airway—patent as patient able to talk, no evidence of airway obstruction
Breathing—breathing regularly, no visible restriction to lung Page 154
movement, give oxygen via mask
Circulation—pulse rapid
— Request Aboriginal Health Practitioner to check blood pressure; BP
92/64 mmHg
— Put on heart monitor and continue to monitor
— Request and interpret ECG
Patient assessed as having supraventricular tachycardia and is
haemodynamically stable.
Focused history
Current symptoms—especially chest pain (nature of pain, radiation, etc.),
shortness of breath, nature of palpitations (skipped beats, extra beats, etc.)
Risk factors for pulmonary embolism—recent travel, previous DVT
Any previous similar episodes of dizziness and chest discomfort and their
treatment
Any family history of heart disease
Medication
Nil taken
Allergies
Nil known.

Treatment1, 2
Valsalva manoeuvre and leg raise—no effect
Carotid sinus massage—no effect
Cold stimulus—not available
Insert IV cannula—take blood for FBC, UEC, troponin (TFTs can be
delayed), magnesium, calcium
Intravenous adenosine as bolus of 6 mg, 12 mg, plus 20 mL saline flush
(Note: if the candidate does not know the dose regime, it is acceptable for
them to check this in a formulary)
Warning to patient of what to expect (sense of impending doom, or heart
stopping)
Returns to sinus rhythm with resolution of symptoms after second bolus.

Further management
Finish secondary survey
Review history—history of palpitations
Explain what happened Page 155
Assess likely precipitants—caffeine intake, current stressors
Advice about manoeuvres to try if it recurs
Advise regarding work—unlikely to need time off
Driving—check in Austroads book/website3
ECG when in sinus rhythm to check for delta wave of Wolf–Parkinson–
White (WPW) syndrome.

Future management
Arrange follow-up and, if recurrent, other treatments available such as
medication or ablation
Health promotion—smoking cessation.

CASE COMMENTARY

This scenario tests the doctor’s ability to cope with an


emergency situation. A logical and calm approach is needed and
the doctor should proceed through DRABC in sequence. Some
doctors who trialled this case argued that assessing Danger
should not be needed when working in a clinic. They make a
good point, but clinicians should consider staff and patient
safety when responding to an emergency, especially as the
realities of weekend on-call work when reduced staffing may
mean omission of checks of emergency equipment.
This case also tests a doctor’s ability to work well with the
Aboriginal Health Practitioner and give clear instructions.
The doctor is expected to recognise the pattern of the SVT on
the ECG and immediately commence vagal stimulation to try to
revert the rhythm.1 The order of pharmacological treatment is
adenosine and then verapamil.2 If these are ineffective,
telephone advice should be sought about other options such as
flecainide, sotalol or amiodarone4 but availability of these drugs
may be a problem. Not all clinics stock adenosine and so it is
acceptable for a doctor to stabilise Catriona and arrange for the
retrieval service to give adenosine on arrival.
DC reversion is reserved for patients who are
haemodynamically unstable and require sedation. Longer-term
recurrent SVT may be treated by ablation therapy.5
Once the emergency is under control, the doctor needs to
consider other important aspects of care such as her fitness for
work, driving, what to do if she has another episode and when
she should be reviewed. Doctors are not expected to Page 156
remember the detail of the Austroads Assessing Fitness
to Drive handbook3 but must know to look at it for advice.
Giving Catriona a copy of the arrhythmia captured on ECG is a
practical tip that ensures it is available for health professionals in
the future.4

References
1. Royal Australian College of General Practitioners 2018, ‘Modified
Valsalva manoeuvre for supraventricular tachycardia’, Handbook of Non-
Drug Interventions (HANDI), RACGP, Melbourne, Vic.
2. Medi, C, Kalman, JM & Freedman, SB 2009, ‘Supraventricular
tachycardia’, Medical Journal of Australia, vol. 190, pp. 255–60.
3. Austroads and National Transport Commission 2017, Assessing Fitness to
Drive, 5th ed, Sydney.
4. Whinnett, ZI, Sohaib, SM & Davies, DW 2012, ‘Diagnosis and
management of supraventricular tachycardia’, British Medical Journal,
vol. 345, p. e7769.
5. Lee, G, Sanders, P & Kalman, JM 2012, ‘Catheter ablation of atrial
arrhythmias: state of the art’, Lancet, vol. 380, pp. 1509–19.
Page 157

Case 31
Carrie Patterson

Instructions for the doctor


This is a short case.
Please manage this emergency.

Scenario
You are driving to work along the local high street. You see a cyclist
swerve across the road and fall off her bicycle. You stop your car and
grab your emergency equipment and drugs.

Instructions for the patient, Carrie Patterson


You are 28 years old and have had insulin-dependent diabetes for 15 years.
You have just moved to this area for work as a barmaid. You enjoy the work
but the late hours make it difficult to control your blood sugar.
You wear a medical alert bracelet on your wrist that says you are an
insulin-dependent diabetic.
This morning you set off on your bike to do some shopping. You began to
feel hypoglycaemic but did not have any snacks with you. One minute you
were riding along, the next thing you know is that you’re lying on the
pavement next to your bicycle. This case begins with you lying beside your
bicycle on the side of the road. A local GP comes up to you. You are able to
say your name but mumble it slowly. If you are asked if you are diabetic, you
can mumble ‘Yes’.
You will recover quickly when the doctor gives you either:
• an injection of intravenous glucose
• an injection of glucagon IM, IV or SC, or
• oral/rectal glucose.

The following information is on your medical record at your own Page 158
GP’s surgery:
Past medical history
Insulin-dependent diabetes since age 13
Medication
Long-acting insulin glargine (Lantus 25 u at night)
Long-acting insulin mixed with short-acting insulin (Mixtard 30/70 42 u
mane, 18 nocte)
Short-acting insulin (Novorapid 6–12 u before meals tds)
Allergies
Nil
Immunisations
Up-to-date
Social history
Employed as a barmaid.

Suggested approach to the case


Primary survey
DRABCDE
Danger—check for any danger at the scene, park car safely, use passers-by to
divert traffic if repositioning the patient, make comment on spine
precautions
Response—check for any response, person able to mumble their name
Airway—maintain cervical spine precautions
— Airway clear, as person able to mumble name
— Apply neck collar or manual in-line head and neck stabilisation
Breathing and ventilation—breathing regular and symmetrical with no added
sounds
Circulation with haemorrhage control—ability to mumble name suggests
brain perfusion
— No evidence of external haemorrhage
Disability—Glasgow Coma Scale 14
— Eye opening response—opens eyes spontaneously (4)
— Verbal response—says name, but not sure where they are and what
happened (4)
— Motor response—does everything you ask (6)
Exposure/Environmental control
— Remove patient’s clothes to enable a secondary survey, respecting
their dignity
— Consider ambient temperature.

Secondary survey Page 159

History

AMPLE
A–no Allergies
M–Medication—insulin
P–Past history, injuries, hospital admissions
L–Last time ate or drank
E–Event
Top-to-toe examination including check for medical alert bracelet or
identification
As soon as the doctor realises that the person is diabetic and on insulin, they
should proceed with management of possible hypoglycaemia.

Treatment
— Glucagon IV, IM or SC—adult dosage: 1 mg and/or
— Glucose 50% IV at 3 mL/min via large vein—adult 20–50 mL
depending on response
— Offer slow-acting oral carbohydrate, such as bread, milk or fruit, once
patient able to eat1
The scenario should finish with clearance from the scene and arrangement for
transport to a hospital/clinic, preferably by ambulance
Full physical examination will be required in the hospital or clinic.

CASE COMMENTARY

GPs should be equipped to cope with simple medical


emergencies such as hypoglycaemia.2 In urban areas, emergency
care can often be provided more quickly by paramedics; in rural
and remote Australia, the GP may be required to attend minor
and major incidents at any time of night or day.3 Tight
glycaemic control reduces the long-term complications of
diabetes but increases the risk of hypoglycaemia.4

COMMON PITFALLS

Carrie’s public hypoglycaemic episode means that emergency


care needs to be given in a public space. However, the doctor
needs to distinguish carefully between emergency care and
routine ongoing care. The latter should be given by Carrie’s GP
of choice and in the privacy of the consulting room; this Page 160
should include finding out reasons for the
hypoglycaemic episode and assessing her fitness to drive5 and
work.
Apart from an imbalance between food intake and insulin
dose, common causes of hypoglycaemia are unexpected extra
exertion, alcohol and recreational drugs. Hypoglycaemia
unawareness is more common in longer duration of diabetes,
and in men and smokers.1

References
1. Craig, ME, Donaghue, KC, Cheung, NW, Cameron, FJ, Conn, J, Jenkins,
AJ & Silink, M 2011, for the Australian Type 1 Diabetes Guidelines
Expert Advisory Group. National Evidence-Based Clinical Care
Guidelines for Type 1 Diabetes for Children, Adolescents and Adults.
Australian Government Department of Health and Ageing, Canberra.
2. Baird, A 2008, ‘Emergency drugs in general practice’, Australian Family
Physician, vol. 37, pp. 541–7.
3. The Royal Australian College of General Practitioners 2017, Managing
emergencies in general practice: A guide for preparation, response and
recovery. RACGP, East Melbourne, Vic, pp. 4–7.
4. Atkinson, MA, Eisenbarth, GS & Michels, AW 2014, ‘Type 1 diabetes’,
Lancet, vol. 383, pp. 69–82.
5. National Diabetes Services Scheme 2011, Diabetes and Driving, Diabetes
Australia.
Page 161
Section 10
Endocrinology
Page 162

Case 32
Veronica Richards

Instructions for the doctor


This is a short case.
Please take a focused history from Veronica. Then ask the facilitator for
the results of a relevant clinical examination and initial investigations.
Outline the diagnosis and your management plan to Veronica.

Scenario
Veronica Richards is 49 years old and has two teenage children. She
grew up in the Philippines and has lived in Australia for the last 30 years.
She came to you four weeks ago complaining of tiredness, at which time
you provided some lifestyle advice.

The following information is on her summary sheet:


Past medical history
Two normal deliveries
Medication
Nil
Allergies
Nil
Immunisations
Up-to-date
Social history
Housewife, trained as a teacher
Non-smoker, no alcohol.

Instructions for the patient, Veronica Richards


You are 49 years old. You grew up in the Philippines and trained as a teacher.
You have been in Australia for the past 30 years. You have two teenage
children and are happily married. You are not depressed.
You saw your doctor four weeks ago complaining of tiredness, at Page 163
which time your doctor suggested making some changes to diet and
regular exercise. They suggested coming back for review if the tiredness
continued, hence why you are here today—‘I’m still tired, Doc’. You sleep
well but don’t have the energy you used to have. You have noticed the
tiredness for about three months and it is getting worse.
Your periods are regular, not particularly heavy and you have no
menopausal symptoms.
You eat a balanced diet, including red meat two or three times a week, and
take your dog for a walk every day after the advice given last visit. You have
put on a few kilos in the past year, including a further kilo since your last
visit (despite eating healthier and exercising regularly). You have no night
sweats or changes to your thirst, urine or bowel symptoms. If asked, you have
noticed feeling the cold a little more than you usually would at this time of
year.

The following information is on your summary sheet:


Past medical history
Two normal deliveries
Medication
Nil
Allergies
Nil
Immunisations
Up-to-date
Social history
Housewife, trained as a teacher
Non-smoker, no alcohol.

Instructions for the facilitator


Please give to the doctor on specific request.

Examination
Hydration good, no cyanosis, anaemia or jaundice
Weight 70.8 kg
Height 1.62 m
BMI 27 kg/m2
Pulse 66 reg
BP 130/82 mmHg
No thyroid enlargement
No thyroid eye signs
Chest clear Page 164
Heart sounds normal
Abdomen soft, no masses, no organomegaly
Neurology—grossly normal, reflexes slow
Urine dipstick—negative.

Investigations
FBE, ESR or CRP—normal
UEC including calcium and magnesium—normal
LFTs—normal
Fasting blood sugar—normal
Urine for MCS—normal
Iron studies—normal
TSH 42.80 (0.3–4.0) mIU/L*
Free T4 8.0 (9.0–25.0) pmol/L*
Antithyroid peroxidase 345 (< 5.5) IU/mL*
Antithyroglobulin 23 (< 4.5) IU/mL*

Suggested approach to the case


Establish rapport.

History
Tiredness—character, duration
Exclude common/serious causes
— Anaemia—diet, occult bleeding, heavy periods
— Diabetes—excessive thirst, polyuria, nocturia
— Menopause—skin changes, regularity of periods, flushing, sweats
— Psychological—mood, affect, sleep, enjoyment of/satisfaction with
life, relationships, stress
— Thyroid—weight changes, skin changes, heat/cold intolerance, bowel
changes
— Occult malignancy—weight loss, cough, haemoptysis, date of last
cervical screening test, breast symptoms, bowel changes, family
history of cancer
Lifestyle factors
— Ask about exercise, smoking, alcohol, drug use
Assess impact of tiredness on her, what treatments has she tried, is there
anything she is worried about?
Request permission to examine.

Examination Page 165

General appearance
Height/weight/BMI
BP and pulse
Thyroid examination
Cardiovascular and respiratory exam
Abdominal examination
Neurological exam
Urine dipstick.

Investigations
FBE, ESR or CRP
UEC including calcium and magnesium
LFTs
Fasting blood sugar
Urine for MCS
Iron studies
Thyroid function tests, request antibodies after abnormal thyroid function
established.

Diagnosis
Hypothyroidism—auto-immune.

Management
Explain condition
Commence thyroxine
Need for regular blood tests and follow-up.

CASE COMMENTARY

Tiredness is a common problem in general practice for which


doctors need a system for assessment. This should be a stepped
approach focusing on the common causes initially.1 Australian
research shows that testing is high2 and few (3%) of patients
have a significant clinical diagnosis based on an abnormal
pathology test.3 More elaborate tests risk false positives.4 A
watchful-waiting approach prior to ordering investigations is
appropriate in the absence of red flags.5

Veronica has hypothyroidism. The doctor should explain that this Page 166
is a common problem and can be treated with replacement thyroxine.
Veronica will need ongoing follow-up to ensure that the correct dose is given.
The most common cause of hypothyroidism in Australia is autoimmune
chronic lymphocytic thyroiditis, characterised by raised circulating levels of
thyroid peroxidase antibody.6

COMMON PITFALLS

The doctor should avoid prematurely attributing the tiredness to


Veronica’s ‘time of life’. Equally it is realistic to tell patients
that the chance of finding treatable cause is low. This lays the
foundation for taking a holistic approach to address
psychological issues,7 work/life balance and expectations, sleep,
diet and exercise.8

References
1. Murtagh, J 2003, ‘Fatigue: a general diagnostic approach’, Australian
Family Physician, vol. 32, no. 11, pp. 873–6.
2. Harrison, M 2008, ‘Pathology testing in the tired patient: a rational
approach’, Australian Family Physician, vol. 37, no. 11, pp. 908–10.
3. Gialamas, A, Beilby, J & Pratt, NL et al. 2003, ‘Investigating tiredness in
Australian general practice. Do pathology tests help in diagnosis?’
Australian Family Physician, vol. 32, no. 8, pp. 663–6.
4. Koch, H, van Bokhoven, MA et al. 2009, ‘Ordering blood tests for
patients with unexplained fatigue in general practice: what does it yield?
Results of the VAMPIRE trial’, British Journal of General Practice, vol.
59, no. 561, pp. e93–100.
5. Wilson, J, Morgan, S, Magin, PJ & van Driel, M 2014, ‘Fatigue—a
rational approach to investigation’, Australian Family Physician, vol. 43,
no. 7, pp. 457–61.
6. So, M, MacIsaac, RJ & Grossmann, M 2012, ‘Hypothyroidism—
investigation and management’, Australian Family Physician, vol. 41, no.
8, pp. 556–62.
7. Dick, ML & Sundin, J 2003, ‘Psychological and psychiatric causes of
fatigue. Assessment and management’, Australian Family Physician, vol.
32, no. 11, pp. 877–81.
8. Rosenthal, TC, Majeroni, BA, Pretorious, R & Malik K 2008, ‘Fatigue:
an overview’, American Family Physician, vol. 78, no. 10, pp. 1173–9.
Page 167
Section 11
Eyes
Page 168

Case 33
Edward Galloway

Instructions for the doctor


This is a short case.
Please take a history from Edward, examine him and tell him the most
likely diagnosis and your recommended management.

Scenario
Edward Galloway is a 66-year-old retired fireman. He spends much of
his time fishing and playing golf and he has noticed increased watering
of both eyes when he goes outside. He finds this frustrating and
embarrassing and is thinking of stopping golf as sometimes it is so hard
to see the ball.
He has decided to come to his usual GP for help.
The following information is on his summary sheet:
Past medical history
Transurethral resection of the prostate two years ago
Skin graft following extensive burn to left lower leg in a fire 10 years ago
Medication
Nil
Allergies
Nil
Immunisations
Up-to-date
Family history
Both parents died in their mid-80s
Mother had glaucoma
Social history
Retired fireman
Non-smoker, alcohol consumption unknown.

Instructions for the patient, Edward Page 169

Galloway
You are a 66-year-old retired fireman. You spend much of your time fishing
and playing golf and have noticed increased watering of both eyes when you
go outside. You find this frustrating and embarrassing and are thinking of
stopping golf as sometimes it is so hard to see the ball.
You decide to see your usual GP for help.
In the latter part of her life your mother was partially sighted due to
glaucoma. You have heard that this runs in the family and are worried that
this is what’s causing the watering. You will only admit to this if the doctor is
sensitive and empathic in their approach to you.
You have difficulty seeing at times because of the watering but your
vision is 6/6 bilaterally when you wear your glasses.

The following information is on your summary sheet:


Past medical history
Transurethral resection of the prostate two years ago
Skin graft following extensive burn to left lower leg in a fire 10 years ago
Medication
Nil
Allergies
Nil
Immunisations
Up-to-date
Family history
Both parents died in their mid-80s
Mother had glaucoma
Social history
Retired fireman
Non-smoker, alcohol consumption unknown.

Suggested approach to the case


Introduction
Establish rapport
Open questions to allow Edward to tell his story.

Specific questions
Eyes watering—timing
Any discharge, eyes sticky or gritty in the morning
Exclude pain or visual disturbance
Treatments tried so far Page 170
Date of most recent optometry check-up—including screening for
glaucoma
Confirm not on any medication, check OTC medications, e.g. decongestants
Request permission to examine.

Examination
Visual acuity and fields to confrontation
Conjunctivae and sclerae
Eyelids/lashes (exclude blepharitis), if lid eversion attempted facilitator to
state this is normal and not necessary to perform
Pupil size, shape and reaction to light
Fluoroscein stain—need to ask to do but not expected to perform in the time
available
Fundoscopy—need to ask to do but not expected to perform in the time
available
All findings normal apart from mild conjunctival injection both eyes.

Most likely diagnosis


Dry eyes—age-related.

Management
Explain dry eyes1
Reassure unlikely to be glaucoma but still needs at least annual checks with
optometrist
Recommend
— Blink more
— Trial of humidifiers
— Regular use of tear ointment at night and drops during the day
— Bathing of eyelid margins to encourage flow of oily tears
— Arrange follow-up
— Future options for severe symptoms could be discussed, e.g.
tetracyclines, punctal plugs.2

CASE COMMENTARY

Dry eyes are a common problem in the elderly that can readily
be diagnosed and treated in general practice. The GP will be able
to get Edward back to regular golf and reassure him that these
symptoms do not indicate that he is getting glaucoma. Page 171
However, Edward will require regular optometry checks on his
eye pressure and visual fields.
As people age, less oily tears are produced. This reduced
lubrication is felt particularly when outside, in arid areas and in
air-conditioned rooms. The surface of the eye senses the dryness
and reflex tearing (watery tears) occurs. Treatment is aimed at
optimising the production of oily tears by bathing the eyelids,
clearing any duct blockages and replacing the oily tears with
artificial tears.
It is counter-intuitive to offer artificial tears to someone with
watery eyes but is a simple way of improving someone’s quality
of life. Sometimes the preservatives in eye drops can irritate
eyes further. If this is the case, or if frequent use is required, a
preservative-free preparation should be recommended.3

COMMON PITFALLS

Edward reports that he sometimes cannot see when he is playing


golf. It is easy to assume that this is a problem with his visual
acuity when in fact it is just that the excess watery tears reduce
his vision. Careful questioning and assessment of visual acuity is
needed to clarify the problem.

References
1. Hodge, C & Sutton, G 2003, ‘Dry eyes: eye series 3’, Australian Family
Physician, vol. 32, no. 4, pp. 265–6.
2. Lemp, MA 2008, ‘Advances in understanding and managing dry eye
disease’, American Journal of Ophthalmology, vol. 146, no. 3, pp. 350–6.
3. British Medical Journal 2016, ‘The management of dry eye’, vol. 353,
pp. i2333.
Page 172

Case 34
Henrik Schneider

Instructions for the doctor


This is a short case.
Please take a focused history from Henrik and look at a photograph of his
eyes. (Refer to Figures 6 and 7, centre insert page D.) Please describe the eye
signs on the photograph to the facilitator. You can ask the facilitator for other
examination findings. You are expected to tell Henrik the most likely
diagnosis and outline your initial management.

Scenario
You are working as a GP locum at a well-equipped clinic five hours
drive away from the nearest base hospital. An Austrian tourist Henrik
Schneider, aged 25, telephones you at 6.30 am saying he needs an urgent
consultation for his painful, red, blurry right eye and you agree to see
him. He is leaving in an hour for the town where the base hospital is
located.
Henrik does not take any medication, has no known allergies and no
significant past medical history.

Instructions for the patient, Henrik Schneider


You are a 25-year-old student backpacking around Australia. Yesterday your
right eye became painful, the vision was a bit blurred and the eye looked red
when you looked in the mirror at the hostel. You thought it might be due to
your contact lenses, and so started to wear your glasses.
This morning you are due to travel five hours in a coach to the nearest
town with a hospital. However, your eye is now more painful and the vision
more blurred so you decide to see the resident GP as soon as possible. Bright
lights make the eye more painful.
You have not had this eye problem before and have not injured the Page 173
eye or got anything in it.
You do not take any medication, have no known allergies and have no
significant past medical history.

Instructions for the facilitator


Clinical examination findings
When the doctor asks to examine the eye, please show them the photographs.
(Refer to Figures 6 and 7 (with fluorescein), centre insert page D.)
Visual acuity
— Left eye 6/12 uncorrected, 6/6 corrected through pinhole
— Right eye 6/18 uncorrected, 6/18 corrected through pinhole
Pupils—as per photograph, photophobia on examination
Eversion of lids: no foreign body.

Suggested approach to the case


Establish rapport
Open-ended questions to explore Henrik’s ideas, concerns and expectations.

Specific questions
Pain
Vision
Redness
Photophobia
Exclude pus-like discharge
Exclude itching
Exclude foreign body or eye injury
Confirm normally wears contact lenses
Previous history of eye problems
Request permission to examine.

Examination
Visual acuity
Observe photograph, candidate should note
— Distribution of erythema—circumcorneal
— Anterior chamber clear/no hypopyon
— Cornea—no fluorescein uptake (excludes dendritic ulcer or Page 174
corneal abrasion secondary to extruded foreign body or
contact lens problem)
— Pupil size and reactions to light—pupil smaller on right and irregular
Check for foreign body, including everting upper lids
Check for photophobia.

Most likely diagnosis


Iritis (acute anterior uveitis) of right eye.

Management
Explain condition
Discuss with ophthalmologist at base hospital and arrange referral and review
Commence topical steroids if available
Not to wear contact lenses until ophthalmologist confirms it is safe to do so.

CASE COMMENTARY

‘Beware the unilateral red eye.’ Most common eye problems in


general practice affect both eyes so the history of a unilateral red
eye is significant. Henrik also has the three sentinel warning
signs of pain, photophobia and blurred vision. The presence of
one or more of these symptoms should prompt urgent
assessment. Misdiagnosis of anterior uveitis/iritis is common,
and a delay of even a few days can lead to irreversible vision
loss.
The doctor should ask about a history of a foreign body or
eye injury. Contact lenses increase the risk of corneal ulcers,
which can become infected.
A pinhole creates a very narrow light path that requires little
focusing prior to hitting the retina and so demonstrates the
potential corrected visual acuity. Henrik’s reduced vision in the
right eye is not corrected by using a pinhole. This shows the loss
of vision in the right eye is not due to a refractive error, so the
doctor must look carefully for an ocular problem.
Starting topical steroids in the eye is potentially dangerous
and it is essential that the doctor excludes a herpetic cause of the
problem before commencing treatment, specifically by Page 175
looking for a dendritic ulcer on fluorescein staining. If
possible discuss the case with a specialist ophthalmologist.
However, the distance between the patient and the base hospital
means that the GP needs to start steroids to prevent further
exacerbation of the iritis.

Further reading
Cronau, H, Kankanala, RR & Mauger, T 2010, ‘Diagnosis and management
of red eye in primary care’, American Family Physician, vol. 81, no. 2,
pp. 137–44.
Durkin, SR & Casey, TM 2005, ‘Beware of the unilateral red eye: don’t miss
blinding uveitis’, Medical Journal of Australia, vol. 182, no. 6, pp. 296–
7.
Statham, MO, Sharma, A & Pane, AR 2008, ‘Misdiagnosis of acute eye
diseases by primary healtrh care provides: incidence and implications’,
Medical Journal of Australia, vol. 189, no. 7, pp. 402–4.
Page 176

Case 35
Roger Chin

Instructions for the doctor


This is a short case.
Please take a focused history. Request the findings of an appropriate
physical examination. Explain your management to Roger.

Scenario
Roger Chin is a 57-year-old financial planner who is a regular patient of
the practice. He is generally healthy and is on perindopril (Coversyl) for
hypertension. He is happily married with three grown children and two
grandchildren. Along with most of his family he does get classic
migraines but rarely in recent years. He called early this morning and
asked to be seen urgently.
The following information is on his summary sheet:
Age
57
Past medical history
Hypertension
Migraine with aura
Medication
Perindopril 5 mg mane
Eletriptan (Relpax) 40 mg stat prn
Allergies
Nil known
Immunisations
Up-to-date
Social history Page 177
Married to Dorothy
Non-smoker
Occasional alcohol
Family history
Hypertension
Migraine.

Instructions for the patient, Roger Chin


Last night you noticed intermittent bright flashes in the temporal field of your
left eye, ‘a bit like lightning bolts’. It was provoked by moving your eye and
was worse when you turned the light out. You were looking out the window
and thought it was lightning but then noticed it inside the house as well.
The bright flashes have gone, but you now have a blurry floating opacity
in the middle of your left eye’s vision. It wafts around with the movement of
your eye. You called the surgery for an urgent appointment as you were
worried.

In answer to specific questioning:


There is no pain; no headache; you have not had this before. The only trouble
you have had with your eyes is that you are short-sighted and wear glasses.
They were last checked about 18 months ago and your prescription adjusted
slightly. There has been no trauma or injury. Apart from your ‘floater’ you
are not aware of any visual problem. Your migraine auras generally consist of
bright zigzag lines in your vision that disappear after 10–15 minutes: they are
unlike what you experienced last night.
If ophthalmological review is suggested, say something like: ‘My office is
really busy at the moment. Can it be delayed until next week?’
Ask what the doctor thinks is going on and (if referred) what the specialist
will do. If surgery is mentioned, ask if you will need a general anaesthetic or,
if not, whether it will hurt.

The following information is on your summary sheet:


Age
57
Past medical history
Hypertension
Classic migraine
Medication
Perindopril 5 mg mane
Eletriptan (Relpax) 40 mg stat prn
Allergies Page 178
Nil known
Immunisations
Up-to-date
Social history
Married to Dorothy
Non-smoker
Occasional safe alcohol
Family history
Hypertension
Migraine.

Suggested approach to the case


The key tasks are to take a focused history to separate the different causes of
flashes and floaters, followed by a relevant physical examination, and a
simple, clear explanation of the likely condition and its management for the
patient. The most likely diagnosis in this setting is posterior vitreous
detachment (PVD) with a possible retinal tear and associated vitreous
haemorrhage. Prompt treatment is required to prevent progression to retinal
detachment. Candidate’s explanation should include the distinction between
PVD and retinal detachment.
Roger’s requests to defer because of a busy work schedule should be met
with a clear explanation of the possible implications of deferring treatment.
Points to address in the history include:
• details of the flashes and floater
• visual change especially field loss
• pain
• risk factors (myopia)
• brief general health screen.
Physical examination should include:
• general appearance
• vital signs
• visual acuity +/− glasses
• visual fields
• pupil response for relative afferent pupillary defect.
Management should include:
• explanation of the significance of his symptoms and the likely diagnosis
i.e. PVD with vitreous haemorrhage and possible retinal tear. The risk
and significance of retinal detachment should be discussed
• urgent referral (same day) to ophthalmologist Page 179
• an outline of the likely treatment i.e. retinal tear surrounded with
laser burns to create a chorio-retinal scar that prevents fluid seeping into
the sub-retinal space. This can be as an outpatient under topical
anaesthesia and is relatively painless
• follow-up and safety netting.

Physical examination
General appearance: ‘As you see him’
Vital signs
— HR 70/min regular
— BP 130/78 mmHg
— Temp 36.1°C
— BMI 26 kg/m2
— RR 12/min
Eye examination: (each item to be asked for individually)
Appearance: normal
Red reflex present bilaterally
Pupils equal, round and reactive to light and accommodation with no relative
afferent pupil defect
Visual acuity
— Uncorrected: L = R = both = 6/24
— Corrected: L = 6/12; R = 6/7.5 both = 6/7.5
Visual fields by confrontation: normal
Fundoscopy (un-dilated pupil) is unremarkable
Cardiovascular exam: normal
Remainder of physical examination is normal
Surgery tests: BSL (random) 5.6 mmol/L
U/A: normal.

CASE COMMENTARY

Acute onset of monocular floaters and/or flashes commonly


present to GPs and the most likely diagnosis in this setting is
posterior vitreous detachment (PVD). Although the majority of
cases of PVD are benign, a significant proportion of patients
with acute PVD develop an associated retinal tear that can lead
to retinal detachment and permanent vision loss if left untreated.
The sudden appearance of Roger’s floater following Page 180
immediately after his flashes is strongly suggestive of a
retinal tear and associated vitreous haemorrhage. If retinal tears
are not treated, one-third to one-half of cases will progress to
retinal detachment, which can result in permanent visual loss.1, 2
The role of GPs is to make the diagnosis of probable PVD
and to identify patients at increased risk of retinal tear and
detachment based on history and physical examination to
determine the urgency of ophthalmologic assessment.
As a minimum approach, candidates should take a history of
change in vision, check visual acuity, and assess visual fields.
Having recognised Roger’s high-risk features for retinal tear he
should have same-day assessment by an ophthalmologist. PVD
is a common degenerative condition and should not be confused
with retinal detachment. The explanation to Roger should
demonstrate an understanding of the pathophysiology and
difference between the two conditions. An intimate knowledge
of the treatment of retinal tears is not required but Roger’s
simple questions about what is involved should be answered.
If Roger is subsequently found to have an uncomplicated
PVD, he is at risk of subsequently developing a retinal tear and
detachment over the next several weeks and requires follow-up.
One variation on the case would be to make it a long case and
require candidates to conduct the clinical examination.

COMMON PITFALLS

Eye symptoms may represent relatively benign conditions or


serious conditions requiring urgent care. Taking a further history
of eye complaints like flashes and floaters can help establish the
diagnosis and therefore the urgency of further treatment if
required. Not all floaters and/or flashes represent ocular
problems, and non-ocular causes can usually be identified by
taking a careful history. Roger’s past history of migraine with
aura represents a possible masquerade in this case and should be
excluded based on the history.
Roger’s normal visual acuity, fields and normal eye
examination findings are consistent with PVD with associated
vitreous haemorrhage and retinal tear, and should not delay
urgent referral.
Many GPs are aware that there are serious conditions among
these presentations and therefore are quick to refer without
properly exploring the history or performing an appropriate
physical examination. Patient explanation should clearly
distinguish PVD from a retinal tear and retinal detachment.

References Page 181

1. Kahawita, S, Simon, S & Gilhotra, J 2014, ‘Flashes and floaters. A


practical approach to assessment and management’, Australian Family
Physician, April, vol. 43, no. 4, pp. 201–3. Available at:
www.racgp.org.au/afp/2014/april/flashes-and-floaters/, accessed 25
February 2019.
2. Hollands, H, Johnson, D, Brox, AC, Almeida, D, Simel, DL & Sharma, S
2009, ‘Acute-onset floaters and flashes: is this patient at risk for retinal
detachment?’, Journal of the American Medical Association, 25
November, vol. 302, no. 20, pp. 2243–9. Page 182
Page 183
Section 12
Gastroenterology
Page 184

Case 36
Jenna Banks

Instructions for the doctor


This is a short case.
Please take a history from Jenna and then ask the facilitator for the results
of a focused examination. Tell Jenna the most likely diagnosis and your
initial management plan.

Scenario
Jenna Banks is a 25-year-old tax manager studying for her chartered
accountancy exams. You last saw her for contraception. She is coming to
see you today because of recurrent abdominal pain and bloating.

The following information is on her summary sheet:


Past medical history
Fracture left humerus aged eight, from a rollerblading accident
Medication
On contraceptive pill
Levonorgestrel 150 mcg, ethinyloestradiol 30 mcg (Levlen ED)
Allergies
Nil known
Immunisations
Up-to-date
Cervical screen
Normal a few months ago
Family history
Parents both alive and well
Social history
Non-smoker
Alcohol—two standard drinks on two occasions per week.

Instructions for the patient, Jenna Banks Page 185

You are a 25-year-old tax manager studying for your chartered accountancy
exams. Your abdominal pains started after you contracted traveller’s
diarrhoea during a trip to Bali six months ago. You saw a GP when your pain
symptoms weren’t improving and stool tests were negative for infection.
Things have been worsening over the last few months as your stress
regarding your accountancy exams has increased. Most days of the week you
experience bouts of abdominal pain, bloating and flatus. The pain can be so
severe that you need to lie down but mostly you can continue at work. The
pain comes on at any time during the day, and is a squeezing, cramp-like
pain, usually in the left or right side of your abdomen. The pain is sometimes
relieved by defecating. You have never woken at night with symptoms.
You have not lost weight, been nauseated or vomited. Your bowel habit
alternates between constipation (hard, rabbit-like pellets) and diarrhoea. You
do not pass any blood or mucus rectally. You have not noticed any particular
food triggers. You have tried some herbal remedies which did not improve
your symptoms.
You are stressed and anxious regarding your exams, but are not depressed.
You have no past history of anxiety or depression.
Your periods are regular on the contraceptive pill, and your bowel
symptoms do not worsen with your menstrual cycle.
You want the doctor to make sure that you have nothing seriously wrong
with you and to tell you what the problem is.

The following information is on your summary sheet:


Past medical history
Fracture left humerus aged eight, from a rollerblading accident
Medication
Levonorgestrel 150 mcg, ethinyloestradiol 30 mcg (Levlen ED)
Allergies
Nil known
Immunisations
Up-to-date
Cervical screen
Normal a few months ago
Family history
Parents both alive and well
Social history
Non-smoker
Alcohol—two standard drinks on two occasions per week.

Instructions for the facilitator Page 186

All physical examination findings are normal. Please give the following
clinical examination findings on specific request:
• temperature, pulse, BP all normal
• looks well
• no hand or nail changes
• no jaundice or anaemia
• abdominal examination—no tenderness, no masses, normal rectal
examination.
Suggested approach to the case
Establish rapport
Open questions to explore Jenna’s ideas, concerns and expectations.

Specific questions
Pain
Bowel habit, including symptoms of urgency and faecal incontinence
Bloating
Exclude PR blood or mucus
Flatulence/belching
Nocturnal symptoms
Explore specific dietary triggers for symptoms, e.g. lactose, wheat/gluten,
onions/garlic
Exclude other systemic symptoms including weight loss
Menstrual history—exclude relationship of symptoms to menstrual periods
Check current medication use, including over-the-counter and
complementary medicines
Recent travel or gastroenteritis
Determine impact of symptoms on Jenna’s life and impact of stress on
symptoms
Exclude relevant family history, e.g. bowel cancer, inflammatory bowel
disease
Request permission to examine.

Examination
Hands, face, mouth—normal
Abdominal and rectal examination—normal.

Most likely diagnosis


Irritable bowel syndrome.

Management Page 187


Provide explanation of the condition and reassurance, without minimising the
impact of symptoms on quality of life
Targeted investigations: FBE, iron studies, ESR or CRP, coeliac serology
including IgA
Suggestions for symptomatic management
— Modify diet—trial of increase in soluble fibre, trial of low FODMAPs
diet
— Increase exercise
— Reduce volume of carbonated drinks
— Medication—antispasmodics (including peppermint oil or
mebeverine), psyllium/ispaghula if constipation predominant,
probiotics if diarrhoea or bloating predominant, tricyclic
antidepressants1
— Psychological and behavioural therapy, such as cognitive behavioural
therapy, relaxation therapy and mindfulness
Referral—consider referral to allied health professionals to help address
above recommendations. Specialist gastroenterologist referral not
required in the absence of red flags
Arrange follow-up.

CASE COMMENTARY

Irritable bowel syndrome (IBS) is a common problem that


accounts for 0.3 per 100 patient presentations in Australian
general practice.2 IBS is diagnosed using the Rome IV criteria,
based on the clinical features of recurrent abdominal discomfort
or pain related to defecation or associated with altered bowel
habits. Symptoms must be chronic, occurring at least once per
week, on average in the previous three months, with a duration
of at least six months.1
The candidate should enable Jenna to tell her story about the
symptoms, and check that there are no alarm symptoms, such as
weight loss, nocturnal symptoms or rectal bleeding. The
examination should focus only on a check of the hands, the face
and the abdomen. Limited investigation to exclude relevant
differential diagnoses (including coeliac disease) is appropriate.
A poor candidate may initiate a large number of investigations
or suggest early referral to a gastroenterologist.
Traditionally, IBS has been a diagnosis of exclusion, however
current recommendations suggest that in a patient who has
symptoms meeting Rome IV criteria without alarm Page 188
symptoms, a positive diagnosis should be made without
resorting to a battery of tests.2, 3 Early provision of a diagnosis
which is clearly communicated is important to avoid a prolonged
search for an alternative explanation and delay of effective
management.3
The pathophysiology of IBS is complex and heterogeneous.
Postulated mechanisms include disorders of the gut–brain axis;
diet; genetic factors; infections and disturbances in the intestinal
microbiota; low-grade mucosal inflammation, immune
activation, and altered intestinal permeability; disordered bile
salt metabolism; abnormalities in serotonin metabolism; and
alterations in brain function.4 While traditionally conceptualised
as a brain–gut disorder due to the high association with
coexisting psychological conditions and childhood trauma, it has
now been shown that in at least a subset of patients the
gastrointestinal changes happen prior to psychological distress
developing later, suggesting the association happens both ways.
Intestinal inflammation, the cytokine response and the
microbiome have been suggested to precipitate such gut-to-brain
changes in IBS.1, 4
The heterogeneity in both the pathogenesis and the
symptomatology of IBS makes a ‘one size fits all’ approach to
management difficult. A modified exclusion diet and stepwise
reintroduction of foods containing gluten or fermentable dietary
oligosaccharides, disaccharides, monosaccharides and polyols
(FODMAPs) may be useful. This is best done under the
guidance of a dietician.5 Soluble fibre such as psyllium can help
some patients (noting that insoluble fibres such as bran can
exacerbate pain and bloating).1 Antispasmodics (including
peppermint oil and mebeverine), probiotics, tricylic
antidepressants, exercise and psychological therapies can be
effective in the treatment of IBS.1, 3

References
1. Ford, AC, Lacy, BE & Talley, NJ 2017, ‘Irritable bowel syndrome’, New
England Journal of Medicine, vol. 376, pp. 2566–78.
2. Charles, J & Harrison, C 2006, ‘Irritable bowel syndrome in Australian
general practice’, Australian Family Physician, vol. 35, pp. 840–1.
3. Linedale, EC & Andrews, JM 2017, ‘Diagnosis and management of
irritable bowel syndrome: a guide for the generalist’, Medical Journal of
Australia, vol. 207, pp. 309–15.
4. Holtmann, GJ, Ford, AC & Talley, NJ 2016, ‘Pathophysiology of irritable
bowel syndrome’, Lancet Gastroenterology & Hepatology, vol. 1, pp.
133–46.
5. Bolin, T 2009, ‘IBS or intolerance?’, Australian Family Physician, vol.
38, pp. 962–5.
Page 189

Case 37
Enrico Castallani

Instructions for the doctor


This is a long case.
Please take a history, conduct an appropriate examination and then outline
your management plan to Mr Castallani.

Scenario
Enrico Castallani is the licensee of the local hotel. He rarely comes to the
surgery, although you know that he has been in town for over two years
now. Today he has asked for an urgent appointment because of bad
abdominal pain. You agree to see him and note the following
information on his medical records.

The following information is on his summary sheet:


Past medical history
MVA 2015—sustained no injuries, cause of accident unclear
Medications
Nil recorded
Allergies
Nil known
Immunisations
Nil known
Social history
Local hotelier
Lives with de facto partner.

Instructions for the patient, Enrico Castallani


You have been the licensee of a hotel for the last two years. You rarely see
the doctor. Today you have asked for an urgent appointment because of bad
abdominal pain.
For the last 24 hours you have experienced constant severe Page 190
abdominal pain, which radiates through to your back. The pain has
been so bad that you can’t work or walk. You have come today because your
partner has persuaded you. You have been vomiting and are off your food.
You cannot get comfortable, and you are sweating and feel generally weak.
You have not vomited blood and have no symptoms or signs of liver
failure. You are very sensitive about your alcohol problem and will only open
up about this if the doctor approaches the issue in a non-judgemental manner.
You have a long-term drinking problem. Both your parents were
alcoholics and although you saw the impact of alcohol on them, you started
drinking at a young age. You have had a series of jobs working in the
hospitality industry. You left your last job in a hurry as you had a car
accident while drunk. No one was injured in the crash, but there was
significant damage to the car and public property; the police were never able
to establish a cause, but you left the town to escape suspicion.
For the past few months you have switched to drinking port rather than
wine. You have about a cask a day.
The following information is on your summary sheet:
Past medical history
MVA 2015—sustained no injuries, cause of accident unclear
Medication
Nil recorded
Allergies
Nil known
Immunisations
Nil known
Social history
Local hotelier
Lives with de facto partner.

Instructions for the facilitator


This is a case of acute pancreatitis. Please give the candidate the following
examination findings on specific request.

Clinical examination findings


Temperature 38.0°C, sweating
Pulse 110 regular
BP 116/80 mmHg
Abdomen—guarded, with tenderness maximal in periumbilical region.

Suggested approach to the case Page 191

Establish rapport
Open questions about abdominal pain.

Specific questions
Nature of pain and radiation
Absence of haematemesis
Vomiting and nausea—present
Absence of melaena or blood in stool or change in bowel habit
Absence of symptoms of liver disease—no jaundice
Absence of symptoms of gallstones
Explore alcohol consumption and motivation for change—negotiate a follow-
up plan
Discuss need for overall health check at some stage
Request permission to examine.

Examination
Temperature 38.0°C, sweating BP 116/80 mmHg
Pulse 110
Height 1.78 m
Weight 92 kg
BMI 29 kg/m2
Hands normal, no liver flap, no jaundice
Examination of abdomen
— Severe abdominal pain with guarding, rebound and rigidity
— Pain hypogastrium
— Absence of ascites, hepatomegaly
— Bowel sounds—reduced
Lungs clear
BSL 4.5 mmol/L
Urine—NAD.

Management
Arrange analgesia
Nil by mouth
Admission to hospital—transfer by ambulance essential
Commence intravenous fluids
Give oxygen
Explain to patient procedure for admission and rationale.

Initial investigations Page 192


Lipase levels; complete blood count with differential; biochemistry (blood
urea, creatinine, glucose, liver function tests and calcium levels); triglyceride
level; urinalysis; and arterial blood gases and imaging (ultrasound scan +/−
CT/MRI)1, 2
Explore social impact of admission—who will run the pub?

CASE COMMENTARY

The doctor is expected to make the provisional diagnosis of


pancreatitis and arrange for urgent admission to hospital via
ambulance. A useful acronym for possible causes of pancreatitis
is GET SMASHED.3
Gallstones
Ethanol
Trauma
Steroids
Mumps
Auto-immune
Scorpion venom
Hyperlipidaemia, hypothermia, hypercalcaemia
ERCP (endoscopic retrograde cholangiopancreatography)
and emboli
Drugs, e.g. azathioprine
The commonest causes in the Australian population are
alcohol and gallstones.
A good doctor will establish rapport with Mr Castallani and
explore his alcohol consumption in a non-judgemental manner.
The CAGE questionnaire is a useful tool for diagnosing
alcoholism. The questions focus on Cutting down, Annoyance
by criticism, Guilty feeling and Eye-openers.4 Arrangements for
another appointment should be suggested to discuss Mr
Castallani’s drinking. Even if the admission shows that the
pancreatitis was due to another cause, such as gallstones, this
level of drinking is harmful.

References
1. Basnayake, C & Ratnam, D 2015, ‘Blood tests for acute pancreatitis’,
Australian Prescriber, vol. 38, no. 4, pp. 128–30.
2. Nesvaderani, M, Eslick, G & Cox, M 2015, ‘Acute pancreatitis: update
on management’, Medical Journal of Australia, vol. 202, no. 8, pp. 420–
3.
3. Wilkinson, I, Raine, T, Wiles, K, Goodhart, A, Hall, C & O’Neill, H
2017, Oxford Handbook of Clinical Medicine, 10th ed, University Press,
Oxford.
4. Ewing, J 1984, ‘Detecting alcoholism: the CAGE questionnaire’, Journal
of the American Medical Association, vol. 252, pp. 1905–7.
Page 193

Case 38
Kirrilee DeMarco

Instructions for the doctor


This is a long case.
Please take a history and conduct an appropriate examination. The
observing examiner will give you specific examination findings and the
results of your initial investigations on request.
Discuss your differential diagnosis and negotiate a management plan with
Kirrilee.

Scenario
Kirrilee is a 23-year-old fashion designer. She presents to you with a
three-week story of abdominal pain.
She is a new patient at the practice and so no past medical history is
available.
Instructions for the patient, Kirrilee DeMarco
You are a 23-year-old fashion designer. You live with your female partner,
Natalie, who is a nurse.
In the last three weeks you have had abdominal pain. At times the pain is
severe. Since the pain started, you have needed to rush to the toilet, and have
been passing loose bowel motions at least three times a day. On a couple of
occasions, you have been woken from sleep with pain and a need to pass
bowel motions. Last week you noticed some blood mixed in with the stool
and that was when you decided to make a doctor’s appointment.
You feel the pain in your right iliac fossa. It is worse after you eat.
You are nauseated but have not vomited. You have lost your appetite and
have two painful ulcers in your mouth. You have lost 2 kg in weight and feel
weak. You have not had fevers.
Your periods are generally regular and non-painful. Your last Page 194
period was last week and normal. There is no vaginal discharge. You
have no dysuria, frequency or haematuria. You have no history of any eye
issues or joint pains.
You work in the city as a fashion designer. You and Natalie have been
together since you met at university.
You have never smoked. Most Saturday nights you drink four to five
glasses of wine.
You recently visited your brother on the central New South Wales coast
but have not travelled overseas.

Medication
Nil prescribed
A friend gave you ginger tablets from the herbal shop but they had no
effect, nor did the ranitidine or antacids that you bought from the chemist
Allergies
You have no allergies
Family history
Both your parents are alive and well
Clinical examination findings
Tenderness without guarding to be demonstrated in the right iliac fossa.
Instructions for the facilitator
Clinical examination findings
To be given if requested/examination demonstrated:
Temperature 37.0°C
Hands normal
BP 114/80 mmHg
Pulse 64
Height 171 cm
Weight 76 kg
BMI 26 kg/m2
No jaundice or pallor
Mouth ulcers × two
Examination of chest and cardiovascular system—normal
Examination of abdomen
— Abdominal pain without guarding in right iliac fossa
— Bowel sounds—normal
— Rectal examination and proctoscopy—normal
Urine dipstick—NAD.

Investigations Page 195


To be given to the candidate on specific request by the observing
examiner.
Full blood count
Haemoglobin 116 (115–155) g/L
MCV 85 (82–99) fL
RDW 14 (<16) %
Platelets 311 (150–400) × 109/L
White cells 10.3 (4.0–11) × 109/L
Neut 70% 7.2 (1.8–7.5) × 109/L
Lymph 22% 2.27 (1.0–4.0) × 109/L
Mono 6% 0.41 (0.1–1.2) × 109/L
Eosin 2% 0.2 (<0.7) × 109/L
ESR 43 mm/hr
CRP 19 (<10) mg/L
Electrolytes/liver function tests
Na 136 (134–146) mmol/L
K 4.2 (3.4–5.5) mmol/L
Cl 106 (95–108) mmol/L
HCO3 24 (22–32) mmol/L
Urea 4.9 (3.0–8.0) μmol/L
Creatinine 54 (30–100) μmol/L
Bilirubin 11 (<16) μmol/L
ALP 42 (20–105) μmol/L
GGT* 49 (<36) μmol/L
ALT 15 (<31) μmol/L
Albumin 42 (38–50) g/L
Total protein 74 (65–85) g/L

Ironstudies
Iron 9 (9–30) umol/L
Transferrin 3.2 (2.0–3.6) g/L
T. saturation* 11 (15–45) %
Ferritin* 240 (10–80) ug/L

Coeliac serology
— Deamidated gliadin IgG/tissue transglutaminase IgA both negative,
normal IgA level
Stool tests
— Microscopy—semi-formed faecal matter
— Normal faecal bacterial growth, pcr negative
— No ova, cysts or parasites Page 196
— Viral pcr negative
— Clostridium difficile toxin negative.

Suggested approach to the case


Establish rapport
Open questions about abdominal pain
Closed questions to determine cause and exclude differentials
— Nature of pain, relieving and alleviating factors
— Appetite—decreased
— Nausea—present, but no vomiting
— No haematemesis
— Blood in stool and change in bowel habit
— Nocturnal symptoms
— Weight loss, fevers
— Absence of symptoms of liver disease—no jaundice
Absence of symptoms of gallstones
Genitourinary symptoms—no dysuria, no haematuria, no frequency, no
discharge
LMP last week, female partner, pregnancy not possible
No history of painful or heavy periods
Travel—nil relevant
Medication—has tried ranitidine and antacids and ginger tablets
Allergies—nil known
Family history—nil known.

Examination
Vital signs including temperature, BP/pulse
Height, weight and BMI
General inspection for pallor and jaundice
Hand signs
Cardiovascular/respiratory examination
Examination of abdomen
— Palpation, auscultation, percussion
— Ask to do rectal examination and proctoscopy
Urine dipstick.

Initial investigations
FBC
ESR or CRP
UEC Page 197
LFTs
Iron studies
Coeliac serology
Stool for MCS/pcr, ocp, viral pcr and clostridium difficile toxin.

Summary of key findings


Abdominal pain, nausea, reduced appetite, weight loss, mouth ulcers, rectal
bleeding
Results—confirm inflammation, no infectious cause
No features of severe acute disease such as hypotension, tachycardia or fever.

Differential diagnosis
Crohn’s disease
Ulcerative colitis
Appendicitis—much less likely given symptom duration
Irritable bowel syndrome—in the presence of alarm symptoms the above
differentials need exclusion prior to making this diagnosis.

Management
Explanation of possible cause and plan of investigation
Recommend referral for colonoscopy
Phone contact with a gastroenterologist may be appropriate to ensure early
review
Arrange follow-up—with result of colonoscopy
Discuss safety-netting to ensure early review if symptoms worsen
Discuss impact of illness on ability to work, assess need for medical
certificate
High alcohol intake at weekends, raised GGT—motivational interviewing
regarding willingness/ability to consider change.

CASE COMMENTARY
Abdominal pain is a common problem with a wide differential
diagnosis. A thorough history and examination in this case
should give the doctor the clues needed to consider Crohn’s
disease in the differential diagnosis. More common causes of
pain and bleeding, such as infection, need to be excluded, prior
to referral for endoscopy.
It is estimated that over 75 000 people are living with Page 198
inflammatory bowel disease in Australia, with over 1622
new cases being diagnosed every year. Inflammatory bowel
disease occurs at any age, with a typical age of onset in the
twenties. It is a chronic disease with a high degree of morbidity
and commonly affects young people at a time when they are
trying to establish careers and relationships. Psychological co-
morbidity is common, and it is important to consider the impact
of this diagnosis on Kirrilee at future follow-up consultations.

COMMON PITFALLS

Pregnancy should be considered in the differential diagnosis but


can be discounted when Kirrilee states she has a female partner
and has never had heterosexual intercourse.
Faecal calprotectin may be considered by some candidates as
part of investigations. Faecal calprotectin is a useful test due to
its high negative predictive value in the absence of alarm
symptoms. In this case, Kirrilee has alarm symptoms including
PR bleeding, weight loss and nocturnal symptoms, which
necessitate endoscopy irrespective of this result. It is worth
noting that faecal calprotectin does not have a Medicare rebate,
leading to a cost to the patient if this is requested.
Likewise, faecal occult blood testing is useful as a screening
test for asymptomatic patients and is inappropriate in a
presentation of overt PR bleeding, as a negative or positive
result will not change management.
Further reading
Alex, G, Andrews, JM, Bell, S, Connor, S, Moore, G, Ward, M & van
Langenberg, D 2018, ‘Clinical update for general practitioners and
physicians: inflammatory bowel disease’, Gastroenterological Society of
Australia, Melbourne. Available at: www.gesa.org.au/resources/clinical-
guidelines-and-updates/inflammatory-bowel-disease, accessed 9
December 2018.
Knight, A 2008, ‘I’ve been bleeding from the bowel’, Australian Family
Physician, vol. 37, no. 11, pp. 918–21.
Morrison, G, Headon, B & Gibson, P 2009, ‘Update in inflammatory bowel
disease’, Australian Family Physician, vol. 38, no. 12, pp. 956–61.
Mozdiak, E, O’Malley, J & Arasaradnam, R 2015, ‘Inflammatory bowel
disease,’ British Medical Journal, vol. 351, p. h4416.
Page 199

Case 39
Mohammed Noor

Instructions for the doctor


This is a long case.
Please take a history and conduct an appropriate physical examination.
The facilitator will give you the results of initial tests on request. Outline the
most likely and differential diagnoses to Mohammed and negotiate a
management plan with him.

Scenario
Mohammed Noor is a 23-year-old student studying business at
university.
He presents today complaining of epigastric burning.

The following information is on his summary sheet:


Past medical history
Nil recorded
Medication
Nil recorded
Allergies
Nil known
Immunisations
Up-to-date
Social history
Lives with mother and five younger siblings
Studying business
Rohingya refugee born in Burma, spent two years in refugee camp in
Bangladesh, arrived in Australia 10 years ago
Alcohol—nil
Non-smoker
Family history Page 200
Mother—migraines
Father killed in Burma prior to rest of family fleeing to Bangladesh.

Instructions for the patient, Mohammed Noor


You are 23 years old and are studying a business degree at university. You
are a Rohingya refugee and arrived in Australia 10 years ago. You speak
English very well. You completed high school here, achieving good marks,
and have almost finished your university studies. Over the past month or so,
you have noticed some indigestion after meals. Particularly after spicy foods,
you notice burning pain in your upper abdomen. It seems to be worse on
university days when you drink three cups of coffee and feel more stressed.
Indigestion tablets that you bought from the pharmacy have helped a little to
relieve the symptoms. You have felt a little nausea but have not vomited. You
have noticed a need to burp associated with times you are feeling pain, which
is embarrassing when in a group.
Your bowels are unchanged and there is no rectal bleeding or black
motions. You have no trouble swallowing and no sensation of reflux or bitter
taste in your mouth. You have not lost any weight (and in fact have probably
gained a few kilos over the last couple of years due to eating at the university
cafeteria). You do not take any over-the-counter medications other than the
indigestion tablets (including no anti-inflammatories).
You are fit and active, playing soccer twice per week with no shortness of
breath or chest pain. Your diet consists of traditional food at home (often
heavy on the chilli which you grow in your garden) and fast food from the
university cafeteria.
You are worried as a friend’s mother just passed away from stomach
cancer, which has shocked your small community as she was relatively
young. You remember her always complaining of symptoms after eating
spicy foods, like yourself. You have no personal family history of gastric
cancer.

The following information is on your medical record:


Past medical history
Nil recorded
Medication
Nil recorded
Allergies
Nil known
Immunisations
Up-to-date
Social history Page 201
Lives with mother and five younger siblings
Studying business
Rohingya refugee from Burma, two years in refugee camp in
Bangladesh, arrived in Australia 10 years ago
Alcohol—nil
Non-smoker
Family history
Mother—migraines
Father killed in Burma prior to rest of family fleeing to Bangladesh.

Examination
You display epigastric tenderness on examination of the abdomen. Physical
examination is otherwise normal.

Instructions for the facilitator


Examination
BP 123/74 mm Hg
P 65 bpm
Height 165 cm
Weight 78 kg
BMI 28.7 kg/m2
The patient should demonstrate epigastric tenderness. Physical examination is
otherwise normal.

Results of tests
Blood tests must be requested item per item; do not give a result unless it has
been requested
FBC confirms a normal Hb 131 g/L, and normal white cell count CRP
normal
Iron studies: ferritin 44 μg/dL (normal 20–150 μg/dL)
Helicobacter pylori (H. pylori) serology positive, urea breath test positive,
stool antigen positive
UEC, LFTs and all other tests normal.

Suggested approach to the case


Establish rapport
Open questions to explore patient’s ideas, concerns and expectations.

Specific questions Page 202

Gastrointestinal
— Clarify nature, frequency, severity and duration of symptoms
— Relationship to meals, identification of specific triggers (spicy foods,
caffeine, alcohol, smoking, fatty foods, stress)
— Relieving factors
— Acid reflux or waterbrash
— Nausea or vomiting
— Early satiety, bloating, belching
— Changes in bowels
Exclude alarm features
— PR bleeding, melaena/haematemesis
— Dysphagia or odynophagia
— Unintentional weight loss
— Family history of gastrointestinal malignancy
Discuss the impact of symptoms on day-to-day life
Brief systems review
Past medical history
Drugs—NSAIDs, prescribed medication, OTC, recreational/illicit
Allergies
Family history
Social history—alcohol consumption, smoking
Request permission to examine.

Examination
Pulse, BP
BMI
Check for: jaundice, pallor
Abdominal examination
— Inspection
— Palpation
— Examine for liver, spleen, kidneys
— Percussion
— Auscultation.

Investigations
Request results from the facilitator
FBC
CRP
UEC
LFTs Page 203
Iron studies
H. pylori serology (useful only if negative)
Urea breath test (gold standard, can be requested instead of serology) or stool
antigen test if serology positive.

Most likely diagnosis


H. pylori–associated dyspepsia (gastritis +/− peptic ulcer disease).

Differential diagnoses
Malignancy unlikely in the absence of alarm features.

Management
Explain most likely diagnosis
Discuss general measures—avoiding triggers (spicy food, caffeine etc.);
smaller, more frequent meals; weight reduction; over-the-counter antacid
or alginate medications
Commence triple therapy for H. pylori
Plan review, repeat H. pylori testing after six weeks to ensure eradication
If symptoms persist after eradication, consider acid-suppression therapy
Sensitively address and reassure patient regarding fear of malignancy.

CASE COMMENTARY

H. pylori infection is a major cause of morbidity and mortality


worldwide. Differences in prevalence exist within and between
countries, with higher prevalence among people with lower
socioeconomic status. Mohammed was born overseas in a
country with likely higher prevalence and has also grown up in a
large family and spent time in a refugee camp (which frequently
have issues with overcrowding and poor sanitation), increasing
his risk of having contracted H. pylori.
In those under 55 years old, a ‘test and treat’ policy (in the
absence of alarm symptoms such as dysphagia, haematemesis or
weight loss) is considered safe practice because of the high
effectiveness of H. pylori treatment plus the low yield for upper
GI malignancy on scoping.
Arrangements for a follow-up are essential to ensure that
persisting dyspepsia despite eradication and adequate acid-
suppression trial is investigated with endoscopy. Mohammed
has likely experienced some trauma in his childhood, Page 204
leading to a higher likelihood of persisting symptoms
and a diagnosis of functional dyspepsia.

Further reading
Duggan, AE 2007, ‘The management of upper gastrointestinal symptoms: is
endoscopy indicated?’ Medical Journal of Australia, vol. 186, pp. 166–
7.
Mitchell, H & Katelaris, P 2016, ‘Epidemiology, clinical impacts and current
clinical management of Helicobacter pylori infection’, Medical Journal
of Australia, vol. 204, pp. 376–80.
Talley, NJ 2017, ‘Functional dyspepsia: advances in diagnosis and therapy’,
Gut and Liver, vol. 11, pp. 349–57.
Yaxley, J & Chakravarty, B 2014, ‘Helicobacter pylori eradication—an
update on the latest therapies’, Australian Family Physician, vol. 43, pp.
301–5.
Page 205

Case 40
Annie Nguyen

Instructions for the doctor


This is a short case.
Please take a history from Annie. Then outline to the facilitator what
examination and investigations you would normally do in your clinical
practice and indicate the most likely diagnosis.

Scenario
Annie Nguyen is 19 years old and in her first year at university. She has
not been to the clinic before. She has made the appointment because she
has been nauseated for the past three days.

While waiting to see the doctor she completed a new patient questionnaire
in which she gave the following information:
Past medical history
Glandular fever Year 12 of high school
Medication
Nil
Allergies
Nil known
Immunisations
Fully immunised
Family history
Nil relevant
Social history
Lives on campus in hall of residence
Non-smoker
Alcohol—three standard drinks per week.

Instructions for the patient, Annie Nguyen Page 206


You are 19 years old and in your first year at university. You have not been
to this GP before. You have been nauseated for the past three days but have
not vomited. You had diarrhoea for the first two days but this has now
settled. You do not feel like eating. You experienced colicky abdominal pain
just before each episode of diarrhoea.
Your last menstrual period was one week ago. You have no other
symptoms. You are not currently sexually active.

You have filled out the new patient questionnaire as follows:


Past medical history
Glandular fever Year 12 of high school
Medication
Nil
Allergies
Nil known
Immunisations
Fully immunised
Family history
Nil relevant
Social history
Lives on campus in hall of residence
Non-smoker
Alcohol—three standard drinks per week.

Suggested approach to the case


Establish rapport
Open-ended questions to explore Annie’s ideas, concerns and expectations.

Specific questions
Gastrointestinal
— Anorexia
— Nausea and vomiting
— Haematemesis
— Suspect food prior to onset of symptoms
— Abdominal pain
— Diarrhoea/constipation
— Blood or melaena pr
Associated features, such as headache
Systems review—fever, energy level Page 207
Genitourinary
— Frequency, haematuria, discharge
— Menstrual cycle, LMP, risk of pregnancy
Infectious contacts—are any other students or staff at hall of residence
unwell? Potential public health issues may need consideration if history
suggests others with similar symptoms
History of recent travel
Substances—medication prescribed elsewhere, OTC, alcohol, drugs of abuse
Request permission to examine.

Examination
Pulse
Temperature
Hydration
Jaundice
Abdominal examination—for tenderness, masses or organomegaly
Urine specimen dipstick.

Most likely diagnosis


Gastroenteritis.

Investigations
Nil needed.

CASE COMMENTARY

Nausea is a common presenting symptom in general practice and


the list of potential causes is extensive. This case tests the
doctor’s ability to adopt a systematic approach to the diagnosis
and follow the dictum that ‘common things are common’. It is
essential to explore Annie’s risk of pregnancy.
The question asks the doctor to tell the facilitator what
physical examination and investigations they would normally do
in their clinical practice. The history is strongly suggestive of
gastroenteritis and there are no alarming symptoms so a rectal
examination and investigations are not needed at this
consultation.

Further reading Page 208

Anderson 3rd, WD & Strayer, SM 2013, ‘Evaluation of nausea and vomiting:


a case-based approach’, American Family Physician, vol. 88, pp. 371–9.
Murtagh J 2018, ‘Nausea and vomiting’. In: Murtagh, J, Rosenblatt, J,
Coleman, J & Murtagh, C (eds), Murtagh’s General Practice, 7th ed,
McGraw-Hill, Sydney, pp. 685–9.
Page 209

Case 41
Kathy Jones

Instructions for the doctor


This is a long case.
Please take a history from your patient, Kathy Jones, and request the
findings of an appropriate physical examination. The facilitator will give you
the results of any relevant investigations on request. Then negotiate a
management plan with Kathy.

Scenario
Kathy Jones is a 55-year-old woman who has been attending your
practice for several years. She is mostly healthy but has hypertension and
dyslipidaemia, which are well-controlled with telmisartan and
atorvastatin. She is married with three grown children and works as a
school librarian. She went through menopause about five years ago and
found some relief from her hot flushes with some over-the-counter
natural remedies. She is quite health conscious, enjoys a good diet and
remains physically active.

The following information is on her summary sheet:


Past medical history
Hypertension
Dyslipidaemia
Medication
Telmisartan 40 mg od
Atorvastatin 20 mg od
Allergies
Nil known
Immunisations
Up-to-date
Social history Page 210
Lives with husband
Non-smoker
Alcohol—one to two units on rare special occasions.

Instructions for the patient, Kathy Jones


You are coming to the doctor because of three to four months of tiredness.
Your fitness is OK and your interest is fine; you just feel lethargic. Your
mood is good and you remain active in your work and various interests.
You’ve never felt like this before and have no idea what could be going on.
You have not had any cold or flu symptoms apart from some vague aching of
your joints—mainly the hands and wrists—over the past couple of months.
You have taken some paracetamol when your joints have been sore. None of
your joints are red, swollen or tender. You’ve not had a temperature, your
appetite is good and your weight is stable. You sleep well and you don’t
snore.
You are happily married with three grown children and four
grandchildren, who are all healthy. You took some over-the-counter natural
remedies for hot flushes when you went through menopause about five years
ago. You still have some issues with vaginal dryness and your libido has been
low in recent months.
You’ve been on medication for blood pressure and to lower cholesterol for
several years and you are extremely conscientious in taking these. You have
no known allergies, have never smoked and are not aware of any significant
family history.
Your cervical screening test, mammogram and FOBT are all up-to-date.

The following information is on your summary sheet:


Past medical history
Hypertension
Dyslipidaemia
Medication
Telmisartan 40 mg od
Atorvastatin 20 mg od
Allergies
Nil known
Immunisations
Up-to-date
Social history
Lives with husband
Non-smoker
Alcohol—one to two units on rare special occasions.

Information for the facilitator Page 211

Clinical examination findings


Please give the candidate the following examination findings on specific
request.
General appearance: well-dressed woman in no distress
BMI 22 kg/m2
Pulse 70/min
Temperature 36.5°C
BP 124/82 mmHg
BSL 5.5 mmol/L (random)
Urinalysis—normal
All physical examination findings are normal.

Investigation results
Please give the candidate the following investigation results on specific
request.
FBC, ESR—normal
EUC—normal
LFTs—normal
TSH—normal

Iron Transferrin TIBC Transferrin Ferritin


saturation
(9–31 (2.0–3.7 (45–80 (16–60%) (30–300
μmol/L) g/L) μmol/L) μg/L)
40.6* 2.8 46 90* 920*

HFE gene—C282Y homozygous (only to be given if candidate has


requested)
Fasting BSL—normal
Fasting lipids—normal
MSU—normal.

Suggested approach to the case


Establish rapport
Open questions to explore fatigue, how she experiences it and the impact on
her function.

Specific questions
Exclude cardiac symptoms (no SOB, palpitations, etc.)
Mood/energy/appetite/sleep/libido
Weight loss Page 212
Fevers/night sweats
Medications, including over-the-counter
Allergies
Systems review—should elicit arthralgia
Social history—alcohol, drugs
Occupational history
Family history
Preventative health, e.g. cervical, breast and bowel cancer screening.

Examination
Request permission to examine
General appearance (no jaundice, pallor or pigmentation)
Vital signs
Cardiovascular and respiratory examination
Thyroid examination
Check lymph nodes
Abdominal examination
Examination of hands/wrists
Urine dipstick
BSL.

Investigations
FBC
EUC
LFTs
TSH
ESR/CRP
Ferritin/iron studies
HFE gene test.

Management
Explanation of hereditary haemochromatosis
Commence therapeutic venesection—initially weekly, then maintenance
schedule to keep ferritin within normal range
Avoid vitamin C (and iron) supplements
Avoid alcohol until iron stores are normalised
Will need regular monitoring of iron studies and venesections to maintain
normal indices
Screening of adult family members.

CASE COMMENTARY Page 213

Fatigue is a common presentation in general practice and is a


good test of our ability to sort out all the possible causes with a
thorough history, appropriate examination and rational
investigation. Kathy is over 50, previously well, and gives a
history of new-onset fatigue and arthralgia—this warrants the
initial investigations listed above.
Hereditary haemochromatosis is the most common genetic
condition of Caucasian populations, with more than one in 200
individuals at genetic risk of having this disease. Importantly,
early diagnosis and treatment of hereditary haemochromatosis
prevents complications and results in a normal life expectancy.
Being able to interpret iron studies is an important skill for GPs
to have. The transferrin saturation (ratio of serum iron and iron
binding capacity) reflects increased absorption of iron, which is
the underlying biological defect in hereditary
haemochromatosis. In very early disease, transferrin saturation
may be elevated before a rise in ferritin. A fasting transferrin
saturation >45% is the most sensitive test for detecting early iron
overload, but a raised fasting transferrin saturation or ferritin is
not diagnostic of hereditary haemochromatosis. Hereditary
haemochromatosis is unlikely if the ferritin is high and the
transferrin saturation is normal. An elevated serum ferritin
reflects an increase in body iron stores but is also an acute phase
reactant and can be elevated non-specifically in the presence of
alcohol consumption, infection or inflammation, as well as some
malignancies.
Kathy presents with the classic triad of fatigue, arthralgia and
low libido at a typical age for women to present with hereditary
haemochromatosis. Before menopause, physiological blood loss
from menstruation and pregnancy provides a degree of
protection from clinical disease. With a ferritin below 1000
μg/L, Kathy does not require a liver biopsy and should proceed
to venesection, which can be expected to improve her tiredness
but possibly not her arthralgia or libido. She should be cautioned
about avoiding vitamin C supplements, which increase
absorption of dietary iron and also facilitate release of iron from
storage.
There is some controversy regarding the ideal target range for
ferritin from venesection therapy. Previous guidelines have
suggested that venesection should continue until ferritin is <50
μg/L. However, with genetic testing and less severe disease on
presentation as a result of earlier detection, some experts now
recommend normalisation of ferritin to <300 μg/L for men and
postmenopausal women, and <200 μg/L for pre-menopausal
women. More aggressive management may be required for those
presenting with more severe iron overload and ferritin >1000
μg/L, when the risk of cirrhosis is higher and the aim is Page 214
to iron deplete extrahepatic organs in addition to the
liver to minimise risk of haemochromatosis.
Hereditary haemochromatosis is unusual in that treatment by
venesection helps other patients who require blood transfusions.

Further reading
Allen, K 2010, ‘Hereditary haemochromatosis: diagnosis and management’,
Australian Family Physician, vol. 39, no. 12, pp. 938–41.
Delatycki, M & Allen, K 2013, ‘Hereditary haemochromatosis: how to treat’,
Australian Doctor, vol. 12, April.
Goot, K, Hazeldine S, Bentley, P, Olynyk, J & Crawford, D 2012, ‘Elevated
serum ferritin: what should GPs know?’ Australian Family Physician,
vol. 41, no. 12, pp. 945–49.
Wilson, J, Morgan, S, Magin, P & van Driel, M 2014, ‘Fatigue—a rational
approach to investigation’, Australian Family Physician, vol. 43, no. 7,
pp. 457–61.
Page 215

Case 42
Jack Kingsley

Instructions for the doctor


Please take a focused history from Jack and answer his questions.

Scenario
Jack is a 43-year-old man whose wife Julie and two children attend the
practice. He saw you last week for a full check-up, including a physical
examination, which was normal. He feels he can trust you and so made
another booking to discuss something else.

The following is on his summary sheet:


Past medical history
Nil recorded
Medications
Nil
Immunisations
ADT 2019 after stepping on a nail last week
Unsure about childhood immunisations
Family history
Brother died MVA aged 35
Mother died of cirrhosis aged 57
Social history
Lives with his wife Julie and two children
Works as a manager at an electrical store
Non-smoker
Drinks four to six full-strength beers at the weekend.

Instructions for the patient Jack Kingsley Page 216

You have known you were hepatitis C positive since 2001 when you were
diagnosed. You injected drugs in your twenties so think this is when you
became infected.
Julie and the children know about your diagnosis. You take precautions
around them and they have never had any exposure to your blood. Julie is
your only sexual partner since diagnosis and you no longer use drugs.
You were diagnosed when in prison for car theft and were commenced on
interferon. You only had about four weeks of treatment as it made you unwell
and after leaving prison you wanted to get your life together and leave your
past behind.
The treatment didn’t work and you accepted that you would live with
hepatitis C until it killed you as it did your mum. One of your friends from
your drug days recently died of liver cancer. At his funeral another friend told
you of his cure on a new treatment.
You had a difficult childhood and ended up in foster care. As a young
adult you were homeless and used IV drugs.
You know you should have seen a doctor regularly but you have lived in
fear of your past coming back to haunt you and just preferred to get on with
your life. But now, with talk of a new treatment, you think it’s worth
investigating the options and you want to ask the doctor about it.
You feel fit and run three times a week and ride 10 km to work and back
five times a week. You don’t smoke. You drink four to six stubbies of full-
strength beer on the weekends.
You have avoided your favourite sport of AFL due to the risk of injury
and blood exposure for your teammates but would love to play again.
You don’t take any other drugs and vow never to do that again.
You are very careful with any blood exposure and would never dream of
getting a tattoo or anything similar.
If there is a new treatment you are happy to undergo any testing the doctor
suggests.
If the treatment sounds like the interferon treatment you are more reluctant
as you remember how bad it made you feel.
If the doctor asks about mood, you are in good spirits. The trauma of your
childhood will never go away but most days you feel lucky to be part of a
loving family, have a good job and a safe home. You do have your moments,
but you had counselling in prison, which gave you some strategies to deal
with nightmares and jumpy moments. You have no thoughts of hurting
yourself.

Suggested approach to the case Page 217

History
Re-establish rapport
Elicit patient’s concerns
Explore diagnosis and possible cause
Ask about past treatment attempts
Approach in a sensitive, non-judgemental manner
Reassure and encourage his newly proactive approach
Brief screen for symptoms of depression and recognise that there is a risk of
co-morbid depression/post-traumatic stress disorder
Check for possible exposures for family members
Brief history regarding any ongoing high-risk activities—tattoos, IV drug
use, sexual partners
SNAP—smoking, nutrition, alcohol, physical activity assessment.
Management
Pre-treatment work-up for hepatitis C treatment
— FBC, EUC, INR
— Liver biochemistry
— Blood glucose
— HCV RNA and genotype
— Upper abdominal ultrasound
— Serology for HIV, hepatitis A virus and hepatitis B virus
For immunisation if not immune to hepatitis A and/or B
Treatment can be commenced by GPs unless there is cirrhosis (needs
gastroenterologist referral)
Offer a GP management plan (prefilled plan available on ASHM website)
Advise to reduce alcohol intake and warn that if there is evidence of cirrhosis
alcohol should stop completely
Ensure understanding of the need for compliance
Reassure patient about improved side effect profile and efficacy of treatment
compared with interferon.

Follow-up
Follow–up to discuss results and decide on best course of treatment
Discuss further exploration of mental health at next visit.

CASE COMMENTARY Page 218

In Australia there are approximately 230 000 people affected by


chronic hepatitis C (HCV) and it is the most common disease
requiring liver transplant. Recent advances in treatment mean
that HCV is curable, whereas treatment with interferon was
poorly tolerated and had limited efficacy. Direct antiviral agents
have minimal side effects and up to 95% cure rate after 12–24
week treatment courses. They can be initiated by GPs if there is
no cirrhosis or other complications. Those with cirrhosis have
the most to gain from treatment and should be referred to a
hepatolgist/gastroenterologist for treatment.
75% of acute HCV infections will become chronic, leading to
increased risk of liver failure, cirrhosis and hepatocellular
carcinoma. 20–30% of people will develop cirrhosis generally
20–30 years after infection.
All Australians living with HCV should be considered for
treatment, so it is important for GPs to know about the treatment
and the pretreatment work-up.

COMMON PITFALLS

Jack has not had adequate medical care for his chronic HCV.
However, he is now motivated. Discussing the care he has not
received will threaten the therapeutic relationship; focusing on
his new proactive approach is preferable.
Jack may already have cirrhosis or possibly hepatocellular
carcinoma. At this visit it is important to think about these
complications when ordering investigations, but there is cause
for optimism with the new direct antiviral agents even if his test
results show elements of cirrhosis. Candidates are not expected
to know the details of the medications but should be able to
describe that they are oral for a 12–24 week course and have a
high chance of cure.

Further reading
Khoo, A & Tse, E 2016, ‘A practical overview of the treatment of chronic
hepatitis C virus infection’, Australian Family Physician, vol. 45, no.
10, pp. 718–20.
Strasser, S 2017, ‘Managing hepatitis C in general practice’, Australian
Prescriber, vol. 40, no. 2, pp. 64–9.
Therapeutic Guidelines Ltd 2017, ‘Hepatitis C’. In: eTG complete Page 219
[Internet]. Therapeutic Guidelines Ltd, Melbourne.
Hepatitis C Management and Treatment. Available at: www.ashm.org.au/HC
V/management-hepc/, accessed 26 February 2019.
Hepatitis C Virus Infection Consensus Statement Working Group 2017,
Australian recommendations for the management of hepatitis C virus
infection: a consensus statement, August. Gastroenterological Society of
Australia, Melbourne, Vic.
Page 220

Case 43
Neil Dawson

Instructions for the doctor


This is a short case.
Please read the following scenario. No further history or examination are
required. Please discuss the results of your initial investigations with the
patient and negotiate a management plan with him.

Scenario
Neil Dawson is a 61-year-old insurance broker who has spent most of his
life avoiding doctors. His wife, worried about him, finally convinced him
to come and see you last week for his first doctor appointment in at least
a decade. She was worried about his increasing fatigue and his ankles
that sometimes swell.
Your further history and systems review elicited no alarm symptoms.
You established that Neil has had some psychosocial stressors (caring for
his mother with dementia who passed away 12 months ago, plus some
financial concerns regarding his home business), but has no signs of
depression or anxiety.
You ascertained that Neil is not particularly bothered by his swollen
ankles, but that he thought this was a sign that it was time to come to get
things checked up.

You collected the below information:


Past medical history
Bilateral inguinal hernia repairs aged 45
Medications Page 221
Nil
Allergies
Nil known
Family history
Mother—deceased (dementia, aged 85)
Father—deceased (bowel cancer, aged 79)
Social history
Lives with wife
Owns insurance brokerage business
Preventative health
Non-smoker
Alcohol—four to six full-strength beers most nights
Nutrition—typical daily intake consists of three slices of toast with jam
plus a glass of orange juice for breakfast, a pie and a can of coke for lunch,
half a packet of biscuits and cheese with a few beers after work,
meat/potatoes/beans/carrots for dinner and a large bowl of ice-cream while
watching TV at night
Physical activity—mows the lawn fortnightly, sedentary job
Bowel cancer screening—non-participant.

Examination
BP 138/85, pulse 80 and regular
Weight 132 kg
Height 177 cm
BMI 42.1
Chest clear, good air entry
Heart sounds dual, nil added
JVP not elevated
Mild peripheral oedema to ankles
Respiratory examination normal
Thyroid examination normal
No lymphadenopathy
Abdominal exam difficult due to body habitus, some distension, shifting
dullness
Few spider naevi upper chest
Palmar erythema noted
You arranged for some initial blood tests and asked Neil to return for a
follow-up appointment.

Initial investigations Page 222

Full blood count


Haemoglobin 142 (130–170) g/L
MCV 98 (82–99)fL
Platelets* 132 (150–400) × 109/L
White cells 10.3 (4.0–11) × 109/L
Neutrophils 70% 7.2 (1.8–7.5) × 109/L
Lymphocytes 22% 2.27 (1.0–4.0) × 109/L

Monocytes 6% 0.41 (0.1–1.2) × 109/L

Eosinophils 2% 0.2 (<0.7) × 109/L

Electrolytes and liver function tests


Na 134 (134–146) mmol/L
K 4.2 (3.4–5.5) mmol/L
Cl 106 (95–108) mmol/L
HCO3 24 (22–32) mmol/L

Urea 4.9 (3.0–8.0) μmol/L


Creatinine 77 (30–100) μmol/L
Bilirubin (total)* 36 (< 16) μmol/L
ALP* 117 (20–105) U/L
GGT* 92 (<36) U/L
ALT* 109 (< 40) U/L
AST* 198 (<35) U/L
Albumin* 30 (38–50) g/L
Total protein* 60 (65–85) g/L

Iron studies
Iron 15 (9–30) umol/L
Transferrin 2.8 (2.0–3.6) g/L
Transferrin saturation 30 (15–45) %
Ferritin* 180 (10–80) ug/L

HbA1c, thyroid function tests, CRP and B12 are normal.

Information for the patient


You are a 61-year-old insurance broker and have spent most of your life
avoiding doctors. Your wife booked your initial appointment last week as she
was worried about your increasing fatigue over the last six months and had
noticed that your ankles are sometimes a bit swollen. This was your Page 223
first doctor appointment in at least a decade.
You are otherwise well and have not noticed any other symptoms. Your
mum passed away from dementia 12 months ago, and prior to this you had a
very stressful couple of years caring for her in your house. Your business
suffered over this time and is still recovering, but you are feeling alright
about this now, and are not depressed.
You are not overly bothered by the swollen ankles, but wished you had
some more energy. You agreed with your wife that it was time to get a check-
up and are a little apprehensive about the results.

The following information is on your medical record:

Past medical history


Bilateral inguinal hernia repairs age 45
Medications
Nil
Allergies
Nil known
Family history
Mother—deceased (dementia, aged 85)
Father—deceased (bowel cancer, aged 79)
Social history
Lives with wife
Owns insurance brokerage business
Preventative health
Non-smoker
Alcohol—four to six full-strength beers most nights
Nutrition—typical daily intake consists of three slices of toast with jam
plus a glass of orange juice for breakfast, a pie and a can of coke for lunch,
half a packet of biscuits and cheese with a few beers after work,
meat/potatoes/beans/carrots for dinner and a large bowl of ice-cream while
watching TV at night
Physical activity—mows the lawn fortnightly, sedentary job
Bowel cancer screening—non-participant.

Suggested approach to the case


Establish rapport
Explore patient’s ideas, concerns and expectations regarding this consultation
Explain results clearly and sensitively
Discuss that results are highly suggestive of liver failure/liver Page 224
cirrhosis, and explain what this is
Arrange further investigations to confirm diagnosis and rule out other causes
— Ultrasound of liver and abdomen
— INR
— Liver screen including hepatitis B serology (surface antibody, surface
antigen and core antibody) and hepatitis C serology
— Check immunity to hepatitis A
— Check AFP
Hepatology referral once above investigations are complete
Discuss likely aetiology of alcohol and obesity to liver disease
Address alcohol—abstinence recommended given there is evidence of liver
failure
Address nutrition and physical activity
Arrange follow-up
Consider GP management plan and allied health referral including dietician
and exercise physiologist
Extras for the case, most likely in future consultations—other preventative
health including bowel cancer screening, check of fasting lipids and
absolute cardiovascular risk assessment, look for and address other
complications of obesity (e.g. obstructive sleep apnoea), immunisations.

CASE COMMENTARY

There is a lot of written information to take in during the three-


minute reading time. Try not to be flustered by this; it is OK to
take a few short moments while in the exam room to gather your
thoughts if necessary. This is a management case that requires
candidates to accurately interpret results, convey a new
diagnosis of a chronic disease in a sensitive and informative
manner, and provide motivational interviewing to initiate
lifestyle changes essential for the management of this condition.
Cirrhosis and chronic liver failure are common causes of
morbidity and mortality and are in most cases preventable. They
are often diagnosed at a late stage, as patients remain
asymptomatic until the occurrence of decompensation. Non-
alcoholic fatty liver disease (NAFLD) and alcohol are becoming
an increasingly important cause of cirrhosis (especially as
incidence of hepatitis C–related cirrhosis is expected to reduce).
Management of NAFLD predominantly involves addressing
lifestyle factors, including nutrition, alcohol and physical
activity, something we as general practitioners are well-placed to
achieve.

Further reading Page 225

Duggan, AE & Duggan, JM 2011, ‘Alcoholic liver disease: assessment and


management’, Australian Family Physician, vol. 40, pp. 590–3.
Heidelbaugh, JJ & Bruderly, M 2006, ‘Cirrhosis and chronic liver failure:
part I. Diagnosis and evaluation’, American Family Physician, vol. 74,
pp. 756–62.
Iser, D & Ryan, M 2013, ‘Fatty liver disease: a practical guide for GPs’,
Australian Family Physician, vol. 42, pp. 444–7.
MacDonald, G 2009, ‘Nonalcoholic fatty liver disease’, Medicine Today, vol.
10, pp. 68–9.
Pattullo, V & Strasser, SI 2017, ‘Managing the rising burden of chronic liver
disease: nonalcoholic fatty liver disease and alcoholic liver disease’,
Medicine Today, vol. 18, pp. 28–35. Page 226
Page 227
Section 13
Men’s health
Page 228

Case 44
Kim Hosking

Instructions for the doctor


This is a short case.
Please take a history. The facilitator will give you the results of the
examination on request. Outline the most likely diagnosis to the patient and
negotiate a management plan with him.

Scenario
Kim Hosking is a 70-year-old retired architect. He comes to see you
every few months for a blood pressure check. At the last check Kim
mentioned that he was having ‘waterworks trouble’ and you suggested
he make an appointment to discuss this further.

The following information is on his summary sheet:


Past medical history
Hypertension—diagnosed 2009
Medication
Perindopril 5 mg od
Allergies
Nil
Immunisations
Up-to-date, including influenza
Social history
Married
Three children, now grown-up
Ex-smoker.

Instructions for the patient, Kim Hosking Page 229

You are a 70-year-old retired architect. Over the past few years your urine
stream has become weaker. You pass urine more often and now have to get
up about three times each night. You get frustrated that, having got out of
bed, you then have to wait a while before the stream starts. It embarrasses
you that you often dribble after passing urine, which wets your clothes.
Your blood pressure is currently well-controlled and except for the urinary
symptoms, you enjoy being retired. Two weeks ago, at your blood pressure
check, you mentioned to your GP that you had ‘waterworks trouble’. The GP
suggested that you make an appointment to discuss this, which is why you
are here.

The following information is on your summary sheet:


Past medical history
Hypertension—diagnosed 2009
Medication
Perindopril 5 mg od
Allergies
Nil
Immunisations
Up-to-date, including influenza
Social history
Married
Three children, now grown-up
Ex-smoker.

Information for the facilitator


Clinical examination findings
Temperature, pulse and blood pressure are within normal limits
Abdominal examination is normal
Rectal examination reveals a smooth, non-tender, symmetrically enlarged
prostate.

Suggested approach to the case


Establish rapport
Open-ended questions to gain understanding of Kim’s ideas, concerns and
expectations.

Specific questions Page 230

Details—urinary frequency, hesitancy, stream, nocturia, incomplete


emptying, incontinence, terminal dribbling
Pain or dysuria
Haematuria
Fever
Systemic malaise, weight loss
Sexual function
Urethral discharge.

Ask for the examination findings


Temperature
Pulse
BP Abdominal examination
— Inspection
— Palpation—exclude palpable bladder
Genitalia
Rectal examination to assess the prostate.

Most likely diagnosis


Obstructive lower urinary tract symptoms due to benign prostatic
hypertrophy.

Investigations
Bladder diary
Urine for MCS
UEC, BSL
PSA—needs informed consent.

Management
Modify caffeine and fluid intake if excessive
Encourage bladder training
For benign prostatic hypertrophy, alpha-adrenoreceptor antagonists or 5-
alpha-reductase inhibitors or phosphodiesterase inhibitors (private script),
or refer for surgery (transurethral resection or minimally invasive)
Arrange follow-up.

Extras for this case Page 231

Discuss alcohol consumption.

CASE COMMENTARY

Kim has obstructive lower urinary tract symptoms, a common


problem for older men. The doctor will need to take a history of
these symptoms and must not be too embarrassed to ask about
sexual function. The most likely cause is benign prostatic
hypertrophy, but the doctor needs to ask about haematuria,
weight loss, fatigue and bone pain, which would suggest
malignancy.
The examination will focus on the abdomen, including a
rectal examination, and exclude an enlarged bladder. Practise
rectal examinations on a simulated model if you can.
Testing for prostatic specific antigen (PSA) is controversial in
asymptomatic men. In Kim’s case, PSA testing might reveal an
advanced prostate cancer, or might be useful in deciding on and
monitoring treatment with 5-alpha-reductase inhibitors.
Kim’s history of hypertension increases the risk of renal
failure, so it is reasonable for doctors to check his renal function
tests. Because of the nocturia and frequency, Kim needs a
fasting BSL to exclude diabetes. An alpha-blocker relieves
symptoms straight away but also has a hypotensive effect. Kim’s
blood pressure will need monitoring for hypotension and a
reduction or cessation of the perindopril dose may be indicated.
5–alpha-reductase inhibitors, such as finasteride, can take up to
six months to work. Longer acting phosphodiesterase type 5
inhibitors, for example, tadalafil, may be effective for lower
urinary tract symptoms but are only available on a private script.
Patients can purchase saw palmetto from health food shops, but
a Cochrane review2 has not confirmed any superiority over a
placebo.
Renal ultrasound is only indicated in patients with moderate-
to-severe symptoms, or an abnormal serum creatinine, to assess
bladder capacity, post-void urine residual volume, prostate
volume and exclude hydronephrosis.1

References
1. Jiwrajka, M, Yaxley, W, Perera, M, Roberts, M, Dunglison, N, Yaxley, J
& Esler, R 2018, ‘Review and update of benign prostatic hyperplasia in
general practice’, Australian Journal of General Practice, vol. 47, no. 7,
pp 471–5.
2. Cochrane Complementary Medicine, Prostate Enlargement, Page 232
Cochrane Collaboration. Available at:
https://cam.cochrane.org/prostate-enlargement#sawpalmetto_prostate,
accessed 20 February 2019.

Further reading
Arianayagam, M, Arianayagam, R & Rashid, P 2011, ‘Lower urinary tract
symptoms. Current management in older men’, Australian Family
Physician, vol. 40, pp. 758–67.
Woo, HH, Gillman, MP, Gardiner, R, Marshall, V & Lynch, WJ 2011, ‘A
practical approach to the management of lower urinary tract symptoms
among men’, Medical Journal of Australia, vol. 195, pp. 34–9.
Page 233

Case 45
Jock Palmer

Instructions for the doctor


This is a short case.
Provide and explain the appropriate information to Jock and answer any
questions he has. You do not need to take any history or examine Jock.

Scenario
Jock Palmer is a 52-year-old executive whose employer offers annual
screening tests to their employees. This year they have included the
prostate specific antigen (PSA) test. The company sent round an
information letter but this left Jock confused. Jock trusts you as his GP
and wants your opinion as to whether he should have this PSA test.
Please discuss this with Jock.
The following information is on his summary sheet:
Past medical history
Nil significant
Medication
Nil
Allergies
Nil
Immunisations
Up-to-date
Family history
Nil significant
Social history
Business executive
Divorced
Non-smoker
Alcohol—six standard drinks per week.

Instructions for the patient, Jock Palmer Page 234

You are a 52-year-old executive. Your employer offers you annual screening
tests. This year you received a letter informing you that the tests would
include the prostate specific antigen (PSA). The company sent round an
information letter but this left you confused. You trust your GP and want the
GP’s opinion as to whether you should have this PSA test. You have made
this appointment with your GP to discuss the PSA test.
You are well and have no urinary symptoms.
During the consultation you will ask the following questions:
1. What is the PSA test?
2. What are the advantages of having the test?
3. Is there any reason not to have the test done?
4. If the test is high, does that mean that I have cancer?
5. What would happen if the result was high?
6. Should I have the test?
The following information is on your summary sheet:
Past medical history
Nil significant
Medication
Nil
Allergies
Nil
Immunisations
Up-to-date
Family history
Nil significant
Social history
Business executive
Divorced
Non-smoker
Alcohol—six standard drinks per week.

Suggested approach to the case


Re-establish rapport
Acknowledge that this is a complex and controversial issue
Question Jock about his understanding of PSA tests
Question Jock about his general health—exclude weight loss, fatigue, fever,
bone pain, family history of prostate cancer

Specific questions Page 235

To exclude urinary symptoms


— Frequency
— Stream
— Nocturia
— Haematuria
— Dysuria
Tailor explanation of PSA test to address Jock’s concerns
— PSA comes from the prostate
— Test has limited accuracy and can be affected by a number of
conditions/scenarios
— Low result does not mean that there is no cancer
— Raised result does not mean that there is cancer
— Trend of results provides some clinical information
If PSA is raised Jock will need to have further tests with a specialist,
including prostatic biopsies
Early treatment of prostate cancer has not consistently been proved to
increase life expectancy, and there are expected harms of false-positives,
overdiagnosis and overtreatment1
PSA testing does not fulfil criteria for a screening test, and there is no
national screening program in Australia.2 The Royal Australian College
of General Practitioners3 does not recommend screening for prostate
cancer unless the man specifically asks for it and he is fully counselled on
the pros and cons.

CASE COMMENTARY

This case tests the doctor’s ability to advise patients on a


controversial topic. The hope was that PSA testing would enable
early detection of prostate cancer and, by allowing early
treatment, improve prognosis. The evidence is that prostate
cancer screening can reduce risk of death from prostate cancer
but the absolute benefit is small, and the chance of prostate
cancer being diagnosed and treated (even if biologically
unimportant) is increased by a much larger amount.1 Jock’s
confusion at the information that he was given is not surprising
and the doctor needs to try to explain the principles of screening,
the concepts of false-negatives and false-positives and enable
Jock to make the right decision for himself.
The doctor needs to have a clear understanding of the Page 236
criteria for a useful screening test. The WHO4 defined these as
follows:
1. The condition should be an important health problem
2. The natural history of the condition should be understood
3. There should be a recognisable latent or early symptomatic
stage
4. There should be a test that is easy to perform and interpret,
acceptable, accurate, reliable, sensitive and specific
5. There should be an accepted treatment recognised for the
disease
6. Treatment should be more effective if started early
7. There should be a policy on who should be treated
8. Diagnosis and treatment should be cost-effective
9. Case finding should be a continuous process.
The National Health and Medical Research Council Information
for Health Practitioners summarises the evidence that shows
why PSA test does not fulfil these criteria, as follows:

Possible benefit of PSA testing5


• For every 1000 men tested, two men will avoid death from
prostate cancer before 85 years of age because of PSA
testing. This benefit might be greater for men at high risk of
prostate cancer, such as those with a strong family history of
the disease.
• For every 1000 men tested, two men will avoid metastatic
prostate cancer before 85 years of age because of PSA
testing.
Expected harms of PSA testing
False-positive results:
• For every 1000 men tested:
— 87 men who do not have prostate cancer will have a
false positive PSA test that will lead to a biopsy
— 28 men will experience a side effect from the biopsy
that they consider to be a moderate/major problem that
may require healthcare, and one will require
hospitalisation.
Overdiagnosis:
• For every 1000 men tested, 28 men will have prostate cancer
diagnosed as a result of the PSA test, many of whom would
have remained asymptomatic for life (i.e. are
overdiagnosed). Overtreatment:
• For every 1000 men tested:
— 25 men will choose to undergo treatment (surgery or
radiation) because of uncertainty about which cancers
need to be treated, many of whom would do well
without treatment (i.e. are overtreated)
— 7–10 of these 25 men will develop persistent Page 237
impotence and/or urinary incontinence, and
some will develop persistent bowel problems, due to
treatment.
• For every 2000 men tested, one man will experience a
serious cardiovascular event, such as myocardial infarction,
due to treatment.1

(One option for running this case is to provide candidates with a


copy of the NHMRC four-page summary of information for
health professionals1 or the RACGP patient information sheet.5
This would test the candidate’s ability to interpret medical
information for patients, rather than test their memory on the
outcomes of PSA testing.)

References
1. National Health and Medical Research Council 2014, PSA Testing for
Prostate Cancer in Asymptomatic Men. Information for Health
Practitioners. Available at:
https://nhmrc.gov.au/sites/default/files/documents/reports/clinical%20gui
delines/men4d-psa-testing-asymptomatic.pdf, accessed 20 February 2019.
2. Cancer Council Australia 2018, ‘Prostate Cancer Detection’. Available at:
www.cancer.org.au/about-cancer/early-detection/prostate-cancer-
screening.html, accessed 1 December 2018.
3. Royal Australian College of General Practitioners 2018, ‘Guidelines for
preventative activities in general practice, Prostate Cancer’, RACGP,
Melbourne, Vic. Available at: https://www.racgp.org.au/clinical-
resources/clinical-guidelines/key-racgp-guidelines/view-all-racgp-
guidelines/red-book/early-detection-of-cancers/prostate-cancer, accessed
20 February 2019.
4. Wilson, J & Junger, G 1968, ‘The Principles and Practice of Screening
for Disease’, World Health Organization, Geneva.
5. Royal Australian College of General Practitioners 2015, Should I have
prostate cancer screening? RACGP, Melbourne, Vic. Available at:
https://www.racgp.org.au/download/Documents/Guidelines/prostate-
cancer-screening-infosheetpdf.pdf, accessed 1 December 2018.

Further reading
Prostate Cancer Foundation of Australia and Cancer Council Australia PSA
Testing Guidelines Expert Advisory Panel 2016, ‘Draft clinical practice
guidelines for PSA testing and early management of test-detected
prostate cancer’, Prostate Cancer Foundation of Australia and Cancer
Council Australia, Sydney.
Page 238

Case 46
Costa Rinaldi

Instructions for the doctor


This is a long case.
Please take a history. The facilitator will give you the results of the
examination on request. Then negotiate a management plan with the patient.

Scenario
Costa Rinaldi is a 36-year-old man. He rarely comes to see the GP but
has booked an appointment today because his girlfriend sent him. They
want a family but have not achieved a pregnancy after a year without
using contraception. Costa’s girlfriend has a child from a previous
relationship.

The following information is on his summary sheet:


Past medical history
Chickenpox aged 11
Medication
Nil
Allergies
Nil
Immunisation
Nil recorded
Social history
Works as an engineer.

Instructions for the patient, Costa Rinaldi


You are 36 years old and work hard as an engineer. Over the last few years
you have put on weight and now have a BMI of 35.5 kg/m2. You have always
been keen to have a family and were delighted to meet Brenda. You want to
start a family with her.
You have a great sex life but it is getting you down that Brenda is Page 239
not pregnant. You have not used contraception for a year. You have a
constant reminder that the lack of pregnancy is probably your problem, as
Brenda’s 6-year-old daughter from a previous relationship lives with you.
This problem is beginning to affect your work and your relationship with
Brenda. You do not smoke and you drink at least five bottles of wine a week.
You have heard that some childhood infections can cause infertility and you
are worried that the chickenpox is to blame. Brenda has sent you to this
appointment.
You have no genitourinary symptoms.

The following information is on your summary sheet:


Past medical history
Chickenpox aged 11
Medication
Nil, no supplements
Allergies
Nil
Immunisation
Nil recorded
Social history
Works as an engineer.

Instructions for the facilitator


Clinical examination findings
On specific request please give the doctor the following results:
Pulse 74
BP 136/82 mmHg
Height 1.75 m
Weight 108.8 kg
BMI 35.5 kg/m2
No gynaecomastia
Abdominal examination—normal
Hair distribution—normal
Genitalia—normal testicular and penile examination; no varicocele.

Suggested approach to the case


Establish rapport
Open-ended questions to explore Costa’s ideas, concerns and Page 240
expectations about the problem and the consultation.

Specific questions1
General health—fever, systemic malaise, weight loss
Sexual function/genitalia
— Frequency of coitus
— Libido, exclude erectile dysfunction, ejaculatory failure, retrograde
ejaculation
— Urethral discharge.
Past medical history
History of undescended testes, trauma, orchitis
History of mumps, other infections—STIs, TB
Chemotherapy/radiotherapy.

Drugs
Prescribed, OTC, drugs of abuse (e.g. anabolic steroids).

Social history
Alcohol consumption, smoking, recreational drugs.

Occupational history
Impact of current issue on relationship and self-esteem
Seek permission to examine.

Ask for the examination findings


Pulse
BP
BMI
Gynaecomastia
Hair distribution
Abdominal examination
Genitalia, in particular size of testes (Klinefelter’s syndrome), varicocele.

Most likely diagnosis Page 241

Subfertility.

Differential diagnosis
Female subfertility
— Cannot assume that it is a male problem
— Need to address the issue as a couple.
Investigations1
Sperm count × two—ideally fresh specimen after three days abstinence, two
to three weeks apart
LH, FSH and morning testosterone1 +/− total testosterone, prolactin
Screening for STI and HIV.

Advice
Discuss impact on relationship and consider need for relationship counselling
Reassurance subfertility not caused by chickenpox
Reduction in weight and alcohol consumption needed
Timing of intercourse
Plans for follow-up with partner.

Extra for this case


Check smoking status.

CASE COMMENTARY

Infertility affects 15–20% of couples, with up to 50% having


male contributing factors; 5% of male infertility is potentially
correctable. It is unrealistic in a single consultation to find the
cause of the problem but Costa needs to leave this consult
sensing that he is supported and that the doctor has set up a
management plan.
It is important for the doctor to reassure Costa that
chickenpox has not caused infertility. He is confusing
chickenpox with mumps.
Hormone measurements can help determine whether Page 242
Costa has gonadotropin deficiency (low testosterone and
low or inappropriately normal LH and FSH), primary testicular
failure (low testosterone, elevated LH and FSH), spermatogenic
failure (normal testosterone and LH, elevated FSH), or androgen
resistance (high testosterone, elevated LH). A majority of
infertile men have normal testosterone, LH and FSH levels.
Obstruction should be ruled out in azoospermic men with
normal testosterone, LH and FSH levels. Yq microdeletions,
specifically in the azoospermia factor (AZF) region, are the most
prevalent cause of spermatogenic failure in men with
azoospermia or severe oligozoospermia. Infertile men with
azoospermia or severe oligozoospermia should be referred to
male fertility specialists for further investigations such as
karyotyping and testing for Yq microdeletions.1

COMMON PITFALLS

A pitfall to avoid is agreeing with Costa that he is without doubt


the cause of the problem. Brenda’s previous pregnancy is not
proof of current fertility. Joint consultations with the GP
emphasise that this is an issue for Brenda and Costa as a couple,
not just for one partner. This can be suggested for the future.
The opportunities in this consultation are to conduct a physical
examination, discuss Costa’s drinking (alcohol can suppress
spermatogenesis) and obesity, and recommend increased
physical activity and arrange follow-up.

Reference
1. Katz, D, Teloken, P & Shoshany, O 2017, ‘Male infertility—the other
side of the equation’, Australian Family Physician, vol. 46, no. 9, pp.
641–6.

Further reading
Hirsh, A 2003, ‘Male subfertility: ABC of subfertility’, British Medical
Journal, vol. 327, pp. 669–73.
Cissen, M, Bensdorp, A, Cohlen, BJ, Repping, S, de Bruin, J & van Wely, M
2016, ‘Treatments for male subfertility’, Cochrane Collaboration, 26
February.
Page 243
Section 14
Mental health
Page 244

Case 47
Phyllis Brown

Instructions for the doctor


This is a short case.
Please take a history from Phyllis and outline to her the most likely
diagnosis. Negotiate an initial management plan with Phyllis.

Scenario
You have been called to see Phyllis Brown, aged 74, at home. She has
not attended the practice since her husband, Ernie, died two years ago.
Ernie and Phyllis were very close and did everything together. They
were involved in the local bowling club and seemed to almost live there.
Ernie died suddenly of a heart attack at the club one day and Phyllis
took it badly. She now relies on most of her shopping being delivered or
done by her daughter. The daughter has asked for this visit as she feels
that her mother needs a good check-up.
The following information is on her summary sheet:
Past medical history
Cholecystectomy 1994
Medication
Nil
Allergies
Nil known
Immunisations
Nil known
Social history
Widowed
Non-smoker.

Instructions for the patient, Phyllis Brown Page 245

You are 74 years old and were widowed two years ago. You and Ernie had a
good marriage and you find it hard without him. Most of the time you
manage at home on your own but going out is impossible. As soon as you
know you have to go out, you get anxious. Your heart races and you get
diarrhoea. You start to shake and feel a lump in your throat. At times it even
becomes difficult to breathe. As soon as you decide not to go out, everything
calms down and you feel much better.
As a result you are staying home more and more. Life is just easier when
you don’t try to go out. You have adapted by using the phone to pay bills and
your daughter has been good about helping with the shopping. Your daughter
is concerned that you have not had a check-up with the doctor and so has
persuaded the GP to do a home visit.
You would like to go out as you miss your friends but feel helpless and do
not know how to get over your problems.

The following information is on your medical record:


Past medical history
Cholecystectomy 1994
Medication
Nil
Allergies
Nil known
Immunisations
Nil known
Social history
Widowed
Non-smoker.

Suggested approach to the case


Introduction, establish rapport
Open-ended questions to explore Phyllis’s concerns about her situation.

Specific questions
Establish history of agoraphobia (anxiety regarding public places)
Explore bereavement and possible prolonged grief reaction
Exclude depression
Confirm diagnosis and possible link to grief.

Suggestions for therapy Page 246

Psychological—grief counselling, relaxation techniques, anxiety


management, cognitive behavioural therapy, graded exposure
Medication—beta-blockers, selective serotonin reuptake inhibitors (SSRIs)
Referral to local mental health team
Mental health care plan
Plan follow-up to include review of physical health.

CASE COMMENTARY
Agoraphobia is a disabling condition that GPs may have limited
experience of—the illness itself prevents sufferers from
attending for help. The illness can be improved with treatment.
Initially home visits may be the only way to provide assistance
for the agoraphobia, preventive health care and the management
of other acute or chronic problems.
In this case, it is important for the doctor to establish rapport
with Phyllis and demonstrate empathy for her problem.
Cognitive behavioural therapy (CBT) is more effective—and
more cost-effective—than medication. Phyllis may be able to
access CBT via a mental health care plan. Free online access to
CBT training is another option, for example, MoodGYM (http://
moodgym.anu.edu.au) or www.emhprac.org.au. Developing new
skills such as mindfulness meditation can help.
SSRIs and TCAs are equal in efficacy with SSRIs and are
more often used because of their relative lower side effect
profile and safety in overdose. Benzodiazepines are not
recommended first line due to side effect profile and the risk of
dependence. However, they are as effective as SSRIs and may
have a place in specific situations, such as enabling Phyllis to
leave the house for an important event.

COMMON PITFALLS

A doctor who is not able to establish rapport with Phyllis will


fail to elicit the full list of symptoms from her. This most often
occurs when doctors bombard patients with questions rather than
listening carefully to their story. Another pitfall is telling Phyllis
that she must come to the surgery for check-ups soon. This fails
to recognise her illness and risks making her feel worse.
Similarly, advising that she should pull herself together or that a
cure is guaranteed are unrealistic and unhelpful.
Note: in the time available for a short case the doctor Page 247
is unlikely to be able to give details of the treatment options as
detailed above. It would be sufficient to mention referral,
medication and psychological assistance.

Further reading
Andrews, G et al. 2018, ‘Royal Australian and New Zealand College of
Psychiatrists clinical practice guidelines for the treatment of panic
disorder, social anxiety disorder and generalised anxiety disorder’,
Australian and New Zealand Journal of Psychiatry, vol. 52, no. 12, pp.
1109–72.
Lampe, L 2013, ‘Drug treatment for anxiety’, Australian Prescriber, vol. 36,
no. 6, pp. 186–9.
Taylor, CB 2006, ‘Panic disorder’, British Medical Journal, vol. 332, pp.
951–5.
Page 248

Case 48
Shirley Hill

Instructions for the doctor


This is a long case.
Please take a history from Shirley. You are expected to tell her your
diagnosis and negotiate a management plan with her.

Scenario
A young mum, Shirley Hill, has arranged for a neighbour to look after
her children so she can see a GP. She has been feeling tired all the time
since the birth of her second child. She has just moved to the area. Just
before she moved she went to her previous GP who did the following
tests for tiredness which were all normal: FBC, ESR, UEC, TFTs, BSL,
iron studies and urine for MCS.
The following information is on her summary sheet, which she brought
from her previous GP:

Past medical history


First child, two years ago
Second child, six months ago
Medication
Nil
Allergies
Nil known
Immunisations
Up-to-date
Cervical screening test
Normal this year
Social history
Ex-nurse
Husband—Bill, a businessman, travels interstate regularly Page 249
Non-smoker
Alcohol intake—30 standard drinks per week recorded three years ago.

Instructions for the patient, Shirley Hill


You used to work as a nurse. You enjoyed your work as a clinical nurse
consultant at a major teaching hospital. You met Bill when he was just
starting up his own business. You brought in the regular income so he could
spend time promoting the company before it became profitable. Over the last
few years the business has thrived but has taken up more and more of his
time. Because he is away so much you have had to give up your casual shifts
at the local hospital.
Since the birth of your second child you have been feeling exhausted. You
are getting some sleep but not as much as you need. You fall asleep as soon
as your head hits the pillow. You are not refreshed when you wake up. You
often get up around 3–4 am, sometimes it’s for a breastfeed, but lately your
baby is sleeping through and you’re still waking up. It’s difficult to get to
sleep after this. You are finding it difficult to concentrate and have forgotten
to pay a phone bill and power bill lately, which is really out of character.
You always wanted to be a mum, but it feels like more of a chore at
present, which you feel guilty about. Other mums seem to have it all together.
You don’t want to admit it, but often you shout at the children and spend
hours wondering what’s gone wrong in life. You have a beautiful home and
wonderful children but feel miserable and resentful and just can’t be bothered
any more.
You don’t have much of an appetite lately and are still losing your baby
weight.
You have no friends to share child care with as you did before. Your
relationship with Bill is almost non-existent because he is away so much. If
asked sensitively you will admit that you have thought that the children
would be better off without you but have not made any suicide attempts or
plans.
If the doctor asks you if you could get extra help, you think maybe your
Mum would come from interstate for a while but you haven’t really
considered anyone helping. You also have enough money to get some paid
help.
You’d be keen to try medication if this is suggested by the doctor.

The following information is on your medical record, which you brought


from your previous GP:
Past medical history
First child, two years ago
Second child, six months ago
Medication Page 250
Nil
Allergies
Nil known
Immunisations
Up-to-date
Cervical screening test
Normal this year
Social history
Ex-nurse
Husband—Bill, a businessman, travels interstate regularly
Non-smoker
Alcohol intake—30 standard drinks per week recorded three years ago.

Instructions for the facilitator


Explain full physical examination is normal and ask the doctor to continue
with the scenario
Edinburgh Postnatal Depression Score—16 (high likelihood of depression)
If asked for a different tool give findings ‘high likelihood of depression’.

Suggested approach to the case


Establish rapport
Open questions to explore Shirley’s ideas, concerns and expectations.

Specific questions
Mood
Tearfulness
Function and energy
Sleep—terminal insomnia present
Appetite
Interest in life
Relationships
Libido
Social concerns—money, social supports
History of abuse
Use of drugs or alcohol
Brief systems review—weight changes, thirst, fever
Question about delivery, feeding and current health of baby
Risk assessment—suicidal ideation Page 251
Ask about and assess the safety of the children
Past or family history of depression.
Ask for the examination findings
All normal
EPDS or other depression scale.

Management
Explain most likely diagnosis—postnatal depression
Investigations—no more needed than as above
Psychoeducation regarding diagnosis
Options for treatment
— Counselling, support groups, online resources
— Greater interaction with community, e.g. mothers’ group, playgroups
etc.
— Mobilising social/family support in the short term
— Drug therapy—SSRIs first line (fluoxetine and paroxetine best
avoided in the peripartum period)
— Cognitive behavioural therapy with psychologist
— Regular exercise
Discuss situation with Bill
Consider referral to psychiatrist if not improving
Offer mental health plan
Arrange early review.

CASE COMMENTARY

The doctor needs to diagnose and assess the severity of Shirley’s


depression, consider her safety and that of her children. Next is
negotiation with Shirley regarding the different management
options and exploring supports that Shirley can access. The
doctor can be optimistic that the situation will improve but must
be realistic that it will take time.
Studies in Australia show one in ten women experience
depression during pregnancy and one in seven during the year
after pregnancy. Mental health conditions in the perinatal period
often go undiagnosed and can have deleterious effects on both
the Mum, baby and family unit. Screening of all pregnant and
post-partum women will help increase rates of diagnosis, using
screening tools such as the Edinburgh Post Natal Page 252
Depression Scale (EPDS). Women scoring 13 or higher
on the EPDS require further assessment. Consider more
culturally appropriate tools where available, such as the
Kimberley Mum’s Mood scale, a tool validated for use with
Aboriginal women in the Kimberley region of Western
Australia.

COMMON PITFALLS

A poor or inexperienced doctor will miss that this is postnatal


depression and organise further tests or a specialist referral to
chase a physical cause for the tiredness. This is an inappropriate
use of resources and denies Shirley access to the assistance she
needs to improve her mental health and the short-term and long-
term health of her children.

Further reading
Austin, M-P, Highet, N & the Expert Working Group 2017, ‘Mental health
care in the perinatal period: Australian clinical practice guideline’,
Centre of Perinatal Excellence, Melbourne.
Goldin Evans, M, Phillippi, S & Gee RE 2015, ‘Examining the screening
practices of physicians for postpartum depression: implications for
improving health outcomes’, Women’s Health Issues, vol. 25, no. 6, pp.
703–10.
Marley, JV, Kotz, J, Engelke, C, Williams, M, Stephen, D, Coutinho, S, et al.
2017, ‘Validity and acceptability of Kimberley mum’s mood scale to
screen for perinatal anxiety and depression in remote Aboriginal health
care settings’, PLoS ONE vol. 12, no. 1, pp. e0168969. http://doi:10.137
1/journal.pone.0168969.
Therapeutic Guidelines Ltd 2014, ‘Psychiatric conditions in pregnancy and
the postpartum’. In eTG Complete. Available at: http://online.tg.org.au.
Page 253

Case 49
Monica Middlethorpe

Instructions for the doctor


This is a short case.
Please take a history and suggest your management plan to Monica. You
are Monica’s usual GP.

Scenario
Monica Middlethorpe is 36 years old and comes to see you for assistance
with claustrophobia. She is planning a trip to Europe and is worried
about the flight.

The following information is on her medical record:


Past medical history
Asthma
Medication
Fluticasone/salmeterol 250 mcg/25 mcg per dose (Seretide) inhaler
1 puff twice daily
Salbutamol 100 mcg/dose (Ventolin) inhaler 2 puffs as required
Allergies
Nil
Immunisations
Nil recorded
Family history
Nil recorded
Social history
Administrative assistant (ex-tour guide)
Non-smoker.

Instructions for the patient, Monica Page 254

Middlethorpe
You are 36 years old. You work as an administrative assistant and used to be
a tour guide.
You now get anxious when in enclosed spaces. It starts with butterflies in
your stomach, then your chest feels tight, your heart races, you feel
frightened and you have a strong urge to run out into an open space. You
recently flew interstate and your husband was quite upset with you when you
almost didn’t get on the plane home. You drank a couple of glasses of wine
in the bar before take-off and somehow managed to fly.
You are planning a trip to Europe in six weeks and are worried about the
flight. You have come to see your usual GP for help.
If the doctor asks sensitively, you will reveal that three years ago you had
an asthma attack when leading a tour group in a cave. You did not have any
salbutamol with you and became unwell and anxious. You had to change jobs
because it was so awful. Since then you have been afraid of closed spaces and
realised, when planning this trip, how much it is affecting your life.

The following information is on your medical record:


Past medical history
Asthma
Medication
Fluticasone/salmeterol 250 mcg/25 mcg per dose (Seretide) inhaler
1 puff twice daily
Salbutamol 100 mcg/dose (Ventolin) inhaler 2 puffs as required
Allergies
Nil
Immunisations
Nil recorded
Family history
Nil recorded
Social history
Administrative assistant (ex-tour guide)
Non-smoker.

Suggested approach to the case


Establish rapport
Explore Monica’s ideas, concerns and expectations.

Assessment of the problem


Ask about recent event on flight
Sensitively probe for possible precipitating event Page 255
Explain correlation between significant event and current symptoms
Exclude underlying generalised anxiety or depression or substance misuse.

Management
Psychoeducation and reassurance that help is available
Short term—can use benzodiazepines for specific situations but risk of
addiction, not suitable for long-term use
Longer term—cognitive behavioural therapy (CBT)
Offer online resources available via online portal
www.mindhealthconnect.org.au
Review or arrange for review of chronic disease
— Asthma control, inhaler technique
— Need for flu vaccine.

CASE COMMENTARY

Monica describes anxiety attacks when in confined spaces


(claustrophobia) that started when she was underground and had
an asthma attack without having her inhaler nearby. Since that
incident she has changed jobs and avoided closed spaces
whenever possible. However, she now has the chance to travel
and does not want to miss out.
In the short-term, benzodiazepines are effective anxiolytic
drugs. However, patients can become reliant on them, so
alternative ways to manage the anxiety in the long-term are
preferred.
CBT is first-line treatment in anxiety disorders and can be
delivered in a variety of ways, depending on Monica’s
preference. A GP Mental Health Care Plan would assist with the
cost of face-to-face CBT with a psychologist, and online
programs or telephone CBT have been proven effective also.
Anxiety is common and disabling with an estimated 1.6
million presentations to Australian GPs per year. It is important
for the doctor to establish that Monica’s anxiety is specific to a
situation and is not part of a more pervasive mental health
problem. Strategies used to cope with anxiety include ‘self-
medicating’ with alcohol or illegal drugs, such as marijuana, or
avoidance of likely precipitating events or functions. It is worth
checking that Monica’s couple of drinks prior to her most recent
plane trip was a one-off, not a routine part of life, and assessing
how much she has altered her life to avoid an anxiety attack.
COMMON PITFALLS Page 256

Beta-blockers can be effective in reducing the physical


symptoms of anxiety and are a useful first-line therapy EXCEPT
in asthmatics like Monica, for whom they can cause fatal
bronchoconstriction.

Further reading
Andrews, G, et al. 2018, ‘Royal Australian and New Zealand College of
Psychiatrists clinical practice guidelines for the treatment of panic
disorder, social anxiety disorder and generalised anxiety disorder’,
Australian and New Zealand Journal of Psychiatry, vol. 52, no. 12, pp.
1109–72.
Bassilios, B, Pirkis, J, King, K, Fletcher, J, Blashki, G & Burgess, P 2014,
‘Evaluation of an Australian primary care telephone cognitive
behavioural therapy pilot’, Australian Journal of Primary Health, vol.
20, no. 1, pp. 62–73.
Handbook of Non Drug Intervention (HANDI) Project Team 2013, ‘Internet-
based cognitive behaviour therapy for depression and anxiety’,
Australian Family Physician, vol. 42, no. 11, pp. 803–4.
Lampe, L 2013, ‘Drug treatment for anxiety’, Australian Prescriber, vol. 36,
no. 6, pp. 186–9.
Orman, J, O’Dea, B, Shand, F, Berk, M, Proudfoot, J & Christensen, H 2014,
‘e-Mental health for mood and anxiety disorders in general practice’,
Australian Family Physician, vol. 43, no. 12, pp. 833–7.
Page 257

Case 50
Tom Newton

Instructions for the doctor


This is a short case.
Please read the scenario below. Ann Newton has made an appointment
with you to discuss her son’s recent hospital admission. Please respond to
Ann’s questions as you would in clinical practice.

Scenario
Last week you were on call and admitted Tom Newton to the local
mental health unit. Tom is 19, lives with his parents and is in second
year, studying engineering at university.
Tom’s parents called you as his behaviour was markedly disturbed;
they suspect Tom has been using illicit drugs. For the last week the
television had been telling Tom that it was his role to save the world
from itself. When Tom said that the TV had told him that he had to die to
save the world, his parents sought help.
The hospital has told Tom’s parents that he has had an acute
psychosis. Tom’s mother, Ann, has made this appointment to talk with
you about psychosis; she has heard from a friend that Tom is at risk of
schizophrenia.

Instructions for Tom’s mother, Ann Newton


You are nearly 60 years old and about to retire from your position as a school
secretary. When you retire you are planning to travel around Australia.
Your eldest daughter is married and lives nearby. Tom, your son, is 19 and
still at home. He is in his second year of an engineering degree. Some of
Tom’s university friends seem weird to you and you suspect Tom has been
trying illicit drugs.
Over the past few weeks Tom’s behaviour had become increasingly
worrying—he was saying that the television wanted him to save the world.
When Tom announced that the TV had told him that he had to die to Page 258
save the world, you and your husband decided to take Tom to the
local GP.
Tom was admitted to the mental health unit at the hospital. The
psychiatrist has said that Tom has had an acute psychosis. A friend told you
that acute psychosis can lead to schizophrenia. You have made this
appointment with the GP to find out more about this illness. You would like
to know the answers to the following questions:
1. What is an acute psychosis?
2. What is schizophrenia?
3. Can it be cured?
4. What causes it?
5. Was it my fault?
6. What will happen to Tom now?
7. Will I still be able to travel around Australia as planned?
Suggested approach to the case
Establish rapport
Enquire about Tom’s mother’s ideas, concerns and expectations regarding
acute psychosis and schizophrenia
No clear diagnosis yet—one psychotic episode is not diagnostic of
schizophrenia, but acknowledge that Tom is at risk of schizophrenia
Explain that you can only talk in general terms, not specifically about Tom,
because of confidentiality requirements
Target answers to Ann’s concerns
— Dispel myths, e.g. not split personality
Diagnostic features
— Gradual change (prodrome usually about two years)
— Changes in thinking, mood and behaviour
Positive symptoms
— Hearing voices or seeing things that are not there (hallucinations)
— Feeling controlled by someone or something else (passivity
phenomena)
— Believing things to be true that are not (delusions)
— Thought disorders
Negative symptoms
— Abnormal affect
— Talking less (poverty of speech and poverty of content of speech)
— Reduced motivation
— Social withdrawal
Causes Page 259
— Multifactorial, combination of genetic susceptibility and
environmental factors
— Link with illicit drugs such as cannabis and amphetamines
Advise of improved outcomes with early intervention
Advise of current therapies
Treatment
— Aim to reduce symptoms, prevent relapse, early intervention of
relapse
— Antipsychotics
— Family support
— Community mental health team
Acknowledge any emotional response, e.g. shock and sadness
Assure Ann of your ongoing interest and support, and support groups for
families caring for the mentally ill
Acknowledge planned trip around Australia, too early to make definite
decision, continue to make plans to go.

CASE COMMENTARY

Acute psychosis is frightening for all involved. Tom’s and his


family’s world has suddenly changed; his mother, Ann, needs
support and space to express her concerns and adjust to the new
situation. Ann also needs help trying to decide what to do about
her plans for travelling around Australia after retirement.
Without Tom’s permission the GP cannot give specific
information to Ann but instead has to give general advice.
Evidence-based guidelines on early psychosis suggest
supporting the family with information about psychotic
disorders and recovery.1
A single episode of psychosis does not necessarily lead to
schizophrenia. Early intervention can help people maintain
cognitive and social function.2 Relapse prevention includes
stopping illicit drug use 3 and promoting a healthy lifestyle
through social support, physical care and vocational
rehabilitation.4

COMMON PITFALLS
In practice, especially when as a GP you care for whole families,
it can be hard to remember who has given permission for you to
say what to whom. Things can never be ‘unsaid’ so, if Page 260
anything, it is wise to be cautious rather than overly
open. Conversely, saying nothing at all is unhelpful, as evidence
suggests that Tom will benefit from his family’s support.1, 3

References
1. Early Psychosis Guidelines Writing Group and EPPIC National Support
Program 2016, Australian Clinical Guidelines for Early Psychosis, 2nd
ed, update, Orygen, The National Centre of Excellence in Youth Mental
Health, Melbourne, Vic.
2. Stafford, MR, Jackson, H, Mayo-Wilson, E, Morrison, AP & Kendall, T
2013, ‘Early interventions to prevent psychosis: systematic review and
meta-analysis’, British Medical Journal, vol. 346, p. f185.
3. Lee, HE & Jureidini, J 2013, ‘Emerging psychosis in adolescents—a
practical guide’, Australian Family Physician, vol. 42, pp. 624–7.
4. Fraser, R, Berger, G, Killackey, E & McGorry, P 2006, ‘Emerging
psychosis in young people: Part 3—key issues for prolonged recovery’,
Australian Family Physician, vol. 35, pp. 329–33.
Page 261
Section 15
Musculoskeletal
medicine
Page 262

Case 51
Anthony Campbell

Instructions for the doctor


This is a short case.
Please take a history from Anthony and conduct a focused examination.
Outline the most likely diagnosis and your proposed management to the
observing examiner.

Scenario
Anthony Campbell is a 46-year-old Aboriginal man who is complaining
of pain in his right foot. He sprained his ankle when he was leaving a
council meeting a few months ago. For the last few weeks he has
experienced pain first thing in the morning, and after standing or walking
for a period of time.
The following information is on Anthony’s summary sheet at your
suburban general practice:
Past medical history
Hypertension
Medication
Ramipril 5 mg od
Allergies
Nil known
Immunisations
Nil recorded
Family history
Nil known
Social history
Married Works as a finance officer for the local town council
Non-smoker
Infrequent alcohol consumption.

Instructions for the patient, Anthony Page 263

Campbell
You are a 46-year-old Aboriginal man. You work as a finance officer for the
local council. You have come to see the GP today because of a pain in your
right heel. You experience sharp intense pain when you put your heel down
first thing in the morning or after sitting.
The pain eases after you have been up for a while, but it gets worse after
walking any distance. It is worse when you walk around barefoot.
The pain is getting worse, not better. You first noticed it a few weeks ago.
A few months ago, you sprained your ankle when coming out of a council
meeting late at night. Your ankle discomfort had seemed to have completely
resolved, but you wonder if the two problems are connected (they turn out
not to be).

The following information is on your medical record:


Past medical history
Hypertension
Medication
Ramipril 5 mg once a day
Allergies
Nil known
Immunisations
Nil recorded
Family history
Nil known
Social history
Married
Works as a finance officer for the local town council
Non-smoker
Infrequent alcohol consumption.

Suggested approach to the case


Introduction
Establish rapport.

History
Use open questions to explore Anthony’s ideas, concerns and expectations.

Specific questions1
Location of the pain—plantar fascia insertion right foot
Duration of the pain
Timing Page 264
What’s been tried so far?
Injury to ankle—sprain after council meeting, location of current pain
suggests no connection
Exclude other musculoskeletal/systemic symptoms1
Request permission to examine.

Examination
Height 1.85 m
Weight 99 kg
BMI 29 kg/m2
Expose both feet and ankles to the knees
Look
— Alignment and range of movement normal
— No scars or skin changes
— Observe gait—some pain on putting down right heel
Feel
— Acutely tender right calcaneum at insertion of plantar fascia, no
lateral/medial calcaneal tenderness
— No other tenderness or increased temperature
Move
— Normal foot and ankle movements
— No excess joint laxity.

Most likely diagnosis


Plantar fasciitis.

Management
Explanation
Reassurance not caused by ankle injury
Treatment
— Stretching exercises—of plantar fascia and Achilles tendon2
— Arch supports and heel cushions
— Ice
— Non-steroidal anti-inflammatory medication, if needed, but caution
because on Ramipril for hypertension
— Avoid walking barefoot or wearing flat shoes
If not settling, consider referral/steroid injection or trial of custom-made night
splints.
CASE COMMENTARY Page 265

It is important to take the history of the recent fall but to realise


that this is not the actual cause of the problem and explain this to
Anthony. Diagnosis of plantar fasciitis is primarily based on
history and physical examination.3 The presence or absence of
heel spurs is not helpful in diagnosing plantar fasciitis; imaging
is needed only if other diagnoses seem more likely.
Most patients with plantar fasciitis improve with conservative
or no therapy. Anthony can try rest, shoe inserts, ice massage
and techniques that stretch the plantar fascia. His higher risk of
renal disease suggests caution with non-steroidal anti-
inflammatories and there is limited evidence of their
effectiveness with the condition.4 Custom-made orthotics and
night splints can be useful, as may shock therapy and steroid
injections, although these may risk longer-term tendon rupture
and pain relief may only last four weeks.5 Failed initial
treatment is an indication for specialist surgical referral. Some
cases persist despite surgery.
Anthony may benefit from encouragement regarding activity
modification and suggestions on exercise he can do that will not
exacerbate his heel pain, such as cycling or swimming. Losing
weight will reduce the load on the ankle and can help. A
thorough candidate will arrange a follow-up to check Anthony’s
blood pressure, BMI, other cardiac risk factors, kidney function
and immunisation status.

References
1. Rio, E, Mayes, S & Cook, J 2015, ‘Heel pain: a practical approach’,
Australian Family Physician, vol. 44, no. 3, pp. 96–101.
2. The Royal Australian College of General Practitioners 2018, ‘Stretching
exercises for plantar fasciitis’, Handbook of Non-Drug Interventions
(HANDI), RACGP, Melbourne, Vic.
3. Goff, JD & Crawford, R 2011, ‘Diagnosis and treatment of plantar
fasciitis’, American Family Physician, vol. 84, no. 6, pp. 676–82.
4. Covey, CJ & Mulder, MD 2013, ‘Plantar fasciitis: how best to treat?’
Journal of Family Practice, vol. 62, no. 9, pp. 466–71.
5. McMillan, AM, Landorf, KB, Gilheany, MF, Bird, AR, Morrow, AD &
Menz, HB 2012, ‘Ultrasound guided corticosteroid injection for plantar
fasciitis: randomised controlled trial’, British Medical Journal, vol. 344,
p. e3260.
Page 266

Case 52
Martin Chatterjee

Instructions for the doctor


This is a short case.
Please do a focused clinical examination. You are not required to take any
further history. Tell the examiner your examination findings, your differential
diagnosis and initial plans for management.

Scenario
Martin Chatterjee is a 48-year-old office manager who does little regular
exercise. Last week he played Masters’ cricket and scored a career best
of 32 runs. He does not recall specifically injuring his shoulder, but has
since had pain in his left (non-dominant) shoulder. He is finding it
difficult to move his left arm and to sleep, as the pain wakes him up
whenever he tries to move. He has applied ice and is taking paracetamol
at night. This is his first experience with a shoulder problem.
The following information is in Martin’s medical record:
Medical history
Type 2 diabetes diagnosed two years ago
Height 1.85 m
Weight 106 kg
BMI 31 kg/m2
Waist circumference 95 cm
Medication
Metformin 500 mg bd Paracetamol 500 mg 2 nocte prn
Allergies
Nil
Social history Page 267
Office manager
Stopped smoking two years ago.

Instructions for the patient, Martin Chatterjee


You are a 48-year-old office manager who does little regular exercise. Last
week you played Masters’ cricket and scored a career best of 32 runs. Since
then you have had pain in your left (non-dominant) shoulder. It is difficult to
move your left arm and to sleep as the pain wakes you up whenever you try
to move. You have applied ice and are taking paracetamol at night. This is
your first experience with a shoulder problem.

The following information is on your medical record:


Medical history
Type 2 diabetes diagnosed two years ago
Height 1.85 m
Weight 106 kg
BMI 31 kg/m2
Waist circumference 95 cm
Medication
Metformin 500 mg bd Paracetamol 500 mg 2 nocte prn
Allergies
Nil
Social history
Office manager
Stopped smoking two years ago.
Please wear a shirt that buttons all the way down the front. You have no
difficulty unbuttoning the shirt but struggle to take your shirt off (removing
your arms from the sleeves) because of the pain in your left shoulder.
You have no bony tenderness. You have difficulty abducting your left
shoulder. When the doctor gets your shoulder beyond the 120° angle you can
then raise your arm and the pain eases.
All other shoulder movements are normal.

Instructions for the facilitator


Please report to the doctor that neck/back movements and axillae are normal
when they start to examine these. This is to save time.

Suggested approach to the case Page 268

Introduction
Brief summary of the case by the candidate leading to request to examine the
patient.

Examination1
Ask patient to remove shirt completely and observe for pain and restricted
movement.

Look
Inspect anterior and posterior for
— Asymmetry—expect non-dominant shoulder to be higher than
dominant
— Bruising
— Scars
— Muscle wasting.

Feel
Bones and joints
— Sternoclavicular joint
— Clavicle
— Acromioclavicular joint
— Glenohumeral junction
— Humerus
— Scapula.
Muscles and tendons
— Subscapularis muscle, teres minor muscle
— Supraspinatus and infraspinatus muscles
— Long head of biceps
— Pectoralis muscle
— Deltoid muscle.

Move
Active and passive examination
— Forward flexion
— Extension
— Abduction—difficult to initiate abduction, painful arc Page 269
between 60° and 120°
— Adduction
— Internal rotation
— External rotation
— Circumduction—limitation at lateral arc
— Neck/back movements and axillae (facilitator, report to doctor results
are normal to save time).
Repeat with movement resisted
— Finding: pain on resisted abduction suggesting supraspinatus problem
— Test supraspinatus—resist abduction with thumb pointing upwards
— Test infraspinatus—resist abduction with thumb pointing downwards.

Apprehension test
Patient lying down, supine, arm externally rotated with elbow flexed to 90°.

Differential diagnosis
Most likely diagnosis—supraspinatus tendinopathy causing impingement
(candidate will still pass if says rotator cuff inflammation/tendinopathy)
Consider rotator cuff tear or adhesive capsulitis
Ex-smoker—consider risk of lung tumour.

Management
Continue rest, ice
Analgesia
Explain—inflammation should settle and resolve
Physiotherapy2
Prevent frozen shoulder (more common in diabetics and 40–60 year olds)2
Discuss safety driving
Need for certification for work
No indication for other investigations at this stage, do CXR if pain
persists/worsens
Follow-up, encourage continued fitness and weight loss and monitor diabetes
Consider subacromial corticosteroid injection if symptoms persist.3

CASE COMMENTARY Page 270

Shoulder pain is a common presentation in general practice.


Most often the pain is musculoskeletal in origin but it can also
signify serious visceral pathology. General practitioners must be
able to conduct a thorough shoulder examination, report their
findings clearly and synthesise the information from the history
and examination to form a differential diagnosis.1, 2

COMMON PITFALLS

Finding pathology on ultrasound is common and does not


necessarily correlate well with the clinical picture. Research
suggests that GPs are over-reliant on shoulder imaging.4 In
many cases a focused but thorough history and examination
provide sufficient information to form a differential diagnosis
and a management plan.5

References
1. Brun, S 2012 ‘Initial assessment of the injured shoulder’, Australian
Family Physician, vol. 41, no. 4, pp. 217–20.
2. Masters, S 2007, ‘Shoulder pain’, Australian Family Physician, vol. 36,
no. 6, pp. 414–20.
3. Arroll, B & Goodyear-Smith, F 2005, ‘Corticosteroid injections for
painful shoulder: a meta-analysis’, British Journal of General Practice,
vol. 55, no. 512, pp. 224–8.
4. Awerbuch, MS 2008, ‘The clinical utility of ultrasonography for rotator
cuff disease, shoulder impingement syndrome and subacromial bursitis’,
Medical Journal of Australia, vol. 188, no. 1, pp. 50–3.
5. Johal, P, Martin, D & Broadhurst, N 2008, ‘Managing shoulder pain in
general practice: assessment, imaging and referral’, Australian Family
Physician, vol. 37, no. 4, pp. 263–5.
Page 271

Case 53
Sarah Cosgrove

Instructions for the doctor


This is a short case.
Please examine Mrs Cosgrove, who presents with hip pain. You are not
required to take any further history. Please tell the examiner your
examination findings, your differential diagnosis and initial plans for
management.

Scenario
Sarah Cosgrove is 71 years old and has come to see you complaining of
pain in her right hip. This pain has been gradually getting worse over the
past few years. The pain is beginning to interfere with her daily activities
and she gets some stiffness after she has rested. She still cycles to do her
shopping.
Mrs Cosgrove has pain in both first metacarpophalangeal joints, both
knees and her neck.

The following information is on her summary sheet:


Past medical history
Appendicectomy as a child
Medication
Nil
Allergies
Nil
Immunisations
Up-to-date
Cervical screening test and breast screen
Normal one year ago
Social history
Married
Two children, one overseas
Alcohol—two standard drinks per week Page 272
Non-smoker.

Instructions for the patient, Sarah Cosgrove


You are a 71-year-old retired personal assistant. You have always been active
and still cycle to do your shopping. You have come to see the GP today
because of pain in your right hip. This pain has been gradually getting worse
over the past few years. Your symptoms now interfere with your daily
activities and you experience some stiffness after resting.
You also have pain in both first metacarpophalangeal joints, both knees
and your neck.
Both your older brothers have had hip replacements because of
osteoarthritis. You suspect this is what you have but you are concerned about
possible surgery because one brother had serious postoperative complications
and nearly died.

Clinical examination findings


On examination you will indicate that the pain is felt centrally in the right
inguinal canal. Please demonstrate minimal tenderness on palpation over the
greater trochanter of the right hip and reduced hip movement on abduction,
internal rotation and extension. When the doctor asks to examine your back,
other hip and knees please report that this is not necessary.

The following information is on your medical record:


Past medical history
Appendicectomy as a child
Medication
Nil
Allergies
Nil
Immunisations
Up-to-date
Cervical screen test and breast screen
Normal one year ago
Social history
Married
Two children, one overseas
Alcohol—two standard drinks per week
Non-smoker.

Information for the facilitator Page 273

Alternative scenario
The doctor is told that Mrs Cosgrove has osteoarthritis of the right hip. They
are required to focus on management. The emphasis would be on ensuring
Mrs Cosgrove understands the problem and chooses with the doctor the
options for treatment. The doctor should notice hesitation when surgery is
discussed and explore the rationale for her concern.

Suggested approach to the case


Introduction
Request permission to examine.

Examination
161cm, 68 kg
BMI 26 kg/m2
Timed Up and Go1
Gait
Posture, look for Trendelenburg’s sign

Hip and knee


Look—scars, swelling, deformity
Feel—temperature, crepitus, effusion
Move
— Active then passive
— Rotation internal and external
— Flexion/extension
— Abduction/adduction.

Most likely diagnosis


Osteoarthritis of hip.

Management2
Aim to minimise pain and maintain function
Maintain fitness and exercise
Symptomatic treatment
— Analgesia: paracetamol as first line, NSAIDs as second line
Physiotherapy, hydrotherapy, walking stick
Referral for joint replacement when pain is intractable
Arrange follow-up.
CASE COMMENTARY Page 274

The doctor needs to establish a rapport with Mrs Cosgrove and


request permission to examine her. Some doctors I’ve watched
have achieved this by briefly summarising the story to show Mrs
Cosgrove that they understand what the issues are.
The doctor must conduct the examination efficiently, moving
with ease from one aspect to the next and not cause unnecessary
pain. The examination should appear as a well-rehearsed
routine; it is obvious to observers when doctors are thinking
what to do next rather than being able to focus on the findings
and their interpretation.
The most likely diagnosis is osteoarthritis of the right hip,
which does not need further investigations at this stage. The
doctor should suggest a range of non-pharmaceutical,
pharmaceutical and surgical options for treatment.2, 3
Strengthening and flexibility exercises can reduce pain and
maintain function.4 Glucosamine and acupuncture2 may not be
more effective than a placebo, and there is evidence that the
small-to-moderate beneficial effects of non-tramadol opioids are
outweighed by large increases in the risk of adverse events.5

References
1. Waldron, N, Hill, A & Barker, A 2012, ‘Falls prevention in older adults.
Assessment and management’, Australian Family Physician, vol. 41, no.
12, pp. 930–5.
2. The Royal Australian College of General Practitioners 2018, ‘Diagnosis
and management of hip and knee osteoarthritis algorithm’. In: Guideline
for the management of knee and hip osteoarthritis, 2nd ed, RACGP, East
Melbourne, Vic, p. 64.
3. McKenzie, S & Torkington, A 2010, ‘Osteoarthritis. Management options
in general practice’, Australian Family Physician, vol. 39, no. 9, pp. 622–
5.
4. Uthman, OA, van der Windt, DA, Jordan, JL, Dziedzic, KS, Healey, EL,
Peat, GM et al. 2013, ‘Exercise for lower limb osteoarthritis: systematic
review incorporating trial sequential analysis and network meta-analysis’,
British Medical Journal, vol. 347, p. f5555.
5. Nuesch, E, Rutjes, AW, Husni, E, Welch, V & Juni, P 2009, ‘Oral or
transdermal opioids for osteoarthritis of the knee or hip’, Cochrane
Database of Systematic Reviews, CD003115.

Further reading
The Royal Australian College of General Practitioners 2018, Guideline for
the management of knee and hip osteoarthritis, 2nd ed, RACGP, East
Melbourne, Vic.
Page 275

Case 54
Jeremy King

Instructions for the doctor


This is a short case.
Please take a history, conduct an appropriate examination and then outline
your management plan to Jeremy.

Scenario
Jeremy King is a 25-year-old plumber who comes to see you for review
following a car accident. He was driving to work when a truck went very
slowly into the back of his car while he was stationary at traffic lights.
He attended the hospital emergency department where he was examined.
The doctor did not think X-rays were needed1 and only did them
following pressure from Jeremy. No fractures were identified.
Jeremy has returned for review a week after the accident. The brake
and indicator light of Jeremy’s work ute needed replacement but
otherwise it was fine.

The following information is on his summary sheet:


Past medical history
Nil
Medication
Nil
Allergies
Nil known
Immunisations
Up-to-date
Social history
Self-employed plumber.

Instructions for the patient, Jeremy King Page 276

You are a 25-year-old self-employed plumber. Your business has been going
badly and you have been getting increasingly tired and frustrated at work.
You have wanted to take time off but cannot afford to.
Last week you were driving to work when a truck went very slowly into
the back of your work ute while you were stationary at traffic lights. You
attended the hospital emergency department where you were examined. The
doctor said that you did not need X-rays but you insisted that they do some;
no fractures were identified.
You have come to see your GP for review a week later.
The brake and indicator light of your work ute needed replacement but
otherwise it was fine.
The truck driver’s insurance company has already arranged for the repairs
to your car, so you could get back to work next week. You have had one
week off but want to take more time out. Your neck was a bit sore after the
accident and you get mild pain at the end of the day. You have no other
residual symptoms.
You want the GP to sign you off for more sick leave.
Clinical examination findings
Clinical examination is normal. There is a full range of pain-free movement.

The following information is on your medical record:


Past medical history
Nil
Medication
Nil
Allergies
Nil known
Immunisations
Up-to-date
Social history
Self-employed plumber.

Suggested approach to the case


Establish rapport
Open-ended questions to establish Jeremy’s ideas and concerns about his
injury, and expectations of this consultation.

Specific questions Page 277

Further history regarding neck pain


— Interference with sleep, activities of daily living
— Treatment so far
Enquire about other associated symptoms, such as headache, paraesthesiae,
weakness
Explore potential impact of neck pain on work as a plumber Request
permission to examine neck.

Examination
Look—normal
Feel—no bony tenderness, no muscular spasm or increased tone
Move—full range of movement.

Management
Explain neck strains, reassure that symptoms will resolve, with no benefit
from further time off
Suggest physiotherapy, simple analgesics; aim to maintain full range of
movement
Empathise regarding business problems, screen for underlying mood disorder
A good doctor will also explore whether Jeremy smokes, drinks or uses
drugs.

CASE COMMENTARY

This case tests the GP’s applied knowledge and skill and also
their professional and ethical role. The car accident and resulting
insurance claim has given Jeremy his first paid time off in years.
He is still tired, and he thinks that more paid leave from the
truck driver’s insurance company would really help him.
A clinical assessment of the neck injury is required plus a
decision about whether further sick leave is indicated. The
minimal residual pain and absence of physical signs demonstrate
that Jeremy is fit to return to work. The doctor must avoid
collusion by endorsing further unjustified sick leave.
Psychological, physiological and financial compensation factors
all influence recovery from whiplash. The GP will need to make
Jeremy feel supported and understood with regard to his Page 278
injury and his business pressures, but at the same time
be clear that there is no justification for further sick leave.
The GP can offer follow-up about the accident, the business
pressures and preventive health.
Reference
1. Ackland, H & Cameron, P 2012, ‘Cervical spine. Assessment following
trauma’, Australian Family Physician, vol. 41 no. 4, pp. 196–201.

Further reading
Ferrari, R 2014, ‘Predicting recovery from whiplash injury in the primary
care setting’, Australian Family Physician, vol. 43, no. 8, pp. 559–62.
Russell, G & Nicol, P 2009, ‘“I’ve broken my neck or something!” The
general practice experience of whiplash’, Family Practice, vol. 26, no.
2, pp. 115–20.
Teichtahl, A & McColl, G 2013, ‘An approach to neck pain for the family
physician’, Australian Family Physician, vol. 42, no. 11, pp. 774–7.
Page 279

Case 55
Geoff Sharp

Instructions for the doctor


This is a short case.
Please take a history from Geoff and conduct an appropriate focused
examination. Outline to the facilitator your findings and suggestions for
management.

Scenario
Geoff Sharp is a 54-year-old teacher. His wife has booked this
appointment as she is fed up with listening to him complain about a pain
in his right elbow. He says that he cannot make her cups of tea, as he
can’t pour the kettle.

The following information is on his medical record:


Past medical history
Nil significant
Medication
Nil
Allergies
Nil recorded
Immunisations
Up-to-date
Social history
Nil recorded.

Instructions for the patient, Geoff Sharp


You are a 54-year-old teacher. Normally you are fit and well and do not go to
the doctor. For the last few weeks, you have noticed a pain in your right
elbow. This makes it hard to carry books at school and you experience pain
when trying to pour the kettle or open doors.
You have not sustained an injury to the elbow but do remember Page 280
that the pain came on after you had spent the weekend gardening.
You have tried some paracetamol but it did not make much difference.

Clinical examination findings


You will demonstrate pain on pressure over the right lateral epicondyle and
on active extension of the wrist. You have tennis elbow—extensor
tendinopathy.

The following information is on your medical record:


Past medical history
Nil significant
Medication
Nil
Allergies
Nil recorded
Immunisations
Up-to-date
Social history
Nil recorded.

Suggested approach to the case


Establish rapport
Open questions to establish Geoff’s ideas, concerns and expectations.

Specific questions
Duration of pain
Onset of pain
Precipitants of pain
General health
Treatment so far
Impact of problem on Geoff’s life
Request permission to examine.

Examination
Ensure full view of both arms
Look
— Exclude deformity or swelling
— Muscle mass
Feel
— Elicit tenderness Page 281
— Exclude temperature increase in right arm
Move
— Test range of movement of elbow joints—active, then passive
— Flexion, extension and pronation/supination
— Test movement against resistance
Pain is maximal on wrist extension against resistance.
Diagnosis
Tennis elbow—extensor tendinopathy.

Management
Education about the diagnosis
Rest
Range of treatment options starting with the least invasive1, 2
— Wringing exercises3, 4
— Topical non-steroidal anti-inflammatory drugs
— Oral non-steroidal anti-inflammatory drugs—exclude
contraindications prior to recommendation/prescription
— Support bandage—epicondylitis brace
— Referral for physiotherapy, ultrasonography treatment
— Steroid/local anaesthetic injection5
— Surgery—tendon transfer2
Occupational therapy review of work situation
Advice about the safety of driving
Arrange follow-up
If time allows, discuss preventive health measures such as smoking, blood
pressure, exercise, diet, alcohol consumption, immunisation status.

CASE COMMENTARY

Geoff’s symptoms should suggest the diagnosis to the doctor


almost before the scenario begins. The doctor can then
demonstrate their skills by excluding other serious pathology
and clinch the diagnosis by identifying the epicondylar pain and
reproduction of the pain on active resistance to wrist extension.
A good doctor will give a clear explanation of the Page 282
problem in simple terms and then outline the options for
treatment, carefully excluding any pre-existing
contraindications. Tennis elbow typically presents after minor
and often unrecognised trauma of the extensor muscles of the
forearm.1 It is a degenerative overuse–underuse tendinopathy of
the common extensor origin of the lateral elbow.2 Rehabilitation
(exercise) based treatment is helpful,3, 4 but to be effective
patients must usually remove tendon overload.2 Many cases of
tennis elbow resolve spontaneously over 6–12 months. Steroid
injection can give rapid pain relief in the short-term but is no
longer recommended following evidence that it results in lower
partial or full complete recovery, and greater recurrence after
one year compared with a placebo injection.5

COMMON PITFALLS

There is time to complete the history, examination and


management but only if the doctor focuses on the elbow. A poor
doctor might be unnecessarily extensive with history-taking and
examination, leaving no time for discussion regarding
management. No investigations are needed for this diagnosis,
nor is early referral to an orthopaedic surgeon.

References
1. The Royal Australian College of General Practitioners 2016, ‘Exercise for
tennis elbow’. In: Handbook of Non-Drug Interventions (HANDI).
Available at: www.racgp.org.au/handi, accessed 1 December 2018.
2. Orchard, J & Kountouris, A 2011, ‘The management of tennis elbow’,
British Medical Journal, vol. 342, p. d2687.
3. Murtagh, J, Rosenblatt, J, Coleman, J & Murtagh, C 2018, Murtagh’s
General Practice, 7th ed, McGraw-Hill Education, Sydney, pp. 726–8.
4. American Academy of Family Physicians 2007, ‘Information from your
family doctor. Exercises for tennis elbow’, American Family Physician,
vol. 76, no. 6, pp. 849–50.
5. Coombes, BK, Bisset, L, Brooks, P, Khan, A & Vicenzino, B 2013,
‘Effect of corticosteroid injection, physiotherapy, or both on clinical
outcomes in patients with unilateral lateral epicondylalgia: a randomized
controlled trial’, Journal of the American Medical Association, vol. 309,
no. 5, pp. 461–9.
Page 283

Case 56
Anna Wong

Instructions for the doctor


This is a short case.
Please take a focused history from Anna Wong, and then request
appropriate examination and investigation findings from the facilitator. Based
on the history, examination and investigation results, please outline your
diagnostic impressions and initial management plan to Anna.

Scenario
Anna Wong is a 32-year-old mother of two. During the appointment for
her four-month-old son’s immunisation last week, she mentioned that
she had been feeling tired and had aching joints. You recommended she
try some simple analgesics and make an appointment to see you. She has
come back today to discuss her symptoms.
The following information is on her summary sheet:
Past medical history
Nil significant
Medication
Nil
Allergies
Nil known
Immunisations
Up-to-date
Social history
Married, two children (four months and two years)
Hairdresser, currently on maternity leave
Non-smoker, non-drinker.

Instructions for the patient, Anna Wong Page 284

Six weeks ago, your left wrist became painful. Initially, you put it down to
the way you were holding your baby to breastfeed. During the following
fortnight your other wrist plus feet, ankles and hands started to hurt and have
been steadily getting worse. You are also feeling really tired and run-down—
a bit like the flu—although you suspect this is due to breastfeeding, lack of
sleep and trying to look after your family. You have not noticed a fever.
Paracetamol does not provide much relief. You have avoided anti-
inflammatories, as the pharmacist said they weren’t safe to take when
breastfeeding.
On specific questioning:
• You think your wrists and feet have been a bit swollen but you’re not
sure. The pain is worse on waking and you have stiffness for the first one
to two hours of the day. You have lost three kg in two months without
specifically trying, which you are very pleased about (getting rid of the
‘baby bulge’).
• You have not travelled anywhere recently and have no other contacts with
similar symptoms.
• You have no personal or family history of auto-immune disease or
allergies. Your only knowledge of arthritis is that it is a ‘wear and tear
disease that old people get’.

The following information is on your summary sheet:


Past medical history
Nil significant
Medication
Nil
Allergies
Nil known
Immunisations
Up-to-date
Social history
Married, two children (four months and two years)
Hairdresser, currently on maternity leave
Non-smoker, non-drinker.

Information for the facilitator


Clinical examination findings
Each aspect needs to be asked for specifically:
Looks tired but has normal colour
BP 125/75 mmHg Page 285
Pulse 82
Temperature 36.8°C
BMI 23.5
No thyroid enlargement
Cardiovascular, abdominal and respiratory examinations unremarkable
Musculoskeletal examination: no joint deformities; wrists, second and third
MCP joints and second, third and fourth MTP joints slightly swollen and
tender bilaterally, and painful on both active and passive movement;
ankles painful but not swollen
No extra-articular signs.
Investigation findings
Each aspect needs to be asked for specifically:
FBC Hb 105, normocytic
ESR 38 (3–9 mm/hour)
CRP 22 (normal <5 mg/L)
Rheumatoid factor 28 (negative <30 IU/L)
Anticyclic citrullinated peptide (anti-CCP) antibody levels positive
Iron studies normal
Ross River virus serology negative
ANA low titre positive
TSH normal
No joint erosions on plain X-rays of hands and feet.

Suggested approach to the case


Develop rapport and show empathy regarding effects of symptoms on
activities of daily living.

Specific questions for patients presenting with arthralgia


Onset—acute, subacute vs chronic
Temporal pattern
— Migratory
— Additive
— Episodic/intermittent
Number of joints affected—1, 2–4, >5
Distribution of joint involvement
— Proximal vs distal
— Small vs large
— Symmetrical vs asymmetrical Page 286
— Spinal vs peripheral
Inflammatory vs non-inflammatory
Systemic symptoms
Extra-articular
— Ocular involvement
— Skin lesions
— Nail dystrophy
— Rheumatoid nodules.

Examination
Systematic approach
General examination plus specific examination of thyroid, skin (rash, bruises,
nodules), nails and eyes
Joint examinations should include all affected joints, looking for swelling,
deformity, tenderness and range of motion (active and passive).

Management
Recognise that clinical picture plus anti-CCP positive indicates diagnosis of
rheumatoid arthritis
Arrange rheumatological referral
Relief of joint symptoms (seeking rheumatological opinion if needed)—e.g.
paracetamol, NSAIDs/COX-2, prednisone
May consider starting DMARD (e.g. methotrexate), in conjunction with
specialist advice if early appointment with rheumatologist is not possible
If medication is recommended, provide appropriate advice about
breastfeeding
Patient information and education (e.g. referral to patient information and
support service organisations, such as Arthritis Australia and the
Australian Rheumatology Association)
Patient support regarding coping with diagnosis
Provide opportunity for questions
Arrange follow-up appointment.

CASE COMMENTARY
This case assesses whether candidates:
• have an efficient and structured approach to conducting the
history and examination on a patient presenting with
polyarthralgia
• rationally and appropriately investigate suspected Page 287
rheumatoid arthritis, and correctly interpret the test
results
• provide appropriate patient education and support, after
sensitively explaining the diagnosis
• refer this patient for specialist care in addition to considering
immediate treatment.
The key features1 that together raise suspicion of rheumatoid
arthritis in Anna are:
• persistent joint pain and swelling (>6 weeks)
• joint pain and swelling affecting at least three joints
(including at least one small joint)
• symmetrical involvement of MCP/MTP joints
• morning stiffness for more than 30 minutes.
Together with a positive anti-CCP and raised inflammatory
markers, these allow a definitive diagnosis of rheumatoid
arthritis (as per 2010 classification criteria2).

COMMON PITFALLS

• Not diagnosing rheumatoid arthritis on basis of negative


rheumatoid factor.
• Not treating joint symptoms while waiting for appointment
with rheumatologist.

References
1. The Royal Australian College of General Practitioners 2014, ‘Early
diagnosis and management of rheumatoid arthritis’. Available at:
www.racgp.org.au/guidelines/musculoskeletal-diseases, accessed 1
December 2018.
2. Wilsdon, T & Hill, C 2017, ‘Managing the drug treatment of rheumatoid
arthritis’, Australian Prescriber, vol. 40, no. 2, pp. 51–8. Page 288
Page 289
Section 16
Neurology
Page 290

Case 57
Wilma Burns

Instructions for the doctor


This is a short case.
Please take a history, conduct a focused examination, outline the most
likely diagnosis and negotiate a management plan with Wilma.

Scenario
Mrs Wilma Burns is 54 years old and has been a patient at the practice
for several years. She is a keen gardener.

The following information is on her summary sheet:


Past medical history
Two children, normal pregnancies and delivery
Vaginal hysterectomy for prolapse aged 45
Mammogram normal this year
Medication
Nil
Allergies
Nil
Immunisations
Nil known
Social history
Non-smoker.

Instructions for the patient, Wilma Burns


You are a 54-year-old housewife and a keen gardener. For the last few weeks
you have noticed pins and needles in your right hand on waking in the
morning. This affects the pulps of the thumb, index, middle and half of the
ring finger. Sometimes you get this after you have been gardening Page 291
and in the last week the tingling sensation has woken you up at night.
When you shake your right hand you can make some of the tingling go away
and by doing this you can get back to sleep.
If the doctor asks, please say that you do not drink alcohol. You last had
immunisations when you were at school.

Clinical examination findings


You have carpal tunnel syndrome. Please demonstrate an area of paraesthesia
in the distribution of the right median nerve. Sensation is normal but power
of thumb abduction is reduced. Tinel’s sign* and Phalen’s sign† are positive.
Neck and shoulder movements are normal.

The following information is on your medical record:


Past medical history
Two children, normal pregnancies and delivery
Vaginal hysterectomy for prolapse aged 45
Mammogram normal this year
Medication
Nil
Allergies
Nil
Immunisations
Nil known
Social history
Non-smoker.

Suggested approach to the case


Establish rapport
Open questions to explore patient’s ideas, concerns and expectations.

Specific questions
Relevant to carpal tunnel syndrome
— Distribution of paraesthesiae
— Weakness of thumb movements Page 292
— Symptoms worse at night and early in the morning
Identify possible cause of carpal tunnel syndrome
— Diabetes, obesity, rheumatoid arthritis, hypothyroidism, employment,
hobbies
Exclude other causes
— Neck or shoulder pathology, OA
— Malignancy—Pancoast tumour, bone tumour
Request permission to examine.

Examination
Confirmation of area of paraesthesiae
Decreased sensation over the palm
Wasting of thenar eminence
Reduced power thumb abduction
Tinel’s sign
Phalen’s sign
Examination of the neck and shoulder.

Most likely diagnosis


Carpal tunnel syndrome.

Management
Discuss/explain diagnosis.

Investigations
TFTs
Glucose
Consider EMG depending on availability.

Treatment
Night splints
Consider corticosteroid injection
Refer for surgical decompression if conservative methods fail.

Opportunistic health promotion


Tetanus status—keen gardener
Alcohol intake.

CASE COMMENTARY Page 293

The history is suggestive of carpal tunnel syndrome. Most


doctors suspect the diagnosis very quickly; the focus of the case
becomes confirming this suspicion and excluding other causes.
The condition should be explained to Mrs Burns and then
which investigations are needed and the treatment options. Some
doctors will have easy access to nerve conduction studies but for
doctors from rural areas, such investigations may be unavailable,
and treatment will commence based on clinical grounds.
Tinel’s sign and Phalen’s sign are of doubtful value and so a
doctor who does not perform these tests will not be penalised.
The signs that correlate best with nerve conduction studies are
the distribution of paraesthesiae and reduced strength of thumb
abduction.1
Treatment of carpal tunnel syndrome usually starts with
removing or modifying any underlying precipitants (such as use
of vibration tools or keyboards, obesity or hypothyroidism), then
trying night splints and/or local steroid injection before referring
for surgery.2, 3

References
1. D’Arcy, CA & McGee, S 2000, ‘The rational clinical examination. Does
this patient have carpal tunnel syndrome?’ Journal of the American
Medical Association, vol. 283, pp. 3110–7. Erratum appears in Journal of
the American Medical Association 2000, vol. 284, no. 11, pp. 1384.
2. Wipperman, J & Goerl, K 2016, ‘Carpal tunnel syndrome: diagnosis and
management’, American Family Physician, vol. 94, no. 12, pp. 993–9.
3. Simpson, MA & Day, B 2011, ‘Painful numb hands’, Medical Journal of
Australia, vol. 195, pp. 388–91.

*Tinel’s sign: tapping the flexor retinaculum on proximal part of palm


reproduces median nerve paraesthesiae.
†Phalen’s sign: flexion of both wrists for 30 seconds reproduces paraesthesiae

in median nerve.
Page 294

Case 58
Sybil Clarke

Instructions for the doctor


This is a long case.
Please take a history, examine this patient appropriately and discuss with
her a management plan.

Scenario
A 65-year-old woman, Sybil Clarke, has booked a long appointment. She
has noticed a tremor and her family have told her that they can no longer
read her writing.

The following information is on her summary sheet:


Past medical history
Glaucoma, sees ophthalmologist regularly
Vaginal hysterectomy eight years ago for prolapse
First child, Terry, now aged 42
Second child, Malcolm, now aged 39
Medication
Timolol 0.25% 1 drop bd
Allergies
Nil
Immunisations
Up-to-date
Social history
Retired school secretary
Husband—retired credit union manager
Non-smoker
Alcohol intake—nil.

Instructions for the patient, Sybil Clarke Page 295

You are a 65-year-old retired school secretary. You have always been a very
meticulous sort of person. Much to your embarrassment you now have a
shake. You have made the appointment because your family said that they
could not read your writing on the Christmas card that you sent them.
The shake has been getting worse gradually over the last couple of years,
but it’s now at a point that it is making it hard to complete day-to-day tasks
such as doing up buttons. Your writing has deteriorated, becoming smaller
and harder to read. You are finding it harder to move, and sometimes it can
take a bit longer to get out of a chair. You are often fatigued. You had put this
down to getting older but are starting to worry that you’re getting older much
quicker than your friends. You haven’t had any falls. If asked, your voice has
become softer and your family have complained they can’t hear you when
you’re talking on the phone. You have had no issues with dribbling but do
occasionally find it a little tricky to swallow foods like steak and bread. You
are mostly continent of urine but have noticed some increased urgency over
time. There have been no changes in your bowels.
You are mostly managing at home with cooking, cleaning, shopping and
so forth, but are noticing that things take a bit more of a toll on you than in
the past.
You have no mood symptoms or excessive anxiety, but you are worried
about the cause of your tremor and deteriorating health.

Clinical examination findings


Please try to display a resting pill-rolling tremor, an expressionless face,
persistent blinking on glabellar tap, cogwheel rigidity, a shuffling gait with a
lack of arm swing, micrographia, poor balance and slow, stiff movements.

The following information is on your medical record:


Past medical history
Glaucoma, sees ophthalmologist regularly
Vaginal hysterectomy eight years ago for prolapse
First child, Terry, now aged 42
Second child, Malcolm, now aged 39
Medication
Timolol 0.25% 1 drop bd
Allergies
Nil
Immunisations
Up-to-date
Social history Page 296
Retired school secretary
Husband—retired credit union manager
Non-smoker
Alcohol intake—nil.

Suggested approach to the case


Establish rapport
Open-ended questions to explore Sybil’s ideas, concerns and expectations.

Specific questions
Writing—what has changed?
Tremor—when does it occur?
Gait
Bradykinesia—slow movements, e.g. difficulty getting out of a chair, rolling
over in bed
Falls
Dribbling/excess salivation
Dysphonia/dysphagia
Continence
Impact of symptoms on function and wellbeing
Mood symptoms—anxiety/depression
General health to exclude other causes—headaches, weight loss, fever,
memory
Medication—exclude drug-induced Parkinsonism
Request permission to examine.

Examination
Pulse
BP
Neurological examination
— Gait—shuffling
— Balance—retropulsion test (the patient stands vertically and the
doctor pulls them backwards to check for the speed of balance
recovery. Parkinson disease patients are slower to recover)1
— Tremor—at rest
Peripheral nervous system
— Coordination—normal
— Tone—cogwheel rigidity
— Power—slow movements but normal strength, writing shows Page 297
micrographia
— Reflexes—normal
— Sensation—normal
Cranial nerve examination
— Normal except for expressionless face and glabellar tap—persistence
of blinking reflex typical of Parkinson disease.

Most likely diagnosis


Parkinson disease.

Management
Explain condition
Demonstrate empathy and observe response to this diagnosis
Information—patient information leaflet/websites/support groups.

Investigations
TFTs2
UEC
LFTs.

Treatment
Drug therapy
— Contraindication for anticholinergic drugs present
— Aim to preserve quality of life3
— Start with a low dose of levodopa/dopa-decarboxylase inhibitors
(carbidopa or benserazide)4—for example, Sinemet
— Dopamine agonist (e.g. pramipexole) or a monoamine oxidase type B
inhibitor (e.g. selegiline) can be added later if needed
Consider neurology referral
Maintain function—healthy diet, regular exercise4
Consider occupational therapy/physiotherapy/exercise physiology/speech
therapy referrals
Arrange follow-up.

CASE COMMENTARY
Given the story of a new tremor plus difficulty writing,
Parkinson disease is the most likely problem. Doctors who think
of Parkinson can ask specific questions to confirm this Page 298
and then demonstrate the cardinal signs of resting
tremor, bradykinesia and rigidity on examination. Doctors who
do not suspect Parkinson disease will need to take a more
comprehensive history as well as doing the full neurological
examination.
The doctor should consider other common causes of tremor,
such as essential tremor, physiological tremor, hyperthyroidism,
and potentially serious causes such as cerebellar disorders or a
cerebral tumour. A good doctor will be able to demonstrate their
clinical reasoning by asking relevant questions and conducting
their examination appropriately so as to conclude that these
disorders are less likely.
Mrs Clarke’s history of glaucoma means that anticholinergic
medication for the Parkinson disease is contraindicated.
Telling Mrs Clarke the likely diagnosis needs to be done
sensitively. A good doctor may have been able to elicit her
concerns at Parkinson being a possible cause. Such doctors will
then be confirming her suspicions rather than breaking the news
to her.
The doctor should consider the impact of the diagnosis on
Mrs Clarke and provide appropriate support and follow up.
Referral for assistance at home is not needed now but may be in
the future. Likewise, Sybil’s driving capacity will need to be
assessed once her functioning on medication is known.
(The list of ‘instructions for the patient’ looks daunting, but it
can be done. I found that playing Sybil was not only excellent
revision for Parkinson but also gave me new insight into what it
might feel like to have the ‘shaking palsy’.)

References
Samii, A, Nutt, JG & Ransom, B 2004, ‘Parkinson’s disease’, Lancet, vol.
1. 363, pp. 1783–93.
2. Sirisena, D & Williams, DR 2009, ‘My hands shake: classification and
treatment of tremor’, Australian Family Physician, vol. 9, pp. 678–83.
3. Hayes, MW, Fung, VS, Kimber, TE & O’Sullivan, JD 2010, ‘Current
concepts in the management of Parkinson disease’, Medical Journal of
Australia, vol. 192, pp. 144–9.
4. Gazewood, JD, Richards, DR & Clebak, K 2013, ‘Parkinson disease: an
update’, American Family Physician, vol. 87, pp. 267–73.

Further reading
Sellbach, A & Silburn, P 2012, ‘Management of Parkinson’s disease’,
Australian Prescriber, vol. 35, pp. 183–8.
Page 299

Case 59
Rosie Inkamala

Instructions for the doctor


This is a long case.
Please take a focused history from Rosie. Outline to Rosie your
differential diagnosis and negotiate a management plan with her.

Scenario
Rosie Inkamala is a 36-year-old Indigenous woman. Rosie has come to
see you because two days ago, on the weekend, she could not move her
right side for about half an hour. She thought she was having a stroke
and was relieved that her movement came back. She has come to see you
for a check-up now that the clinic is open.

The following information is on her summary sheet:


Past medical history
Rheumatic fever aged 15
Atrial fibrillation (AF)
Alcoholic hepatitis
Hypercholesterolaemia
Medication
Atorvastatin 10 mg od
Aspirin 100 mg od
Digoxin 0.625 mg od
Ramipril 10 mg od
Etonogestrel intrauterine implant six months ago (Mirena)
Allergies
Nil known
Immunisations
Up-to-date
Cervical screening test Page 300
Normal this year
Social history
High alcohol use
Chews tobacco.

Instructions for the patient, Rosie Inkamala


You are a 36-year-old Indigenous woman. Two days ago, on the weekend,
you could not move your right arm or leg for about half an hour. You thought
that you were having a stroke and were relieved when the movement came
back. Today you have decided to come to see the GP for a check-up, now that
the clinic is open. You drink heavily whenever you can afford it, which
works out at more than 10 standard drinks three to four days a week. When
you drink you tend to forget your medications, which are in a Webster pack.
You live with extended family and your two children and feel safe at
home.
You are very scared of having a stroke after seeing your aunt suffer from a
big stroke last year; you are motivated to do what the doctor thinks you
should do. If they suggest stopping drinking, tell them you will need help to
stop.
You are fluent in four Aboriginal languages, but your English is limited.

Clinical examination findings


BP 124/82 mmHg
No evidence of cardiac failure
Rate-controlled atrial fibrillation (AF)
No residual neurological findings.

The following information is on your medical record:


Past medical history
Rheumatic fever aged 15
Atrial fibrillation (AF)
Alcoholic hepatitis
Hypercholesterolaemia
Medication
Atorvastatin 10 mg od
Aspirin 100 mg od
Digoxin 0.625 mg od
Ramipril 10 mg od
Etonogestrel intrauterine implant six months ago (Mirena)
Allergies Page 301
Nil known
Immunisations
Up-to-date
Cervical screening test
Normal this year
Social history
High alcohol use
Chews tobacco.

Suggested approach to the case


Request the assistance of an interpreter
Establish rapport
Open-ended questions to explore Rosie’s ideas, concerns and expectations.

Specific questions
Details about loss of movement—duration, sites of loss of movement
Aura (suggests migraine)
Headache
Any associated loss, such as loss of vision, sensation, consciousness or
bladder control
Any residual problem
Previous episodes
Family history of stroke, heart disease
Systems review, e.g. exclude fever, weight change
Request permission to examine.

Examination
Temperature
Cardiovascular system
— Pulse—rate, rhythm, volume
— BP
— Splinter haemorrhages
— Apex beat
— Heart sounds, added sounds
— Carotid bruits
— Evidence of CCF
— JVP raised
— Pulmonary oedema
— Hepatomegaly Page 302
— Pitting dependent oedema
Neurological system
— Gait
— Balance
— Peripheral nervous system
— Inspection—wasting, tremor, fasciculation
— Tone
— Sensation—light touch
— Power
— Reflexes
— Coordination
— Cranial nerve examination
I questions regarding change sense of smell
II acuity, fields, pupil reflexes, fundi
III, IV, eye movements, exclude diplopia and nystagmus V opening jaw and
VI facial sensation
VII facial movements
VIII hearing, balance
IX, X swallowing
XI shrug shoulders
XII tongue movements.

Summary of findings
BP 124/82 mmHg
No evidence of cardiac failure, soft pansystolic murmur
Controlled AF
No residual neurological deficit
BMI 22 kg/m2.

Most likely diagnosis


Transient ischaemic attack (TIA).

Differential diagnoses
Hemiplegic migraine
Postictal state
Intracranial bleed.
Management Page 303

Explain likely diagnosis


Educate about link with previous rheumatic fever—need to prevent future
stroke and need for control of cardiovascular risk factors
Encourage emergency call if symptoms recur—TIA = Take Immediate
Action.

Investigations
FBC, ESR, UEC
BSL/HbA1c
Digoxin level
Fasting lipids
LFTs and coagulation studies (liver damage)
ECG
CT scan asap to exclude haemorrhagic event1
Carotid duplex ultrasound or CT angiogram
Echocardiogram.

Treatment
Continue current medications
Consider additional anticoagulation post CT scan result—low molecular
weight heparin plus warfarin is recommended; clopidogrel is an
alternative. Doctors need to assess patient’s safety regarding warfarin: the
risks can outweigh the benefits in patients who consume high volumes of
alcohol or cannot reliably attend for monitoring
Culturally appropriate, motivational interviewing regarding alcohol and
tobacco use. Offer support from drug and alcohol team regarding alcohol
cessation
Encourage regular exercise, eating healthy bush tucker
Arrange follow-up.

CASE COMMENTARY
The doctor should ask if an interpreter is available. In this
scenario one is not, so the doctor will need to communicate with
Rosie in simple English. The doctor should avoid using a loud
voice or assuming that an inability to understand English
signifies intellectual disability. These challenges can lead
doctors to give up on communication and thus deny patients
essential information.
Rosie is at risk of a disabling stroke. A TIA is not a Page 304
benign event but should be considered a warning for
stroke. Sensitivity will be needed when encouraging Rosie to be
investigated and to adopt a healthier lifestyle, and when helping
her determine the relative risks and benefits of commencing
warfarin. Rosie’s atrial fibrillation is associated with mitral
regurgitation following her rheumatic heart disease. Prescribing
the newer oral anticoagulants for this situation would be off-
label as these are only approved for non-valvular atrial
fibrillation.
Rosie should be advised to seek assistance immediately if the
paralysis returns. Urgent transfer to a hospital for thrombolytic
therapy can then be arranged.2–4

References
1. Leung, ES, Hamilton-Bruce, MA & Koblar, SA 2010, ‘Transient
ischaemic attacks—assessment and management’, Australian Family
Physician, vol. 39, pp. 820–4.
2. Brieger, D 2014, ‘Anticoagulation: a GP primer on the new oral
anticoagulants’, Australian Family Physician, vol. 43, pp. 254–9.
3. Wang, Y, Wang, Y, Zhao, X, Liu, L, Wang, D, Wang, C et al. 2013,
‘Clopidogrel with aspirin in acute minor stroke or transient ischemic
attack’, New England Journal of Medicine, vol. 369, pp. 11–9.
4. Dhamija, RK & Donnan, GA 2007, ‘Time is brain—acute stroke
management’, Australian Family Physician, vol. 36, pp. 892–5.
Page 305

Case 60
Joe Summers

Instructions for the doctor


This is a short case.
Please read the following history and conduct a focused clinical
examination. Tell the facilitator your findings and what initial investigations
you would request.

Scenario
Joe Summers is a 67-year-old retired truck driver. He has been a patient
at the practice for years but only attends to get his licence renewed. One
year he went to a Pit Stop health promotion stand at the local show and
he recorded a BMI of 39 kg/m2 and a waist circumference of 115 cm.
Joe has noticed over the last few months that he cannot feel so well
with his hands and feet. He assumed this was a normal part of ageing but
he mentioned it when he went to buy some shoes and the shop assistant
was concerned and told him that he had to come to see a doctor.

The following information is on his summary sheet:


Past medical history
Nil
Medication
Nil
Allergies
Nil
Immunisations
Nil known
Family history
Nil known
Social history
Retired truck driver.

Instructions for the patient, Joe Summers Page 306

You are a 67-year-old retired truck driver. You have been a patient at the
practice for years but only attend to get your licence renewed. One year you
went to a Pit Stop health promotion stand at the local show and you recorded
a BMI of 39 kg/m2 and a waist circumference of 115 cm. You already knew
you were overweight and drank too much so did not see the point of going
back to the GP to be told off again.
You have noticed over the last few months that you cannot feel so well
with your hands and feet. You also get funny tingling feelings that are
irritating. You assumed this was a normal part of ageing but you mentioned it
when you went to buy some shoes. The shop assistant was concerned and
told you to come to see a doctor for a check-up.

The following information is on your medical record:


Past medical history
Nil
Medication
Nil
Allergies
Nil
Immunisations
Nil known
Family history
Nil known
Social history
Retired truck driver.
Please mimic the absence of sensation to monofilament testing, and
reduced vibration sense and joint position and pain sense of your feet to the
level of your ankle, and hands to your wrist. Motor function, skin, pulses and
reflexes are normal.

Instructions for the facilitator


The following equipment is needed: tendon hammer, sterile neurology pins,
tuning fork and 10 g monofilament.
In the interests of time, examination of the hands is not required. When the
doctor asks to examine the hands, please say that the signs are the equivalent
to the signs in the feet. If the doctor does not examine the hands, do not
volunteer this information.

Suggested approach to the case Page 307

Establish rapport
Summarise history
Request permission to examine.

Examination
Observe patient walking
Skin intact
No muscle wasting or fasciculation
Tone
Sensation
— 10 g monofilament testing
— Pain sense with neuro-pin
— Vibration sense
— Joint position sense
Motor function
Coordination
Reflexes
Pulses
Random BSL or urinalysis.

Findings
Reduced sensation in both feet to ankle level and hands to wrists, i.e. glove
and stocking sensory neuropathy.

Initial investigations
Fasting blood glucose
FBC, ESR, B12, folate
Liver function tests, renal function, TSH
Fasting lipids and ratio
Further assessment
— Drug and alcohol history.

CASE COMMENTARY

Joe has developed a glove and stocking symmetrical sensory


peripheral neuropathy. There are multiple potential causes but
the most common are diabetes or alcohol.1 A stepped Page 308
approach to the investigations is appropriate, starting
with routine blood tests. If these are normal, serum protein
electrophoresis for a monoclonal gammopathy, and studies for
HIV, sarcoidosis, heavy metal poisoning and vasculitis may be
indicated.2 The underlying cause of the neuropathy needs
treatment, plus Joe needs advice on preventing damage to his
hands and feet.
Foot ulcers are a significant burden for the individuals
affected and for the health care system. Prevention is imperative.
Testing for peripheral neuropathy is a key component of
prevention.3 Loss of ability to feel a standardised 10 g
monofilament correlates with a risk of ulceration, making this
test arguably more relevant in clinical practice than the more
expensive nerve conduction test. Loss of vibration sense also
correlates with the risk of ulceration but less so than the
monofilament test.4

References
1. Azhary, H, Farooq, MU, Bhanushali, M, Majid, A & Kassab MY 2010,
‘Peripheral neuropathy: differential diagnosis and management’,
American Family Physician, vol. 81, pp. 887–92.
2. Pascuzzi, RM 2009, ‘Peripheral neuropathy’, Medical Clinics of North
America, vol. 93, pp. 317–42.
3. Ogrin, R & Sands, A 2006, ‘Foot assessment in patients with diabetes’,
Australian Family Physician, vol. 35, pp. 419–21.
4. Singh, N, Armstrong, DG & Lipsky, BA 2005, ‘Preventing foot ulcers in
patients with diabetes’, Journal of the American Medical Association,
vol. 293, pp. 217–28.
Page 309
Section 17
Palliative care
Page 310

Case 61
Liz Ross

Instructions for the doctor


This is a short case.
Please manage the patient’s request as appropriate. Physical examination
and investigations are not required.

Scenario
Liz Ross is 81 years old and a regular patient of yours. She has recently
been diagnosed with metastatic ovarian cancer and her treatment is
supportive rather than curative. She is currently feeling quite well
physically and is pain free. Liz has come in to see you today because her
daughter has told her to ‘get her affairs in order’.

The following information is on her summary sheet:


Past medical history
Ovarian cancer
Mild hypertension
Medication
Paracetamol 1 g qid (antihypertensive stopped after cancer diagnosis)
Allergies
Nil
Immunisations
Influenza and pneumococcal vaccinations up-to-date
Social history
Widowed, four children; lives with her eldest daughter, Rosie
Non-smoker
Does not consume alcohol.

Instructions for the patient, Liz Ross Page 311

Your opening statement is ‘As you know doctor my days are numbered, my
daughter thinks I should get my affairs in order and talk to you about
paperwork.’
You are an 81-year-old woman with recently diagnosed ovarian cancer.
Your treatment will not cure your cancer but aims to keep you comfortable.
At the moment you feel very well and live independently. You have limited
health literacy and don’t understand the legal or practical aspects of advance
care planning. You don’t need anything else today as you saw your specialist
yesterday.
You have come to terms with your diagnosis and feel satisfied that you’ve
lived a long and good life. You are not depressed and don’t have any
questions regarding your prognosis or treatment at this time.
You have heard there is a form that will stop hospital doctors ‘giving you
the paddles’ if your heart stops. You think this is a reasonable idea, as you
don’t want to prolong your life if you are very ill with little chance of
recovery.
Your understanding from what your daughter has told you, is that these
forms will mean you don’t have to make any future decisions, handing over
the management of your health and financial affairs to your daughter. While
this appeals in one sense, and you trust that your daughter will act in your
best interests, you also feel like you would like to have a say in what happens
while you still have your faculties and feel less comfortable with signing
something that hands over decision-making at this point.
You have a simple will, leaving your estate to your children and your
grandchildren and you are happy with that.
Initially answer questions about your understanding with a hesitant ‘yes’;
however, if asked to explain in your own words or for other appropriate
assessments of understanding, you respond in a way that shows your
understanding is very limited. If the matter is explained again in a clear and
helpful way, your response indicates increased understanding of the issues.
Your reading level is mid-primary school and you don’t use a computer.
Once the doctor explains the idea of an advance care plan or a living will,
you want to pursue this. If the doctor indicates that they will give you the
legal forms (Guardianship and Power of Attorney), you don’t feel you need
this right away. You would like another appointment to organise a living will
and, if the doctor suggests it, you think it would be best if your daughter
attends.

The following information is on your summary sheet:


Past medical history
Ovarian cancer
Mild hypertension Page 312
Medication
Paracetamol 1 g qid (antihypertensive stopped after cancer diagnosis)
Allergies
Nil
Immunisations
Influenza and pneumococcal vaccinations up-to-date
Social history
Widowed, four children; lives with her eldest daughter, Rosie
Non-smoker
Does not consume alcohol.
Suggested approach to the case
Establish rapport
Inquire into feelings regarding her recent diagnosis and prognosis
Assess level of understanding of advance care planning (‘living will’)
Provide information about advance care planning pitched at appropriate level
and checking for understanding
Explore Liz’s views towards advance care planning and her wishes for end-
of-life care
Ask whether Liz has a legal will
Explain the difference in simple terms between an advance care plan,
enduring guardianship and enduring power of attorney
Explain your ability to assist in advance care planning
Provide opportunity for questions in a supportive and non-confrontational
way
Provide educational resources/hand-outs
Offer another appointment and suggest that her daughter (and/or another
family member) attends with her.

CASE COMMENTARY

The legislation for advance care planning and power of attorney


documents vary between states and territories. Good candidates
would be expected to have a reasonable working knowledge of
the legislative requirements in their own jurisdiction. Key points
include explaining the concept of guardianship and that enduring
power of attorney/enduring guardianship (medical) and enduring
power of attorney (financial) are separate to advanced Page 313
care planning. Many resources are now online, however,
this will be of little help to Liz who does not use computers.
Knowing reputable sites to access information relevant to your
state and printing them for the patient would be the best
approach.
COMMON PITFALLS

Poor knowledge of advance care planning


Not recognising the patient’s limited health literacy
Asking ‘Do you understand?’ and accepting a response of ‘Yes’
at face value.

Further reading
Advanced Care Planning Australia 2018, Factsheet for health professionals.
What is advance care planning? Retrieved from: www.advancecareplann
ing.org.au/docs/default-source/acpa-resource-library/acpa-fact-sheets/ac
pa_healthcare-professionals-factsheet-online_aug2018.pdf?sfvrsn=1,
accessed 5 March 2019.
Bird, S 2014, ‘Advance care planning’, Australian Family Physician, vol. 43,
no. 8, pp. 526–8.
Koay, K, Schofield, P & Jefford, M 2012, ‘Importance of health literacy in
oncology’, Asia Pacific Journal of Clinical Oncology, vol. 8, no. 1, pp.
14–23.
Johnson, CE, McVey, P, Rhee, JJ et al. 2018, ‘General practice palliative
care: patient and carer expectations, advance care plans and place of
death—a systematic review’, BMJ Supportive and Palliative Care,
Published Online First: 25 July 2018. doi: 10.1136/bmjspcare-2018-
001549.
Page 314

Case 62
Frank Stanley

Instructions for the doctor


This is a short viva.
Please discuss your management of this situation with a GP colleague.

Scenario
Frank is a 68-year-old retired postman who is dying from bowel cancer.
He is now receiving palliative care and has a prognosis of weeks to
months. He lives with his wife in a country town; their daughter lives
nearby and helps a lot.
You visit Frank at home and glean the following:
• Frank is taking 20 mg oxycodone (Oxycontin) BD and 5 mg
oxycodone (Endone) for breakthrough pain two to three hourly
• he sleeps badly as he is waking throughout the night to take
breakthrough pain medication. It relieves the pain for about 90
minutes.
Please answer the examiner’s questions about Frank.

Instructions for the facilitator


This is a viva. The doctor is expected to talk with you as a professional
colleague.
Please ask the following questions:

1. How would you manage Frank’s pain?


2. What are some of the symptoms you may need to consider in Frank’s
end-of-life care?
Prompts if needed: Page 315
Do you think he is on the right pain medication?
What do you think of the dose?
Are there any non-opioid options to consider?
Are there any non-pharmacological options?
3. Who else might be involved in Frank’s care?

Suggested approach to the case


Overall aim is a ‘good death’ by managing Frank’s pain and any other
symptoms and taking a holistic approach to palliative care for him and his
family.

1. Ask about the detail of his pain to determine best options for
management:
• consider if the pain is from the tumour, metastases, constipation or
another new condition
• Frank most likely needs a higher dose of slow-release pain medication
• calculate his current total daily dose from the slow-release oxycodone
and the total breakthrough doses of oxycodone
• increase the slow-release oxycodone dose to the current total daily
dose
• provide breakthrough oxycodone at a sixth to a twelfth of the total
dose
• if a patient is on two different opioids, use an opioid conversion chart
to calculate current total 24-hour dose of morphine, and then calculate
the slow-release and breakthrough doses
• ask Frank to document his breakthrough doses so that you can
increase slow-release doses as needed in future
• reassure that using opioids is appropriate
• provide scripts for laxatives, as well as asking about constipation
• add paracetamol regularly as this potentiates the action of opioids
• consider the role of palliative radiotherapy, chemotherapy or nerve
blocks, but these would require hospital care away from home
• heat/cold packs, antispasmodics, nifedipine (for tenesmus) and
corticosteroids could also be tried.

2. Palliative care patients can experience many complex symptoms and it is


important to consider your patient and their family holistically.

A helpful mnemonic is PAIN DOCTORSSS1

Pain Page 316


Anorexia
Insomnia
Nausea
Dyspnoea
Oedema
Constipation
Tiredness
Oral hygiene/mouth care
Restlessness
Spiritual
Social
pSychological
Managing pain is a vital first step in relieving suffering; so too is open
discussion about how Frank and his family are coping. What are their fears or
concerns? Are there long-term worries or issues that could be addressed
before he dies? Is his will up-to-date? Does he want to give his wife enduring
power of attorney so that she can manage his financial affairs? Does he want
to write an advanced health directive regarding his care? Are they receiving
financial assistance through Centrelink? These topics can be addressed over
time during visits to check on his pain and other physical symptoms.

3. Palliative care, given its many facets, is done best with a team. Others
you may want to involve in end-of-life care include, but are not limited
to:
• spiritual care—pastor/priest/chaplain
• palliative care teams, either locally or remotely. Most states have
palliative care advice lines etc.
• community care—visiting nurses, carers
• home help, meals-on-wheels
• respite for his family—hospital, local nursing home, local volunteers
• psychologist/social worker.

CASE COMMENTARY

This is a fairly straightforward case of end-of-life care. Frank


appears to be on the right medication as it does work, but the
need for regular breakthrough pain relief and waking from sleep
suggests he is not on the right dose. You would not need to
come up with actual doses in a viva, rather a structured approach
to more effective pain management and care for Frank.
COMMON PITFALLS Page 317

Despite evidence that GP involvement in end-of-life care is


beneficial, there are some GPs who opt out. Sometimes doctors
are optimistic prognosticators and do not undertake end-of-life
care planning early enough, resulting in late referrals to
palliative care. It is important we have a solid understanding of
end-of-life care in general practice and use help from palliative
care teams and hotlines to continue providing excellent care.

Reference
1. Hart, A, Palliative care consultant, Western Australia (personal
communication).

Further reading
Mitchell, G 2014, ‘End-of-life care for patients with cancer’, Australian
Family Physician, vol. 43, no. 8, pp. 514–19.
Opioid Calculator—FPM ANZCA. App produced by the Faculty of Pain
Medicine of the Australian and New Zealand College of Anaesthetists.
Available for Apple and android.
www.palliaged.com.au. PalliAGED app available on android and Apple.
Australian Government Department of Health palliative care resource.
Tait, P, Morris, B & To, T 2014, ‘Core palliative medicines: meeting the
needs of non-complex community patients’, Australian Family
Physician, vol. 43, no. 1–2, pp. 29–34.
Therapeutic Guidelines Ltd, Palliative Care Expert Group 2016, Therapeutic
Guidelines: Palliative Care. Version 4. Therapeutic Guidelines Ltd,
North Melbourne, Vic.
Page 318

Case 63
Katrina Carroll

Instructions for the doctor


This is a short case.
Read the following scenario. Discuss the essential issues that arise with
Katrina and suggest appropriate management. If you are considering anything
that you do not wish to discuss with the patient at this stage, you should
inform the observing examiner.
No physical examination is required and no further investigations are
available.

Scenario
Katrina Carroll, a long-term patient of yours, is attending for an urgent
appointment after scheduled routine follow-up with her breast surgeon.
Your receptionist has obtained Dr Fisher’s report from her recent review
as follows:
Dear Doctor,
Re: Mrs Katrina Carroll, I saw Katrina for routine annual review
following her T2 N1 M0 receptor negative left breast cancer four years
ago. She has been well and had no specific complaints, but clinical
examination revealed an enlarged, firm node in the left supraclavicular
area. I organised an excision biopsy, which unfortunately returned results
consistent with metastatic breast cancer. I organised staging scans which
suggested she has metastasis in her T7 and L1 vertebrae, as well as two
lesions in her liver.
We discussed her results today and I outlined that her condition is not
operable. I have organised for her to see the oncologist next week.
Regards

The following information is on her summary sheet: Page 319


Past medical history
Mild episodic asthma
Left breast cancer—mastectomy, radiotherapy and chemotherapy four
years ago
Medication
Nil regular
Allergies
Nil known
Immunisations
Up-to-date
Social history
Non-smoker
Married to Paul
Six children
Works as a librarian.

Instructions for the patient, Katrina Carroll


You are Katrina Carroll, a 54-year-old woman who has been coming to this
GP for several years. You are married to Paul (a high school English teacher)
and you have six children. The youngest of your children is 17 and is the only
one still at home. Your oldest two are married with children of their own.
You present to the doctor today following a visit to your surgeon, Dr
Fisher, last week.
You came to this doctor with a breast lump four years ago and ended up
having your left breast surgically removed because of breast cancer.
Following this, you had radiotherapy to the left armpit and shoulder area and
a course of chemotherapy. All this went pretty well and you’ve been healthy
since then—back at work and feeling well.
About three weeks ago you went to see Dr Fisher for your annual check-
up.
You weren’t bothered by anything, but he found a lump around your left
collarbone and cut it out a couple of days later. It came back as cancer in a
lymph node and he then sent you for more tests—mostly scans. You saw him
for the results of these last week, and he explained that your breast cancer has
come back and has spread to your backbone in a couple of places, as well as
your liver. He has explained that your situation is not one he can help with
and has referred you to the oncology clinic. You’re feeling abandoned by him
and are glad to see your regular GP today.
You’re scared but are prepared to concentrate on making the most Page 320
of the time you have left. In contrast, your husband Paul is eager to
pursue surgical or other cures and does not want to talk about death or dying.
Paul has told you of an experimental light treatment in Mexico and is
suggesting you sell the house to give it a go. You really don’t think this is a
good idea but are keen to know what the GP thinks.
You knew if it ever came back it would end your life and you are keen to
know how long you’ve got. If your GP is reluctant to give any time frame,
ask for their ‘best guess’ or a ‘ballpark’ idea. You’re currently feeling well
and if it is explored, you would like to spend the time you’ve got left with
your children and grandchildren and you’ve always wanted to see Rome.
You’re not sure how long you would be prepared to keep having
chemotherapy as it made you so very ill the first time around.

The following information is on your summary sheet:


Age
54
Past medical history
Mild episodic asthma
Left breast cancer—mastectomy, radiotherapy and chemotherapy four
years ago
Medication
Nil regular
Allergies
Nil known
Immunisations
Up-to-date
Social history
Non-smoker
Married to Paul
Six children
Works as a librarian.

Suggested approach to the case


Establish rapport
Gently clarify Katrina’s understanding of her condition and her prognosis
Recognise that Katrina and Paul are at different stages of acceptance.

Management
A good candidate should explore Katrina’s concerns and recognise her
husband’s denial
Sensitively dismiss surgery as a curative option Page 321
Gently discourage the pursuit of an unorthodox cure at great expense
to the family
Emphasise the GP’s role in supporting Katrina and her family through the
next stage of Katrina’s care
Give compassionate but realistic information about Katrina’s prognosis
Be prepared to sensitively encourage Katrina to face her mortality and
maximise her time remaining
In response to the questions about prognosis you could introduce the concept
of survival data and their value, given that each case is different
Katrina’s five-year survival could be up 25% but it is dependent on receptors
and her response to chemotherapy.

CASE COMMENTARY

Palliative care represents an unparalleled privilege for the GP.


To care for your patient and their family through the last stage of
life is both challenging and rewarding. Most are familiar with
the stages of grief made popular by American psychiatrist
Elisabeth Kübler-Ross. We now do not consider progress
through these stages (in any order) as normative, but allow for a
wide range of responses in our patients facing grief and loss.
This case allows for practice in some important communication
skills in a setting complicated by the husband’s reluctance to
accept his wife’s prognosis.
While patients are free to elect to pursue alternative or
unconventional treatment, there remains an important advocacy
role for the GP in this setting. Encouraging the Carrolls to
mortgage their house to expose Katrina to a non-evidence-based
treatment is to betray that role. There are valuable skills to be
practised in the discussion about her prognosis, with the
extremes of being overly specific or overly evasive to be
avoided.

COMMON PITFALLS

It is tempting to allow Katrina to join Paul in his optimism for a


quick fix or miracle cure and, if you do, the case runs along a
very positive line. Even Katrina can eventually be drawn into
this. However, you have not been honest and she will be less
prepared for what lies ahead as a result.
Sometimes we are quick to forcefully ‘burst the bubble’ of
those we see in denial, forgetting that it can be a defence
mechanism to allow continued functioning in the face of great
loss. It’s easy to side with Katrina as if she has the correct
response and Paul must quickly ‘catch up’ with her. Page 322
Helping Katrina to see that Paul’s attitude is just as valid
as hers is valuable.
When we raise referring our patients to various services—
oncology, palliative care and so on—it can sound to the patient
as though we are abandoning them and we have to work hard to
emphasise our continued and central role in their care.
Michael Barbato’s book, Caring for the Palliative Care
Patient, deals with the role of denial and what to advise about
alternative therapies.

Further reading
Barbato, M 2002, Caring for the Palliative Care Patient, McGraw-Hill,
Sydney.
Kübler-Ross, E 1969, On Death and Dying, Routledge, London.
National Health and Medical Research Council 2011, ‘An Ethical Framework
for Integrating Palliative Care Principles into the Management of
Advanced Chronic or Terminal Conditions’. Available at: www.nhmrc.g
ov.au/_files_nhmrc/publications/attachments/rec31_ethical_framework_
palliative_care_terminal_110908.pdf, accessed 24 September 2014.
Page 323
Section 18
Preventive
health
Page 324

Case 64
Bill Ferguson

Instructions for the doctor


This is a short case.
Please consult with Bill Ferguson, who wants help in giving up smoking.

Scenario
Bill Ferguson is a 60-year-old carpenter. He has smoked 30 cigarettes
per day since leaving school at the age of 14. Yesterday he was
diagnosed as having had a transient ischaemic attack. The doctor at the
Emergency department told him that he had to stop smoking.
Bill has come to see you today to ask for advice about stopping
smoking. He has previously resisted any discussion about giving up
smoking whenever he attends for regular blood pressure and cholesterol
checks.
The following information is on his medical record:
Past medical history
Hypertension five years
Hypercholesterolaemia three years
Medication
Perindopril 5 mg od
Atorvastatin 80 mg per day
Aspirin 100 mg per day
Allergies
Nil known
Immunisations
Up-to-date
Family history
Father died of a stroke, aged 64
Mother aged 93, moderate dementia living in a nursing home
Social history Page 325
Smokes 30 cigarettes per day
Alcohol intake—not known.

Instructions for the patient, Bill Ferguson


You are a 60-year-old carpenter. You have smoked 30 cigarettes per day
since leaving school at the age of 14. Yesterday you had a funny turn. You
suddenly could not use or feel your right arm and right leg. It only lasted
about five minutes but it scared you and you went to the hospital. The letter
from the hospital said it was a transient ischaemic attack or ‘ministroke’. The
doctor at the Emergency department told you that you had to stop smoking or
you may have another attack or even a full stroke. Additionally, you could
lose your driving licence.
You have come to your usual GP for advice on stopping smoking. You
have previously resisted any discussion about giving up smoking when you
come for regular blood pressure and cholesterol checks.
Lighting a cigarette has been the first activity of your day for over 40
years. You have tried to give up smoking before by going ‘cold turkey’ and
only stopped for two weeks. You have never tried nicotine replacement
therapy or tablets. Yesterday’s funny turn has petrified you and you are now
willing to do anything the doctor suggests.

The following information is on your medical record:


Past medical history
Hypertension five years
Hypercholesterolaemia three years
Medication
Perindopril 5 mg od
Atorvastatin 80 mg per day
Aspirin 100 mg per day
Allergies
Nil known
Immunisations
Up-to-date
Family history
Father died of a stroke, aged 64
Mother aged 93, moderate dementia living in a nursing home
Social history
Smokes 30 cigarettes per day
Alcohol intake—not known.

Suggested approach to the case Page 326

Introduction
Establish rapport and confirm reason for consult
Use the 5 A’s approach:1
1. Ask
How many cigarettes per day, for how long
2. Assess
Readiness to quit
Previous attempts/experience of quitting, longest period of abstinence
Assess nicotine dependence
— Minutes after waking to first cigarette
— Cravings or withdrawal symptoms in previous quit attempts
Personal preference of using nicotine replacement therapy (NRT) or
medication, such as bupropion or varenicline
3. Advise
Specific advice on how to use NRT or medication
4. Assist
Talk through strategies when craves a smoke
— 4 D’s: delay, deep breathe, drink, distract
— Give information about Quitline
5. Arrange
Arrange follow-up
Relapse prevention advice.

CASE COMMENTARY

Assisting patients to stop smoking is a core skill for general


practitioners, with clear evidence of health benefits. The
research evidence is that Bill will have greater chance of success
with quitting if he uses either NRT, bupropion (Zyban) or
varenicline (Champix). He will also benefit from the support of
those around him at work and at home, from agencies such as
Quitline and from planned review with his GP.
All forms of NRT (gum, patch, spray or inhaler) are equally
effective and have been shown to increase quit rates at 5 to 12
months approximately two-fold compared with a placebo. The
choice of NRT product depends on personal preference and
combining NRT with oral medication may improve success
rates. Bill has smoked 30 cigarettes per day for many years and
has his first cigarette within 30 minutes of waking up, so he has
a high likelihood of nicotine dependence. Initially he Page 327
should use the high-dose patch (21 mg/24 hr or 15
mg/16 hr patch) or gum (4 mg) and then follow the
manufacturer’s instructions on switching to the lower dose.2
Electronic cigarettes (e-cigarettes) are battery-powered
devices that deliver nicotine in a vapour without tobacco or
smoke. Some trials suggest that e-cigarettes could play a role in
smoking cessation,3 but long-term safety and efficacy data are
needed. Acupuncture and hypnosis are not effective, but
mindfulness is being explored to assist smoking cessation,
especially in smokers with mental illness.

References
1. Zwar, N, Mendelsohn, C & Richmond, R 2014, ‘Tobacco smoking:
options for helping smokers to quit’, Australian Family Physician, vol.
43, no. 6, pp. 348–54.
2. Mendelsohn, C 2013, ‘Optimising nicotine replacement therapy in
clinical practice’, Australian Family Physician, vol. 42, no. 5, pp. 305–9.
3. McRobbie, H, Bullen, C, Hartmann-Boyce, J & Hajek, P 2014,
‘Electronic cigarettes for smoking cessation and reduction’, Cochrane
Database of Systematic Reviews, vol. 12:CD010216.

Further reading
Australian Medical Association, Tobacco Smoking and E-cigarettes 2015.
The AMA Position. Available at: https://ama.com.au/position-statement/
tobacco-smoking-and-e-cigarettes-2015, accessed 1 December 2018.
The Royal Australian College of General Practitioners 2011, Supporting
smoking cessation: a guide for health professionals, RACGP,
Melbourne, Vic.
Page 328

Case 65
Taylor Jordan

Instructions for the doctor


This is a short case.
Please take a history, assess this patient appropriately and discuss a
management plan with her.

Scenario
Taylor Jordan is a 30-year-old woman not previously seen by you. She
has made an appointment to ask you for advice on losing weight.
Three months ago, she attended the surgery and was examined and
tested to exclude any organic cause for her obesity.

The following information is on her summary sheet:


Past medical history
Irritable bowel syndrome
Medication
Ethinyloestradiol 30 mcg, levonorgestrel 150 mcg (Microgynon 30 ED)
Allergies
Nil
Cervical smear
None recorded
Immunisations
Recall due for Hep A and for Hep B second injection
Social history
Single
Non-smoker
Alcohol intake not recorded.

Instructions for the patient, Taylor Jordan Page 329

You are 30 years old. You have always had a problem with your weight. You
used to play hockey at state level but no longer exercise regularly. You work
at a call centre about an hour’s drive from home. Life is a constant series of
diets but your height is 1.70 m, your weight is 90 kg and BMI is 30.2 kg/m2
with an 80 cm waist. Three months ago, you attended the surgery and were
examined and tested to exclude any organic cause for your obesity.
A friend at work lost weight through a very low-calorie diet but now she is
putting the weight back on. You had thought of trying this diet but are now so
uncertain about what to do that you have decided to see another GP for
advice.

The following information is on your medical record:


Past medical history
Irritable bowel syndrome
Medication
Ethinyloestradiol 30 mcg, levonorgestrel 150 mcg (Microgynon 30 ED)
Allergies
Nil
Cervical smear
None recorded
Immunisations
Recall due for Hep A and Hep B second immunisation
Social history
Single
Non-smoker
Alcohol intake not recorded.

Suggested approach to the case


Establish rapport
Open questions to explore Taylor’s ideas, concerns and expectations.

Specific questions
Review current weight, BMI, waist circumference and trend1
Review current diet and alcohol intake
Review current exercise level and attitude towards exercise
Exclude eating disorder
Brief review of Taylor’s previous weight loss strategies.

Management2 Page 330

Reassure that there is no need for further tests


Need for diet/activity balance and long-term sustainable change
Refer to reputable information sources regarding healthy eating, consider
dietitian referral
— Five serves of vegetable and two serves of fruit per day
— Drink 500 mL water half an hour before food3
Advise about use and interpretation of food labels, e.g. low in fat may mean
high in sugar
Increase activity—approximately an hour per day for weight loss
Set realistic goals—focus on health gains, not weight loss
Plan for regular follow-up and review
Plan for future consultations including hepatitis immunisations and cervical
screen.

CASE COMMENTARY

This case tests the doctor’s ability to engage with Taylor and
encourage her with the difficult task of lifestyle change. GPs
need to skilfully support and motivate patients into action by
exploring their readiness for change and by being non-
judgemental.2 The formula for losing weight is simple, but
counteracting the multiple environmental, cultural, social and
personal factors3 that have contributed to obesity in Australia is
not.
Taylor’s recent physical examination and tests mean that
these do not need to be repeated. The doctor should review
Taylor’s daily routine, her diet and activity. Obesity results from
a chronic imbalance between energy intake from food and
energy expenditure. Changes to her diet and energy use
(exercise) are needed. The doctor should encourage Taylor to
consider realistic changes that fit around her work. For example,
is there an option to park the car 15 minutes’ walk away from
work; can she exercise during her lunch breaks; what exercise
can she do at weekends? Picking activities she enjoys will turn
exercise from a chore to a hobby. Can she gain needed support4
by teaming up with a work colleague or friend who also wishes
to lose weight?
One pitfall is linking exercise with a reward of food and
actually putting on more weight. To burn off the kilojoules in
one banana (365 kJ), the average person would need to cycle for
11 minutes; to burn off the kilojoules in one beer (585 kJ), they
would need to walk for 33 minutes—and one jam doughnut
(1360 kJ) is equivalent to
References Page 331

1. The Royal Australian College of General Practitioners 2018, Guidelines


for preventive activities in general practice, 9th ed, updated, RACGP,
East Melbourne, Vic, pp. 69–72.
2. Grima, M & Dixon, JB 2013, ‘Obesity—recommendations for
management in general practice and beyond’, Australian Family
Physician, vol. 42, pp. 532–41.
3. Handbook of Non-Drug Interventions (HANDI), Pre-meal water
consumption for weight loss, Royal Australian College of General
Practitioners.
4. Russell, HA, Rufus, C, Fogarty, CT, Fiscella, K & Carroll, J 2013, ‘“You
need a support. When you don’t have that . . . Chocolate looks real good”.
Barriers to and facilitators of behavioural changes among participants of a
healthy living program’, Family Practice, vol. 30, pp. 452–8.

Further Reading
The Royal Australian College of General Practitioners 2015, Smoking,
nutrition, alcohol, physical activity (SNAP): A population health guide
to behavioural risk factors in general practice, 2nd ed, RACGP,
Melbourne, Vic.
Page 332

Case 66
Ali Turnbull

Instructions for the doctor


This is a short case.
Please take a history and negotiate a management plan with Ali.

Scenario
Ali Turnbull is a 32-year-old artist who comes to see you requesting
sleeping tablets. This is the first time that she has been to this surgery.

Prior to seeing you she completed the new patient questionnaire as below:
Past medical history
Recurrent tonsillitis
Medication
Nil
Allergies
Nil
Immunisations
Can’t remember, will ask Mum
Cervical screen
Normal two years ago
Family history
Mum and Dad both fine, live interstate
Social history
Artist
Single Smokes 15 cigarettes per day
No recreational drugs.

Instructions for the patient, Ali Turnbull Page 333

You are a 32-year-old artist who has come to the GP to request sleeping
tablets. You used to combine your artwork with work as a Community
Development Officer at the downtown youth centre. This year you sold
enough paintings to be able to afford to paint full-time. You love the freedom
and chance to express yourself and are happy to be earning enough money
doing what you love to do.
Your timetable is flexible enough to fit around your inspiration to paint
and you often end up painting all night if you are working well. You fit
shopping, washing and cleaning in between painting sessions, sometimes
taking your work outside for more inspiration. You drink about eight cups of
coffee per day and do no regular exercise. You drink two to three beers a day.
Your ideal evening is painting until late and then catching up with friends on
social media from the comfort of your bed.
Your mood is stable and you have no weight loss or other symptoms
suggestive of an organic illness.
The only frustration is that you cannot sleep and you have decided to see if
the local GP can help. This is your first visit to this practice as you previously
saw the GP near the youth centre.

Before seeing the GP you completed the new patient questionnaire as


below:

Past medical history


Recurrent tonsillitis
Medication
Nil
Allergies
Nil
Cervical screen
Normal two years ago
Immunisations
Can’t remember, will ask Mum
Family history
Mum and Dad both fine, live interstate
Social history
Artist
Single
Smoke 15 cigarettes per day
No recreational drugs.

Suggested approach to the case Page 334

Establish rapport
Open-ended questions to establish Ali’s ideas, concerns and expectations.

Specific questions1
Sleep pattern and quality, bedtime regime
Lifestyle
— Work
— Family or community commitments
— Alcohol and other drugs
— Smoking
— Exercise
General health
— Brief systems review to exclude organic illness
Mental health
— Mood, energy levels—exclude mania or depression
Strategies and treatments tried so far.

Diagnosis
Primary insomnia, secondary to lifestyle.

Management1, 2
Explain problem—need for significant lifestyle change to improve sleep
quality in short-term and long-term health
No indication for medication
Appropriate sleep environment—quiet, dark, appropriate temperature,
comfortable bedding
Regular routine—consistent daytime rising important
Develop regular night-time routine—keep bed for sleep, not the internet
Cut down caffeine
Stop smoking
Cut down alcohol consumption
Regular exercise (at least two hours ahead of bedtime)
If can’t sleep, get up and try again
Bedtime restriction—limiting duration in bed to average calculated sleep
time.

CASE COMMENTARY Page 335

Ali’s success in her painting career has meant freedom from the
restrictions imposed by paid external employment. Her love of
her work means that it seems effortless to her to stay up for
much of the night painting. However, this freedom comes at a
price of poor sleep and a lifestyle that has the potential for harm
in the long term. Ali needs specific advice on how to restore her
circadian rhythms to promote good quality sleep and a non-
judgemental space to discuss the implications of her smoking,
alcohol consumption and lack of exercise.
The last thing that Ali needs is medication so that she can
mask the problem of not sleeping and continue to push her body
beyond the design specification!

COMMON PITFALLS

Poor sleep can be a sign of mental illness such as mania or


depression, or might herald a new onset of an organic illness
such as diabetes mellitus causing nocturia. While Ali’s story
seems immediately related to her lifestyle, it is vital to avoid
‘premature closure’ of the diagnosis and exclude the symptoms
indicative of more sinister causes.

References
1. Cunnington, D, Junge, M, Fernando & A 2013, ‘Insomnia: prevalence,
consequences and effective treatment’, Medical Journal of Australia, vol.
199, no. 8, pp. S36–40.
2. Therapeutics Guidelines Australia 2013, ‘Patient information sheet:
advice on good sleep practices’.

Further reading
Berk, M 2009, ‘Sleep and depression—theory and practice’, Australian
Family Physician, vol. 38, no. 5, pp. 302–4.
Fernando 3rd, A, Arroll, B & Falloon, K 2013, ‘A double-blind randomised
controlled study of a brief intervention of bedtime restriction for adult
patients with primary insomnia’, Journal of Primary Health Care, vol.
5, no. 1, pp. 5–10.
Mansfield, D & McEvoy, R 2013, ‘Sleep disorders: a practical guide for
Australian health care practitioners’, Medical Journal of Australia, vol.
199, issue 8 supplement, p. 8.
The Royal Australian College of General Practitioners, ‘Brief behavioural
therapy: insomnia in adults’, Handbook of Non-Drug Interventions
(HANDI). Page 336
Page 337
Section 19
Professional
practice
Page 338

Case 67
Vincent Butler

Instructions for the doctor


This is a short viva.
Please discuss your management of this situation with a GP colleague.

Scenario
You are on call and it is the end of the Saturday morning surgery.
Your receptionist gives you a message from the laboratory to say that Mr
Vincent Butler’s INR level is 5.7.

You find the following information in Mr Butler’s medical record:


Age
62
Past medical history
Mitral valve replacement six years ago
Hypertension
Medication
Warfarin 4 mg, alternate day 5 mg
Ramipril 10 mg per day
Atorvastatin 20 mg per day
Allergies
Nil recorded
Immunisations
Nil recorded
Social history
Retired chef.

Instructions for the facilitator


This is a viva.
The doctor is expected to talk with you as a professional colleague. Page 339
During the viva please ensure you ask the following questions:
1. What is your management of Mr Butler today?
2. What are the common causes of poor INR control?
3. What systems should there be in a practice to monitor the INR of patients
on warfarin?
4. You decide to do a locum in a remote Aboriginal community.
Would that make any difference to your management of patients on
warfarin?

The following information is in Mr Butler’s medical record:


Age
62
Past medical history
Mitral valve replacement six years ago
Hypertension
Medication
Warfarin 4 mg, alternate day 5 mg
Ramipril 10 mg per day
Atorvastatin 20 mg per day
Allergies
Nil recorded
Immunisations
Nil recorded
Social history
Retired chef.

Suggested approach to the case


1. Management of the high result
Clarify result with laboratory—preferably in writing/fax
Check that it is the correct result for the correct patient on the correct date
Search notes for:
• contact details—inform patient by phone if possible or via text
message or through friends/family. If not by direct contact candidate
must ensure that Mr Butler contacts you back to confirm receipt of the
message and management plan. If not contactable may need to do
home visit
• clinical information—reason for being on warfarin, target INR Page 340
1, any recent medication or dietary change likely to alter INR,

pattern of INR control.2


Management plan3—to discuss with Mr Butler
• Plan for temporary cessation of warfarin, next test date and how to get
results to find out next dose of warfarin
• Reiterate indications that would require hospital assessment:
prolonged bleeding from a cut or unexpected bleeding
• Recommend quiet weekend, avoid contact sports
• Question to determine what affected control.
2. Common causes of poor INR control
• Poorly calibrated machine/incorrect use of machine on point-of-care
testing
• Omissions or commissions taking warfarin, either deliberate or
accidental
• Medication interaction: patients should inform any prescriber that they
are on warfarin, as NSAIDs and antibiotics (ciprofloxacin,
metronidazole, doxycycline, erythromycin) are common causes of
raised INR
• Herbal products known to interact with warfarin include gingko,
ginger, ginseng, garlic, St John’s wort
• Change to a different formulation—the formulations of warfarin are
not interchangeable
• Dietary change: increase in consumption of foods high in Vitamin K,
such as green vegetables, can reduce INR; conversely a decrease in
consumption can increase INR
• Alcohol consumption.
3. What systems should there be in a practice to monitor the INR of
patients on warfarin?
Practices need systems that are not dependent on one person; they should
be ‘locum-proof’ and ‘holiday-proof’.
The systems can be electronic or paper-based and should include the
following:
• a list of all patients on warfarin
• for each patient on warfarin, their notes should say the indication,
planned duration of treatment and target INR
• each patient should be advised about the need for regular monitoring,
to alert any prescriber that they are on warfarin, to maintain a stable
diet, and of symptoms that would require urgent attention. A medical
alert bracelet should be considered
• method of checking that each person being prescribed Page 341
warfarin has regular INRs. The minimum recommended
interval is monthly. Tests can be done at a laboratory or onsite using
point-of-care testing equipment
• a result is received for each INR requested
• each INR result is checked on the day that the sample was taken
• patients get or receive their INR results on the day that the sample was
taken. Each result to include the INR result, the current and
recommended new dose and date of next test
• each result is signed, dated and filed
• if using point-of-care testing, calibrating and using instrument as per
manufacturer’s instructions
• system should include protocol for weekend and on-call handling of
results.
4. You decide to do a locum in a remote Aboriginal community. Would
that make any difference to your management of patients on
warfarin?
Management may need to take into consideration the following:
• use of point-of-care testing very common
• if lab is used, potential delay in getting specimen to lab and getting
results
• limited ability to contact patients by telephone. May need to liaise
with clinic staff about how results are normally passed to patients, e.g.
clinic bus or via Aboriginal liaison officer
• limited use of English. May need to use interpreters or in clinic use
warfarin tablets to demonstrate how many of each colour to take
• intermittent access to food, including fruit and vegetables, may affect
INR control
• lack of secure facilities to store warfarin
• patient may choose a nomadic lifestyle—need to assess risk/benefits
of warfarin in this situation
• potential high use of alcohol—particularly in binges. What is the
patient’s individual risk/benefit equation?
• cultural factors—warfarin may need to be ceased ahead of some
ceremonies/pay-back.

CASE COMMENTARY
This case requires the GP to talk with a professional colleague
about a common clinical situation. The doctor will be assessed
on their ability to listen and then talk concisely, clearly Page 342
and appropriately answer the questions. Medical jargon
can be used provided it is clear that both doctors use the same
jargon or abbreviations to have the same meaning.
Warfarin is indicated for primary and secondary prevention
of cardiovascular disease following evidence from randomised
controlled trials of its benefits.1 While there is evidence of
benefit at the population level for those at high risk, it is up to
the GP to assess the risks and benefits for an individual patient,
taking into consideration that patient’s psychological, social,
cultural and biomedical factors and the clinic’s ability to monitor
the drug safely.2, 3 The new anticoagulants that do not require
INR monitoring are changing the treatment of non-valvular atrial
fibrillation but are not licensed for use in patients like Mr Butler
who have had valve replacements.4

COMMON PITFALLS

Medico-legal advice is that the prescription of warfarin requires


patients ‘who are well instructed, able to communicate clearly,
cognitively intact, cooperative and compliant’.5
The duty of care that GPs have for following up and taking
action on results depends on the clinical significance of the test
results. Clinical significance is determined by the probability
that the patient will be harmed if further medical advice is not
obtained and the likely seriousness of the harm.6 Warfarin and
INR problems accounted for the largest proportion (18%) of
medication errors in the Threats to Australian Patient Safety
(TAPS) study.7
References
1. Whitlock, R, Sun, J, Fremes, S, Rubens, F & Teoh, K 2012,
Antithrombotic and thrombolytic therapy for valvular disease:
Antithrombotic Therapy and Prevention of Thrombosis, 9th ed, American
College of Chest Physicians Evidence-Based Clinical Practice Guidelines,
141 (2 supplement) e:5765–e005.
2. Tideman, P, Tirimacco, R, St John, A & Roberts, G 2015, ‘How to
manage warfarin therapy’, Australian Prescriber, vol. 38, no. 2, pp. 44–8.
3. Tran HA, Chunilal SD, Harper PL, Tran H, Wood EM & Gallus AS
2013, ‘An update of consensus guidelines for warfarin reversal’, Medical
Journal of Australia, vol. 198, no. 4, pp. 198–9.
4. Brieger, D & Curnow, J 2014, ‘Anticoagulation: a GP primer on the new
oral anticoagulants’, Australian Family Physician, vol. 43, no. 5, pp. 254–
9.
5. Bird, S 2003, ‘Medication errors: warfarin’, Australian Family Physician,
vol. 32, no. 12, pp. 1003–4. Page 343
6. The Royal Australian College of General Practitioners 2017,
‘Follow-up systems’, Standards for general practices, 5th ed, RACGP,
East Melbourne, Vic.
7. Makeham, MAB, Saltman, DC & Kidd MR 2008, ‘Lessons from the
TAPS study. Warfarin: a major cause of threats to patient safety’,
Australian Family Physician, vol. 37, no. 10, pp. 817–18.
Page 344

Case 68
Stephanie Clark

Instructions for the doctor


This is a short case.
You have arranged a meeting with your receptionist, Stephanie Clark, to
manage the following situation.

Scenario
Stephanie Clark works as a receptionist at your single-doctor practice.
Her 3-year-old child, Letitia, attends the local childcare centre. Last
week another child, Daisy, attended the practice because of diarrhoea.
Daisy’s mum, Sally, was advised to keep Daisy at home until the results
of the stool tests were available.
Sally brings Daisy to the practice today for a follow-up. Sally
mentioned that she already knew that the stool test showed an infection
that needed treatment, as Stephanie had told her when they met in the
supermarket yesterday.
Breach of patient confidentiality is a reason for instant dismissal in all
your staff contracts. Your practice manager is currently on leave and so
you have to meet with Stephanie on your own.

Instructions for the receptionist, Stephanie


Clark
You work as a receptionist at the local general practice. Your 3-year-old
daughter, Letitia, attends the local childcare centre.
Children at the childcare centre have had a ‘run’ of diarrhoea. You have
been hoping that Letitia would not catch the infection as you would have to
take time-off work to care for her.
Letitia’s best friend is Daisy. Last week Daisy attended the practice
because of diarrhoea. Daisy’s mum Sally was advised to keep Daisy at home
until the results of the stool tests were available. When you chatted Page 345
with Sally at the supermarket yesterday you told her the result
showed Daisy needed treatment. Sally was happy to know this, as she could
then reschedule her work.
You are a bit surprised that the GP has asked to see you this morning.
When the GP tells you about the breach of confidentiality and implications
for your job, you react defensively. You consider that the rules are ‘over-the-
top’ and stupid—Sally is your friend and you have a right to tell her what you
know about her own child.
You show no appreciation of the seriousness of your error.

Suggested approach to the case


Arrange the meeting in a quiet, private office and ask Stephanie if she wishes
to bring a support person
Explain the reason for meeting—Sally knew the results of Daisy’s stool test
prior to seeing the doctor
Sensitively ask for Stephanie’s side of the story
Demonstrate active listening to Stephanie
Explore Stephanie’s understanding and response to a reminder of the rules of
patient confidentiality
Remind Stephanie that her employment can be terminated immediately if a
breach of confidentiality occurs
State that Stephanie has breached confidentiality based on information from
Sally and herself
Tell Stephanie that her employment is being ceased and that she will be paid
outstanding leave entitlements. Ask her to leave the practice and remind
her that her obligation to confidentiality continues after leaving the job
Document the meeting and its outcome in Stephanie’s personnel folder.

CASE COMMENTARY

This case tests the doctor’s knowledge of the legal and


organisational domain of general practice and their skill in
applying this knowledge in a difficult situation. All GPs need
this core competence, even if many delegate such tasks to
practice managers or choose to be an employee in a corporate
entity.
Privacy of health information is a legislative requirement.1
The Medical Board of Australia considers that ‘Patients or
clients have a right to expect that practitioners and their staff
will hold information about them in confidence, unless Page 346
information is required to be released by law or public
interest considerations’.2 Confidentiality is a particular issue for
rural and remote practices where everybody knows everybody,
but this does not mean that everybody has a right to know
everybody’s business.
Stephanie has committed a serious offence that threatens the
reputation and potentially the future viability of the practice. The
GP is fortunate that the patient has not lodged a formal
complaint. Stephanie’s response shows that she has little insight
into the seriousness of her misconduct. Continuing her
employment risks further breaches and so the GP must take the
necessary step of terminating her employment immediately.

References
1. The Royal Australian College of General Practitioners National Expert
Committee on Standards for General Practices. Standards for General
Practices, 5th ed, RACGP, Launched 26 October 2017. Criterion C6.3:
Confidentiality and privacy of health and other information. Available at:
www.racgp.org.au/your-practice/standards/standards-for-general-
practices-(5th-edition)/, accessed 3 September 2018.
2. Medical Board of Australia. Good medical practice: A code of conduct
for doctors in Australia. 3.4 Confidentiality and privacy,
www.medicalboard.gov.au/Codes-Guidelines-Policies/Code-of-
conduct.aspx, accessed 3 September 2018.
Page 347

Case 69
Debra and Declan Poole

Instructions for the doctor


This is a short case.
You are expected to discuss a clinical problem with the facilitator role
playing your senior colleague.

Scenario
Last Friday you did your first evening on call for the practice. It was
much busier than you had expected. As it was your wedding anniversary
you were keen to get home as soon as possible.
You are a bit puzzled as to why the practice principal has asked to
chat with you after this morning’s surgery about a problem. Your
memory is that you saw mostly kids with runny noses and one with an
ear infection.
Instructions for the practice principal
Last weekend you were on call when you received a phone call from an
angry and anxious mother, Debra Poole. Her son Declan, aged four, had a
painful ear and they had come to see the new doctor at the practice. Debra
said that the doctor seemed to be in a hurry and had a quick look in his ears
but no further examination before prescribing amoxycillin.
Debra gave Declan the first dose of the medication but then looked more
carefully at the packet and saw the drug was similar to penicillin. Two years
previously Declan had had an allergic reaction to penicillin requiring him to
stay overnight in hospital. Once she realised what had happened she went
straight to the emergency department. As a precaution he was admitted to
hospital for observation. Fortunately, Declan did not react to the single dose
of amoxycillin but Debra lost a day’s pay because she monitored him the
following day.
When you talked with Debra on the weekend you promised that Page 348
you would discuss this with your colleague on Monday.

Suggested approach to the case


Doctor to listen carefully to the practice principal’s concerns
Doctor demonstrates understanding of different issues involved
— Problem of a cursory clinical examination and risk of missing other
more serious pathology
— Impact of hurrying the consultation
— Inappropriate prescribing of amoxycillin despite allergy—an
avoidable and potentially fatal mistake
— Financial impact of mistake on patient’s family
— Impact of the error on the practice reputation/business
Doctor acknowledges mistakes and outlines plans for action
— Apologise to the family for the mistake
— Apologise to the practice principal for impact on practice/reputation
— Conduct more appropriate full physical examinations in the future
— Check with patients about allergies and use information in the records
— Check if Declan’s file includes his penicillin allergy and update it if
required
— Undertaking prescribing modules from RACGP or NPS
MedicineWise
— Seek assistance with consultation techniques such as asking senior
doctor to sit in or observe videotaped consultations
— Ask practice principal if there is anything else that should be done
— Inform their medical defence organisation.

CASE COMMENTARY

This case tests a doctor’s ability to talk to a colleague, to


recognise a mistake and to learn from it. The Medical Board of
Australia’s code of conduct states that ‘When adverse events
occur, you have a responsibility to be open and honest in your
communication with your patient, to review what has occurred
and to report appropriately’.1 In this instance, formal reporting
seems unnecessary as no severe reaction occurred, but the doctor
should apologise to the family and the practice principal for the
error. Next, there should be reflection on the precipitants of the
problem. In this case, it was the tension between a work Page 349
and a home commitment—a common scenario for GPs.
This does not mean that GPs should not have a home life but
that, in most instances, it should take second place when on call.
A useful acronym is HALT: a GP should take extra care when
Hungry, Angry, Late or Tired.
The doctor should outline to the practice principal what they
intend to do differently in the future to prevent another similar
event. Regular reviews of critical incidents and complaints are
recommended, so that individuals and practices can learn from
errors and modify processes to reduce risks.2 A counterbalance
to negative feedback can be ‘compliments received’, so that
good practice is also acknowledged and reinforced.
Computerised alerts embedded in electronic health records
have the potential to reduce prescribing errors by warning
prescribers of possible risks such as allergies, but many studies
report that doctors override computerised alerts up to 95% of the
time.3–5 ‘Alert fatigue’ resulting from excessive numbers of
alerts has been identified as the primary reason for this but there
is evidence that the timing of the alerts (often in the final stages
of prescribing) also contributes to an alert being regarded as
intrusive and unwelcome, and subsequently being ignored.6

COMMON PITFALLS

The doctor in this scenario must avoid defending their actions


and blaming the mother for not mentioning the penicillin
allergy. Checking an allergy history is good practice whenever a
medication is being prescribed. Absence of a documented
allergy is not evidence of an absence of allergy. Such errors in
the process of care are more common than errors due to lack of
knowledge or skills.7

References
1. Medical Board of Australia. Good medical practice: A code of conduct
for doctors in Australia. 3.10 Adverse events,
www.medicalboard.gov.au/Codes-Guidelines-Policies/Code-of-
conduct.aspx, accessed 3 September 2018.
2. Steer, N 2010, ‘Managing clinical risks—tips from the toolkit 9’,
Australian Family Physician, vol. 39, no. 10, pp. 791–2.
3. Weingart, SN, Toth, M, Sands, DZ, Aronson, MD, Davis, RB & Phillips,
RS 2003, ‘Physicians’ decisions to override computerised drug alerts in
primary care’, Archives of Internal Medicine, vol. 163, pp. 2625–31.
4. van der Sijs, H, Aarts, J, Vulto, A & Berg, M 2006, ‘Overriding Page 350
of drug safety alerts in computerized physician order entry’,
Journal of the American Medical Informatics Association, vol. 13, no. 2,
pp. 138–47.
5. Sweidan, M, Reeve, JF, Brien, JE, Jayasuriya, P, Martin, JH & Vernon,
GM 2009, ‘Quality of drug interaction alerts in prescribing and
dispensing software’, Medical Journal of Australia, vol. 190, pp. 251–4.
6. Hayward, J, Thomson, F, Milne, H, et al. 2013, ‘Too much, too late:
mixed methods multi-channel video recording study of computerized
decision support systems and GP prescribing’, Journal of the American
Medical Informics Association, vol. 20, pp. e76–e84.
7. Makeham, M, Stromer, S, Bridges-Webb, C, Mira, M, Saltman, D,
Cooper, C, et al. 2008, ‘Patient safety events reported in general practice:
a taxonomy’, Quality & Safety in Health Care, vol. 17, no. 1, pp. 53–7.

Further reading
Australian Commission on Safety and Quality in Health Care 2013,
Implementing the Australian Open Disclosure Framework in small
practices. ACSQHC, Sydney.
Page 351
Section 20
Respiratory
medicine
Page 352

Case 70
Andrew Bond

Instructions for the doctor


This is a long case.
Please take a history and conduct an appropriate clinical examination.
Outline to Andrew your differential diagnosis and plans for initial
management.

Scenario
Mr Andrew Bond is a 65-year-old man who has not been to the surgery
for a while. He is usually well. You spent time with him a year ago when
he was giving up smoking. You are not sure why he has booked this
appointment with you today.

The following information is on his summary sheet:


Past medical history
Vasectomy aged 42
Medication
Nil
Allergies
Nil
Immunisations
Up-to-date
Social history
Retired bus driver
Gave up cigarette smoking last year—was 20 per day for 40 years.

Instructions for the patient, Andrew Bond


You are a 65-year-old retired bus driver who gave up cigarette smoking last
year. The final spur to giving up was when your older brother was diagnosed
with lung cancer. Like you, he had smoked since leaving school. You Page 353
had a bit of a cough for the weeks after you gave up but then you
were fine and felt better than you had for years.
You have developed a new cough in the last three to four weeks. It is
worst first thing in the morning. You have coughed up some blood but mostly
it is a dry cough. You have lost weight and your appetite is not so good. You
get puffed more easily than you used to but you are still managing to do the
wood-turning you enjoy.
At the back of your mind is the thought that this might be cancer. You are
not going to tell the GP that this is your concern but will talk about it if the
GP mentions it.

Clinical examination findings


Height 1.7 m
Weight 67 kg (weight five years ago 73 kg)
BMI 22.1 kg/m2
No abnormal clinical findings.
The following information is on your medical record:
Past medical history
Vasectomy aged 42
Medication
Nil
Allergies
Nil
Immunisations
Up-to-date
Social history
Retired bus driver
Gave up cigarette smoking last year—was 20 per day for 40 years.

Suggested approach to the case


Establish rapport
Open-ended questions to establish Andrew’s ideas, concerns and
expectations.

Specific questions
Cough
Haemoptysis
Sputum Page 354
Pain Shortness of breath
Fever
Family history
Social history
— Confirm smoking history
— Occupational history—exposure to asbestos
— Travel
Systems review—to include weight loss/appetite/energy/sleep
Request permission to examine.

Examination
General signs
— Weight loss
— Cyanosis
— Clubbing
Respiratory system
— Respiratory rate
— Use of accessory muscles
— Trachea
— Chest expansion
— Percussion
— Auscultation
— Lymph node enlargement
Abdominal examination looking for evidence of metastatic spread to the
liver.

Differential diagnosis
Lung cancer—doctor must mention this as a possibility
Acute respiratory tract infection
Chronic obstructive pulmonary disease
Tuberculosis (less likely without risk factors).

Initial investigations
FBC
ESR or CRP
UEC
LFTs Page 355
Chest X-ray
Consider spirometry
Sputum for cytology and MCS.

Management
While not an initial investigation, Mr Bond has risk factors for lung cancer
and will require a chest CT and specialist referral irrespective of initial
results
Assure Mr Bond of ongoing support
Non-urgently need to find out about alcohol consumption and check of
immunisation status
No proven benefit for cough medicine
Arrange follow-up.

CASE COMMENTARY

Mr Bond’s story of past long-term smoking, loss of weight and


appetite, and a new cough and haemoptysis indicates lung
cancer until proved otherwise. Haemoptysis is the only symptom
that in isolation consistently predicts lung cancer.1 As there is
high clinical suspicion of lung cancer, a CT chest and urgent
referral to a specialist linked with a lung cancer
multidisciplinary team is indicated even if the initial chest X-ray
is negative.2, 3
It is essential that the doctor includes lung cancer in the
differential diagnosis. In clinical practice the timing of when to
share the potential diagnosis of cancer is determined by many
factors. These include the GP’s prior relationship to the patient
and the patient’s level of interest in finding out the diagnosis.
Patients have the right to information and evidence suggests that
patients are keen to find out more than doctors are willing to tell
them. True patient-centred medicine gives the right level of
information, in understandable terms, at the right pace and at the
right time, as determined by the patient.
Poor doctors will either tell Mr Bond a definite diagnosis of
cancer with blunt disregard for his feelings or be too
embarrassed or anxious to mention the word cancer. Try to
avoid these extremes. Mr Bond is already concerned regarding
cancer. A good doctor will explore these pre-existing concerns
and sensitively talk about the differential diagnosis in the light
of this and the clinical situation. Empathy with Mr Bond’s
reactions will also be demonstrated.

References Page 356

1. Shim, J, Brindle, L, Simon, M & George, S 2014, ‘A systematic review of


symptomatic diagnosis of lung cancer’, Family Practice, vol. 31, no. 2, pp.
137–48.
2. Earwood, JS, Dwight, D & Thompson, TD 2015, ‘Hemoptysis: evaluation
and management’, American Family Physician, vol. 91, no. 4, pp. 243–9.
3. Cancer Australia. Investigating symptoms of lung cancer: a guide for GPs,
2012. Available at: www.canceraustralia.gov.au/publications-and-resource
s/cancer-australia-publications/investigating-symptoms-lung-cancer-guide
-gps, accessed on 23 June 2018.
Page 357

Case 71
Kerrie Griffiths

Instructions for the doctor


This is a short case.
Please take a history from Kerrie and conduct an appropriate focused
clinical examination. Outline your differential diagnosis to the facilitator.

Scenario
Kerrie Griffiths is a 32-year-old woman whose first child was born by
caesarean section a week ago. Baby Eloise is doing fine and Kerrie was
discharged home yesterday.
This morning Kerrie has noticed a sharp pain on the left side of the
chest. The pain is worse on breathing in. You are called to see Kerrie at
home.
The following information is on her medical record:
Past medical history
Asthma
Medication
Fluticasone 250 mcg 1 puff bd
Salbutamol inhaler prn
Allergies
Nil
Immunisations
Up-to-date, rubella booster pre-pregnancy
Family history
Nil significant
Social history
Non-smoker.

Instructions for the patient, Kerrie Page 358

Griffiths
You are 32 years old and your first child was born by caesarean section a
week ago. Baby Eloise is doing fine and you were discharged home
yesterday. This morning you have noticed a sharp pain on the left side of
your chest. This is worse on breathing in and you feel short of breath. You
have asked the doctor to see you at home.
This is a new pain. It is preventing you from breathing comfortably. You
have had a cough this morning and have coughed up a bit of blood. You do
not have a temperature and do not have any pain or symptoms suggestive of a
deep vein thrombosis in your legs. You are breastfeeding without difficulty.

The following information is on your medical record:


Past medical history
Asthma
Medication
Fluticasone 250 mcg 1 puff bd
Salbutamol inhaler prn
Allergies
Nil
Immunisations
Up-to-date, rubella booster pre-pregnancy
Family history
Nil significant
Social history
Non-smoker.

Clinical examination findings


When the doctor examines you, please tell them the following:
• Temperature 37.2°C
• Pulse 106 regular
• BP 112/78 mmHg
• Respiratory rate 20 per minute.
You will experience pain in your left lower lobe on inspiration and be
reluctant to take deep breaths
The chest is clear
Legs are normal (the most likely diagnosis is that you have a pulmonary
embolus).

Suggested approach to the case Page 359

Establish rapport
Ask after the baby, feeding, sleep etc.
Open questions to explore Kerrie’s ideas, concerns and expectations.

Specific questions
Details about the pain
Check for haemoptysis, dizziness, syncope, shortness of breath
Enquire about asthma control Exclude infection as likely cause—ask about
fever, systemic upset
Leg pain or swelling
Past history of thromboembolism or known thrombophilia disorders
Exclude mastitis/feeding problem
Reason for caesarean section—epidural or general anaesthetic?
Social history—does she smoke?
Family history of thromboembolism or thrombophilia disorders
Request permission to examine.

Examination
Temperature
Pulse
BP
Respiratory rate
Chest
— Use of accessory muscles
— Palpation—chest expansion
— Percussion
— Auscultation
Heart sounds
Legs—for signs of a DVT.

Differential diagnosis
Must mention pulmonary embolus
Basal atelectasis
Pneumonia
Other options including musculoskeletal chest wall pain—less likely.

CASE COMMENTARY Page 360

This case provides doctors an opportunity to practise, time and


demonstrate their skills in taking a history and conducting an
examination of both the cardiovascular and respiratory systems.
The doctor must sensitively ask permission to examine the
breasts, as mastitis is one of the potential diagnoses. The need
for a chaperone should be discussed even though arranging this
can be difficult on a home visit. The facilitator will explain that
examination of the breasts is normal and that this is not required
in this case.
The combination of new pleuritic chest pain, shortness of
breath, a cough, haemoptysis and recent delivery requires the
doctor to consider pulmonary embolus as a potential cause.1
Pulmonary embolus is more common (relative risk over four)
during pregnancy and is 15 times more common in the
postpartum period,2 hence this serious condition requires careful
consideration during these times.
Although this case does not ask the doctor for management, it
is worth noting that pulmonary embolism cannot be excluded
from just a history and examination. Kerrie requires transfer to
hospital for further tests. Risk stratification tools such as the
Wells score and PERC rule3 are less useful in the pregnancy and
postpartum setting. Likewise, D-Dimer testing is inappropriate
postpartum and post-surgery, and indeed her high risk of
pulmonary embolism means that a negative D-Dimer test would
NOT rule out this diagnosis.4

References
1. Chapman, NH, Brighton, T, Harris, MF, Caplan, GA, Braithwaite, J &
Chong, BH 2009, ‘Venous thromboembolism—management in general
practice’, Australian Family Physician, vol. 38, pp. 36–40.
2. Tapson, VF 2008, ‘Acute pulmonary embolism’, New England Journal of
Medicine, vol. 358, pp. 1037–52.
3. Doherty, S 2017, ‘Pulmonary embolism: an update’, Australian Family
Physician, vol. 46, pp. 816–20.
4. Agnelli, G & Becattini, C 2010, ‘Acute pulmonary embolism’, New
England Journal of Medicine, vol. 363, pp. 266–74.
Page 361

Case 72
Paul Jackson

Instructions for the doctor


This is a short case.
Please read the following history. The results of the examination will be
available from the facilitator. Using this history and the examination findings,
outline your diagnosis and management plan to Paul.

Scenario
Paul Jackson is a 20-year-old man who is about to start a painting and
decorating apprenticeship. Paul had asthma as a child but has been well
since. Four weeks ago, he had a viral upper respiratory tract infection.
Since then he has had a cough, which is keeping him awake at night. He
has to stop playing soccer after only 10 minutes due to shortness of
breath and wheeze. He is noticing some wheeze occasionally during the
day, maybe two to three times per week.
The following information is on his summary sheet:
Past medical history
Childhood asthma
Medication
Nil
Allergies
Nil
Immunisations
Up-to-date
Family history
Maternal osteoporosis
Social history
Lives with de facto partner
Non-smoker.

Instructions for the patient, Paul Jackson Page 362

You are 20 years old and will soon start a painting and decorating
apprenticeship.
As a child you had asthma. Four weeks ago, you had a cold and since then
you have had a cough. The cough is dry and worse at night; your girlfriend is
getting really irritated by it. You are worried because you are short of breath
and wheezy after exercise. You can manage only around 10 minutes of
playing soccer before coming off. You have occasionally (two to three times
per week) felt a little wheezy at other times during the day. It feels as though
your asthma has come back.

The following information is on your medical record:


Past medical history
Childhood asthma
Medication
Nil
Allergies
Nil
Immunisations
Up-to-date
Family history
Maternal osteoporosis
Social history
Lives with de-facto partner
Non-smoker.

Instructions for the facilitator


Please give the doctor the following information when requested specifically:
• Height 1.85 m
• Pulse 92
• Blood pressure 106/70 mmHg
• No cyanosis
• Talking in sentences
• Respiratory
— Respiratory rate 14/min
— No use of accessory muscles
— Chest expansion normal
— Percussion note normal
— Widespread wheeze on auscultation
— Oxygen saturation 99% in room air.

Peak expiratory flow rate actual 350 L/min Page 363


Peak expiratory flow rate predicted 580 L/min
Peak expiratory flow rate after salbutamol 450 L/min

Please hand these results to the doctor if spirometry is requested:


Actual: FEV1 3.5 L FVC 5.78 L Ratio FEV1/FVC 60.5%
Predicted: FEV1 5.05 L FVC 6.09 L Ratio FEV1/FVC 82.9%
After salbutamol: FEV1 4.3 L FVC 5.82 L Ratio FEV1/FVC 73.9%
Suggested approach to the case
Establish rapport
Brief summary of history—for example, ‘So I have read that you used to
have asthma. You had a cold recently and you are now wheezy and out of
breath. I’m going to find out the results of the physical examination and
then we can chat about what to do.’
Request permission to examine.

Ask for the examination findings


General appearance and colour
Heart rate and blood pressure
Oxygen saturation
Ability to talk in sentences
Respiratory examination
— Respiratory rate
— Use of accessory muscles
— Chest expansion
— Percussion
— Auscultation
— Oxygen saturation 99% in room air
— Peak expiratory flow rate
— Spirometry.

Diagnosis
Poorly controlled asthma.

Management
Explain likely recurrence of asthma
Asthma can recur in adulthood
Reassure Paul that asthma can be treated
Poorly controlled asthma—start both preventer (low-dose inhaled Page 364
corticosteroid) and reliever (e.g. salbutamol)
Discuss rationale and mechanisms of action of both medications
Demonstrate/explain the use of inhalers and spacer
On suggestion of starting inhaled steroids, watch for emotional reaction and
respond appropriately
Enquire sensitively about the reasons for the concern about steroids, for
example, Paul’s mother developed osteoporosis
Outline risks and benefits of inhaled steroids, in particular addressing any
concerns raised by Paul
Plan follow-up and development of an asthma action plan
Assure Paul that asthma should not stop him starting his apprenticeship but to
observe any impact from paint fumes
Advise on when to return for follow-up, and to contact urgently if condition
worsens
Discuss asthma first aid and emergencies
Other health promotion—use of alcohol and other drugs
Advice regarding influenza vaccination.

CASE COMMENTARY

The key issue in this case is to explain to Paul that it is most


likely his asthma has returned.
According to the National Asthma Council’s Australian
Asthma Handbook, Paul has poorly controlled asthma, requiring
management with preventer therapy. Most patients can be well-
managed on a low-dose inhaled corticosteroid, e.g. 100–200
mcg fluticasone propionate or beclomethasone dipropionate or
250–400 mcg budesonide.1, 2
Optimal management will involve an inhaled beta-agonist
administered through a spacer, commencement of inhaled
steroids as above and early review to ensure best lung function is
achieved, preferably assessed by spirometry.1, 2 Subsequent
visits can be used to maintain best lung function, minimise the
dose of medication while encouraging adherence3 and give Paul
an asthma action plan.1, 2 Paul can be encouraged by pointing
out that avoiding cigarette smoke and maintaining his fitness by
returning to soccer training should help to control his asthma.4
Paul can commence his apprenticeship but will need to observe
any impact of paint fumes on his breathing.

References Page 365

1. National Asthma Council 2017, Australian Asthma Handbook, Version


1.3. National Asthma Council Australia, Melbourne. Available at: www.as
thmahandbook.org.au, accessed 5 July 2018.
2. Hancock, K 2014, ‘Long-term management of asthma: the new Australian
guidelines’, Medicine Today, vol. 15, no.11, Suppl, pp. 6–19.
3. Martinez, FD & Vercelli, D 2013, ‘Asthma’, Lancet, vol. 382, pp. 1360–
72.
4. Carson, KV, Chandratilleke, MG, Picot, J, Brinn, MP, Esterman, AJ &
Smith, BJ 2013, ‘Physical training for asthma’, Cochrane Database of
Systematic Reviews 9:CD001116.
Page 366

Case 73
Nicholas Morris

Instructions for the doctor


This is a long case.
Please take a history and conduct an appropriate clinical examination.
Discuss with the patient the most likely diagnosis, your differential diagnoses
and negotiate a plan for management.

Scenario
Nicholas Morris is 63 years old and has booked in to see you this
morning. The last entry in the notes was five years ago when he attended
after a dog bite.

The following information is on his summary sheet:


Past medical history
Dog bite
Medication
Nil
Allergies
Nil
Immunisations
Nil recorded
Social history
Maths teacher
Single
Smokes 30 cigarettes per day.

Instructions for the patient, Nicholas Morris


You are a 63-year-old maths teacher and a lifelong smoker. You enjoy
smoking and get quite offended when people suggest that you should Page 367
give up. Even the pupils at school keep telling you to stop when they
see you smoking around town at the weekends. You only go to the doctor
when you really have to.
In the last few months you have become increasingly short of breath. You
take your dog for walks in the morning and this is getting more difficult. You
have begun to arrive earlier to school so that you can get a parking space near
the staffroom.
When asked by the GP you will reveal that you have had a productive
cough in the mornings for the last few years. You have never coughed up
blood. Colds ‘always go to your chest’ and when this happens you get a
wheeze.
You have been saving up to visit your relatives overseas and are due to fly
soon. You have made the appointment to discuss your breathlessness with the
GP and want to know if there is any medication that will fix the problem.

Clinical examination findings


Height 1.72 m
Weight 75 kg
BMI 25.3 kg/m2
Tobacco stains on the fingers
No respiratory distress, respiratory rate 16 breaths per minute
Chest shape and expansion normal
Percussion normal
Auscultation reveals scattered wheezes
PEFR 320 L/min, predicted 620 L/min
ECG shows normal sinus rhythm.

The following information is on your medical record:


Past medical history
Dog bite
Medication
Nil
Allergies
Nil
Immunisations
Nil recorded
Social history
Maths teacher
Single
Smokes 30 cigarettes per day.

Information for the facilitator Page 368

If requested, please hand these spirometry results to the doctor:


Actual: FEV1 2.0L FVC 3.3L Ratio FEV1/FVC 60.6%
Predicted: FEV1 3.2L FVC 4.3L Ratio FEV1/FVC 75.1%
After salbutamol: FEV1 2.1 FVC3.3L Ratio FEV1/FVC 63.6%

Suggested approach to the case


Establish rapport
Open-ended questions to establish Nicholas’ ideas, concerns and
expectations.

Specific questions
Shortness of breath, including exercise tolerance
Cough
Sputum
Haemoptysis
Wheeze
Fever
Chest pain
Orthopnoea/paroxysmal nocturnal dyspnoea
Ankle swelling
Impact of symptoms on lifestyle and ability to work
Smoking history
Alcohol history
Family history
Systems review—include energy levels, sleep, weight loss or change in
appetite
Request permission to examine.

Examination
General signs
— Weight and BMI
— Cyanosis
— Clubbing
— Tobacco stains on the fingers
Respiratory system
— Respiratory rate
— Use of accessory muscles
— Trachea
— Nodes
— Chest shape
— Chest expansion Page 369
— Percussion
— Auscultation
Cardiovascular system
— Pulse
— Blood pressure
— Heart sounds
— Signs of heart failure.

Most likely diagnosis


Chronic obstructive pulmonary disease

Differential diagnoses
Carcinoma of the bronchus
Tuberculosis
Infection
Heart failure.

Investigations
FBC
ESR or CRP
UEC
LFTs
Fasting lipids
Fasting BSL
Chest X-ray.

Management
Using the COPD-X management plan:
Confirm diagnosis and assess severity
Explain most likely diagnosis chronic obstructive pulmonary disease
Inform that this illness is related to smoking
Assess ability to travel overseas.
Optimise function
Inhaled bronchodilators
Pulmonary rehabilitation.
Prevent deterioration
Use motivational interviewing techniques to discuss smoking
Outline options for assistance in giving up smoking
Immunisations—influenza, pneumococcal vaccines as per Page 370
Australian Immunisation Handbook.
Develop support network and self-management plan
GP and primary care team follow-up
Develop written action plan to aid in recognition and response to
exacerbations.
Manage eXacerbations (cover in subsequent consultations)
Increased bronchodilators
Systemic steroids
Early treatment with antibiotics if signs of infection.

Other issues
Check alcohol consumption
Offer follow-up to address other preventative health issues.

CASE COMMENTARY

This case tests the doctor’s ability to identify chronic obstructive


pulmonary disease (COPD) as the most likely diagnosis and to
engage with Mr Morris about the importance of giving up
smoking. The COPD-X frame-work provides a useful mnemonic
for the key information that should be covered in the next few
consultations with Mr Morris. Further detail may be relevant to
some patients and is available in the full guideline.
COMMON PITFALLS

A poor doctor might display a judgemental attitude to the


smoking and ignore the issue of the forthcoming holiday. A
good doctor will use motivational interviewing to assess Mr
Morris’s readiness to change and give practical advice about
smoking cessation tailored to his readiness to quit (see Case 64
).
A good doctor will acknowledge the importance of the
holiday and assess Mr Morris’s fitness to fly. If Mr Morris can
climb a flight of 15 stairs and walk 50 metres without
symptoms, he should not experience problems during the flight
(see Case 79). He is unlikely to get travel insurance to cover his
respiratory condition, so buying a flexible ticket so that he can
travel when medically stable is recommended.

Further reading Page 371

Lim, ML, Brazzale, DJ & McDonald, CF 2012, ‘“Is it okay for me to. . .?”
Assessment of recreational activity risk in patients with chronic lung
conditions’, Australian Family Physician, vol. 41, no. 1, pp. 852–4.
Seccombe, L & Peters, M 2010, ‘Patients with lung disease. Fit to fly?’,
Australian Family Physician, vol. 39, no 3, pp. 112–5.
Walters, J 2010, ‘COPD—diagnosis, management and the role of the GP’,
Australian Family Physician, vol. 39, no. 3, pp. 100–3.
Yang, IA, Brown, JL, P George, J, Jenkins, S, McDonald, CF, McDonald, V,
et al. 2017, ‘COPD-X Concise Guide for Primary Care’. Available at: ww
w.lungfoundation.com.au, accessed 20 November 2018.
Page 372

Case 74
Jonty McLeod

Instructions for the doctor


This is a short case.
Please take a history from Jonty and his father, Matt. The results of a
focused physical examination will be available on specific request from the
facilitator. Outline the most likely diagnosis to Jonty and Matt and discuss
your suggested management plan with them.

Scenario
Jonty McLeod is 12 years old. Following his parents’ divorce, he spends
half his time with his mum, Elspeth, and half with his dad, Matt. Jonty
has mild persistent asthma, which is well controlled with inhaled
corticosteroids.
The following information is on Jonty’s medical record:
Past medical history
Asthma
Medication
Fluticasone 100 mcg bd
Salbutamol 200 mcg prn via spacer
Allergies
Nil known
Immunisations
Up-to-date
Family history
Mother has severe eczema
Social history
Attends high school.

Instructions for Jonty and Matt Page 373

Jonty
You are 12 years old. You are quite settled in the routine of spending half
your time with your father, Matt, and half with your Mum, Elspeth. You
enjoy being in your Mum’s city flat, which has a nearby cinema and ice-
cream shop. Time with your Dad is also enjoyable but more chaotic, as his
house is packed with half-finished custom-made furniture, plus his ever-
loyal, kelpie-cross Homer.
You have had asthma since you were a baby and it is well controlled. For
the last few months you have noticed it has been worse when you are at
Dad’s and you also seem to sneeze a lot. Your nose is often itchy, runny and
blocked, so that you can find it difficult to sleep.
You are in grade seven at a school half-way between Mum and Dad’s and
are generally a happy kid. There are no problems at either home or at school,
other than finding blowing your nose all the time annoying and embarrassing!

Matt
You are a self-employed carpenter. Your marriage to Elspeth fell apart
because she wanted to live in the city, whereas you wanted a more relaxed
country lifestyle. You have both been much happier since the divorce and
Jonty seems to thrive on the variety of having two very different homes.
In recent months you have been nagging Jonty to blow his nose and often
hear him sneezing and awake during the night. When you finally caught up
with Elspeth after one of her overseas trips, you were surprised to find out
that Jonty sleeps through when he is at her house and seems well most of the
time.
You have now come to see your own doctor to plan what to do.
Note: If you are unable to persuade someone to play Jonty’s role, please
change the scenario so that the consultation takes place with Matt on his own.

The following information is on Jonty’s medical record:


Past medical history
Asthma
Medication
Fluticasone 100 mcg bd
Salbutamol 200 mcg prn via spacer
Allergies
Nil known
Immunisations Page 374
Up-to-date
Family history
Mother has severe eczema
Social history
Attends high school.

Information for the facilitator


Please give the candidate the results of examination findings on specific
request.

Examination
Apyrexial, pulse 84, respiratory rate 14/minute
Ears normal
Throat normal
Nose—blocked, with serous discharge
Chest—no respiratory distress, clear on auscultation
Height 1.55 m (over 50th percentile)
Weight 51 kg
BMI 21 kg/m2
Predicted PEFR rate 323 L/min
Actual PEFR 255 L/min = 79% of predicted.

Suggested approach to the case


Establish rapport with both Jonty and Matt
Open questions to explore their ideas, concerns and expectations
Closed questions
— Nasal symptoms: itch, sneeze, nature of discharge, blockage or
bleeding, facial pain
— Asthma: wheeze, cough, shortness of breath, exercise tolerance, need
for salbutamol
— Eyes: itch, discharge, swelling, erythema
— Brief systems review: sleep, energy levels, appetite, growth
— Timing of current symptoms
— Exclude acute illness: no fever, no sore throat, no headache, no ear
pain
— Treatment tried so far.

Examination Page 375

Temperature, pulse, respiratory rate


Ears, nose, throat
Chest: movement, breath sounds, percussion note, PEFR
Growth.
Management
Most likely diagnosis of allergic rhinitis
Explain condition and its link with asthma.

Investigations
Observation of environmental precipitants
Consider skin-prick test or serum-specific IgE.

Treatment options
Non-pharmacological
— Identify and avoid allergens
Pharmacological
— Antihistamines
— Intranasal corticosteroids
— Saline nasal spray may help with congestion
Arrange follow-up and consider referral for immunotherapy.

CASE COMMENTARY

Allergic rhinitis is a common problem that is often


underdiagnosed and undertreated. Improving Jonty’s symptoms
is likely to improve his asthma as well as his sleep and thus his
quality of life. The most common environmental allergens to
consider are house dust mites, grasses, pollen, animal dander
and mould. The potential causes in this case include sawdust
from Matt’s carpentry and dander from Homer, the kelpie-cross.
The GP will need to give this information sensitively: if there is
ongoing tension between the two parents about living styles, the
fact that Jonty gets unwell each time he visits his Dad has the
potential to upset the dynamics and arrangements.
Depending on your location, it is worth considering Page 376
the potential for thunderstorm asthma in anyone with allergic
rhinitis, and particularly those with concomitant asthma (as
poorly controlled asthma is associated with worse outcomes).
Risk is greatest in those with seasonal (springtime) allergic
rhinitis, as they can be assumed to be sensitised to ryegrass
pollens. Jonty does not give a history of seasonal rhinitis and
hence ryegrass is less likely a trigger in this case. Risk of
thunderstorm asthma is greatest in south-eastern Australia, hence
the importance of considering this varies by location.
The increased number of children who have two homes under
shared parenting arrangements makes this an area where GPs
need to avoid escalating any difficult situations. If a child has
more than one GP, a shared electronic health record can
facilitate communication and avoid duplication of tests, or
Matt’s GP could give the other GP a hard copy of the record. As
Jonty enters his teenage years, this may become further
complicated by his right to confidentiality and this should be
discussed when using an electronic health record.

Further reading
Australasian Society of Clinical Immunology and Allergy 2017, ‘Allergic
rhinitis clinical update’. Available at: www.allergy.org.au/hp/papers/aller
gic-rhinitis-clinical-update, accessed 23 November 2018.
National Asthma Council 2014, ‘Australian Asthma Handbook, version 1.0’,
National Asthma Council, Melbourne. Available at: www.asthmahandboo
k.org.au, accessed 23 November 2018.
National Asthma Council 2017, ‘Epidemic thunderstorm asthma’, National
Asthma Council, Melbourne. Available at: www.assets.nationalasthma.or
g.au/resources/Thunderstorm-Full-WEB-JRD.pdf, accessed 23 November
2018.
Rueter, K & Prescott, S 2014, ‘Hot topics in paediatric immunology: IgE-
mediated food allergy and allergic rhinitis’, Australian Family Physician,
vol. 43, no. 10, pp. 680–5.
Page 377
Section 21
Sexual health
Page 378

Case 75
Ben Ramsay

Instructions for the doctor


This is a short case.
Please conduct this consultation as you would in your clinical practice.
There is no need to conduct an examination.

Scenario
Ben Ramsay is a 33-year-old engineer who recently ended a long-term
relationship with his partner Tim. He has presented to you today to
discuss taking PrEP to lower his risk of HIV. Please discuss his
suitability for PrEP and answer his questions.

The following information is on his summary sheet:


Past medical history
Rectal gonorrhoea 2000
Medication
Nil recorded
Allergies
Nil known
Immunisations
Nil known
Social history
Homosexual
Smokes five to ten cigarettes per day.

Instructions for the patient, Ben Ramsay Page 379

You are a 33-year-old engineer who recently ended a long-term relationship


with your partner Tim. You think of yourself as pretty sensible and your
friends know you as ‘Mr Safe Sex’ from all the reminders you’ve given over
the years, but have found being back on the single scene difficult. You had
unprotected receptive anal intercourse once after using methamphetamine at a
music festival and a condom broke last month. You had a negative STI check
six months ago. Despite going out more and burning the candle at both ends
you’ve not had any infections recently. You’re happy to have whatever tests
the doctor suggests today.
Some of your friends have started taking PrEP and after these two
episodes you think this might be a good idea. You’ve heard the rates of new
HIV cases are starting to go down with PrEP on the PBS and have some
questions for the doctor:
• Would I be eligible for PrEP?
• Are there any tests I need to have before I start?
• What’s the best way to take it? Some friends just take it if they know they
are going to have a big weekend of drugs and sex.
• What side effects can you expect?
• How often will you need to see a doctor and get tests while taking PrEP?

The following information is on your summary sheet:


Past medical history
Rectal gonorrhoea 2000
Medication
Nil recorded
Allergies
Nil known
Immunisations
Nil known
Social history
Homosexual
Smokes five to ten cigarettes per day.

Instruction for the facilitator


Please provide a hard and/or electronic copy of the Australian Medicines
Handbook or Therapeutic Guidelines.

Suggested approach to the case Page 380

Establish rapport
Demonstrate a non-judgemental empathic response to his request for PrEP
Listen carefully to his ideas and concerns
Exclude symptoms suggestive of recent STI
Assess his current risk of HIV and other STIs
Check details of PrEP using drug resource available and discuss with Ben
Reinforce value of using condoms.

The steps to prescribing PrEP are as follows:

1. Determine eligibility—medium to high risk eligible Those at high risk of


acquiring HIV include men who have sex with men who have a
concurrent STI-rectal gonorrhoea, chlamydia; those who have had
condomless anal intercourse with a HIV positive partner or partner of
unknown status in the previous six months, methamphetamine use.
Confirm HIV status—the patient must be confirmed HIV negative prior
2. to commencing PrEP PrEP should be commenced within seven days of a
negative HIV test being performed. This is because PrEP is inadequate in
the treatment of undiagnosed acute or chronic HIV.
3. STI and hepatitis testing
Screen for hepatitis B, hepatitis C, kidney function and STIs. Treat STIs
as required, and consult an expert in the case of a positive hepatitis B or C
test, or impaired kidney function. (PrEP can negatively impact kidney
function.) Review and update immunisations for HPV, hepatitis A and B
and meningococcus. Measure bone density if there are risk factors for low
bone density, and actively monitor.
4. Prescribe PrEP
Daily, continuing, oral dose PrEP. Patients need to take a daily dose of
PrEP for seven days before high levels of protection are achieved for both
vaginal and rectal exposure to HIV.
There is an ‘on demand’ regime of PrEP whereby the patient takes it
only at high-risk times. This is not as effective and should be discussed
with an experienced prescriber.
5. Review the patient every three months
Review sexual practices and adherence to PrEP to reassess continuation
of PrEP. Test kidney function and test for HIV and STIs. This is an ideal
chance to attend to preventative health and primary health care needs of
the patient.

CASE COMMENTARY Page 381

Ben has recognised his risks and is taking steps to proactively


reduce them. Asking for PrEP may be difficult and so the doctor
must not judge him.
This is a new area of medicine and you may not know the
details of PrEP prescribing but this will not prevent you taking
an appropriate history and forming a management plan with
Ben. When new medicines become available doctors need to be
able to use appropriate resources in the consultation to guide
patient care. Being able to check drug details while showing
empathy and maintaining patient contact are important skills.
Australian data from 2015 show that men who have sex with
men accounted for 73% of new HIV diagnoses. The Aboriginal
and Torres Strait Islander population is over represented in new
cases of HIV and yet is not getting access to PrEP at the same
level as their non-Indigenous counterparts.
GPs can prescribe PrEP (Truvada) as HIV pre-exposure
prophylaxis to eligible patients. People without Medicare cards
can access PrEP under the TGA personal importation scheme.
PrEP is effective at preventing new HIV infections in those who
take it correctly. San Francisco has seen a 50% drop in new HIV
infections since PrEP was approved by the US Food and Drug
Administration in 2012.
There are concerns that PrEP may change sexual practices
and lead to more STIs. Regular follow-up to prescribe PrEP is
an opportunity to discuss overall health, including preventing
other STIs and use of drugs, for example, methamphetamines in
Ben’s case.

COMMON PITFALLS

In cases discussing new developments in medicine, if you are


not across the developments, you may freeze momentarily.
Remember to return to first principles and you will realise there
is a lot you will be able to do, while sourcing the up-to-date
information.

Further reading
Cornelisse, V 2018, PrEP on the PBS: An opportunity in HIV prevention.
Retrieved from: www.nps.org.au/news/prep-on-the-pbs-an-opportunity-in
-hiv-prevention, accessed 25 February 2019.
Decision making in PrEP Australasian Society for HIV, Viral Page 382
Hepatitis and Sexual Health Medicine 2018. Retrieved from: ww
w.Ashm.Org.Au/Products/Product/3000100092, accessed 25 February
2019.
Ward, J, Hawke, K & Guy, RJ 2018, ‘Priorities for preventing a concentrated
HIV epidemic among Aboriginal and Torres Strait Islander Australians’,
Medical Journal of Australia, vol. 209, no. 1, pp. 5–6.
Wright, E, Grulich, A, Roy, K, Boyd, M, Cornelisse, V, Russell, D,
Zablotska, I 2017, ‘Australasian Society for HIV, Viral Hepatitis and
Sexual Health Medicine HIV pre-exposure prophylaxis: clinical
guidelines’, Journal of Virus Eradication, vol. 3, no. 3, pp. 168–84.
Page 383

Case 76
Vinay Singh

Instructions for the doctor


This is a long case.
This consultation may be conducted as if it is more than one session.
Please take a history from Vinay. Examination findings are available from
the facilitator. When you are ready, outline your diagnostic impressions and
initial management to Vinay. Investigation results will then be available from
the facilitator. Discuss these results with Vinay, together with any further
management, if required.

Scenario
Vinay Singh is a 35-year-old management consultant whom you’ve seen
a few times for minor ailments. His wife and children are regular patients
of yours. His wife rang the surgery this morning to say Vinay is sick
with the flu and has a strange non-itchy rash on his palms and soles.
Upon arrival Vinay is put in a spare consulting room for quarantine
purposes. His wife and children are in the waiting room.

The following information is on his summary sheet:


Past medical history
Nil significant
Medication
Nil
Allergies
Nil
Immunisations
None recorded
Social history
Married, two children (aged four and six), management consultant.

Instructions for the patient, Vinay Singh Page 384

A week ago you started feeling unwell—joint pains, fatigue, malaise—as


though you were getting the flu. You weren’t particularly worried until a rash
developed on your palms and soles. It is not itchy or painful. You looked on
the internet and self-diagnosed hand, foot and mouth disease but the rash
does not look like the picture on the internet and has now started spreading to
your chest. Having returned from a business trip to the northern United States
a month ago, you were also wondering about Lyme disease.

On specific questioning
You travel overseas about five times per year, for one to two weeks, to either
the United States or South-East Asia for work. Your family does not
accompany you on these trips. Not uncommonly on these overseas trips
you’ll have sex with male and/or female sex workers (vaginal, oral and anal
—receptive and penetrative). You use condoms ‘most’ of the time. You have
never had a sexually transmitted infection (STI) screening or any previous
STI (to your knowledge).
You see these trips as well-deserved opportunities to let your hair down
and have some fun. You compartmentalise your life: you see yourself as a
‘different person’ when working overseas and believe that your trips away
enhance your ability to be a good husband and father, as they allow you to
‘blow off steam’. You categorically see yourself as a heterosexual loving
father and husband. You deny any mood problems or guilty feelings—while
life has its ups and downs, you see yourself as a relatively happy and
confident person.
You were born in India and have lived in Australia since the age of 10.
You are not religious and don’t have any particularly strong cultural or
extended family ties. You have not been back to India for more than 10 years.
You did not visit any wooded/rural areas during your recent trip to the
United States and have no particular reason for thinking you have Lyme
disease except that you have heard that it is ‘common over there’.
You do not smoke and rarely drink alcohol, as it tends to give you a
headache and to make you feel sleepy. If asked about other drugs, you pause,
look uncomfortable and say ‘not really’. If asked to expand in a sensitive and
non-judgemental way, you admit to using methamphetamine (‘meth’/‘ice’) as
a party drug when overseas for work. You also use it to help you to keep
awake and alert when jetlagged (e.g. before a big meeting). You have used it
in tablet form but prefer ‘snorting’ it (intranasally). You’ve never used IV
drugs. You used marijuana a few times in college but not since then. You do
not consider yourself an addict and you’re not interested in giving it Page 385
up. You’ve never used drugs in front of your wife or kids (they are
unaware of your drug use). You purchase drugs from local dealers (usually in
nightclubs). You never carry drugs while travelling and you’ve never been in
trouble with the law. You have had some dental decay requiring crowns and
veneers to be fitted, but otherwise no adverse effects from the meth use.
About three months ago (two to three weeks after returning from a trip to
Thailand), you noticed a painless ulcer around your anus. It healed in about
three weeks. You have been otherwise well until last week. You have noticed
some new, painful lumps around your anus.
Suggested prompt: ‘Is there any chance this could be something I got
overseas?’
When the diagnosis is given
You are devastated by the syphilis diagnosis and initially disbelieving:
‘Could there be some mistake?’
You thought syphilis was an old disease that had been eradicated, like
polio. You thought you may have been vaccinated against it in the past. You
know nothing else about it, other than it can ‘send you crazy’.
You are initially resistant to the idea of informing your wife, but you agree
that if the reasons (including the health risks to your wife) are explained
sensitively and with empathy you will sit down and talk with her this
evening. You would prefer to talk to her alone initially, rather than her
finding out another way. If suggested, you are willing to return for a follow-
up visit with your wife and to follow an appropriate management plan.
Suggested prompts:
• ‘What should I tell my wife?’
• ‘Do I need tests to see if it’s cured?’
• ‘You did say that this consultation was confidential, didn’t you?’

The following information is on your summary sheet:


Past medical history
Nil significant
Medication
Nil
Allergies
Nil
Immunisations
None recorded
Social history
Married, two children (aged four and six), management consultant.

Information for the facilitator Page 386

Clinical examination findings


Each finding needs to be asked for specifically.
General appearance—looks mildly unwell
Temperature 37.8°C
BP 120/75 mmHg
Pulse 80 and regular
Pale red, discrete, round, symmetrically bilateral macules on palms, soles and
trunk; lesions measure 5–10 mm in diameter
No oral lesions
No alopecia
Anogenital area—reddish-brown tender papular lesions coalescing into
plaques in places
Generalised non-tender lymphadenopathy (cervical, axillary and inguinal)
Rest of examination is unremarkable.

Investigation results
FBE within normal limits
Urea and electrolytes within normal limits
LFTs within normal limits
First pass urine PCR—negative for Chlamydia and gonorrhoea
Oral and anal swabs PCR—negative for Chlamydia and gonorrhoea
Swabs of lesions on palms/soles PCR—negative for enteroviruses
Anogenital lesions, wet mount preparation of expressed exudate—dark field
microscopy reveals syphilitic spirochaetes
Syphilis serology—positive ELISA, followed by positive results on
confirmatory testing
Hepatitis A, B and C serology negative, non-immune
HIV Ab negative
Lyme disease serology—result pending (three-to-four week turnaround)
Chest X-ray normal.

Suggested approach to the case


Establish rapport
Empathic, open and non-judgemental approach
Thorough history of presenting complaint, past history, social history and
systems review.

Specific questions Page 387

Sexual history should include partners (number and gender), sexual practices,
protection from STIs, previous STIs
Drug history should include drug type/s, frequency of use, route of
administration and readiness to modify behaviour
Relationship with wife/family, beliefs and self/cultural identity, mood.

Examination
Ask the facilitator for specific examination findings as per above list
Must ask for description of skin lesions
Must ask for examination of anogenital area.

Investigations
As per above list
Key investigations include
— First pass urine PCR
— Oral and anal swabs: PCR and wet mount preparation
— Syphilis/Hepatitis B/HIV serology.

Management
Sensitively break news of secondary syphilis infection
Patient education about syphilis, including explaining cause, transmission,
stages (primary, secondary and tertiary) and correcting any
misconceptions
Treat infection
— Benzathine penicillin G 1.8 g IM single dose OR
— Procaine penicillin G 1.0 g IM daily for 10 days OR
— Doxycycline 100 mg twice daily for 14 days (if allergic to penicillin)
Vaccinate against hepatitis A and B
Talk to sexual health unit/clinic
Tracing of all sexual partners, if possible
Wife needs to be informed and tested (encourage Vinay to inform her, but
offer assistance: yourself and/or contact tracing service)
Refrain from sexual activity until seven days after treatment of both partners
is complete
Safe sex messages
Follow-up—regular STI screening
Follow-up—clinical and lab testing (syphilis serology ) at 3, 6 and 12 Page 388
months
Harm reduction regarding methamphetamine use. Aim to move from
precontemplation to contemplation.

CASE COMMENTARY

This case assesses a candidate’s ability to suspect, identify and


manage sexually transmitted infections and ‘party drug’ use. A
detailed and sensitively taken sexual and drug-taking history is
crucial. The approach should be open and non-judgemental, and
questions should be normalised/depersonalised whenever
possible, for example: ‘There are some personal questions I ask
all patients with symptoms like these to help lead us to a
diagnosis. Some of them might be quite confronting but I would
appreciate any information you’re willing to share with me.’
Sensitively and successfully negotiating the issues of
confidentiality with the patient (balancing doctor–patient
confidentiality with the need for contact tracing to reduce harm
to others) is another key feature of this case.
People taking ‘party drugs’ at social functions may not be
ready to change their behaviour. The best outcome in such cases
is likely to be achieved by using a non-judgemental approach
and focusing on informing patients on how to reduce harm and
risk when using substances. During this consultation, Vinay
realises that, while he compartmentalises his life, infections and
illness transcend the compartments he has established. Not only
is his health threatened, but so is his marriage and his wife’s
health; skill is needed to help him to use this realisation to
motivate change.

COMMON PITFALLS

Many patients prefer to see their GP for sexually acquired


infections rather than to attend sexual health clinics. However,
sexual health is an aspect of clinical practice where a GP’s
background and personal values can significantly affect their
professional effectiveness. It is easy to jump to erroneous
conclusions regarding sexuality, belief systems and/or cultural
identity based on a patient’s marital status, socioeconomic status
and racial background. GPs who are uncomfortable asking about
sexual or drug practices risk missing important clues in the
history. Practising asking open and specific questions and
learning to respond to the answers is time well spent.

Further reading Page 389

Australasian Sexual Health Alliance 2018, ‘Australasian STI management


guidelines for use in primary care; Syphilis’. Retrieved from http://www.s
ti.guidelines.org.au/sexually-transmissible-infections/syphilis.
Cornelisse, V, Fairley, CK & Roth, NJ 2016, ‘Optimising healthcare for men
who have sex with men: A role for general practitioners’, Australian
Family Physician, vol. 45, no. 4, pp. 182–5.
Frei, M 2010, ‘Party drugs: use and harm reduction’, Australian Family
Physician, vol. 39, no. 8, pp. 548–52.
Hopwood, M, Cama, E & Treloar, C 2016, Methamphetamine use among
men who have sex with men in Australia: A literature review. Sydney:
Centre for Social Research in Health, UNSW Australia. http://doi.org/10.
4225/53/58d4418d0a14b.
Ong, JJ & Towns, JM 2017, ‘Occult syphilitic chancres in the rectum and
oropharynx’, Australian Family Physician, vol. 46, no. 9, pp. 673–5.
Page 390

Case 77
Samantha Heyward

Instructions for the doctor


This is a short case.
Please take a history from your regular patient, Samantha Heyward. The
facilitator will give you the results of any relevant physical examination on
request. Then outline a management plan with Samantha.

Scenario
Samantha Heyward is 21 years old and her family have been your
patients for many years. She has mostly enjoyed good health and has
been on the oral contraceptive pill for about five years, initially for
irregular periods and then for contraception.
She has also had some depression, which got quite severe during her
final year of school and she has been on sertraline since then. She has
tried a couple of times to stop taking it, but within a month or so her
mood drops and she recommences. Sam has a close relationship with her
parents and has two older brothers who are both married with children
but live nearby. You see Sam every several months and at her last
appointment she was doing well. She told you she had a new boyfriend,
Glen, and was enjoying her work as a clerk in the courthouse.

The following information is on her summary sheet:


Age
21
Past medical history
Depression
Medication
Sertraline 100 mg od
Ethinylestradiol 30 mcg/levonorgestrel 150 mcg (Microgynon 30) one
daily
Allergies Page 391
Nil known
Immunisations
Up-to-date
Social history
Non-smoker Non-drinker.

Instructions for the patient, Samantha


Heyward
Your name is Samantha (Sam) Heyward and you are a 21-year-old long-term
patient of this doctor. You’ve had some trouble with depression in the past
but have generally been healthy. You have come to the doctor because you
are confused. You are having problems with your partner, Glen, and are not
sure which one of you is normal and who is ‘to blame’. Your opening line
can be something like, ‘Glen and I are having problems and I don’t know if
there’s something wrong with me or him’.
Let the candidate ask questions to draw the story out. You are initially
somewhat embarrassed and reticent but respond to the doctor’s approach. If
the doctor seems embarrassed, you can become more shy and awkward.
Alternatively, if the questions are being asked skilfully in a relaxed manner,
you can also relax and open up more.
You’ve been living with Glen for about nine months now and have been
together for a little over a year. He seems really nice but you’re having
problems in the ‘bedroom department’ and this is starting to affect your
relationship. He wants to have sex a couple of times a day and perpetually
seems excited and ready. At first you didn’t mind because you thought it
would settle down with time, but it hasn’t.
As far as you are concerned, sex is enjoyable and you feel you have a
healthy interest and libido, but most days you’d rather read a book or watch
TV than have intercourse again. You’ve had some talks with Glen about the
problem and as a compromise you have sex once a day. Neither of you are
very happy about this. Glen says you have something wrong with your libido,
and you’ve had a brief look at some women’s magazines (like Cleo and
Cosmo), which do seem to suggest you might have a problem. Women that
you read about in those magazines (and see on TV and in movies) all seem to
have incredible libidos, and this seems a long way from your own experience.
You talked with your Mum about it and she seemed embarrassed, saying
dismissively, ‘Don’t worry about those magazines—all the women in our
family have very low libidos’. You’re wondering if it is as simple as that.
If asked: Page 392
You had three partners before Glen, each of them monogamous at the
time and each lasting three to six months. Your first partner was when you
were 17 (he was 18).
You’ve never been sexually abused or pressured into having sex. You’ve
never been tested for STIs. You don’t feel scared of Glen or pressured, you
just feel you would like to satisfy his desires.
You used condoms in your previous relationships but after a few months
you and Glen decided it would be OK not to use them because you were on
the pill.
You have no concerns about Glen’s faithfulness, but do worry that if you
had sex less often he might start looking elsewhere.
You do get aroused when having sex but have never climaxed. This
doesn’t really bother you—sex is enjoyable and you’re not bothered about a
lack of orgasm.
Sometimes lubrication seems to be an issue but you think this is probably
normal and is just because he is always ready to go and you don’t have time
to ‘warm-up’.
If asked more detail about your sexual practices, you can be a bit reluctant
and ask if that is really necessary but if the doctor pushes for details you can
say that it is vaginal and oral sex.
You’ve never been pregnant and are not keen to be pregnant at the
moment. You have good cycle control on your contraceptive pill and if it is
suggested you stop it you are quite anxious about pregnancy and irregular
periods. You suspect Glen would not be keen to have to go back to using
condoms.
If ceasing your antidepressant is raised, you are very reluctant because you
have tried to get off it a few times and each time your mood plummets and
your anxiety skyrockets.
You’ve been on both the oral contraceptive pill and the antidepressant
since before you were sexually active and hadn’t considered that either could
be affecting your interest or performance negatively.
Your mood is good, your appetite, energy and sleep are fine and you are
enjoying your work at the courthouse.

The following information is on your summary sheet:


Age
21
Past medical history
Depression
Medication Page 393
Sertraline 100 mg od
Ethinylestradiol 30 mcg/levonorgestrel 150 mcg (Microgynon 30) one
daily
Allergies
Nil known
Immunisations
Up-to-date
Social history
Non-smoker
Non-drinker.
Information for the facilitator
Clinical examination findings
General appearance: Young adult female in no distress
BMI 21 kg/m2
BSL 5 (random) mmol/L
Urinalysis normal
All physical examination findings are normal.

Suggested approach to the case


Establish rapport
Open questions to elicit Sam’s concerns.

Specific questions
Explore libido mismatch
Explore the relationship
Exclude abuse
Explore arousal, orgasm and dyspareunia
Explore general health and medications.

Examination
General appearance BMI
BSL
Urinalysis.

Management Page 394

Explanation of libido mismatch


Reassurance
Explanation that anorgasmia likely to be related to SSRI use
Resources and simple suggestions regarding libido mismatch
Offer to see Sam and her partner together
Follow-up.

CASE COMMENTARY

Sex is an important part of human experience, but often patients


(like Sam in this case) can find themselves having problems and
feeling that they have no one to help them. A GP does not have
to be a trained sex therapist, but the basic tools of taking a
sexual history, giving appropriate reassurance, offering simple,
common sense advice and exploring this vital but sensitive area
should be in the GP’s armament. Female sexual issues include
painful intercourse, lack of libido (up to four in ten women) and
anorgasmia (up to one in three women). Sam should be
reassured that there is nothing ‘wrong’ with her but that as a
couple she and her partner are experiencing libido mismatch.
Libido mismatch (also referred to as desire discrepancy) is the
most common reason for a person to seek out a sex counsellor.
Both her SSRI and the OCP can dampen libido, but a careful
history should reveal that Sam’s libido is healthy and she should
be reassured about this. Libido is typically classified as high or
low, but it is actually much ‘more complex than how often a
person desires sex. It has elements including what triggers
arousal and what dampens it, the importance of sex compared to
other parts of a relationship, the meaning of sex for each
individual, what is pleasurable during sexual activity’. When a
couple has mismatched libidos, a ‘pursuer–distancer’ cycle can
develop that will threaten the relationship if it continues.
Australia’s Bettina Arndt is a prolific author and researcher
on attitudes towards sex and sexuality. She has followed a large
number of Australian couples and encouraged them to document
their thoughts and experiences regarding the sexual aspect of
their relationship. She provides in-depth understanding into the
interpersonal dynamics of desire discrepancy, as well as some
excellent practical advice for couples trying to manage it.
Having identified the issue, the GP can give patients education
and resources about the condition, as well as some Page 395
management principles, and should offer to counsel the
couple together in order to facilitate communication.
As an added issue, Sam describes primary anorgasmia, an
adverse effect of the SSRI. It should be noted that this is not
why she has presented and is not a problem for her. She is
anxious about ceasing her sertraline but a watchful trial of a
lower dose might be discussed.

COMMON PITFALLS

GPs can be uncomfortable taking a sexual history and may skip


important issues as a result. This will lead to an inadequate
understanding of the situation, which in turn will mean less
helpful management suggestions. Diagnostic algorithms exist for
primary anorgasmia, and it is easy to progress through these and
forget that this is not a problem for Sam. Her instructions are to
respond to the doctor’s manner: relaxing and being more
forthcoming if the doctor seems relaxed and skilful in their
questioning and vice versa if they are not. This is realistic, but
one of the disadvantages of exam anxiety is that it’s hard to
appear relaxed. Practice can definitely help here.

Further reading
Arndt, B 2009, The Sex Diaries, Melbourne University Press, Melbourne.
Goodwach, R 2017, ‘Let’s talk about sex’, Australian Family Physician, vol.
46, no. 1–2, pp. 14–18
Phillips, N 2000, ‘Female sexual dysfunction: evaluation and treatment’,
American Family Physician, no. 62, no. 1, pp. 127–36. Page 396
Page 397
Section 22
Travel health
Page 398

Case 78
Tanya Hardy

Instructions for the doctor


This is a long case.
Please take a history from Tanya Hardy, who is a 23-year-old patient of
your practice. Conduct any relevant physical examination and discuss an
appropriate management plan with Tanya.

Scenario
Tanya is a nursing student who has generally been very healthy but has
presented in the past for immunisations, an episode of bronchitis and
once for emergency contraception.

The following information is on her summary sheet:


Past medical history
Lower respiratory tract infection
Cervical screen
Normal this year
Medication
Ethinyloestradiol 30 mcg/levonorgestrel 150 mcg (Microgynon 30)
Allergies
Nil known
Social history
Nursing student
Single
Smokes 10–15 cigarettes per day
Immunisations
Up-to-date.

Instructions for the patient, Tanya Hardy Page 399

You are coming to the doctor today because you and a friend are planning a
backpacking holiday after you both graduate. In two months you will go to
Thailand, Laos, Vietnam and then party in the Thai islands before going to
Malaysia and finishing in Singapore.
You have read about a backpacker track where you can get food and
accommodation for a couple of dollars a day and alcohol is ‘super-cheap’. If
asked, you are hoping to do some jungle trekking, see wild monkeys, ride an
elephant, learn to ride a scooter, try scuba diving and do lots of partying.
You have never travelled overseas before and aren’t big on planning.
Nothing is booked apart from your flights. You’re very excited about the trip
but haven’t thought much about any health issues. You are receptive to issues
the doctor raises and listen attentively to their advice, asking the sort of
practical questions you think the real Tanya would ask.
Your health is good, and your only regular medication is your
contraceptive pill. Your family are all healthy but live interstate.
If asked, you are not currently sexually active, having broken up with your
boyfriend six months ago. You’ve heard men in Asia are very friendly and
fun-loving so you’re looking forward to making new friends on your trip.
You have used marijuana in the past and ecstasy on occasion when you’ve
gone dancing. You’ve heard about the magic mushroom shakes you can get
in Asia and can’t wait to try them.
You had various immunisations three years ago when you started nursing.
You’re not exactly sure which ones, but you think they included hepatitis B,
pertussis and measles/mumps/rubella. You currently smoke about 10 to 15
cigarettes a day and have done so since you started nursing. You plan to give
up one day, but certainly not on your big holiday. You don’t drink alcohol
every day but about once a week you go out with your friends and drink up to
10 to 12 drinks. This doesn’t really bother you, apart from the hangovers,
which you plan to sleep through on your holiday. If explored, a couple of
times in the past you have had too much to drink and ended up sleeping with
guys you don’t know very well. About half of the times this has happened
you have not remembered to use condoms. You are forthcoming about all this
if asked, but will clam up if you feel the doctor is lecturing you.

The following information is on your summary sheet:


Past medical history
Lower respiratory tract infection
Cervical screen
Normal this year
Medication Page 400
Ethinyloestradiol 30 mcg/levonorgestrel 150 mcg (Microgynon
30)
Allergies
Nil known
Social history
Nursing student
Single
Smokes 10–15 cigarettes per day
Immunisations
Up-to-date.

Information for the facilitator


Clinical examination findings
General appearance: healthy-looking female
BMI 21 kg/m2
BP 121/78 mmHg
Physical examination is normal.

Suggested approach to the case


Establish rapport
Open questions to elicit the nature of her trip.

Specific questions1
Previous travel history Specific travel questions
— Intended activities
— Precise itinerary
— Style and mode of travel
— Type of accommodation
— Time of year
— Length of stay
— Any travel/health insurance
Drug (including recreational drugs) and alcohol history
Smoking history
Previous vaccinations
General health and systems review
Request permission to examine.

Examination Page 401

General appearance
BMI
Vital signs.

Management
Food and water advice—should be practical and specific
Accidents and safety—alcohol and road safety, wearing helmets,
hiking/jungle safety
Safe sex/contraception
Malaria/dengue prevention, zika advice
Rabies prevention
Drug awareness
Travel insurance
Simple medical/first aid kit
Vaccinations2
— Typhoid
— Hepatitis A +/− Salmonella
— Japanese encephalitis
— Cholera
— +/− Rabies
Advise her to seek medical attention if she becomes unwell within the first
few weeks of her return to Australia
Provide written information as well.

CASE COMMENTARY

People are travelling more and, while travel offers wonderful


new experiences, it comes with health risks, which GPs should
be equipped to manage. The main challenge in this case will be
managing your time as you work your way through a long
agenda of relevant issues for Tanya, a healthy but naïve
traveller. It is good to begin with open questions exploring the
nature of the intended trip, followed by a checklist of specific
questions to further define the key issues.
The backpacker circuit through South-East Asia is well worn
and often appeals to the people seeking cheap food,
accommodation, alcohol and adventures. The associated health
problems include infections transmitted through contaminated
food and water and insect bites, as well as sexually Page 402
transmitted infections and motor vehicle accidents,
either as driver or pedestrian.
Food and water hygiene advice should be delivered in an
efficient and practical way, indicative of lots of previous
experience. Supplement the health advice with written material
to improve recall and compliance. In exams, reference can be
made to these as if they have been provided.
The risk of cholera infection is low, but cholera vaccination
does afford some partial short-term protection against travellers’
diarrhoea caused by enterotoxigenic Escherichia coli (ETEC)
because cholera and ETEC share the same toxin. The combined
Hepatitis A/Salmonella Typhi vaccination has similar benefits.
Tanya’s attitude to alcohol and other drugs should raise
alarms and be addressed sensitively but clearly.
Rabies prevention includes discussion about the risk of the
disease from bites or scratches from infected animals such as
dogs, monkeys, bats and rodents. Tanya’s itinerary is reasonably
high risk and she is spending considerable time in rabies-
endemic areas, some of which are isolated regions where access
to appropriate medical care, such as effective and timely post-
exposure prophylaxis is limited, so vaccination is recommended.
Tanya risks catching malaria on her trip, and a discussion
about prevention should include the pros, cons and costs of
chemoprophylaxis alternatives (probably doxycycline or
mefloquine), as well as mosquito bite prevention measures.

COMMON PITFALLS

Often travel advice focuses on recommended vaccinations but it


includes much more. A common error is to ask only about the
itinerary, but a few additional questions allow us to accurately
assess risk. We also shouldn’t miss the opportunity of preventive
health care by exploring Tanya’s attitude to her smoking,
episodic binge drinking and risky sexual practice. To be
thorough, post-travel illness presentation and management
should be addressed as well.

References
1. Chen, LH, Hochberg, NS & Magill, AJ 2015, ‘The pre-travel
consultation’, Centers for Disease Control and Prevention, Atlanta, GA.
2. Henderson, J, Harrison, C, Bayram, C & Britt, H 2015, ‘Travel advice and
vaccination’, Australian Family Physician, vol. 44, no. 1–2, pp. Page 403
14–15.

Further reading
Batchelor, T & Gherardin, T 2007, ‘Prevention of malaria in travellers’,
Australian Family Physician, vol. 36, no. 5, pp. 316–20.
Centers for Disease Control and Prevention 2017, CDC Yellow Book 2018:
Health Information for International Travel, Oxford University Press,
New York.
Leder, K 2015 ‘Advising travellers about management of travellers’
diarrhea’, Australian Family Physician. Vol. 44, no. 9, pp. 34–37.
Neilson, A & Mayer, C 2010, ‘Cholera—recommendations for prevention in
travellers’, Australian Family Physician, vol. 39, no. 4, pp. 222–5.
Neilson, A & Mayer, C 2010, ‘Hepatitis A: prevention in travellers’,
Australian Family Physician, vol. 39, no. 12, pp. 942–8.
Neilson, A & Mayer, C 2010, ‘Rabies: prevention in travellers’, Australian
Family Physician, vol. 39, no. 9, pp. 641–5.
Page 404

Case 79
Betty Ward

Instructions for the doctor


This is a short case.
Please take a focused history from Betty and manage the issues that arise.

Scenario
During her routine check-up last week, 68-year-old Betty Ward
mentioned that she had just booked to go on an overseas holiday. Today
she has returned with her planned itinerary.

Last week you noted:


Feeling well, no new issues
BP 110/70
Pulse 95 bpm irregularly irregular
CVS—heart sounds dual, nil added, JVP three cm above sternum, no
swelling of ankles
Respiratory—respiratory rate: 26 breaths/min, sats 94% on room air, no
dyspnoea at rest, decreased breath sounds globally on auscultation (long-
standing)
Spirometry—FEV1 54% predicted, no significant reversibility (FEV1 58%
six months earlier)1
FBE, LFTs and electrolytes all in normal range
INR 2.7.

The following information is on her summary sheet:


Past medical history
Atrial fibrillation (non-valvular)
COPD
Hypertension
Medication Page 405
Warfarin (dose variable, currently 3 mg OD)
Perindopril 4 mg
Salbutamol 100 mcg 2 puffs prn
Fluticasone propionate/salmeterol xinafoate 250/50 mcg 1 puff bd
Tiotropium bromide 18 mcg one daily
Allergies
Nil
Immunisations
ADT, influenza and pneumococcal immunisations up-to-date
Social history
Retired school teacher, lives alone
Widowed 18 months ago, no children
Ex-smoker (40 pack/year history; quit three years ago)
No alcohol.

Instructions for the patient, Betty Ward


Last week you booked a last-minute non-refundable special deal to go on a
three-week trip to Europe (Germany, France, Switzerland, Italy), leaving in
four weeks’ time. You don’t have a travel companion but will be joining a
tour group of older travellers the day after you land in Europe. It has been a
lifetime dream to ‘see Europe’ but your husband didn’t like to travel, and
you’ve never been overseas before.

On specific questioning:
Your travel agent reassured you that the pace of the tour will be ‘sedate’ and
that they ‘will take good care of you’. You have not yet investigated travel
insurance options. You live independently but can walk only 30 to 40 metres
without ‘stopping to catch a breath’. You have been on warfarin for four
years and have monthly INR tests. Your INR results have been stable and
you have not required any dose adjustment for more than six months.
You are happy to see a respiratory physician if this is recommended, as
long as it can be done before your trip.

The following information is on your summary sheet:


Past medical history
Atrial fibrillation (non-valvular)
COPD
Hypertension
Medication Page 406
Warfarin (dose variable, currently 3 mg OD)
Perindopril 4 mg
Salbutamol 100 mcg 2 puffs prn
Fluticasone propionate/salmeterol xinafoate 250/50 mcg 1 puff bd
Tiotropium bromide 18 mcg one daily
Allergies
Nil
Immunisations
ADT, influenza and pneumococcal immunisations up-to-date
Social history
Retired school teacher, lives alone
Widowed 18 months ago, no children
Ex-smoker (40 pack/year history; quit three years ago)
No alcohol.
Suggested approach to the case
Develop rapport.

Specific questions2
Previous travel experience
Itinerary (places/activities/length of stay)
Method of travel (flights/rail/bus/vehicle; independent/tour)
Travel companion? Support available?
Travel insurance? Health insurance?
Current exercise tolerance
Immunisation status.

Management3
Medical and medication summary
Letter from GP stating medication is for personal use
Carry list of emergency contacts
Consider medical alert bracelet
Find travel companion, if possible
Advice about importance of travel insurance (warn her that she may have
difficulty finding insurance to cover pre-existing conditions)
Advice about carrying medication (e.g. in original packaging, spare supplies
split between cabin and checked luggage)
Discuss mobility needs with tour group company and airline Page 407
Send for specialist assessment/get specialist advice about need for
inflight supplemental oxygen (Clearance to Fly certificate)4
Test INR just before departure or consider switching to new orally active
anticoagulants5, 6
Warn about INR variations due to unfamiliar diet etc. and possible need for
testing while overseas (provide letter)
Advice about travel medical kit
General safety precautions (e.g. motor vehicle, theft)
Discuss impact of jet lag (tends to be more severe in older travellers; consider
scheduling ‘recovery’ days/stopover if possible).
Register with SmartTraveller website.3

CASE COMMENTARY

With cheaper flights and increased numbers of older Australians


wanting to travel, GPs are commonly in the position of needing
to give travel advice to those with chronic medical conditions. It
is important for GPs to have a good working knowledge of the
special health issues that can arise during travel and how best to
manage these.2
This case is aimed at determining the candidate’s ability to
take a focused pre-travel history, identify risk factors in this
patient (age, travelling alone, COPD and warfarin therapy) and
provide both general and specific travel advice.
For those with chronic lung disease, the effects of decreased
partial pressure of oxygen in an aircraft cabin and, if applicable,
destination altitude (especially if travelling to Nepal, Peru, Tibet
or Bolivia) need to be considered.1 Aircraft cabins are generally
pressurised at 1524–2438 m, which results in a concomitant fall
in oxygen saturation of around 5%. This is generally well
tolerated in those with mild respiratory disease but for those
with dyspnoea or angina at rest, with hypoxaemia (resting
saturation <95%) and/or those who cannot walk 50 m or 15 steps
without breathlessness, such a drop in oxygen saturation may
significantly compromise their function.4 In these patients,
inflight supplemental oxygen should be considered, preferably
following assessment by a respiratory physician. High-altitude
simulation tests (HASTs) can be performed in respiratory
laboratories to determine the need for oxygen during flight. It is
important to discuss the need for comprehensive travel insurance
in detail with patients, highlighting the possible costs if medical
care and/or medical repatriation is necessary while overseas.
References Page 408

1. Yang, IA, Brown, JL, George, J, Jenkins, S, McDonald, CF, McDonald,


V, Smith, B et al. 2018, The COPD-X Plan: Australian and New Zealand
Guidelines for the management of Chronic Obstructive Pulmonary
Disease Version 2.56.
2. Chen LH, Hochberg NS & Magill AJ 2015, ‘The pre-travel consultation’,
Centers for Disease Control and Prevention, Atlanta, GA.
3. Australian Government, Smart Traveller. Available at: www.smartraveller
.gov.au/.
4. Lung Foundation Australia 2016, Fitness to Fly Factsheet.
5. Brieger, D & Curnow, J 2014, ‘Anticoagulation: a GP primer on the new
oral anticoagulants’, Australian Family Physician, vol. 43, no. 5, pp. 254–
9.
6. NPS MedicineWise, NOAC indications and PBS listings. Available at:
www.nps.-org.au/noac-indications-and-pbs-listings, accessed 1 December
2018.

Further reading
Lim, M, Brazzale, D & McDonald, C 2012, ‘“Is it okay for me to. . .?”
Assessment of recreational activity risk in patients with chronic lung
conditions’, Australian Family Physician, vol. 41, no. 11, pp. 852–4.
Henderson, J, Harrison, C, Bayram, C & Britt, H 2015, ‘Travel advice and
vaccination’, Australian Family Physician, vol. 44, no. 1–2, pp. 14–15.
Page 409
Section 23
Women’s health
Page 410

Case 80
Jenny Butterfield

Instructions for the doctor


This is a short case.
Please discuss with Jenny what you think may be causing the problem and
how you plan to manage this situation.

Scenario
Jenny Butterfield is a 35-year-old woman who has heavy periods. Her
periods are regular but last up to 10 days. For the first three to four days
she is leaking despite wearing both tampons and sanitary pads. She has
to change protection every half an hour and takes time off work during
her period because of this. The periods are not painful. She has no other
gynaecological symptoms and she does not pass clots. She is separated
from her husband.
The following information is on her medical record:
Past medical history
Two normal vaginal deliveries
Medication
Nil known
Allergies
Nil
Immunisations
Up-to-date
Pap smear
Normal this year
Family history
Mother died of pancreatic cancer, aged 53
Social history
Separated
Two children Page 411
Non-smoker
Clinical examination findings
Normal abdominal and pelvic examination.

Instructions for the patient, Jenny Butterfield


You are 35 years old and work as a receptionist. It is not easy to leave the
reception desk to go to the toilet. Your last three periods have come on time
but have been very heavy. The periods last up to 10 days. For the first three
to four days you leak despite wearing both tampons and sanitary pads. You
have to change protection every half an hour and because of this take time-off
work during your periods. The periods are not painful. You have no other
gynaecological symptoms and do not pass clots.
You are separated from your husband. You will soon run out of sick leave
and this worries you as you are bringing up two teenage children on your
own.
If specifically asked
— You sometimes have a heavy dragging sensation in your lower
abdomen
— You have felt much more tired than normal. The other day you’d
forgotten to put out the bins and made a mad dash in your pyjamas,
your heart was pounding out of its chest, you were short of breath and
nearly fainted. You assumed you’ve just become really unfit. You are
reluctant to attend your normal gym classes because of the flooding
— Your weight is stable and you don’t particularly feel the heat or the
cold
— Your bowel habit has not changed.
Please make sure you ask the doctor to outline your options for treatment.

The following information is on your medical record:


Past medical history
Two normal vaginal deliveries
Medication
Nil known
Allergies
Nil
Immunisations
Up-to-date
Pap smear
Normal this year
Family history Page 412
Mother died of pancreatic cancer, aged 53
Social history
Separated
Two children
Non-smoker
Clinical examination findings
Normal abdominal and pelvic examination.

Suggested approach to the case


Establish rapport
Open questions to explore Jenny’s ideas, concerns and expectations
Explain menorrhagia—probably idiopathic
Reassure Jenny that symptoms can be improved.

Investigations
FBC and iron studies
Consider clotting factors and thyroid function tests.

Treatment options
Levonorgestrel releasing intrauterine device (Mirena)
NSAIDs—check no contraindication Antifibrinolytic such as tranexamic acid
Oral contraceptive pill—if no other risk factors
Ultrasound scan and endometrial sampling not needed initially
Consider referral for danazol, endometrial resection or ablation or
hysterectomy if medical treatment is not effective
Offer to discuss with employer if Jenny wishes
Arrange follow-up.

CASE COMMENTARY

This case tests the doctor’s ability to manage a common clinical


problem, to recognise the impact of this problem on the patient’s
life and to negotiate management with the patient.
The regularity of the periods, and the absence of pain or any
abnormalities on clinical examination make it most likely that
this menorrhagia is idiopathic. A full blood count and Page 413
iron studies should be done to exclude anaemia. Further
tests such as an ultrasound scan and endometrial sampling are
indicated if initial medical therapy is ineffective.
There is a range of effective options for menorrhagia with the
levonorgestrel releasing intrauterine system (Mirena) now
recommended as first-line therapy in primary care. The doctor
should outline the options, avoiding medical jargon, and listen to
Jenny’s preferences.

Further reading
Australian Commission on Safety and Quality in Health Care 2017, Heavy
Menstrual Bleeding Clinical Care Standard, Sydney, ACSQHC.
Bano, R, Datta, S & Mahmood, TA 2016, ‘Heavy menstrual bleeding’,
Obstetrics, Gynaecology and Reproductive Medicine, vol. 26, no. 6, pp.
167–74.
Gupta, J, Kai, J, Middleton, L, Pattison, H, Gray R & Daniels, J et al. 2013,
‘Levonorgestrel intrauterine system versus medical therapy for
menorrhagia’, vol. 368, pp. 128–37.
Lethaby, A, Duckitt, K & Farquhar, C 2013, ‘Non-steroidal anti-
inflammatory drugs for heavy menstrual bleeding’, Cochrane Database of
Systematic Reviews 1:CD000400.
Lethaby, A, Penninx, J, Hickey, M, Garry, R & Marjoribanks, J 2013,
‘Endometrial resection and ablation techniques for heavy menstrual
bleeding’, Cochrane Database of Systematic Reviews 8:CD001501.
Middleton, LJ, Champaneria, R, Daniels, JP, Bhattacharya, S, Cooper, KG,
Hilken, NH et al. 2010, ‘Hysterectomy, endometrial destruction, and
levonorgestrel releasing intrauterine system (Mirena) for heavy menstrual
bleeding: systematic review and meta-analysis of data from individual
patients’, British Medical Journal, vol. 341, p. c3929.
Page 414

Case 81
Vikki Nicolaides

Instructions for the doctor


This is a short case.
Please conduct this consultation as you would in clinical practice.

Scenario
Vikki Nicolaides is a 34-year-old accountant. She and Nico, her
boyfriend, have been together for four years and they are considering
starting a family. She has made this appointment to find out about when
to stop the pill.

The following information is on her summary sheet:


Past medical history
Glandular fever
Mild anxiety—postgraduate exams 2005
Medication
Ethinyloestradiol 30 mcg/levonorgestrel 150 mcg (Microgynon 30)
Allergies
Nil
Immunisations
Nil recorded
Cervical screening
2018 normal
Social history
Parents migrated from Cyprus in 1960s
Non-smoker
Binge drinking at university.

Instructions for the patient, Vikki Page 415

Nicolaides
You are 34 years old and work long hours as an accountant. Over the past
year you have been discussing with your boyfriend the possibility of starting
a family. You want to find out from the GP when to stop taking the pill and
would be happy to have any other advice that you are offered.
You have regular periods on the pill and your BMI is 24 kg/m2.
Your Mum and sister have diabetes. They were both diagnosed in their
30s. Your maternal grandmother is now on dialysis due to her kidney trouble
and also was diagnosed young with diabetes. But they’re all heavyset and you
think you got your Dad’s genes. You’re hoping this won’t be an issue for
you.

The following information is on your medical record:


Past medical history
Glandular fever
Mild anxiety—postgraduate exams 2005
Medication
Ethinyloestradiol 30 mcg/levonorgestrel 150 mcg (Microgynon 30)
Allergies
Nil
Immunisations
Up-to-date
Cervical screening
2018 normal
Social history
Parents migrated from Cyprus in 1960s
Non-smoker
Binge drinking at university.

Suggested approach to the case


Establish rapport
Open questions to explore Vikki’s ideas, concerns and expectations regarding
this consultation
General health
— Diet, check BMI, BP
— Smoking
— Caffeine and alcohol intake—note past history
— Exercise
Briefly review past medical history—include drug use (prescribed, over the
counter and illicit), exclude diabetes, epilepsy, depression
Cervical screening test.

Specific advice Page 416

Tailor advice to the answers given to the questions above, plus:


Diet and toxin avoidance
— Folic acid (500 mcg od) and iodine supplements (150 mcg od)
— Avoid eating liver
— Eat fish twice a week
— Hygiene—fresh food, care with cat litter
— Avoid environmental toxins
Physical activity
— Aim for 30 minutes/day of physical activity to optimise fitness
Smoking and alcohol
— Enquire about substance use and advise regarding dangers of
smoking, alcohol or other drugs during conception or pregnancy
Optimising fertility
— When to stop contraceptive pill—explore understanding of fertile
time
— Decreased fertility after age 35—information needs to be given
sensitively
Immunisations and blood tests
— Check rubella immunity and herpes zoster immunity
— Consider HIV test, hep B/C test, RPR, TFTs, vitamin D
— Consider need for dTpa booster
Chromosomal, genetic abnormalities
— Need to check if any family history of thalassaemia or other inherited
disorders, and offer screening prenatal carrier screening. The table
below from RANZCOG Prenatal screening and diagnosis of
chromosomal and genetic conditions in the foetus in pregnancy gives
you an idea of which conditions to consider based on ethnicity.
— Availability of foetal anomaly screening tests non-invasive prenatal
testing (NIPT), first trimester screening

Cystic Haemoglobinopathies/ Common Spinal Fragile X


fibrosis thalassaemia Ashkenazi muscular syndrome
mutations atrophy 1
European X X
Ashkenazi X X X
Jewish
Asian X X
African X X
Mediterranean X X
1Only women need be offered FXS screening. FXS screening is particularly
important if there is a family history of intellectual disability.
— Consider health insurance cover Page 417
— Early OGTT given family history of diabetes
— Future travel advise against travel to Zika areas pre-conception and
during pregnancy.

CASE COMMENTARY

Vikki attends this consultation just wanting to know about when


to stop the pill. A good doctor will offer her preconception
counselling and look for any modifiable risk factors to optimise
her health prior to falling pregnant. Fertility issues related to her
age also need to be identified.
Vikki’s personal background that her parents migrated from
Cyprus should alert doctors to ask about a family history of
thalassaemia. Screening can be offered by a full blood count and
red cell indices. A doctor would not necessarily remember the
detail of the screening test but should know that it is available. If
thalassaemia is confirmed Vikki’s partner would require testing.
A leaflet summarising prenatal advice can supplement the
information given during the consultation. The doctor needs to
offer the advice such that Vikki wants to comply with it but is
not so scared that she ignores it.
Pre-conception care can prevent a lifetime of problems for
another human being, such as Foetal Alcohol Syndrome or
congenital rubella, and is worth incorporating into routine care
when discussing contraception.

Further reading
Dorney, E & Black, K 2018, ‘Preconception care’, Australian Journal of
General Practice, vol. 47, no. 7, pp. 424–9.
Royal Australian and New Zealand College of Obstetrics and Gynaecology
2019, Prenatal screening for Fetal Genetic or Structural Conditions.
Accessed at: www.ranzcog.edu.au/RANZCOG_SITE/media/RANZCOG-
MEDIA/Women%27s%20Health/Statement%20and%20guidelines/Clinic
al-Obstetrics/Prenatal-Screening-for-Fetal-Genetic-or-Structural-Conditio
ns-(C-Obs-35)-Review-March-2016.pdf?ext=.pdf, accessed 8 June 2019.
Tan, YL & Kidson-Gerber, G 2016, ‘Antenatal haemaglobinopathy screening
in Australia’, Medical Journal of Australia, vol. 204, no. 6, pp. 226–30.
Page 418

Case 82
Shantelle Kickett

Instructions for the doctor


This is a short case.
Please read the following scenario, and then take a focused history from
Shantelle. Ask the facilitator for the results of a relevant physical examination
and surgery tests. Outline the most likely diagnosis and discuss further testing
required, your initial management, and recommendations with Shantelle.

Scenario
Shantelle Kickett is a 31-year-old Noongar woman from Western
Australia. She has moved in with her long-term partner and is planning a
family soon. She is concerned about her fertility as she only gets five
periods a year. A colleague ordered tests and Shantelle has come to you
for the results today.
Pelvic USS
The uterus and both ovaries were clearly identified. No abnormality
demonstrated. No hydronephrosis.
FSH 6 (5–20) mIU/mL
LH 18 (5–20) mIU/mL
FSH/LH ratio 1:3 (elevated)
FAI 12 (normal 7–10)
SHBG 20 (18–114) nmol/L

The following is on her summary sheet: Page 419


Past medical history
Acne
Medications
On Ethinyloestradiol 35 mcg/cyproterone 2 mg (Diane 35) until 12 months
ago
Tretinoin cream 0.05% (Retin-A) nocte
Immunisations
Childhood immunisations as per schedule.
Cervical screening
Last pap negative September 2017
Family history
Mother and maternal grandparents—type 2 diabetes
Sister—gestational diabetes mellitus
Father—high cholesterol
Social history
Lives with de facto partner David (34)
Works as a school teacher
Non-smoker
Alcohol three to fourth drinks one day a week
Not taking any recreational drugs or OTC medications.

Instructions for the patient, Shantelle Kickett


You and David have started to think about the long-term and planning kids.
You stopped the contraceptive pill 12 months ago as you had irregular
periods in your twenties and wanted to check your natural cycle before trying
to fall pregnant. You have never been pregnant and use condoms for
contraception.
Your period tracker shows you have had five periods in the last year. You
are worried because the app suggests you are not ovulating and you need to
see a doctor.
Your periods started at age 13 and were irregular until you started
ethinyloestradiol 35 mcg/cyproterone 2 mg (Diane) for moderately severe
acne when you were 20. You have always struggled with your weight. Your
family are all big people. You thought that stopping the pill might help with
this but you haven’t lost any weight despite your efforts to do so.
You have a good knowledge of nutrition and eat healthy meals most of the
time but admit there is room for improvement.
You are physically active and play tennis twice a week and walk Page 420
around a lot at school.
Your mood is good. You are not lethargic or tired and get a good amount
of sleep, six to eight hours a night, feeling refreshed when you wake. You do
not snore. You have a good libido and no problems in your sex life.
You had some mild acne while on ethinyloestradiol 35 mcg/cyproterone 2
mg (Diane) but it has worsened since stopping. You are using benzyl
peroxide and vitamin A–derived cream and you feel you are well informed
and managing it adequately (you do not wish to discuss this further today).
You used to have dark hairs on your upper lip and chin but laser treatment
has ‘cured’ you of this. You have new dark hairs appearing around your
nipples and belly button which you’ve been plucking out.

The following is on your summary sheet:


Past medical history
Acne
Medications
On Ethinyloestradiol 35 mcg/cyproterone 2 mg (Diane 35) until 12 months
ago
Tretinoin cream 0.05% (Retin-A) nocte
Immunisations
Childhood immunisations as per schedule
Cervical screening
Last pap negative September 2017
Family history
Mother and maternal grandparents—type 2 diabetes
Sister—gestational diabetes mellitus
Father—high cholesterol
Social history
Lives with de facto partner David (34)
Works as a school teacher
Non-smoker
Alcohol three to four drinks one day a week
Not taking any recreational drugs or OTC medications.

Information for the facilitator


Clinical examination findings
Each aspect needs to be asked for specifically:
BP 120/70 mm Hg
Pulse 75 bpm Page 421
Height 175 cm
Weight 87 kg
BMI 28 kg/m2
Waist circumference 87 cm
Striae (old) on abdomen, upper thighs, breasts and upper arms
Central obesity.

Suggested approach to the case


Establish rapport
Elicit her concerns
Consider her need for an interpreter—none in this case
Take history
— Menstrual and reproductive history
— Current contraception; planning to become pregnant?
— History of acne and hirsutism, treatments tried
— How her symptoms are affecting/have affected her life
— Mental health screen for anxiety and depression
— Family history (polycystic ovarian syndrome (PCOS), diabetes,
cardiovascular disease)
— SNAP (Smoking, Nutrition, Alcohol, Physical activity)
— Social history
An excellent candidate may do the following:
• screen for obstructive sleep apnoea—snoring, daytime somnolence
• screen for eating disorder—prevalence in PCOS 21% with less than half
clinically significant eating disorders picked up (Jean Hailes)
• screen for psychosexual dysfunction.

Examination
Height/weight/BMI
Signs of hyperandrogenism (acne, hirstusim, central obesity, striae)
BSL
Urinary bHCG.

Diagnosis
Polycystic ovarian syndrome (PCOS) with oligomenorrhoea—possibly
anovulatory cycles.

Management Page 422

Metabolic screen—oral glucose tolerance test (OGTT) gold standard—


HbA1c appropriate if OGTT difficult/not practical. No place for fasting
insulin
Cardiovascular risk screen—lipids
Advise lifestyle and benefit of 5% weight loss—likely to help with menstrual
irregularity and decrease cardiovascular risk
Vigorous exercise five times a week for 30 minutes (enough to break a sweat
or not be able to sustain normal conversation due to being out of breath)
Commence metformin and consider referral for clomiphene
Advise need to cease topical retinoid prior to stopping contraception
An excellent candidate will consider preconception counselling and
commencement of folic acid and iodine supplement (see Case 81 Vikki
Nicolaides).

CASE COMMENTARY

This case tests candidates’ ability to diagnose polycystic ovarian


syndrome. A good candidate will screen for its metabolic,
cardiovascular and psychosocial effects.
The Rotterdam diagnostic criteria require two of the
following three features:

1. Hyperandrogenism (biochemical or clinical)


2. Irregular anovulatory menstrual cycles
3. Polycystic ovaries on ultrasound.
Polycystic ovarian syndrome is the most common endocrine
disorder in women of reproductive age (12–18% and up to 21%
in Aboriginal and Torres Strait Islander women). Studies
estimate up to 70% of women remain undiagnosed. It is a
complex and heterogeneous condition with significant
reproductive, metabolic and psychological sequalae. Depression
has a prevalence of 28–64% and anxiety a prevalence of 34–
57%.

COMMON PITFALLS
Doctors need to ask about hirsutism because women often self-
treat to hide their abnormal hair growth. A normal ultrasound
result does not exclude PCOS and patients must be off the
contraceptive pill for three months before biochemistry testing
for PCOS.

Further reading Page 423

Ellis, E, Gibson-Helm, M & Boyle, JA 2018, ‘Polycystic ovary syndrome in


Central Australia: Diagnosis and screening of cardiometabolic risk and
emotional wellbeing’, Australian Journal of General Practice, vol. 47,
no. 4, pp. 227–32.
International evidence-based guideline for the assessment and management of
polycystic ovary syndrome 2018, copyright Monash University,
Melbourne, Vic. Retrieved from: https://jeanhailes.org.au/contents/docum
ents/Resources/Tools/PCOS_evidence-based_guideline_for_assessment_
and_management_pcos.pdf, accessed 26 February 2019.
Jean Hailes PCOS Australian Alliance 2016 Indigenous health, ‘Polycystic
Ovary Syndrome’. Retrieved from: https://jeanhailes.org.au/health-a-z/ind
igenous-health/polycystic-ovary-syndrome-pcos, accessed 26 February
2019.
Norman, RJ & Teede, HJ 2018, ‘A new evidence-based guideline for
assessment and management of polycystic ovary syndrome’, Medical
Journal of Australia, vol. 209, no. 7, pp. 299–300.
Therapeutic Guidelines Ltd 2015, ‘Polycystic Ovary Syndrome’. In: eTG
complete (Internet). October, Therapeutic Guidelines Ltd, Melbourne,
Vic.
Page 424

Case 83
Zahra Mohammed Ibrahim

Instructions for the doctor


This is a long case.
Please take a history, ask for examination findings and discuss your
management plan with the patient.

Scenario
Zahra Mohammed Ibrahim is a 36-year-old Somali refugee who arrived
in Australia 18 months ago. She presents with a two-month history of
missed periods and is asking for a pregnancy test. Your practice nurse
has completed a urine pregnancy test that is positive.

The following information is on her summary sheet:


Past medical history
G9P6M2
Miscarriage (nine months ago)
Hepatitis B (HBeAg negative)
Latent TB (completed nine months isoniazid therapy under guidance of
TB clinic)
Medication
Nil prescribed
Allergies
Nil known
Immunisations
Completed standard refugee catch-up schedule
Family history
Unknown
Social History
Alcohol—nil
Smoking—nil
Lives with husband and six children (ages 2–12) Page 425
Cervical screening
None recorded.

Instructions for the patient, Zahra Mohammed


Ibrahim
You have presented today because you think you are pregnant. Your last
period was about eight weeks ago (you are uncertain of the exact dates). You
speak Somali and have very little English, although attend English classes
three days per week when the children aren’t at home sick. Please accept an
interpreter if offered, and if not, give single-word answers or feign confusion.
You are happy about the pregnancy but a little worried because of your
miscarriage last year. You have had one other miscarriage 10 years ago when
you were living in a refugee camp. You did not seek any preconception
counselling and are not taking any medications or vitamin tablets. Large
families are normal in your culture. You are mildly concerned about where
you are going to fit another child in your three-bedroom housing commission
home.
You have been pregnant many times before but have not delivered in
Australia. Your previous deliveries have been at home or in a refugee camp
without medical assistance. You have delivered naturally without
complications in the past. You have little knowledge regarding what to
expect in an Australian system.
You’ve never heard about Down syndrome screening but would never
terminate a pregnancy. You would accept any child that you were given.
You are feeling well other than occasional mild nausea.
You have hepatitis B and have only a superficial understanding of this,
knowing that it could affect your liver (which you understand is something
inside your belly but don’t really know much more than this). You have had
blood tests and an ultrasound 12 months ago and were told everything was
okay; you have not been back to see your doctor since then.
Your two older children were also diagnosed with hepatitis B on arrival in
Australia. Your husband was told he was immune to hepatitis B, and your
younger children were vaccinated on arrival in Australia. You are aware that
you passed on hepatitis B to your older children and are worried that this may
happen again in this pregnancy.
You do not know what a pap smear/cervical screening test is and have
never had one.
If asked, you have a history of female cutting/female genital Page 426
mutilation (FGM) as a young girl. Your initial delivery was difficult
and prolonged due to this but after this you have not had any obstetric
problems related to FGM.
Your husband is supportive and is also studying English. There is no
history of domestic violence. You receive Centrelink benefits but are hopeful
that your husband will be able to soon find a job.
Prompts:
‘Who will deliver my baby?’
‘Will my baby get hepatitis B?’

The following information is on your summary sheet:


Past medical history
G9P6M2
Miscarriage (nine months ago)
Hepatitis B (HBeAg negative)
Latent TB (completed nine months isoniazid therapy under guidance of
TB clinic)
Medication
Nil prescribed
Allergies
Nil known
Immunisations
Completed standard refugee catch-up schedule
Family history
Unknown
Social history
Alcohol—nil
Smoking—nil
Lives with husband and six children (ages 2–12)
Cervical screening
None recorded.

Information for the facilitator


Clinical examination findings
Well-looking 36-year-old female with dark skin wearing traditional Somali
clothing including hijab
Height 167 cm
Weight 89 kg
BMI 36 kg/m2
BP 125/76 mmHg Page 427
Pulse 78/min regular
Cardiovascular/respiratory/thyroid examinations normal
Abdomen—no peripheral signs of cirrhosis, abdomen soft non-tender, no
masses including no fundus felt
Cervical screening/genital examination—to be deferred to later consult
Urine pregnancy test is positive
Urine dipstick negative
Finger-prick blood sugar level 6.1 mmol/L (non-fasting)
Edinburgh postnatal depression score 0.

Suggested approach to the case


Establish rapport
Level of English and need for interpreter should be established early
Open-ended questions to explore Zahra’s ideas, concerns and expectations
A good candidate will explore Zahra’s pre-existing knowledge/health literacy
rather than assuming a level of knowledge or lack of knowledge.

Specific questions
Determine whether pregnancy is planned and wanted
Date of last menstrual period
Ask about previous pregnancies and complications
Ensure Zahra is well
Clarify past medical history, current medications, allergies, family history
Smoking/alcohol
Ask regarding previous cervical screening and FGM
Explore social history and support network, and mental wellbeing.

Examination
General appearance
Blood pressure and heart rate
Height, weight and BMI
Cardiovascular/respiratory/thyroid examination
Abdominal examination—palpate for fundus and liver examination
Urine dipstick
Finger-prick BSL
Edinburgh postnatal depression scale.

Management Page 428


Explain briefly about models of antenatal care in Australia (noting that Zahra
is inappropriate for shared care due to obesity and hepatitis B)
Discuss need for folate and iodine. As Zahra’s BMI is >35, she will require a
5 mg dose of folate
Advise regarding smoking and alcohol recommendations
Discuss healthy diet in pregnancy and avoiding high-risk foods
Mention cervical screening, this may require an additional specific
appointment to further discuss and explain
Consider influenza vaccination
Dating scan is appropriate as Zahra does not know exact dates
Routine antenatal blood tests including FBC, blood group and antibodies, hep
C, syphilis, HIV serology, rubella serology, urine MCS
Hepatitis B:
• request LFTs and hepatitis B quantitative DNA level given known
hepatitis B
• good candidates will recognise she is also overdue screening for
hepatocellular carcinoma
• candidate should reassure Zahra that her hepatitis B will be monitored
and treated if necessary, and that baby will have immediate vaccination
and hepatitis B immunoglobulin and that it is highly unlikely that she will
transmit hepatitis B to this baby.
Zahra is at risk of vitamin D deficiency and should have this checked
She has risk factors for gestational diabetes, including obesity and ethnicity
and should have an early oral glucose tolerance test or HbA1c depending
on local hospital recommendations
Urine protein: creatinine ratio is appropriate given high BMI
Consider checking TSH and varicella serology
First trimester screening can be mentioned but full discussion may be
deferred to a later consult
Likewise, non-invasive prenatal testing can be mentioned, although it is
unlikely Zahra will have the funds to access this
Candidates should recognise that Zahra has a high-risk pregnancy and should
be referred early for antenatal care and management of her hepatitis B.
CASE COMMENTARY

Candidates require a good knowledge of routine antenatal care,


plus some knowledge and understanding of the challenges
involved in refugee health care. About a third of women Page 429
who give birth in Australia were not born in Australia
and this can provide challenges with regards to differing cultural
expectations or views surrounding the birthing experience. In
particular, women with migrant or refugee backgrounds may
present later for antenatal care and may have other more
complex medical and psychosocial problems.1
A phone interpreter should be offered early in the consult.
Phone interpreters are available free of charge to healthcare
providers via the National Translating and Interpreting Service (
www.tisnational.gov.au). Some tips for working with
interpreters include:
• speak to the patient not the interpreter
• use eye contact and speak in the first person
• allow time for the interpreter to interpret what you say
• ensure that both you and the patient are in proximity to
the speaker phone, and that you are speaking clearly for
the interpreter to hear you.2
Candidates should be mindful that interpreters are not
medically trained and may have low health literacy themselves.
Care needs to be taken when using interpreters to avoid the use
of complex language or jargon that is hard to translate. Small
amounts of information at a time should be given to ensure
things are not forgotten or missed. It can be tempting to use
family members or children as translators, but this should be
avoided when possible.3
Health information handouts can be given in English or
Somali, but it is worth keeping in mind that many refugees
(women in particular) may not have attended school and will not
understand written information in either language.
Female genital mutilation is extremely common in Somalia
and should be considered as it can complicate deliveries and
result in difficult and/or traumatic examinations.4
Chronic hepatitis B is also common throughout Africa,
usually through vertical transmission. More than 90% of infants
infected at birth will go on to have chronic hepatitis B and its
associated health outcomes. In contrast 95% of infected older
children and adults will clear the infection without developing
chronic disease. Fortunately, in Australia, perinatal transmission
can be easily prevented through strategies including antiviral
medication in the 3rd trimester for women with high viral loads,
hepatitis B immunoglobulin and immediate vaccination for
infants at birth, followed by a full hepatitis B vaccination course.
Infants born to mothers with hepatitis B should have hepatitis B
serology at 9–12 months of age.5

References Page 430

1. Department of Health 2018, Clinical Practice Guidelines: Pregnancy Care.


Australian Government Department of Health, Canberra.
2. Phillips, C 2010, ‘Using interpreters—a guide for GPs’, Australian Family
Physician, vol. 39, pp. 188–95.
3. Bird, S 2010, ‘Failure to use an interpreter’, Australian Family Physician,
vol. 39, pp. 241–2.
4. Royal Australian and New Zealand College of Obstetricians and
Gynaecologists 1994, revised November 2017. Female Genital Mutilation.
Available at: www.ranzcog.edu.au/RANZCOG_SITE/media/RANZCOG-
MEDIA/Women%27s%20Health/Statement%20and%20guidelines/Clinic
al%20-%20Gynaecology/Female-Genital-Mutilation-(C-Gyn-1)-Nov17.pd
f?ext=.pdf, accessed 1 July 2018.
5. Mathews, G, Robotin, M & Allard, N (eds) 2014, ‘B Positive—all you
wanted to know about hepatitis B: a guide for primary care providers’,
Australasian Society for HIV, Viral Hepatitis and Sexual Health Medicine,
Darlinghurst. Available at: www.hepatitisb.org.au, accessed 1 July 2018.
Page 431
Section 24
Vivas
Page 432

Case 84
Lori Dalton

Instructions for the doctor


This is a short case.
Mrs Lori Dalton is a 67-year-old retired journalist. She is a keen
bushwalker but is frustrated because of pain in her osteoarthritic knees, for
which she takes oral ibuprofen 200 mg three times daily as needed. She has
brought in the following abstract from an article in Australian Family
Physician on rosehip powder. She will ask you questions about this.

Rosehip powder for arthritis and inflammatory bowel disease


Background: Rosehips—which contain a particular type of galactolipid—
have a specific anti-inflammatory action. A standardised rosehip powder
has been developed to maximise the retention of phytochemicals. This
powder has demonstrated antioxidant and anti-inflammatory activity as
well as clinical benefits in conditions such as osteoarthritis, rheumatoid
arthritis and inflammatory bowel disease.
Objective: To examine the evidence suggesting that standardised rosehip
powder may be a viable replacement or supplement for conventional
therapies used in inflammatory diseases such as arthritis.
Discussion: A meta-analysis of three randomised controlled trials
involving 287 patients with a median treatment period of three months
reported that treatment with standardised rosehip powder consistently
reduced pain scores and that patients allocated to rosehip powder were
twice as likely to respond to rosehip compared to placebo. In contrast to
non-steroidal anti-inflammatory drugs and aspirin, rosehip has anti-
inflammatory actions that do not have ulcerogenic effects and do not
inhibit platelets nor influence the coagulation cascade or fibrinolysis.1

Instructions for the patient, Lori Dalton Page 433

You are a 67-year-old retired journalist. You are a keen bushwalker but get
frustrated because of the pain in your osteoarthritic knees. You have seen an
advert for rosehip powder at your local health food shop and want to know
whether to use this during your planned trekking holiday in Nepal. The advert
quoted an article in the Australian Family Physician,1 so you have brought in
the abstract to ask your GP about it.
You will ask the GP the following questions:

1. What do you think about this article, doctor? Do you think that the
information is reliable? (Supplementary question: Why do you think
that?)
2. The article talks about randomised controlled trials. Please would you
explain what these are. And I’ve heard of some trials being ‘double
blind’. What does this mean?
3. Do you think the rosehip powder is worth trying?
4. Will the rosehip powder be safe for me to take while also taking
ibuprofen?
5. Should I take it with me when I go on my trekking holiday to Nepal?

Suggested answers
1. What do you think about this article, doctor? Do you think that the
information is reliable? (Supplementary question: Why do you think
that?) This article is about using rosehip powder to help with
inflammatory conditions such as arthritis.
Factors to consider2, 3
• Journal: the Australian Family Physician is a peer-reviewed,
professional journal, indexed by Medline. Reputable journals with
high impact factors are regarded as reliable, but they still require
systematic scrutiny.
• Article: sponsored or not? Are there conflicts of interest?
• Level of evidence: meta-analysis, but not clear in abstract who
conducted this, or what methods were used. There is no indication on
the inclusion or exclusion criteria.
• Gold standard is methodology used by the Cochrane collaboration.4
Consider and explain what the levels of evidence are and how these
support recommendations.
• Trials: were all trials of rosehip powder included or only those
published?
• Statistical and clinical significance: in this paper, response is Page 434
described as ‘twice as likely’; it is not clear if this is the
absolute or relative risk of benefit. There is no indication of the
sample sizes or statistical power.
• Author(s): publication history, qualifications, academic affiliations,
financial ties to research topic or conflicts of interest.
2. The article talks about randomised-controlled trials. Please would you
explain what these are. And I’ve heard of some trials being ‘double
blind’. What does this mean?
• Randomised controlled trials test whether treatments work.2
• A group of people, all with the same clinical problem, are randomly
allocated to have different treatments. The effect of a new treatment is
compared to an inert, look-alike pill (the placebo) or another
treatment.
• Ideally trials are ‘double blinded’ so that neither the patient nor the
person who assesses the effects of the treatments know which
treatment the patient is on. This reduces the potential for bias among
the groups.
• Calculations are done before the trial to work out how many
participants are needed (the power of the trial) to be sure any result is
both clinically and statistically significant.
3. Do you think the rosehip powder is worth trying? From the information in
this article it may be worth trying, but it is not something that I have
previously studied or recommended. The Therapeutic Goods
Administration (TGA)—the Australian government organisation that
checks whether medicines work and are safe—lists products that are
approved for use; we can examine the TGA and other sources of
information. Within the article the number of people who have tried it
seems very low compared to the number who have used other anti-
inflammatories and painkillers. Also, the article mentions people who
have taken it for a short time only and so its long-term safety is uncertain
from this information.
It is not clear what type of arthritis the participants in this study had,
whether they were men or women and what age they were. This makes it
more difficult to know if the results apply to you.
4. Will the rosehip powder be safe to take while also taking ibuprofen? I’m
sorry, Mrs Dalton, but I cannot answer that question from this article. The
participants were on either the rosehip powder or a dummy placebo pill.
There is no information on the safety of mixing the rosehip powder with
other medications.
5. Should I take it with me when I go on my trekking holiday to Page 435
Nepal? A general rule is to not start taking a medication for the
first time while overseas. If you do want to take it with you, I would
suggest a trial of several weeks before you go, so that if you have a bad
response to it you will know while you are still in Australia and will have
time to recover before you actually go away. It is also worth checking to
see if rosehip powder can legally be taken into Nepal.
There’s another option. Your dose of ibuprofen is quite low. Providing
you are not getting any side effects from it, you could increase the dose,
add regular paracetamol, and try walking poles and leg strengthening
exercises.

CASE COMMENTARY

This case tests the doctor’s ability to appraise an abstract and


then apply the information to the care of individual patients.
General practitioners are not usually expected to conduct
research but all should be able to read research information and
apply it to their practice. One trap to avoid is for GPs to concur
with the belief that ‘natural’ automatically equates to it being
harmless.5 The origin of a product is much less important than
its proven efficacy and short and long-term safety.

References
1. Cohen, M 2012, ‘Rosehip—an evidence-based herbal medicine for
inflammation and arthritis’, Australian Family Physician, vol. 41, no. 7,
pp. 495–8.
2. Greenhalgh, T 2014, How to read a paper: the basics of evidence-based
medicine, 5th ed, British Medical Journal Books.
3. Margolis, S 2018, ‘Evidence-based medicine’, Australian Journal of
General Practice, vol 4, no. 6.
4. Spurling G, Mitchell B & van Driel M 2018, ‘Unlocking the value of
Cochrane reviews for general practitioners’, Australian Journal of
General Practice, vol. 47, no. 6, pp. 333–6.
5. Smith, A 2002, ‘It’s natural so it must be safe’, Australian Prescriber, vol.
25, pp. 50–1.

Further reading
1. van Driel, M & Spurling, G 2017, ‘Guidelines and systematic reviews:
Sizing up guidelines in general practice’, Australian Family Physician,
vol. 46, no. 6, pp. 438–40
2. The Royal Australian College of General Practitioners 2018, Guideline for
the management of knee and hip osteoarthritis, 2nd ed, RACGP, East
Melbourne, Vic.
Page 436

Case 85
Kaitlin Johansen

Instructions for the doctor


This is a short viva station.
Please discuss your management of this situation with a GP colleague.

Scenario
You are working an evening clinic in general practice and a patient of
your practice, Kaitlin Johansen, comes in unexpectedly, tearful, reporting
she has been raped at a party. Your last patient cancelled, so you can see
Kaitlin now.

The following information is on her summary sheet:


Age
19
Past medical history
Asthma
Medications
Fluticasone 250 mcg/salmeterol 25 mcg (Seretide) 1 puff bd
Salbutamol inhaler 100 mcg 1–2 puffs prn
Social
Studying beauty therapy
Allergies
Nil known
Immunisations
Up-to-date
Cervical screening
None.

Instructions for the facilitator Page 437

This is a viva station.


Please ask the candidate the following questions about the case:

1. What are your immediate medical (including psychological) concerns for


Kaitlin?
2. She tells you that she does not want to involve the police. How would you
respond?
3. You arrange for the Sexual Assault Referral Centre to perform a forensic
examination in two hours. She needs to go to the toilet. Is there anything
you need to advise her?
4. Tell me the important things to consider about your notes regarding this
consult?
5. What follow-up will you arrange for Kaitlin?

Suggested approach to the case


1. Medical priorities
Physical injuries: assess Kaitlin with an initial primary survey and
then more detailed secondary survey for specific injuries.
STI screening and bloodborne viruses: possible prophylactic treatment
for chlamydia, gonorrhoea, HIV and syphilis, and hepatitis B
immunisation depending on her predicted risk.
Pregnancy prevention: offer emergency contraception.
Mental health: assess Kaitlin’s current mental state, and her ongoing
safety and support. She may wish to call a support person and may need
help to find a safe place to go following examination.
Most areas have a Sexual Assault Resource Centre (SARC) and/or
telephone hotline which provide expert support. General practitioners
often follow up patients who have experienced trauma.
2. When alleged assault is disclosed it is important to offer police
involvement. As an adult, this is Kaitlin’s choice. If she would like to
report the offence you can call the police immediately. However, if she
doesn’t wish to involve them, then forensic specimens can still be
collected in case she changes her mind. Forensic specimens have to be
collected promptly.
3. Contact the local SARC services and establish the most appropriate place
for a timely forensic examination. If there will be some waiting time and
Kaitlin is wanting to go to the toilet you might be asked to collect initial
forensic samples, such as gauze swab of perineum and anal Page 438
region, first pass urine and mouth washings, depending on details
of the assault. SARC staff will help you decide necessary initial
specimens.
4. Your notes may be used in legal proceedings so they must be written
objectively and without judgement. Contemporaneous notes are
imperative. It is recommended to write her history verbatim and
distinguish this clearly from your clinical observations and management.
5. Follow-up may include a multidisciplinary team. Follow up two to three
days post-incident to reassess injuries and wellbeing. After two weeks
repeat a pregnancy test and follow up STI checks, assess her coping,
healing and mental state. After three months repeat STI/bloodborne virus
tests. Do a mental health assessment, offer ongoing counselling if
necessary and SARC/legal follow-up as desired.
CASE COMMENTARY

In 2011 there were 17 238 reports of sexual assault in Australia.


This is likely to be lower than the true prevalence as the majority
of victims do not report incidents to the police. The Australian
Bureau of Statistics Personal Safety survey in 2016 showed that
18% of women and 4.7% of men had experienced at least one
episode of sexual violence since the age of 15.
The risk factors for sexual assault include:
• young age
• Aboriginal or Torres Strait Islander people
• female
• substance users
• those suffering mental health issues and/or disability
• previous history of sexual assault
• homelessness
• poverty
• sex workers.
Disclosures of assault may happen immediately or years
after. Other presentations to a GP following assault may be for
emergency contraception or STI check and a thorough history
and good rapport will be essential to exploring the presentation.

COMMON PITFALLS Page 439

It is common to feel unqualified in a presentation such as this,


however, despite specialist services being available, immediate
care often needs to be given by the first treating practitioner. A
general practitioner is often a key part of the ongoing care of
someone post-sexual assault. A history of assault is often
implicated in other presentations such as chronic pelvic pain.
Considering this, it is important to understand the basics in
responding to an alleged assault.

Suggested resources
Daisy App helps people connect to local resources. Available at: https://www
.1800respect.org.au/daisy/
RACGP White Book. Available at: https://www.racgp.org.au/clinical-resourc
es/clinical-guidelines/key-racgp-guidelines/view-all-racgp-guidelines/whi
te-book/interpersonal-abuse
For local resources in your state check this website: https://au.reachout.com/a
rticles/sexual-assault-support

Further reading
The Royal Australian College of General Practitioners 2014, Abuse and
violence: Working with our patients in general practice, 4th ed, RACGP,
Melbourne, Vic.
Guidelines for medico-legal care for victims of sexual violence © World
Health Organization 2003 Retrieved from: http://apps.who.int/iris/bitstrea
m/handle/10665/42788/924154628X.pdf;jsessionid=83BD690CD8FE199
FB8EB39ABA1385216?sequence=1, accessed 26 February 2019.
Tarczon, C & Quadara, A 2012, ‘The nature and extent of sexual assault and
abuse in Australia’, Australia Centre for the Study of Sexual assault,
Melbourne. Retrieved from: https://aifs.gov.au/sites/default/files/publicati
on-documents/rs5.pdf, accessed 26 February 2019.
Australian Bureau of Statistics’ 2016 Personal Safety Survey. Available at: ht
tps://www.abs.gov.au/ausstats/[email protected]/Lookup/4906.0main+features12
016, accessed 9 March 2019. Page 440
Page 441
Section 25
Vulnerable
populations
Page 442

Case 86
Jill Krecher

Instructions for the doctor


This is a long case.
Please take a history from Jill and ask for the physical examination
findings from the facilitator. Talk to Jill about your initial diagnosis or
concerns and discuss an initial plan of management with her.

Scenario
Jill Krecher is a 29-year-old woman who has been to the surgery four
times for minor ailments in the last few months. The receptionist told
you she sounded teary on the phone when she booked for an appointment
today.

The following information is on her summary sheet:


Past medical history
Anxiety—treated 2013–14
Menorrhagia—inactive
Medication
Mirena inserted September 2017
Allergies
Nil known
Immunisations
As per schedule
Social history
Teacher at local primary school
Lives with fiancé Paul, who is an accountant
Non-smoker
Social drinker with friends on the weekend
No recreational drugs
Family history
Dad—hypertension
Mum—osteoarthritis knees.

Instructions for the patient, Jill Krecher Page 443

You have asked to see the doctor urgently as you can’t teach today due to
your bruises. You have a black eye, which you can’t cover with make-up and
bruises on your arms. Your partner Paul has become increasingly violent.
You haven’t been able to tell anyone and have hidden the bruises. This
weekend he has been very violent. He’s never hit you in the face before.
You have had some time off work recently as his behaviour has been
upsetting you so much, but you’ve told the doctors it’s either gastro or a viral
illness.
Today you think the doctor will probably notice the bruises and if they ask
you will tell them what’s going on. You feel so much shame about being in
this situation you can’t bring it up unprompted.
Initially, tell the doctor you’ve had trouble sleeping and don’t think you
can teach today.
If you trust them and they ask questions about your situation you will tell
them you had an argument with your partner and ‘things have been a bit
rough lately’. If they ask about violence or if you feel safe you will confide in
them.
You’ve been with Paul for two years. The first year he was like a dream
come true and he swept you off your feet. All your friends and family loved
him too. But he then became jealous and slowly you’ve stopped seeing
friends and family.
He first hit you six months ago after a male colleague chatted to you at a
local café. He thought you were cheating. His violence is getting more
frequent and last night he strangled you until you almost passed out.
Your friends seem worried about you, but they have stopped ringing as
often as you have been withdrawn.
There are no children involved.
You would like to consider leaving but have no idea where to start and
you are scared about what Paul will do if you leave.

The following is on your summary sheet:


Past medical history
Anxiety—treated 2013–14
Menorrhagia—inactive
Medication
Mirena inserted September 2017
Allergies
Nil known
Immunisations
As per schedule
Social history Page 444
School teacher at local primary school
Lives with fiancé Paul who is an accountant
Non-smoker
Social drinker with friends on the weekend
No recreational drugs
Family history
Dad hypertension
Mum osteoarthritis knees.
Information for the facilitator
Each aspect is to be asked for specifically.
General appearance: pale, teary woman; obvious bruising to right eye and
some conjunctival injection
BP 110/70 mmHg
Pulse 76 bpm
Height 165 cm
Weight 60 kg
BMI 22.1 kg/m2
Waist circumference 76 cm
Urine NAD
If skin exposed, slight bruising to neck
Different-aged bruises to both arms
All other systems NAD.

Suggested approach to the case


Establish rapport
Empathic, open and non-judgemental approach
Start with open questions: ‘How are things at home?’
Specific questions: ‘Are you ever afraid of your partner?’
In response to this patients’ specific injuries you might say something along
the lines of:
‘When I see injuries like this, I wonder if someone could have hurt you?’
‘You seem very anxious and nervous. Is everything alright at home?’

Validate
Use statements such as:
‘You deserve to feel safe’
‘You do not deserve to be treated this way’ Page 445
‘I am here to help you’
‘I am concerned about your safety’
Avoid statements such as:
‘Why don’t you leave?’
‘Why did he hit you?’
‘You should leave’
Traumatised patients often need time. You may not have much time in an
initial consultation, but early follow-up and enlisting the help of a team
help the patient to get the time they may need.

Assess safety
Does the patient feel safe?
Are there any red flags?
Is the violence escalating?
In this case the patient has experienced non-lethal strangulation. Studies show
this increases the risk of attempted and completed homicide four to six
fold in the future. This behaviour needs to be recognised as a significant
risk.

Assist in safety planning


The elements of safety planning include a list of emergency numbers, access
to money, important documents such as passport, birth certificate, driver’s
licence, Medicare card, clothing and keys.
Establish where she would go if she had to leave
How would she get there?
What would she take with her?
Who are the people she could contact for support?
Does she need referral to domestic violence services to assist with
planning?
Document any plans made, for future reference
Organise early follow-up.

CASE COMMENTARY

GPs often comment that they do not see many victims of


domestic violence; however, it is estimated that on average GPs
see five women a week who are affected. 17% of all women and
5.3% of all men aged > 18 years have experienced violence from
a partner.
Evidence shows that four to ten pregnant women out Page 446
of every 100 experience intimate partner violence.
Aboriginal and Torres Strait Islander people are over-
represented in domestic and family violence statistics.
Domestic violence has a devastating effect on individuals and
communities. Victims of domestic violence often suffer adverse
outcomes, including depression and anxiety, post-traumatic
stress disorder, physical injury and disability. Injuries can be
severe and sometimes fatal, which is why increasing resources
are available to deal with this important health issue.
The World Health Organization recommends that GPs ask
women about intimate partner abuse as a part of assessing the
conditions that may be caused or complicated by intimate
partner abuse. These include mental health symptoms, alcohol
and other substance use, chronic pain or chronic digestive or
reproductive symptoms.
GPs can offer frontline support to the victims of domestic
violence, including asking about violence, listening, validating
the patient’s experience and providing non-judgemental practical
support. Directing patients to resources and support helps to
enhance their safety.
When children are involved and at risk, GPs need to practise
according to the mandatory reporting requirements in their
jurisdiction.
A multidisciplinary approach is important; social workers,
nurses, counsellors and psychologists can all provide assistance.
Dedicated domestic violence services within the police
department may also be able to assist, particularly in a case
where patients would like to press charges or seek protective
measures such as restraining orders.
COMMON PITFALLS

Most women don’t mind being asked about family violence by


their GP but a minority are actually asked. Given the high
prevalence of family violence, we need to remember to ask
about the safety of our patients.

Suggested resources
Daisy App helps people connect to local resources. Available at: https://www
.1800respect.org.au/daisy/
RACGP White Book, Chapter 2 Intimate partner violence. Available at: https
://www.racgp.org.au/clinical-resources/clinical-guidelines/key-racgp-guid
elines/view-all-racgp-guidelines/white-book/interpersonal-abuse

Further reading Page 447

Australian Domestic and Family Violence Death Review Network (2018)


Data Report. Retrieved from: www.coroners.justice.nsw.gov.au/Documen
ts/ADFVDRN_Data_Report_2018%20(2).pdf?fbclid=IwAR0Y6Wnr6e6q
zv4IY_PRZPMfJ-OugwkIaF5XhVIN-HQydtA6BDj-aMa42aY, accessed
26 February 2019.
Brooks, M, Barclay, L & Hooker, C 2018, ‘Trauma-informed care in general
practice: Findings from a women’s health centre evaluation’, Australian
Journal of General Practice, vol. 47, no. 6, pp. 370–5.
Glass et al. 2008, ‘Non-fatal strangulation is an important risk factor for
homicide of women’, The Journal of Emergency Medicine, vol. 35, no. 3,
pp. 329–35.
Forsdike, K, Tarzia, L, Hindmarsh, E & Hegarty, K 2014, ‘Family violence
across the life cycle’, Australian Family Physician, vol. 43, no. 11, pp.
768–74.
The Royal Australian College of General Practitioners 2014, Abuse and
violence: Working with our patients in general practice, 4th ed, RACGP,
Melbourne, Vic.
Page 448

Case 87
Marcus Petrovic

Instructions for the doctor


This is a short case.
Please take a history from Marcus and then request appropriate
examination findings from the facilitator. Outline your diagnostic
impressions and problem list to the facilitator. Negotiate your management
plan with Marcus.

Scenario
Marcus Petrovic is 28-year-old man who moved to your practice six
months ago. He has a 10-year history of schizophrenia with frequent
hospital admissions. In the last year he has been relatively stable on
fortnightly Zuclopenthixol (Clopixol) depot injections and nightly
quetiapine (Seroquel).
He is obese, has borderline hypertension and is not very physically
active. He has a family history of diabetes, heart disease and
dyslipidaemia. He smokes but does not drink alcohol. He is unemployed
and is on a disability support pension.
Your previous impression has been that he seems quite blunted in
affect and you suspect he has at least some moderate intellectual and/or
developmental delay, which gives him a child-like manner. Six months
ago, he moved to your town to live with his parents but he has recently
moved into a flat with friends. He is on the waiting list to see a local
psychiatrist but has yet to engage with local mental health services.
He takes his medication, recognising that it has probably contributed
to his good run in recent years. He presents today in between his
scheduled fortnightly appointments.

The following information is on his summary sheet: Page 449


Past medical history
Chronic schizophrenia
Intellectual disability
Gastro-oesophageal reflux disease
Medication
Zuclopenthixol (Clopixol) depot injection IMI 300 mg/mL every two
weeks
Quetiapine (Seroquel) XR 300 mg nocte
Gaviscon tabs po prn
Allergies
Nil known
Immunisations
Unknown
Social history
Smokes 15 cigarettes per day
Non-drinker
Lives with friends.

Instructions for the patient, Marcus Petrovic


The area of redness and soreness in your right elbow began a couple of days
ago and is getting bigger each day. Early on in the discussion you say, ‘I
think I must’ve used a dirty needle’.
You readily volunteer information about your drug use and are not
embarrassed about it. You’ve been using either ice or heroin (‘Harry’) for the
past month, initially irregularly but now daily. You have never used drugs
before but have been introduced to them by your new group of friends. You
have never paid anything as your new friends (from the motorcycle gang)
supply them for free along with the needles and injecting equipment. If
specifically asked, your friends sometimes get you to ‘do a Sydney run’—to
ride your bike to Sydney and back with a package, or to carry packages
around town. You have no idea what you are carrying on these trips.
You have limited understanding about safe injecting or bloodborne
viruses. You are not concerned about the risks of IV drug use but don’t like
this infection because it is painful and you can’t use that area for injecting.
You like the way the drugs make you feel relaxed and happy. Other than this
infection you haven’t experienced any negative effects from your drug use
and haven’t contemplated quitting.
You talk animatedly about your ‘new’ bike (a 1974 Honda CB750) that
your new friends helped you buy and look after. You are proud of your new-
found independence—living in a flat with friends instead of living with your
parents. You are positive about your new group of friends in the Page 450
motorcycle gang and are not aware that they might be taking
advantage of you.
You trust the doctor and have a good relationship but struggle to understand
anything even slightly complex. You don’t understand any medical jargon
and get easily lost if the doctor uses long words or speaks too quickly.

The following information is on your summary sheet:


Past medical history
Chronic schizophrenia
Intellectual disability
Gastro-oesophageal reflux disease
Medication
Zuclopenthixol (Clopixol) depot injection IMI 300 mg/mL every two
weeks
Quetiapine (Seroquel) XR 300 mg nocte
Gaviscon tabs po prn
Allergies
Nil known
Immunisations
Unknown
Social history
Smokes 15 cigarettes per day
Non-drinker
Lives with friends.

Suggested approach to the case


Establish rapport. Use an open and non-judgemental approach
Recognise and manage cellulitis as a result of intravenous drug use
Recognise the broader risks of unsafe injecting
Identify Marcus’s vulnerability with respect to drug trafficking.

Specific questions
Explore the history of drug use—its context and Marcus’ awareness of
risks/safe injecting
Assess mood and thought content as well as any psychotic symptoms and, if
present, their relationship to the drug use
Explore Marcus’s awareness of his current vulnerability with his new friends
and with the police.

Examination Page 451

General appearance
Vital signs
Examine right antecubital fossa for possible abscess; examine both
antecubital fossae for track marks
Cardiovascular examination.
Management
Manage the presenting complaint (cellulitis) with appropriate antibiotics, e.g.
flu/dicloxacillin to cover Staphylococcus aureus, which is the likely
causative organism. Add in a topical skin wash such as chlorhexidine.
Outline the health risks of IV drug use to Marcus in words he understands.
Explain the need for safe injecting as well as testing for bloodborne
viruses and offer immunisation against hepatitis B.
Gently introduce the idea that his friends may be taking advantage of him.
Ensure follow-up.

Physical examination
General appearance: obese man of stated age in no distress
Vital signs
HR 78/min regular
BP 135/82 mmHg sitting
Temp 36.8°C
Height 178 cm
Weight 121 kg
BMI 38 kg/m2
RR 14/min
Right antecubital fossa has a round area of redness, warmth and induration of
the skin with no evidence of any underlying abscess
Both antecubital fossae have several small marks consistent with fresh
puncture wounds
Cardiovascular examination: normal
Respiratory examination: normal
Abdominal examination: normal
Mental state examination: normal apart from a blunted affect and limited
insight
Surgery tests: ECG—sinus rhythm 75/min; BSL (random)—6.2; Urinalysis
—normal
Remainder of physical examination is normal and further investigations are
not available.
Diagnostic impressions/problem list Page 452

Cellulitis—likely related to intravenous drug use with non-sterile equipment


Intravenous use of opiates (heroin) and methamphetamine (‘ice’)
Risk of contracting bloodborne viruses
Risk of drug use causing psychosis
Social vulnerability—currently being used for drug transportation
Existing problems of chronic schizophrenia, intellectual impairment and
obesity At risk of metabolic syndrome in the future.

CASE COMMENTARY

Although research findings vary, it seems most adults with


intellectual disability are less likely to use substances than adults
without intellectual disability.1 This may be related to the fact
that a substantial number of people with intellectual disability
live with families or other caregivers who provide some degree
of supervision.2 With the move to integrate patients with
intellectual disability into the community comes potential
exposure to substances such as alcohol and drugs and these
vulnerable patients can lack the skills to cope with this. Patients
with intellectual disability who do use alcohol and illicit drugs
are more likely to develop an abuse problem.2 Additionally,
many of the intellectual disability population do not engage well
with standard interventions for substance misuse which
frequently fail to meet their needs.3 As a result, intellectual
disability substance abusers are less likely to receive substance
abuse treatment or remain in treatment.3, 4 When intellectual
disability patients also suffer mental illness, the risk of serious
problems with substance abuse increases further.3–5 This group
is at greater risk of complications because they tend to be
prescribed medications for co-existent mental illness that can
negatively interact with alcohol and drugs.3, 5
Patients such as Marcus risk the ‘triple whammy’ of chronic
mental illness, intellectual disability and substance abuse,
resulting in significant disability and real barriers to effective
treatment. The skilled general practitioner can provide
individualised longitudinal care through a trusted relationship
and can support engagement in services and build resilience over
time.6

COMMON PITFALLS

If the doctor adopts a judgemental or punitive role, the


likelihood of Marcus adhering to treatment and remaining
engaged with the GP is reduced. Also, if the candidate fails to
target the advice and information to an appropriate level Page 453
then the patient education is ineffective. Under the
pressure of time in a short case we can try to cover all the
content quickly and forget to check for patient understanding.
The other pitfall here is to focus on the simple condition of
localised cellulitis or to only expand as far as addressing the risk
of bloodborne viruses and safe injecting but to avoid the broader
psychosocial issues as being firmly in the ‘too hard basket’. The
opportunity to list the identified diagnoses (or more broadly,
problems) represents the chance to demonstrate the candidate’s
thinking regarding Marcus’s health even though they will not get
to address all these issues with Marcus within the time frame of
the case.
Marcus is vulnerable and in need of advocacy and a holistic
approach to his care.
The insightful GP will recognise that without significant
intervention the next likely outcome will involve the authorities
and will probably result in Marcus’s incarceration. This would
have further significant negative impacts on his health.
References
1 Carroll-Chapman, SL & Wu L 2012, ‘Substance abuse among individuals
with intellectual disabilities’, Research in Developmental Disabilities, vol.
33, pp. 1147–56.
2 Emerson, E & Brigham, P 2013, ‘Health behaviours and mental health
status of parents with intellectual disabilities: Cross sectional study’,
Public Health, vol. 127, pp. 1111–16.
3 Day, C, Lampraki, A, Ridings, D & Currell, K 2016, ‘Intellectual
disability and substance use/misuse: a narrative review’, Journal of
Intellectual Disabilities and Offending Behaviour, vol. 7, no. 1, pp. 25–34.
4 Lin, E, Balogh, R, McGarry, C et al. 2016, ‘Substance-related and
addictive disorders among adults with intellectual and developmental
disabilities: an Ontario population cohort study’, British Medical Journal
Open, vol. 6, p. e011638.
5 Chaplin, E, Gilvarry, C & Tsakanikos, E 2011, ‘Recreational substance
use patterns and co-morbid psychopathology in adults with intellectual
disability’, Research in Developmental Disabilities, vol. 32, no. 6, pp.
2981–6.
6 Samuel, S 2016, ‘GPs and vulnerable populations’ (editorial), Australian
Family Physician, vol. 45, no. 10, p. 697. Page 454
Page A

Figure 1 Teenage acne (see page 14).


Photo courtesy of iStockphoto
Figure 2 Rash on four-month-old baby (see page 114).
Photo courtesy of the Eczema Association of Australasia Inc.
Page B

Figure 3 Rash on outside of left elbow (see page 120).


Photo courtesy of iStockphoto

Figure 4 Close up of Ken’s lesion (see page 123).


Photo courtesy of Dr Andrew Moreton
Page C

Figure 5 ECG courtesy of Dr Brendan Bell (see page 153).


Page D

Figure 6 Painful, red blurry right eye (see page 172).


Photo courtesy of Dr Andrew Moreton

Figure 7 Close-up of painful, red blurry right eye (see page 172).
Photo courtesy of Dr Andrew Moreton

You might also like